You are on page 1of 124

Station 4 

Communication skills and ethics

CONTENTS
Introduction to communication skills 4.17 Chronic disease 650
and ethics 616 4.18 Discussing an acutely terminal situation
Communication skills 616 with relatives 651
Ethics 617 Confidentiality, consent and capacity 653
Cases 618 4.19 Legal points in confidentiality 653
Discussing clinical management 618 4.20 Breaching confidentiality when a third
party may be at risk 654
4.1 Explaining a diagnosis 618
4.21 Breaching confidentiality in the public
4.2 Explaining an investigation 619
interest 656
4.3 Discussing treatment 622
4.22 Confidentiality when talking with
4.4 Discussing management, prognosis relatives and other third parties 658
and possible complications in a patient
4.23 Consent for investigation or treatment 660
with multiple problems 625
4.24 Consent and capacity 663
4.5 Discussing diagnostic uncertainty 626
4.25 Refusal of consent 668
4.6 Discussing risk and treatment effect 628
4.26 Deliberate self-harm 670
4.7 Negotiating a management plan for
a chronic disease / long-term condition 631 End-of-life issues 673
4.8 Encouraging concordance with treatment 4.27 End of life and palliative care 673
and prevention 633 4.28 Advance decision making 681
Communication in special circumstances 635 4.29 Resuscitation status decision-making –
4.9 Cross-cultural communication 635 discussion with patient 683
4.10 Communicating with angry patients 4.30 Resuscitation status decision-making –
or relatives 636 discussion with relative 688
4.11 Communicating with upset or distressed 4.31 Appropriateness of intensive therapy
relatives 638 unit transfer 690
4.12 Discharge against medical advice 639 4.32 Withholding and withdrawing
life-prolonging treatments – antibiotics
4.13 Delayed discharge 641
and drugs 692
Breaking bad news 643
4.33 Withholding and withdrawing
4.14 Cancer – potentially curable 643 life-prolonging treatments – clinically
4.15 Cancer – probably incurable 646 assisted nutrition and hydration 693
4.16 Cancer – patient not fit for active 4.34 Percutaneous endoscopic gastrostomy
treatment 648 feeding 697

© 2013 Elsevier Ltd 615


Station |4| Communication skills and ethics

4.35 Vegetative state 701 4.43 Fitness to practise – health problems


4.36 Brainstem death 705 in a colleague 722
4.37 Discussing live organ donation 707 4.44 Recruitment to a randomised
controlled trial 723
4.38 Requesting an autopsy (post mortem) 709
Other communication, ethical and
Clinical governance 711 legal scenarios 727
4.39 Critical incident 711
4.45 Genetic testing 727
4.40 Managing a complaint and the question
4.46 HIV testing 729
of negligence 714
4.47 Needlestick injury 730
4.41 Fitness to practise – poor performance
in a colleague 717 4.48 Medical opinion on fitness for anaesthesia 732
4.42 Fitness to practise – misconduct in 4.49 Fitness to drive 734
a colleague 720 4.50 Industrial injury benefits 735

Introduction to communication skills and ethics

We could treat your overactive thyroid with tablets called


COMMUNICATION SKILLS carbimazole or we could use radiotherapy or we could
arrange for you to have an operation. Now which will it be?

Effective communication she would probably respond:


Effective communication in medicine means simply that Well, I don’t know what’s best. You’re the doctor.
doctor and patient (relative, colleague, etc.) understand
each other. We could, broadly, consider two stages to an Patient-centred communication does not mean abdicat-
encounter with a patient. The first stage is concerned with ing decision making to patients. It is the process of provid-
information gathering. The aim is to answer the question: ing patients with the appropriate knowledge and
‘What is the problem or what are the problems?’ The understanding to enable them to make informed deci-
second stage is concerned with developing a management sions and take appropriate responsibility for their care.
plan, aiming to answer the question: ‘How do I solve this Such an approach combines the professional knowledge
problem or these problems?’ Management plans should and opinions of the doctor with respect for the autonomy
be developed not just for patients, but also with patients. of the patient. A better way to approach the thyrotoxic
A plan is seldom a case of saying: patient would be:
You have an overactive thyroid; you will need to take a tablet There are a number of ways to treat an overactive thyroid. One
called carbimazole, would be to take tablets for a few months to see if they settle
but more a case of: it down. They are often very effective, although some patients
do eventually need to have an operation. Another choice
Let’s look at how we could treat your overactive thyroid. would be to consider surgery at the outset, but in my
experience most surgeons prefer to operate if tablets don’t
It is a two-way process of information sharing. The work. A third option would be something called radiotherapy,
but this may not be a good idea if you were planning to
doctor does not so much know the solution to the problem
become pregnant in the near future. I usually prefer patients
(although may know what is clinically best), but reach a to take advice from a doctor who specialises in thyroid
solution through communicating with the patient. This problems before deciding, but wonder if you have any
method of problem solving is more patient centred. immediate thoughts about these treatments.

Doctor- and patient-centred The degree to which we engage in open discussion


depends upon the patient. At one end of the spectrum are
communication
patients with seemingly very little interest in decision 
At first sight, allowing patients to be involved in manage- participation – I’ll go with whatever you think is best, doctor.
ment decisions might seem to be relinquishing medical At the other extreme are patients who ruminate over every-
expertise and merely facilitating what patients want. This thing that happens to them, and want to know everything
would be unprofessional, and we could deservedly ques- – How many stitches would the surgeon need to use, doctor?
tion how patients can know what they want without Patients differ. Yet when we look for evidence-based
medical knowledge and training. Were you to say to the management of disease, we find management plans that
thyrotoxic patient: focus on disease and not the permutations of our patients.

616
Ethics

Traditionally, doctors have been disease centred, but dis-


eases are wrapped in packages that are patients and a ETHICS
fundamental clinical skill is applying evidence-based med-
icine to the spectrum of different patients. Guidelines
might advise against radioactive iodine in pregnancy and The importance of ethical
the patient might say: decision making
I was thinking about having a family soon. Every day in medicine we make ethical decisions. Some of
these decisions are based on documented professional or
But she might also say: legal frameworks. Most are less clearly defined. How much
information about the side effects of a drug should we tell
My mother had a thyroidectomy 30 years ago and had terrible a patient, for example?
swallowing problems afterwards until the day she died. Ethical decision making involves the application of
ethical principles. Often, the decisions we reach seem
Those swallowing problems may not have been associ-
intuitively right. But intuition, whilst valuable, may alone
ated with the thyroidectomy. One could also say that
be inadequate and even detrimental to a patient’s care. For
surgery has moved forward in 30 years. But the patient
one thing, a purely intuitive approach alone may lead to
may nevertheless have decided she does not want to con-
inconsistent behaviour in the way we care for patients,
template surgery. The point is that the optimal management
especially those with complex issues. If we aspire to
of a disease is not automatically the optimal management for
provide the best care, we need to be more analytical.
a patient with that disease.
Taking into account how a patient feels about their
disease and its management is what is meant by patient Ethical principles
centredness. Pure doctor centredness is doctor controlled and Four broad ethical principles underlie ethical decision
tends to be inflexible from patient to patient. Management making (Box 4.1).
decisions are made by the doctor and the patient is
expected to comply. The key to patient centredness is not
to abandon the strengths of disease and doctor-centred Applying ethical principles
practice, but to share information so that decisions about Specific cases have a vital role in ethical argument. Ethics is
management are shared with and acceptable to the patient. not a science where we agree upon a set of principles and then
Patient centredness and doctor centredness should work deduce what is right or wrong from these. Rather, the implica-
together, the former relating to clinical knowledge of tions of the principles in specific cases must be explored.
disease and how it should be managed, the latter a means Ethical reasoning is an interactive process that involves
of harnessing the patient’s perspective to convert that consideration of individual situations and application of
knowledge into an acceptable management plan. Because ethical principles. One logical requirement to this process
patients have a fundamental ethical right to be involved is consistency. If what you believe to be right in one situ-
in decisions about managing their disease, doctors have a ation seems inconsistent with what you believe to be right
duty to be patient centred. in another, you must either identify a morally relevant
Patient centredness is central to helping patients decide difference between the two situations or change your view.
what is best for them. Consider that the surgeon of the When considering individual patients and applying ethical
thyrotoxic patient felt thyroidectomy to be the best course principles, foremost in our minds in decision making
of action. It would certainly be right to explore her under- should be what is deemed in the patient’s best interests. It
standable concerns about the operation. Negotiating the helps, when making a decision, to ask why you have made
best course of action for her should address these con- it and to be able to justify it.
cerns. A balanced explanation by the surgeon may empha-
sise the low incidence of serious complications, despite
her fears, and include an expectation of what may happen
A word on dignity
if she does not proceed to surgery now. Despite this In recent years the notion of dignity, while emerging pri-
process of shared understanding, if the patient, competent marily from palliative care, has been more formally
to decide and aware of the risks, declines surgery, she is applied across the spectrum of medicine. The core values
exercising her autonomy. More likely, she will be reassured of kindness, respect and dignity are indispensable and
and begin to find surgery an acceptable option. invariably, although not automatically, allow ethical prac-
Patients have a right to be involved in decisions that tice to flourish. Awareness of the A, B, C, D of dignity in
affect them. Practising modern medicine is not just about health care is of value to some: attitudes (largely about
knowledge, but also about the skills to share that knowl- appreciating the patient’s situation and your own impact
edge in ways that encourage patients to make sound on it), behaviours (professional and respectful disposition
informed choices. Patients, in general, value our ability to and clinical examination, and facilitation of communica-
guide. When they are guided, rather than pushed, they are tion), compassion and dialogue. The Royal College of
far more likely to keep going along the treatment path we Physicians rightly emphasises the importance of dignity in
suggest. both clinical practice and professional examinations.

617
Station |4| Communication skills and ethics

Box 4.1  Ethical principles

1.  Respect for persons for patients, but raises the question as to who is the judge of what
We have a duty to respect the rights, autonomy and dignity of the is best.
person. This duty incorporates concepts such as honesty, Beneficence is often seen as being applied in practice when a
truthfulness, sincerity and trust. In medicine, respecting a patient’s health professional determines, by objective assessment, what is in
autonomy is a fundamental ethical principle. a patient’s best interests. The patient’s views are encapsulated in
Autonomy is the capacity to think and decide, and act on the the principle of autonomy. Usually, that which the health
basis of such thought and decision, freely and independently. This professional determines and the patient’s views lead to the same
requires health professionals to provide the necessary information conclusion, because most patients choose what is objectively in
to help patients reach decisions for themselves and respect such their best interests.
decisions even if these do not appear to be the best course of In a sense, beneficence is the first duty of a doctor – to alleviate
action. symptoms and suffering, usually, but not always, through diagnosis
and treatment.
2.  Justice
4.  Non-maleficence
Justice refers to our duty of universal fairness or equity. It
incorporates our duty to avoid discrimination, abuse or exploitation The duty to avoid doing harm runs, in most situations, parallel to
of people. the duty to do good. Most treatments carry some risk of doing
more harm than good but it does not follow that such treatments
3.  Beneficence should be avoided on the grounds that avoiding harm takes priority
over doing good. This said, the first rule of medicine traditionally is,
The duty to do good (in the medical context, to our patients) is a
and firmly remains, do no harm.
fundamental guiding ethical principle. It entails doing what is best

Cases

the examiners. Do not take the history again except for


Discussing clinical management details that will help in your discussion with the
patient / subject. You are not required to examine the
patient / subject.
CASE 4.1  EXPLAINING A
DIAGNOSIS Patient / subject information
Mrs Elizabeth Wright is a 45-year-old hospital secretary
Candidate information with two sons. For the last year she has had intermittent
but slowly worsening pain in the joints of her hands and
Role
a facial rash that is sensitive to sunlight. She is previously
You are a doctor in the medical outpatient clinic. very well, takes no medications and is a non-smoker. She
Please read this summary. has been investigated recently in the outpatient clinic and
informed that systemic lupus erythematosus (SLE) is a
Scenario possibility. She is here today for the results of tests. She is
very concerned that she may need to stop working, which
Re: Mrs Elizabeth Wright, aged 45 years she can ill afford as a single mother. She is also concerned
Mrs Wright has systemic lupus erythematosus (SLE). She has had because she is aware of a patient in her workplace needing
a malar rash and pain in her hands for 12 months. Her clinical dialysis, and an internet search advised that SLE was not
presentation, together with results of extensive investigations a curable disease and could cause kidney failure. Other-
including autoimmune serology and radiology, supports the wise she knows very little about SLE.
diagnosis of SLE confined to skin and joints without other
multi-systemic involvement. She is here today for results. She is a
hospital secretary and very concerned she may need to stop working. How to approach the case
Your task is to explain the diagnosis, addressing her concerns.
Communication skills (conduct of interview,
exploration and problem negotiation) and
Your examiners will warn you when 12 minutes  ethics and law
have elapsed. You have 14 minutes to communicate 
with the patient / subject followed by 1 minute of reflec- 1.  Introduction
tion. There will then follow 5 minutes of discussion with Introduce yourself and confirm her identity.

618
Case 4.2  Explaining an investigation

2.  Clarify the task How does it look on the whole, doctor?
Be clear in your own mind what problem or diagnosis It looks all right, on the whole. Most patients with your
needs to be discussed. condition live a very normal life.

The tests confirm that SLE has very likely been the cause of your 7.  Repeat important information
problems.
This can be an effective way of emphasising ‘take home’
messages’:
3.  Establish previous experience
Stronger candidates are guided by patients when giving I should emphasise again that most people do not end up with
explanations. They operate within a patient-centred frame- kidney problems or need dialysis. Most patients have skin and
joint problems similar to yourself, and these tend to get better
work. Weaker candidates think ‘Oh no! What do I know with treatment.
about SLE?’ and proceed to tell their patients very little or
far too much, often in a disorganised fashion. Try to estab- 8.  Confirm understanding
lish what your patient knows about their diagnosis before
launching into your explanation. Rather than saying: It can sometimes be useful to ask the patient to recount
what they feel to be the important points of the
SLE is a condition which … discussion.
a better approach would be one of:
9.  Encourage feedback and invite questions
Have you heard of this condition before? I appreciate we’ve covered rather a lot there. Is there anything
I wonder if you know anything about this condition before we start? you would like me to go over again or are there any
questions you would like to ask me?
Have you any ideas about this condition?
What have you been told so far about this condition? Be honest in your explanations and prepared to admit
uncertainty (with assurance you will seek the answer) if
A specific, uncommon condition like SLE may mean
she asks something you cannot answer.
very little to a patient. Other diagnoses such as multiple
sclerosis, cancer and rheumatoid arthritis are well known
10.  Agree a way forward
but often poorly understood. Establishing prior knowl-
edge or preconceptions helps determine at what level to Ensure that there are arrangements for treatment and
start your explanation. She might not know that SLE can follow up.
be a multisystem disease affecting more than skin and
joints. Alternatively, she might say: Discussion
I was reading that it can affect the kidneys and that some Would you tell her that her condition
patients end up on dialysis. Is that likely to happen to me?
is incurable, if she asked?
4.  Be alert to ideas, concerns and expectations The approach to explaining any diagnosis depends upon
Establishing prior experience can help you understand any the diagnosis itself and should be tailored to the patient.
specific fears that may relate to it. Throughout your expla- Many conditions are potentially serious but treatable, and
nation, try to take account of any ideas, concerns and in this case potentially ‘incurable’ in that it may return
expectations you elicit. Examples of concerns that a patient without treatment but nonetheless it is likely to be fully
with SLE might have include no longer being able to go treatable. Other conditions are more palpably ‘incurable’
out in the sun, an unsightly rash, arthritis, needing to take such as disseminated malignancy. But few if any condi-
drugs or serious complications such as renal failure. Effects tions are untreatable as palliation is always possible.
of a diagnosis on work, finances and home life are often Words such as incurable are unhelpful, and it is more
central concerns. helpful to tell her that her condition can usually be fully
suppressed with treatment.
5.  Frame the explanation
Try to provide a framework or order to your explanation.
You might talk about why SLE occurs (‘misdirected’ CASE 4.2  EXPLAINING
immune system), possible clinical problems and the likely
AN INVESTIGATION
natural history, with and without treatment.

6.  Keep it clear Candidate information


Keep your explanation as clear as possible. Use language
she will understand (without being patronising), avoiding
Role
complex medical terminology. Sometimes it helps to use You are a doctor on the medical ward.
similar phrases to the patient: Please read this summary.

619
Station |4| Communication skills and ethics

Scenario a better approach might be:

The chest specialist feels that we should look more closely into
Re: Mr Roger Thornhill, aged 74 years this problem of coughing up blood. He suggests some further
Mr Thornhill has been losing weight for 5 months, and feeling tests. May I ask you what you’ve been told about the results
slightly breathless on exertion. He has hitherto been a well man, of your tests so far?
playing golf every day until admission to hospital last week with
I know the X-ray wasn’t completely normal. What tests does he
cough and possible pneumonia. He feels the antibiotics have
suggest?
helped. However, his chest X-ray showed mediastinal widening and
a computed tomography (CT) scan confirmed a mass arising from He suggests a test called a bronchoscopy. Have you heard of a
the left hilum, which is likely to be primary lung cancer but possibly bronchoscopy before?
lymphoma. He was aware that the chest X-ray showed an
abnormal shadow but does not know the results of his CT scan. He 4.  Be alert to ideas, concerns and expectations
has not smoked for 20 years.
Your tasks are to explain the results of the CT scan and discuss Establishing prior experience can help you understand 
bronchoscopy as the best next step. any specific fears relating to the potential diagnosis or 
to the test itself. These should be addressed before
proceeding:

Your examiners will warn you when 12 minutes  What do you think the X-ray means, doctor? The other doctor
have elapsed. You have 14 minutes to communicate  said it could be a growth in the lung. I’ve smoked all my life
with the patient / subject followed by 1 minute of  so I wouldn’t be surprised, to be honest with you. Is it bad?
reflection. There will then follow 5 minutes of discussion The X-ray does suggest the possibility of a growth in the lung. A
with the examiners. Do not take the history again except bronchoscopy is the best way of knowing this for certain. It’s
for details that will help in your discussion with the very hard to say what it means for you until we know for sure
what it is. Once we know, we are in a much better position to
patient / subject. You are not required to examine the
know the best way forward.
patient / subject.
5.  Frame the explanation
Patient / subject information
Try to provide a framework for your explanation of the
Mr Roger Thornhill is a 74-year-old man admitted to hos- investigation (Box 4.2).
pital last week with cough and pneumonia. He has been
losing weight for 5 months. Although he feels a little better The bronchoscopy involves passing a flexible telescope with a
after antibiotics, he has been told that his chest X-ray light on the end into the airways, usually under sedation. If
there is anything seen, such as a growth, then the specialist
shows an abnormal shadow that may not be purely attrib-
would take a biopsy, a small piece of tissue, from it for closer
utable to infection. A chest CT scan has been performed examination. It is a very common test these days, and
and he is awaiting the results. He has been worried that generally very safe.
the scan might show something sinister, like cancer. He
has been an active man, playing golf daily. He was widowed 6.  Keep it clear
6 years ago. He gave up smoking 20 years ago.
Keep your explanation as clear as possible. Use language
he will understand.
How to approach the case
Communication skills (conduct of interview, 7.  Repeat important information
exploration and problem negotiation) and This can be an effective way of emphasising ‘take home
ethics and law messages’

1.  Introduction 8.  Confirm understanding


Introduce yourself and confirm his identity. It can sometimes be useful to ask the patient to recount
what they feel to be the important points of the
2.  Clarify the task discussion.
Be clear in your own mind what investigation needs to be
discussed, and the purpose for which it is being
performed. Box 4.2  Explaining an investigation
• The purpose of the investigation (usually to further a diagnosis,
3.  Establish previous experience assess the severity or extent of a disease, or monitor response
to treatment)
Try to establish what he knows about the problem before
explaining the investigation. Rather than saying: • The nature of the proposed investigation
• Any risks that may be involved
You need to have a test called a bronchoscopy,

620
Case 4.2  Explaining an investigation

9.  Encourage feedback and invite questions is negative, the question is whether or not it is good at 
I appreciate we’ve covered rather a lot there. Is there anything not missing disease. Sensitivity and specificity are not
you would like me to go over again or are there any affected by the prevalence of a disease; predictive values are
questions you would like to ask me? (Table 4.1).

10.  Seek consent Sensitivity


Remember that you are explaining the reason for an inves- Sensitivity refers to the ability of a test to detect disease. It
tigation and not telling a patient they must have it. Fol- considers all with disease (disease positives) and is the
lowing explanation, you should seek informed consent to proportion of true disease positives (a + c) that the test
proceed. detects (a). A highly sensitive test is able to detect disease
or target substance in trace amounts and so misses few
Discussion that have the disease, i.e. a low false-negative rate. For
example, bone marrow aspiration and soluble transferrin
An 86-year-old woman with moderately receptor assay are highly sensitive tests for confirming iron
severe dementia has a suspicious lesion on deficiency, whereas ferritin may be misleading.
chest X-ray. Would you recommend
Specificity
bronchoscopy?
Specificity refers to the ability of a test to exclude disease.
Guidelines for investigating and treating suspected lung It considers all without disease (disease negatives) and is
cancer suggest bronchoscopy or percutaneous biopsy to the proportion of true disease negatives (b + d) that the
guide appropriate treatment – surgery, chemotherapy, test correctly identifies as negative (d). A highly specific
radiotherapy or palliation. But guidelines often fail to test is able to detect a single disease or target substance
address significant comorbidity. Here, assessment of cog- and no other and seldom misdiagnoses disease, i.e. a low
nition, functional and social support and symptom false-positive rate. For example, mammography screens a
control are likely to supersede attempts to confirm a diag- defined age population with higher specificity for detect-
nosis, but investigation may be warranted if likely tumour- ing breast cancer. Lowering the age threshold for screening
related symptoms develop to allow more secure planning may lower specificity and increase false-positive results, as
of palliative measures. Her capacity to understand the younger women tend to have lumpier breasts.
issues should be explored, ideally in the presence of next
of kin, and empirical treatment considered if she does not
accept investigation. If she accepts investigation, the least
Positive predictive value (PPV)
likely to cause distress should be arranged, for example a PPV considers all who test positive and is the proportion
computed tomography scan rather than bronchoscopy. of people with a positive test (a + b) who have the disease
in question (a). A high PPV is desirable for diagnostic tests,
What do you understand by the i.e. a low false-positive rate.
terms sensitivity, specificity, positive
predictive value (PPV) and negative Negative predictive value (NPV)
predictive value (NPV)? NPV considers all who test negative and is the proportion
of people with a negative test (c + d) who do not have the
Sensitivity and specificity are to do with finding or not disease in question (d). A high NPV is desirable for diag-
finding disease. Sensitivity is concerned with finding disease, nostic tests, i.e. a low false-negative rate.
and specificity with not detecting disease in those without
the disease. Disease positives and negatives are based  Explain the terms pre-test probability and
on ‘gold standard tests’ (ultimate diagnostic tests as 
the profession sees them). Predictive values are to do  post-test probability
with diagnostic / screening tests. If a test is positive, the ques- Pre-test probability refers to the proportion of people with
tion is whether or not it is good at detecting disease; if a test a target disorder in the population at risk and is simply

Table 4.1  Sensitivity, specificity and predictive values

Disease positive Disease negative


Test positive (abnormal) a b PPV = a / (a + b)
Test negative (normal) c d NPV  =  d /  (c + d)
Sensitivity = a / (a + c) Specificity  =  d /  (b + d)
NPV, negative predictive value; PPV, positive predictive value.

621
Station |4| Communication skills and ethics

another term for prevalence. Pre-test probability may be


calculated using the formula (a + c) / (a + b + c + d). Post- CASE 4.3  DISCUSSING TREATMENT
test probability refers to the proportion of patients with a
particular test result who have the target disorder.
Candidate information
What criteria are important in determining Role
whether or not to implement a screening You are a doctor on the cardiology ward.
test for a disease? Please read this summary.
Ideally, the disease should be common, important and
diagnosable by an accepted method. There must be a Scenario
latent interval between disease onset and important 
manifestations during which effective intervention is pos-
sible. The screening test should be simple, cheap (cost Re: Mr James Bonde, aged 80 years
effective) and performed on a group deemed by policy to Mr Bonde is recovering well from a non-ST elevation
myocardial infarction. Although well before admission, coronary
be at high risk. It should have a high sensitivity and a angiography is not being considered and he is keen to go home.
specificity. He is currently taking aspirin. Beta blockade is deemed risky on
account of sinus bradycardia. His blood pressure is 160 / 90. You
What is meant by a valid and reliable test? think he might benefit from clopidogrel, a statin and an
angiotensin-converting enzyme inhibitor to further modify risk.
A valid test is one that measures what it sets out to measure. Your task is to explain these treatments to him, and allow
A reliable test is one that produces repeatable results. him to make an informed choice about whether or not to take
them.
Which factors may influence a decision to
investigate older people?
Guidelines for investigating conditions can fail to address
individual factors that should be considered when assess- Your examiners will warn you when 12 minutes have
ing the potential benefits and risks of investigation; such elapsed. You have 14 minutes to communicate with the
factors become more common with advancing age. Comor- patient / subject followed by 1 minute of reflection. There
bidity and multiple pathologies become more likely in will then follow 5 minutes of discussion with the examin-
older patients, with an increase in chronic diseases, and it ers. Do not take the history again except for details that
can be more appropriate to identify problems and use will help in your discussion with the patient / subject. You
problem solving than seek single unifying diagnoses,  are not required to examine the patient / subject.
and more appropriate to make judgements as to which
problems are most probably affecting quality of life.  Patient / subject information
Cognitive impairment, either mild or dementia, may pre-
clude informed consent for an investigation. Physical dis- Mr James Bonde is an 80-year-old man who has never
ability such as osteoarthritis can, for example, thwart before been in hospital. He was admitted to hospital 4
exercise stress testing. Patients’ views and wishes are of days ago with a heart attack. His wife died of heart disease
course most valuable in deciding the best way forward. A 6 years ago, having taken many medications for her heart.
common example is the extent to investigate anaemia in  He feels he has had a good life. He does not want to be
an older person once a basic ‘blood screen’ has been kept in hospital for further tests or treatments if he can be
completed. allowed home more quickly without these, and fully
A pragmatic way of decision making for investigating accepts he might be declining potentially optimal, ‘cutting
older people is to select investigations most likely to edge’ approaches. He feels simply that he has done all that
change management or yield prognostic information. If a he wants to in life and ‘will accept the end when it comes,
patient is competent to give informed consent to an inves- with or without treatment’. He will consider treatment that
tigation then it is possible to proceed; if not then it may keeps further chest pains at bay, but does not want ‘any-
be discussed with those close to the patient, and a decision thing too complicated’.
made in the patient’s best interests as judged by the doctor.
If either a competent patient or those close to a patient How to approach the case
not competent refuse investigation then empirical treat-
ment might be offered.
Communication skills (conduct of interview,
Automatic investigations that will not benefit patients exploration and problem negotiation) and
are inappropriate but discrimination against older patients ethics and law
by restricting access to investigations is equally unaccept-
able. Decisions should be made by balancing potential
1.  Introduction
benefits and risks, and not determined by age. Introduce yourself and confirm his identity.

622
Case 4.3  Discussing treatment

2.  Explain the reasons for considering treatment 9.  Seek consent
I’m glad to see that you are recovering from your recent heart Remember that you are explaining the reasons for tablets
attack. Your test results are largely encouraging but your blood and not telling him he must have them.
cholesterol level and blood pressure are still a little high. When
the cholesterol is high and we don’t treat it, this can increase 10.  Respect autonomy
the risk of future heart attack, as can high blood pressure.
Fortunately, the risk can be reduced with medication and I Ultimately, the decision to accept treatment or stay in hos-
think we should discuss starting this medication in your case. pital is his, and his informed decision should be respected.

3.  Give the patient a chance to react to the   Discussion


need for treatment
Are you more reluctant to prescribe
Remember that a lot has happened to him recently. His
world has been changed by the heart attack. He probably medications for older people?
had an enormous fright when admitted, and is still coming Treating older people must aim for maximum benefit (it
to terms with the repercussions of his diagnosis. He will is older people who often derive most benefit from a
have received a great deal of advice from doctors, nurses particular drug) and minimal adverse effects by avoiding
and members of the cardiac rehabilitation team. He will excessive or inappropriate medications. To achieve this
be on a range of tablets indefinitely. It is important to balance, individualised prescribing that considers comor-
recognise that his hopes and yours may not immediately bidity, co-prescription and social aspects of care, and
be the same. There may be a mismatch of agendas. You careful judgement of overall frailty and likelihood of
want him to start more tablets and then see your next benefit, are crucial. Older people are often inappropriately
patient but he desperately wants some good news. Yet under-represented in therapeutic trials, and so the size of
more tablets may not be what he had in mind. Going benefit might be underappreciated in an older person, but
home may be. Give him a chance to react to what you see so might the absolute risk increase. Six rules in prescribing
as the most important next step, and appreciate his. for older people are shown in Box 4.3.

4.  Be alert to ideas, concerns and expectations Is drug treatment for coronary syndromes
Establishing prior experience (e.g. of his wife) can help the same for older as younger patients?
you understand any specific beliefs. Throughout your Thrombolytic agents provide a greater absolute benefit in
explanation, try to take account of any ideas, concerns and older people and should only be withheld if there are
expectations you elicit. contraindications, although these are more likely in older
people. Meta-analyses of aspirin trials show around 25%
5.  Explain the likely benefits of treatment reduction over 2 years in secondary prevention of major
Explain these as accurately as possible, without over- vascular occlusive events, relative risk reduction being
whelming him with outcomes of studies. It often helps to similar in younger and older people but absolute benefit
reassure patients that others are taking the same drug and greatest for high-risk patients over 65 years (but the risk–
that your advice is commensurate with the larger body of benefit ratio tips aspirin out of favour for low-risk older
current medical opinion. people in primary prevention). Beta blockers after myocar-
dial infarction reduce mortality from ischaemia and
6.  Explain what the treatment involves arrhythmias with evidence up to 75 years, but beyond this
Explain the frequency of dosing of tablets, the duration of age benefit can only be extrapolated and older people are
therapy and any special instructions (e.g. when tablets more likely to have relative or absolute contraindications,
should be taken). Any requirements for blood monitoring, such as sinus pauses or postural hypotension. Angiotensin-
if relevant, should be discussed. converting enzyme inhibitors reduce mortality and mor-
bidity in cardiovascular disease but again older patients
7.  Explain likely side effects of treatment are more likely to have contraindications such as hypo­
Mention side effects that are common or serious. Try to tension, moderate aortic stenosis or vulnerable renova­
explain the balance of risk (benefits versus side effects). scular disease.
Explain what should be done if a side effect occurs. Many Is treatment for hypertension the same for
patients know that aspirin can cause ulcers, but carry on
taking it if they have bleeding symptoms. Patients have not
older as younger patients?
worked in medical units. Do not assume that what is Absolute benefit from antihypertensive treatment is much
obvious to you is obvious to your patient. greater in older people.

8.  Encourage feedback and invite questions Is treatment for hyperlipidaemia the same
for older as younger patients?
I appreciate we’ve covered rather a lot there. Is there anything
you would like me to go over again or are there any The benefits of statin therapy are deduced from overall risk
questions you would like to ask me? rather than lipid concentration. The Heart Protection

623
Station |4| Communication skills and ethics

example, a glance at the drug chart might reveal the con-


Box 4.3  Prescribing for older people comitant use of a selective serotonin reuptake inhibitor
1.  Consider the indication (SSRI), a tricyclic antidepressant and tramadol; SSRIs are
enzyme inhibitors that enhance the action of the other
This includes stopping drugs when investigations do not support a drugs and the combination of a SSRI and tramadol can
diagnosis. It is not uncommon for patients whose results do not
support ischaemic heart or other vascular disease to be continued
produce a serotonergic syndrome. Common drugs causing
on antiplatelet, antihypertensive or lipid-lowering treatment and to disease include non-steroidal anti-inflammatory drugs
be re-admitted with anaemia or hyponatraemia. (NSAIDs), aspirin, clopidogrel, warfarin, diuretics,
angiotensin-modifying agents and corticosteroids, but
2.  Be alert to possible side effects   these drugs are commonly prescribed. Over-the-counter
and limit prescribing drug use should also be explored, as around 25% of hos-
Almost any drug can provoke confusion. Extrapyramidal side effects pitalised patients may be taking these yet in only around
are not uncommon with antipsychotics, anti-emetics and 1% of patients do they appear in their case records.
anti-histamines; depression can be triggered by beta blockers and
calcium channel blockers; constipation can be the result of opiates,
diuretics and anticholinergics. Broad-spectrum antibiotics, What do you know about adverse drug
antipsychotics, benzodiazepines, beta blockers and phenytoin are
just some drugs that may be better avoided in older people.
reactions (ADRs)?
ADRs are a challenge for both primary and secondary care,
3.  Do not withhold treatments that work and reflect the value of a strong pharmacist presence in the
Warfarin, for example, is often inappropriately withheld. health-care team. Year on year drugs are withdrawn
Osteoporosis treatment is underprescribed. because of safety concerns, examples in recent years
4.  Stop inappropriate medications including cerivastatin, rofecoxib and co-proxamol; 6.5%
The aim should be symptom relief, for example in patients who are of hospital admissions are due to ADRs, and 14.7% of
dying, who should not be given a diet of drugs to prevent vascular patients experience an ADR in hospital, a major propor-
disease. tion of these thought to be potentially preventable. Drugs
should thus always be considered in the differential diag-
5.  Be imaginative with routes of administration noses of presentations. To assess cause and effect may
Use intramuscular gentamicin, for example, in an agitated, require any of a temporal relationship, re-challenge, exclu-
confused patient with a urinary infection. sion of other causes, novelty or de-challenge (dose reduc-
6.  Consider concordance tion or withdrawal). Errors in omission appear to be more
common than commission. Alternative medicines are a
Ideally patients should be discharged with a list of their treatments
large problem, and for example ginseng, feverfew and
and the reasons for each.
garlic reduce platelet function and St John’s wort induces
drug metabolism of protein pump inhibitors and this
combination may result in gastric bleeding. The Medical
Research Council for Drug Safety Science Hospital Admis-
sion Study revealed the top 10 causative drugs for ADRs 
Study showed significant reductions in first event rates as NSAIDs, diuretics, warfarin, angiotensin-converting
(around 25%, P < 0.0001) for myocardial infarction and enzyme inhibitors, antidepressants, beta blockers, opiates,
stroke each year after the first year and these rates were digoxin, prednisolone and clopidogrel. NSAID-induced
similar in those under or over 70 years. Statins should be peptic ulceration is the most frequent ADR cause of admis-
considered for patients with occlusive arterial disease or sion. Antiplatelet agents are a common example of the
diabetes where life expectancy is greater than 1 year and benefit–harm balance consideration, as they benefit vas-
quality of life is positive. cular disease but may cause gastrointestinal bleeding, the
latter risk increased with NSAIDs and former benefit
Is warfarin justified in patients aged over reduced by some gastroprotective agents.
75 years with atrial fibrillation?
Warfarin in atrial fibrillation reduces stroke risk by 68%,
Are pharmaceutical companies and
compared with 21% with aspirin. The high rate of intra­
cranial haemorrhage with warfarin in patients aged over pharmaceutical representatives to be
75 years does not negate its benefit, but comorbidities considered favourably by doctors?
limiting life expectancy may limit its potential benefit. Absolutely, and yet doctors are often naive as to how 
drugs must be developed, licensed and marketed. The 
What do you know about pharmaceutical industry is an essential one of scientists,
health-care professionals and business people without
drug-induced disease? which we would have no drug development, no drug trials
Any drug may cause disease, and drugs should always be and no drug information with and upon which to treat our
included in the differential diagnosis of symptoms. For patients.

624
Case 4.4  Discussing management, prognosis and possible complications in a patient

What do you know about drug licensing? Your examiners will warn you when 12 minutes have
elapsed. You have 14 minutes to communicate with the
Drug licensing is not able to influence the vigilance all
patient / subject followed by 1 minute of reflection. There
doctors must have to licensed drugs, but is a vital line of
will then follow 5 minutes of discussion with the examin-
defence against drug-induced disease. The British Medical
ers. Do not take the history again except for details that
Journal in 1909 published a report on ‘Beechams Reme-
will help in your discussion with the patient / subject. You
dies’, a popular product at the time which falsely claimed
are not required to examine the patient / subject.
a dazzling range of benefits, but which contained little else
than soap and ginger. Licensing became formalised in
Patient / subject information
1958 in the thalidomide era, and now to gain a licence a
drug must pass the hurdles of quality of manufacture, Mr John Rowe is a 74-year-old man who had an operation
efficacy in the proposed indication and reasonable safety. for gastric cancer 2 years ago. It was diagnosed very early,
The Medicines and Healthcare products Regulatory Agency by investigation for ingestion. After surgery he had no
(MHRA) in the UK, through its Commission on Human further symptoms and his surgeon told him ‘all was clear.’
Medicines and Licensing Authority, has a pivotal role. On He now has recurrence of indigestion. He has also recently
introduction, most drugs go through the phases of initial seen the cardiologists because of suspected angina and
high expectation then subsequent disappointment before had a positive exercise tolerance test and awaits angiogra-
finding their balanced place. Drugs then generally have a phy. He has been started on aspirin, simvastatin and rami-
period of patency protection before coming ‘off-patent’ pril. He has mild emphysema for which he takes inhalers,
and becoming generic, whereupon ‘me-too’ products at and gave up smoking 20 years ago. He also wishes to talk
competitive rates may consume 90% of sales. Pharmaco- about his angina and breathing because he feels the car-
vigilance is vital, and MHRA pharmaco-vigilance groups diologists did not tell him very much. He is worried about 
receive yellow cards from reporting doctors, and these the risks of an angiogram but also worried that without
provide vital cumulative clues of danger; not enough further tests he could have a heart attack. He wonders what
doctors submit these. his risk of a heart attack might be. He recently had two
episodes of fresh rectal bleeding, but is not too troubled
by this.
CASE 4.4  DISCUSSING
MANAGEMENT, PROGNOSIS AND How to approach the case
POSSIBLE COMPLICATIONS IN A Communication skills (conduct of interview,
PATIENT WITH MULTIPLE PROBLEMS exploration and problem negotiation) and
ethics and law
Candidate information 1.  Introduction
Role Introduce yourself and confirm his identity.

You are a doctor in the medical outpatient clinic. 2.  Know how to deal with multiple problems
Please read this summary. It is increasingly common in general internal medicine to
face numerous problems in a single consultation. Patients
Scenario
with multiple problems are often uncertain about what is
really wrong. They might not know why certain tests are
Re: Mr John Rowe, aged 76 years being arranged. They might not know exactly why they are
Mr Rowe had a partial gastrectomy for gastric cancer 2 years taking medications. And in the era of increasing specialisa-
ago. It was diagnosed very early, by oesophagogastroduodenoscopy tion they might be attending multiple specialist clinics,
for dyspepsia. After surgery and a course of Helicobacter pylori sometimes punctuated by acute admissions to hospital
eradication he had no further symptoms and his surgeon told him
after which they are discharged on different tablets, and
all appeared to be clear. He now has recurrence of dyspepsia and
is seeing you urgently in clinic and wants to know if the cancer they might have difficulty organising and retaining some-
might have returned. He has recently seen the cardiologists times seemingly conflicting advice from the different
because of suspected angina and awaits angiography. He has been health professionals they have seen.
started on aspirin, a statin and an angiotensin-converting enzyme
inhibitor but not a beta blocker on account of mild chronic 3.  Take one step at a time
obstructive pulmonary disease for which he takes inhalers. He also Your task is to arrange the puzzle into a series of boxes,
wishes to talk to you about his angina and breathing because he and address each of these on the basis of which you see
feels the cardiologists did not tell him very much. And he has
as priorities. In this case an appropriate way forward is to
recently had two episodes of fresh rectal bleeding, but he is not
too troubled by this. You may assume systematic enquiry otherwise address the dyspepsia first and consider further endoscopy
unremarkable. (which he has had before), and reassure him that you will
Your task is to address his concerns and discuss a way forward. move on to discuss his angina and breathing problems
afterwards, and not forget his rectal bleeding.

625
Station |4| Communication skills and ethics

4.  Obtain the facts


Box 4.4  Issues to be aware of in managing
Dyspepsia is often hard to differentiate from cardiac chest older people
pain. He has cardiac problems but equally a history of
gastric malignancy and recently starting aspirin and so has • Reduced body reserve
two compelling reasons to proceed to oesophagogastrodu- • Atypical presentations
odenoscopy (balanced against minor procedural risks • Multiple problems / pathologies
posed by stable angina). • Polypharmacy
• Social adversities
5.  Explore concerns
Make sure you explore any specific concerns he may have,
for example about proceeding to the angiogram. 10.  Agree a way forward and ensure follow-up
arrangements are in place
6.  Discuss management options with   Reassure him of follow-up arrangements, and that you
a clear framework would happily review sooner at any time at his or his
I think we should strongly consider looking into the stomach general practitioner’s request if there are questions or
again with the endoscopy you had before. This will tell us concerns.
quickly whether or not there is anything we need to be worried
about. I’d hope it would simply show some ulceration that we
can treat with tablets, and this would not be uncommon in
Discussion
people taking aspirin. But I think you should continue the
aspirin for the time being, as it gives some protection to the
What issues should you be generally aware
heart. In the unlikely event that the endoscopy showed of when managing older people?
anything more untoward then we would be in a better position Many of the scenarios in this book deal with older patients,
to know what to do sooner rather than later.
and appropriately so since general internal medicine is
managing an increasingly older population. A mnemonic
7.  Discuss potential prognoses and possible for issues you should be particularly aware of is given in
complications of any diagnoses (with and Box 4.4.
without treatment) or treatments
It can be easy to be drawn into discussion of possible
outcomes but as far as possible try to avoid hypothetical
discussion of what has not been diagnosed. With respect
CASE 4.5  DISCUSSING DIAGNOSTIC
to his angina, it would be best to leave management in the UNCERTAINTY
hands of the cardiologists, but appropriate to comment
on the reasons for angiography (to consider if a procedure
such as stenting would help his angina) and the impor-
Candidate information
tance of medications in reducing the risk of a heart attack. Role
8.  Prioritise … but do not forget   You are a doctor in the medical outpatient clinic.
Please read this summary.
unturned stones
It is important at this stage not to ignore his rectal bleeding, Scenario
which he considers less important. But unturned small
stones can sometimes turn out to be very significant.
I do not think we should ignore the bleeding you have had Re: Miss Melissa Hamilton, aged 24 years
from the tail end, and it may be preferable for us to consider You have been seeing Miss Hamilton in clinic for follow-up of
examining the tail end with a flexible telescope with a light Crohn’s disease. She has been asymptomatic for 6 months. At the
on the end – similar to the endoscopy you have had before end of the consultation she mentions that she has had pain in her
– to be certain we are not missing something. right calf for the past week. She hoped you might take a quick
look at it. Examination is unremarkable.
Assuming rectal examination normal, colonoscopy Your task is to discuss management options with her.
might be a crucial test, and the decision is where to place
it in the overall sequence. One way forward would be to
request it provisionally (waiting lists will preclude rapid Your examiners will warn you when 12 minutes have
access!). elapsed. You have 14 minutes to communicate with the
patient / subject followed by 1 minute of reflection. There
9.  Encourage feedback and invite questions will then follow 5 minutes of discussion with the examin-
I appreciate we’ve covered rather a lot there. Is there anything ers. Do not take the history again except for details that
you would like me to go over again or are there any will help in your discussion with the patient / subject. You
questions you would like to ask me? are not required to examine the patient / subject.

626
Case 4.5  Discussing diagnostic uncertainty

Patient / subject information Box 4.5  Probability and potential seriousness of


Miss Melissa Hamilton is a 24-year-old woman with diagnosis
Crohn’s disease that has been well controlled on treatment
for 6 months. At the end of her outpatient consultation A symptom may have a low probability of representing serious
pathology but a high probability of serious consequences if ignored.
she mentions that she has had pain in her right calf for
Such information should always be shared with a patient. The
the past week. She didn’t want ‘to bother her GP’ and management of disease in theory is often clear cut: investigation A
hoped you might take a look at it. She is not particularly for suspected disease B; treatment Y for disease Z. In practice,
concerned about it but wonders what it might be due to. whether or not to embark on an investigation in the face of limited
She is otherwise well. She started the combined oral con- evidence is a complex decision that should involve sharing your
traceptive pill (COCP) 2 months ago whilst touring in knowledge, concerns and expectations with the patient.
Australia with her boyfriend. They returned home 10 days
ago. She has no family history of thromboembolic disease.
a decision and ‘if you wanted us to’ may be too open
How to approach the case ended to actually help her decide. She might respond:
Well, you’re the doctor, I’ll take your advice. Do you think it’s
Communication skills (conduct of interview, really necessary?
exploration and problem negotiation) and Sharing information is more than showing patients the
ethics and law menu!
1.  Introduction 7.  Give an honest professional opinion
Introduce yourself and confirm her identity. If these decisions are difficult for us, with medical training,
then they can be enormously difficult for patients, given
2.  Clarify the task only small quantities of distilled information. Since there
Confirm that the problem is calf pain, and nothing else. is no clear history of muscle strain, the potential serious-
ness of a DVT may persuade you to say:
3.  Obtain the necessary facts I’d advise a blood test called a D-dimer. If this is negative we can
Gather information that is necessary to discuss manage- be reassured that a blood clot is highly unlikely. If it is positive,
ment. Crucially, this includes additional symptoms and we should go on to arrange a scan of the leg to be sure.
risk factors for venous thromboembolism – personal or
family history, long-haul travel, COCP. 8.  Confirm understanding
It can sometimes be useful to ask the patient to recount
4.  Be alert to ideas, concerns and expectations what they feel to be the important points of the discussion.
She probably has no concerns about a deep venous throm-
9.  Encourage feedback and invite questions
bosis (DVT).
I appreciate we’ve covered rather a lot there. Is there anything
you would like me to go over again or are there any
5.  Outline the possibilities questions you would like to ask me?
She appears to lead an active lifestyle, has no signs of a
DVT, and the absolute risk of thromboembolic disease on 10.  Cast a ‘safety net’
the COCP is very low. Conversely, she has two possible If a patient decides not to pursue tests, always have a safety
risk factors, the COCP and a long-haul flight, and throm- net, such as follow-up by the general practitioner or to seek
boembolic disease, although unlikely, can be life- medical advice if symptoms develop further. Any decision
threatening and is treatable. that involves waiting for something else to happen before
acting should be ‘safety netted’ and patients should be clear
6.  Share the probability and potential about which symptoms should prompt reassessment.
seriousness of a diagnosis
Probability is discussed in Box 4.5. Discussion
This is most likely harmless muscle pain. There is a small An 85-year-old man has had a large cortical
possibility the pain is due to a clot of blood in one of the
deeper veins of the leg. It is difficult to exclude this without stroke causing dysphasia and dense
further tests. Although I think a blood clot very unlikely, there is hemiplegia. Routine tests reveal a
a risk that an untreated blood clot could travel to the lungs and haemoglobin of 9 g / dl with a low ferritin.
cause serious harm. We could arrange a blood test and then
possibly a scan of the leg to be more sure, if you wanted us to. Would you investigate this further or accept
This sounds straightforward, but in practice there can be diagnostic uncertainty?
a problem with sharing information in this way. Patients Oesophagogastroduodenoscopy (OGD) and colonoscopy
frequently do not warm to the idea of being asked to make are the standard investigations to look for a source of

627
Station |4| Communication skills and ethics

chronic blood loss in iron deficiency anaemia. However, investigations, but the problem with each is that they do
the balance of risk of proceeding with these might not be not involve the patient. It would be important to share
in the patient’s best interests immediately. OGD might with him that a small lesion may not show up on chest
increase the risk of hypoxia, colonoscopy impractical and X-ray and that the only way to be certain of excluding this
depending upon the type of dysphasia informed consent would be with further tests. Teamwork is integral to good
could be an issue. Investigations should probably be clinical care and when you feel out of your depth in
delayed until the outcome of the stroke is clearer, and as making a decision then you should seek advice.
an interim measure exclusion of other haematic deficien-
cies, supplemental iron and possible withholding of
antiplatelet treatment is appropriate. If recovery is satisfac-
tory investigations could be instituted a few weeks later, CASE 4.6  DISCUSSING RISK AND
and even if colorectal cancer is diagnosed and inoperable TREATMENT EFFECT
or declined because of operative risk, then such informa-
tion may influence placement or the type of future care
provision. If there is severe residual disability from the Candidate information
stroke, then investigations should probably not be pursued
Role
and perhaps only considered if severe blood loss demands
repeated transfusion. Myelodysplasia is common in older You are a doctor on the acute medical unit.
people and anaemia is not usually a contraindication to Please read this summary.
antiplatelet therapy if monitored closely, or even anti­
coagulation if there is an embolic source and high risk of Scenario
further stroke in a recovering patient.

A 70-year-old woman has new onset Re: Mr Christopher Roberts, aged 47 years
exertional chest pain. She has severe Mr Roberts was admitted with atypical chest pain and is about to
osteoarthritis, walking only a limited be discharged on a statin because of a cholesterol level of
6.5 mmol / l. He is obese with a body mass index of 32, and has
distance, and Parkinson’s disease. Would numerous other risk factors for cardiovascular disease (CVD)
you attempt to further the diagnosis of including smoking, alcohol consumption on the edge of safety and
angina with a stress test? borderline hypertension. You are aware of evidence that stopping
smoking, statin therapy, targeting his hypertension (alcohol
A stress test could define risk more precisely and identify reduction playing a part), exercise and a ‘Mediterranean’ diet will
whether coronary angiography and potential revasculari- significantly reduce his relative risk of CVD and developing type 2
sation is likely to be of benefit. Pharmacological testing diabetes.
with dipyridamole, dobutamine or adenosine would be Your task is to discuss his perceived risks and the benefits of
needed. The likely net befits of investigation and possible these treatments and lifestyle modifications with him.
treatments should be explained and her views sought as
to whether to proceed to testing or a more conservative
strategy.
Your examiners will warn you when 12 minutes have
elapsed. You have 14 minutes to communicate with the
A 48-year-old non-smoker is being patient / subject followed by 1 minute of reflection. There
discharged from your ward today following will then follow 5 minutes of discussion with the examin-
investigation for chronic diarrhoea. No ers. Do not take the history again except for details that
cause has been found. He tells you that a will help in your discussion with the patient / subject. You
are not required to examine the patient / subject.
few weeks ago he had a single episode of
‘coughing up a teaspoonful of blood’ that Patient / subject information
he’d forgotten to mention until now. His
chest X-ray was normal. What are the Mr Christopher Roberts is a 47-year-old travel agent admit-
ted with chest pain initially thought to be angina. He is
important issues? very relieved to be told that investigations have not shown
There is probably no sinister cause. It has not recurred and evidence of a heart attack or angina and that his pain is
he is a non-smoker with a normal chest X-ray. Neverthe- now thought to be muscular resulting from poor posture
less, the haemoptysis remains unexplained. There are and a sedentary lifestyle coupled with a lot of driving and
numerous potential approaches to the problem. At one a recent minor whiplash accident. He is about to be dis-
extreme he might be reassured and told that no further charged on a statin because of a cholesterol level of
tests are needed; at the other extreme he could be advised 6.5 mmol / l. He is obese, with a body mass index of 32,
to have a bronchoscopy to exclude a tumour. Each doctor and has numerous other risk factors for coronary heart
has his or her own ‘risk threshold’ for undertaking disease including smoking, 42 units of alcohol per week

628
Case 4.6  Discussing risk and treatment effect

and high blood pressure. His father died of a heart attack 6.  Risk communication
at the age of 55.
Frame advice around the benefits of risk modification
(Box 4.7).
How to approach the case
Communication skills (conduct of interview, 7.  Optimise the likelihood of concordance
exploration and problem negotiation) and Concordance is more likely when risk perception is real-
ised, and he realises risk can be modified and that he can
ethics and law do it. The message can be reinforced through gentle repeti-
1.  Introduction tion and a plan of action and support.
Introduce yourself and confirm his identity.
8.  Confirm understanding
2.  Review the history Confirm his understanding not just that he is at risk but
that his risk can be substantially reduced.
Recap to him that the tests did not show that he had had
a heart attack but that it was very clear on assessing him
that he had significant risk factors for heart disease. 9.  Encourage feedback and invite questions
I appreciate we’ve covered rather a lot there. Is there anything
3.  Clarify the task you would like me to go over again or are there any
questions you would like to ask me?
Make it clear that reducing his risk of future heart attacks
is the aim of the discussion, and that this is a very achiev- 10.  Agree a way forward and ensure follow-up
able goal.
arrangements are in place
4.  Be alert to ideas, concerns and expectations This might include writing to his general practitioner 
and suggesting follow-up with the practice nurse, seeing 
That his father died relatively young from a heart attack
a dietitian and attending a smoking cessation advice
may be very much on his mind, but he may not know that
service.
it puts him at increased risk or that overall risk relates to
the interplay with other, modifiable risk factors. Risk per-
ception is discussed in Box 4.6.

5.  Clarify risk Box 4.7  Risk communication


Having established his risk perception, clarify exactly what Risk communication involves sharing information about risk, how
his risk factors are – smoking, alcohol, deficient exercise, to reduce it and ensuring that patients have the confidence to
high cholesterol and blood pressure – and potential health change their behaviour.
consequences of these.
Response efficacy
It is helpful to outline not just the benefits in terms of coronary
heart disease, but also the more widespread benefits of
disease-modifying behaviours.
Box 4.6  Risk perception • Stopping smoking will of course also reduce respiratory disease
and exercise intolerance and be financially advantageous.
• Risk perception can determine risk-reducing behaviour. People
who perceive themselves at increased risk of coronary heart • Statin therapy has well-established primary and secondary
disease are more likely to adopt risk-reducing behaviours such preventive outcome benefits.
as smoking cessation, eating a Mediterranean diet, increased • Targeting hypertension reduces stroke, coronary heart disease,
exercise and taking medication. Risk perception consists of two renovascular disease and peripheral vascular disease.
judgements, the perceived likelihood of an event and the • Exercise not only reduces the risk of vascular disease but also
perceived severity of that event. improves physical dexterity and overall well-being.
• Risks can be discussed in verbal categories, such as likely, • A Mediterranean diet produces similar benefits.
probable and possible, and in absolute probabilities, such as a
10% chance of having the event. Verbal categories mean Self-efficacy
different things to different people. People tend to downplay Habits are hard to break. Patients must not only be aware of the
risk and make overly optimistic judgements of risk in benefits of trying to improve their situation, but also should be
comparison with others of the same age and sex. instilled with the belief that they can. Motivation can be assisted
• The perception of being at increased risk is unlikely to motivate by trying to identify which consequences of failing to modify
people to adopt risk-reducing behaviours. It is also necessary to behaviour resonate most with a patient (e.g. not being able to
perceive that one has control over the risk of an event and this walk into the garden without feeling breathless by the age of 55,
has two components – response efficacy is the perception that erectile dysfunction) and support networks and agencies to assist,
risk-reducing behaviour will be effective and self-efficacy is such as specialist cardiac and respiratory nurses, smoking advice
having confidence in one’s ability to adopt it. services and so forth.

629
Station |4| Communication skills and ethics

Discussion risk of an event in the unexposed or control group is


sometimes called the control event rate (CER). Assume the
What is meant by the term prevalence? risk of myocardial infarction in those treated with lipid-
This refers to the proportion of individuals in a population lowering therapy is 1 in 100 (EER) and the risk in the
with the condition of interest at a point in time. control group is 2 in 100 (CER). The relative risk of myo-
cardial infarction with treatment is (1 / 100) / (2 / 100) or
Prevalence = number with the condition / 0.5. Equally, the risk without treatment is doubled or 2.
number in the population
What is meant by the term odds ratio?
What is meant by the term incidence? Odds ratio = odds in exposed group /
Incidence is concerned with the rate of new occurrences. odds in unexposed group
There are three principal measures of incidence – risk, odds
and rate. What is meant by the term rate ratio?
What is meant by the term risk? Rate ratio = rate in exposed group /
Risk is simply the proportion (a 1 in 2 or 50% chance of a rate in unexposed group
coin landing heads or tails) of disease-free individuals
who develop the condition during the study period. If these ratios are 1 there is no difference between
exposed and unexposed groups. If <1, the exposed group
Risk = number of new cases in the period / is at lower risk, and if >1 at higher risk.
number of disease-free individuals at the start
How does relative risk (RR) differ from
It is the probability that a disease-free individual will absolute risk (AR)?
develop the disease over the period. Absolute risk refers to the risk difference between exposed
(e.g. treated) and unexposed.
What is meant by the term odds?
This is the ratio (a 1 to 1 odds of the coin landing heads Risk difference = risk in exposed group
or tails) of the probability of getting the disease to the − risk in unexposed group
probability of not.
similarly:
Odds = number of new cases in the period /
number who remain disease-free during the period Rate difference = rate in exposed group
− rate in unexposed group
Odds against disease simply inverts the formula.
If 20 people have a stroke out of 100 in a population These absolute measures are generally referred to as
studied (i.e. 80 do not), the risk of stroke is 20 / 100 = 0.2 measures of attributable risk.
or 20% and the odds of a stroke is 20 / 80 = 0.25 or 25%. Relative risk should be interpreted in the context of the
We could also think of odds as ‘this versus this’ and risk prevalence of the condition in question. If, for example, a
as ‘this versus the whole population’. study showed that the risk of a disease x were doubled by
a particular drug y (e.g. the risk of venous thromboembo-
What is meant by the term rate? lism is increased by hormone replacement therapy), then
it is important to place that increased risk in the context
Risk and odds assume that all individuals are present of the likelihood of an event (absolute risk). If the natural
throughout the period. In practice, some enter after the prevalence or absolute risk of disease x is 1 in 1 million,
start and some leave before the end. then the absolute risk in the presence of treatment y is still
only 1 in 500 000. A decision to treat will depend upon
Incidence rate = number of cases / the risk–benefit ratio of treatment y.
total person time at risk Conversely, relative risk may not, at first sight, seem
high, but 10 extra strokes per 1000 people, for example,
What is meant by the term risk ratio? would be clinically substantial.
Risk ratio is also referred to as relative risk.
What is meant by the term relative risk
Relative risk = risk in exposed group / reduction (RRR)?
risk in unexposed group RRR is an illustration of how the risk of an event has been
reduced by intervention.
The risk of an event in an exposed or intervention group
is sometimes called the experimental event rate (EER). The RRR = 1 − RR

630
Case 4.7  Negotiating a management plan for a chronic disease / long-term condition

If, for example, RR = 0.2, then RRR = 1 − 0.2 = 0.8 or Your examiners will warn you when 12 minutes 
80%. have elapsed. You have 14 minutes to communicate 
with the patient / subject followed by 1 minute of reflec-
What is meant by the term absolute risk tion. There will then follow 5 minutes of discussion with
reduction (ARR)? the examiners. Do not take the history again except for
details that will help in your discussion with the
ARR refers to the number of additional people who benefit patient / subject. You are not required to examine the
from an intervention out of 100 and is the absolute arith- patient / subject.
metic difference in rates of bad outcomes between control
and experimental groups in a trial. ARR is therefore useful
in determining whether the RRR is important. If CER = 2%
Patient / subject information
and EER = 1%, ARR is therefore 1% and so 1 in 100 people Mr Henry Jackson is a 45-year-old man admitted 14 days
benefit from treatment. If the CER is much smaller, say ago with a large anterior myocardial infarction. He was
0.001%, the ARR achieved by the same RRR of 50% is very unwell for the first few days, and developed subse-
0.0005%. This ARR would warrant a much higher ‘number quent heart failure. He has had a stent inserted in one of
needed to treat’. his coronary arteries. His echocardiogram shows that he
has significant damage to the left ventricle. He is now
What is meant by the term number needed taking six different medications. He is very concerned that
to treat (NNT)? he still cannot walk far around the hospital without feeling
a little short of breath. He wonders if he will get back to
NNT refers to the number of patients who need to be a normal life. He is very concerned that he was a previ-
treated to prevent one bad outcome (or achieve one addi- ously very well man working as a manager for an IT
tional favourable outcome and benefit one additional company. He has a wife and two children and could not
patient). NNT is the reciprocal of ARR, i.e. 1 / ARR. afford to give up work. He is a little overweight. He used
to smoke 30 cigarettes per day.
What is meant by the term number needed
to harm (NNH)?
How to approach the case
NNH refers to the number of patients who need to be
treated to cause one bad outcome. The lower the NNH, Communication skills (conduct of interview,
the more harmful is the treatment. exploration and problem negotiation) and
ethics and law
CASE 4.7  NEGOTIATING A 1.  Introduction
MANAGEMENT PLAN FOR A Introduce yourself and confirm his identity.
CHRONIC DISEASE / LONG-TERM
2.  Clarify the task
CONDITION
Reassure him that your task and expectation is to guide
him back to as normal a life as possible.
Candidate information
3.  Establish the facts
Role
In practice you would establish as much history as possi-
You are a doctor on the medical ward. ble but since your communication skills are being assessed
Please read this summary. you should explore the concerns as soon as possible.

Scenario 4.  Explore concerns


The key word is concerns. It is very easy to assume what a
Re: Mr Henry Jackson, aged 45 years patient’s concerns are likely to be and then address
Mr Jackson was admitted with a large anterior myocardial assumptions. It is not uncommon to overlook a patient’s
infarction and subsequent heart failure, requiring initial ionotropic concerns when giving advice after, for example, a coronary
support. His echocardiogram showed moderate systolic dysfunction
and angiography a critical lesion in his left anterior descending
syndrome, as there are standard areas that are usually
artery for which he received percutaneous coronary intervention. discussed relating to medications. Patients do not always
He has been established on numerous secondary preventive drugs voice their concerns, thinking the doctor ‘too busy’ or
(aspirin, clopidogrel, a statin, an angiotensin-converting enzyme because they are afraid or simply do not know how to. It
inhibitor, a beta blocker and eplerenone). He is being discharged is important to ask.
and has some concerns about going home.
Your task is to discuss his concerns and negotiate a There are a number of things to explain before you go home,
management plan for him following discharge from hospital. but before we start is there anything worrying you in
particular?

631
Station |4| Communication skills and ethics

Concerns will include return to a normal life, return to 10.  Agree a way forward and ensure follow-up
work, impact on family life, likelihood of taking tablets arrangements are in place
indefinitely, whether it could happen again and the way
forward from here. Reassure him of follow-up arrangements with your team,
including the cardiac rehabilitation team, that you will
contact his general practitioner (GP) about his condition
5.  Share management options
and that you would happily review sooner at any time at
Patients vary in the extent to which they wish to be his or his GP’s request if there were any questions or
involved in management decisions, but most want to be concerns.
informed of possible options. It is worth stating your posi-
tion if there appears to be a strongly ‘right’ option clini-
cally and a patient is not keen to concord (e.g. in taking Discussion
secondary preventive drugs). Where there is a range of What do you understand by the term
clinically acceptable options you could deploy ‘thinking
aloud’ skills, observing the patient’s reactions or asking,
chronic disease or long-term condition
encouraging good ideas and gently countering bad ones: (LTC)?
LTCs generally last longer than a year, are incurable,
I wonder…
require ongoing care and often progress. They include
It might help if… diabetes, arthritis, asthma, coronary heart disease and
I expect that if we…(never say what will happen, only what chronic heart failure.
seems likely)

Explicit categorisation can help, telling your patient that


Should LTCs be managed by primary or
there are three things, for example, you want him or her secondary care?
to know, and that they are as follows. Integration is crucial! Multiple areas of care have evolved
separately in the NHS (primary care, secondary care, out-
6.  Frame your suggested management plan patient clinics, social care, mental health, intermediate
Being patient centred is shepherding rather than abdicat- care), but the assumption in the past was that general
ing all responsibility for decisions to the patient. As well practitioners would coordinate all care. But an ageing
as exploring concerns, you will have a framework of things population and proliferation of LTCs make this an increas-
you need to discuss that might include reasons for each ingly difficult task. Ideally, secondary care admissions
medication (offering to write these down), need for should fit as acute episodes within a continuum of care
follow-up (hospital and cardiac rehabilitation nurse) and for a chronic process that is integrated between multiple
secondary preventive modifications to lifestyle. Most services in close liaison. The three broad contexts of man-
patients prefer doctors who are open, informative and aging LTCs are acute care, disease management (monitor-
consider their concerns, but with one proviso – that their ing, treatment adjustments, secondary prevention) and
doctor appears to be actively in control of the problem. A frailty management (recognising when patients, often
good analogy is with active and passive listening. A doctor with multiple LTCs, and often in the last year or two of
who sits forward, makes good eye contact and seems atten- life, can no longer adapt to their environment).
tive is more likely to inspire confidence than one who sits
back and appears uninterested. Equally, a doctor who is Is there a role for community geriatricians?
actively trying to negotiate the best management strategy The proportion of elderly people with LTCs and frail
through considered, informed discussion is more likely to elderly is expanding with a predicted doubling of people
achieve a better outcome. aged over 80 in the next 20 years.
The increasing cohort of older people and greater sur-
7.  Respond to patient cues vival in older people brings with it a greater morbidity in
Throughout, be alert to his reactions or ask what he thinks, LTCs often with acute crises prompting admission to hos-
encouraging good ideas whilst gently countering bad ones. pital. Elderly care expertise in the community clearly helps
to prevent hospital admissions to acute hospitals, to
provide more treatment in community hospitals, and to
8.  Confirm understanding maintain people at home with regular review in their
It can sometimes be useful to ask the patient to recount locality. It allows advance decisions to be made about the
what they feel to be the important points of the appropriateness or otherwise of admission to hospital in
discussion. the event of acute illness. It also allows comprehensive
assessment of people entering long-term care. Publications
9.  Encourage feedback and invite questions such as the Department of Health’s Supporting People with
I appreciate we’ve covered rather a lot there. Is there anything Long Term Conditions, endorsed by the British Geriatrics
you would like me to go over again or are there any Society, make a strong case for the role of geriatricians in
questions you would like to ask me? intermediate and community care.

632
Case 4.8  Encouraging concordance with treatment and prevention

Most people with dementia and other advancing  Scenario


LTCs have not given any advanced indication of their
wishes in the event of life-threatening illness and whether
Re: Mr Marc Davey, aged 46 years
or not they would wish to be transferred to an acute 
Mr Davey was in hospital 2 months ago with severe
hospital should they develop pneumonia or other poten- hypertension (240 / 120 on admission) and renal impairment.
tially terminal illness. The government’s initiative Pre- Investigations did not reveal a secondary cause for hypertension
ferred Priorities for Care (formerly Preferred Place of  and urinalysis revealed isolated proteinuria. The renal team
Care) addresses this, and geriatricians, in consultation decided against renal biopsy and thought it very likely he had
with GPs and other health professionals, must ensure nephrosclerosis from hypertension. He was established on an
these issues are addressed at a stage when a patient’s cogni- angiotensin-converting enzyme inhibitor and calcium channel
tive skills are still intact – at least sufficiently so to discuss blocker. His electrocardiogram showed left ventricular hypertrophy.
with them, their family members and carers potentially His blood pressure on discharge was 140 / 85 but today in clinic is
180 / 100. He told the nurse that he has not been taking his
predictable problems. This is not to deny at all that 
medications because he has felt well.
many care-home residents may benefit from appropriate Your task is to discuss ways to improve his blood pressure
interventions, but where capacity is dwindling extreme control.
care is needed by people attuned to the person’s needs that
their common-law rights are respected, and take priority Your examiners will warn you when 12 minutes have
over defensive actions by care-home managers or deputis- elapsed. You have 14 minutes to communicate with the
ing services that default to acute hospital admission. If patient / subject followed by 1 minute of reflection. There
appropriate and timely sensitive discussions and decisions will then follow 5 minutes of discussion with the examin-
are implemented, and community support is available ers. Do not take the history again except for details that
with geriatrician support, inappropriate admissions to will help in your discussion with the patient / subject. You
hospital and undue distress to patients and relatives  are not required to examine the patient / subject.
could be avoided; and instead of aiming to achieve 
admission in less than 4 hours through the emergency Patient / subject information
department many patients will be better served by 
advance decisions and community care in a familiar and Mr Marc Davey is a 46-year-old man being reviewed in
more comfortable environment rather than the acute clinic. He was in hospital 2 months ago with headaches,
hospital. severe hypertension and resulting kidney damage. Investi-
A major role of day hospitals, where patients could gations did not reveal a secondary cause for hypertension.
receive low-intensity investigation, treatment and care,  He was started on perindopril and amlodipine to control
was to preserve frail people in the community, but this, his blood pressure but stopped taking these soon after dis-
sadly, was largely eroded by the advent of private nursing charge because he developed an irritating cough. He has felt
home care. well since and did not see a need to continue medications.
The hospital team had told him it was important to con-
Are domiciliary visits still important? tinue taking tablets but he does not remember the reason.
He has a wife and two children and enjoys life. He is not
Domiciliary visits generally serve one of three purposes –
overweight, but has successfully given up smoking. He
to assess patients too unwell or frail to visit outpatient
manages a vineyard, but does not consume excess alcohol.
clinics, to assess patients in their own environment and
understand the medical and social context and, rarely
these days, to provide the human face of the hospital – How to approach the case
seen by many as a place of no return to fear – and reas- Communication skills (conduct of interview,
surance that the aim is to enable rather than be the thin
end of the wedge that culminates in nursing home
exploration and problem negotiation) and
placement. ethics and law
1.  Introduction
Introduce yourself and confirm his identity.
CASE 4.8  ENCOURAGING
2.  Clarify the task
CONCORDANCE WITH TREATMENT
Make it clear that achieving better control of his blood
AND PREVENTION pressure is the aim of the discussion. Do not assume at
the start that you are dealing with poor concordance.
Candidate information
3.  Explore whether poor concordance could be
Role a problem
You are a doctor in the medical outpatient clinic. The term concordance is often preferred to compliance
Please read this summary. since the former suggests agreement between patient 

633
Station |4| Communication skills and ethics

and doctor. Concordance should always be checked  7.  Repeat important information
before evaluating the effectiveness of a treatment. Non-
This is especially important when poor concordance 
concordance with antihypertensives is common, and it is
may be due to misunderstanding or previous poor
important not to apportion blame. Patients do not take
explanation.
medications for many reasons and the discussion should
begin with this premise and not by highlighting this
8.  Confirm understanding
patient’s ‘failure’. Before you attempt to influence him in
taking his medications, consider the three questions in It can sometimes be useful to ask the patient to recount
Box 4.8. what he or she feels to be the important points of the
discussion.
4.  Be alert to ideas, concerns and expectations
9.  Encourage feedback and invite questions
His cough is probably the result of the angiotensin- 
converting enzyme (ACE) inhibitor but he may think it a I appreciate we’ve covered rather a lot there. Is there anything
you would like me to go over again or are there any
side effect of all antihypertensives. He may, at the time,
questions you would like to ask me?
have appreciated the severity of his illness but may not
understand that despite feeling well hypertension could
lead to worsening chronic kidney disease and an ultimate
10.  Agree a way forward and ensure follow-up
need for renal replacement therapy. He may also not arrangements are in place
understand the other effects of uncontrolled hypertension, Reassure him of follow-up arrangements and that you
notably stroke. would happily review sooner at any time at his or his
general practitioner’s (GP) request if there are questions
5.  Counter misunderstandings or concerns.
Explain that ACE inhibitors can cause cough, but not
amlodipine, and that there are alternatives (e.g. angi- Discussion
otensin receptor blockers). Explain the risks of uncon-
trolled hypertension.
A 34-year-old man being treated for a
high-grade non-Hodgkin’s lymphoma on
6.  Discuss management options within a clear your haematology ward is 3 days into a
framework chemotherapy regimen involving high-dose
Frame the discussion around advantages that are likely to steroids. This morning, according to your
be important to him – retarding progressive chronic house officer, he was in a strange mood,
kidney disease, reduction in stroke risk, need for future tearful but also declaring he believed
hospital admissions and so on. Such gift wrapping of treat- himself cured. He left the ward in pyjamas
ments can be highly effective. Sometimes illustrating the
point with reference to a similar case can help. Explain and overcoat and has not returned. What
that all blood pressure tablets can have side effects but that should be done?
these are usually mild and if problematic a different tablet Patients have the right to decide how they are going to be
can always be considered. treated and indeed whether they are going to be treated.
Such autonomy should be respected. But where declining
treatment is likely to have serious consequences it is
Box 4.8  Questions about concordance imperative to explore a patient’s reasoning. Here, iatro-
Are you dealing with poor concordance? genic mood disturbance from high-dose steroids must be
considered, and his ‘autonomy’ questioned until further
Often the importance of taking medications has not been carefully
assessment has been made. Informing ward staff, the con-
explained or sufficiently emphasised. Even when explained, patients
may still not fully understand or may forget the importance of sultant, the patient’s GP, relatives, and, on discussion with
medication. the consultant, involving hospital security and the police
(restraint under the Mental Health Act is generally a last
Is poor concordance due to iatrogenic resort but may need to be instituted) are steps which may
symptoms? be needed to ensure his safety.
Always consider drug side effects or drug–drug interactions as a
reason for a patient not taking prescribed medication. How might you help persuade a patient
Is the poor concordance due to another reason to stop smoking?
that needs to be explored? As well as the serious cardiac, respiratory and neoplastic
Very often, poor concordance is due to discrepancy between what risks of smoking, it carries considerable social harms and
the doctor sees as important and what the patient sees as promotes premature ageing. Smoking cessation (Box 4.9)
important. carries immediate benefits, including mortality reduction,
and reduces the risk of many diseases including

634
Case 4.9  Cross-cultural communication

Box 4.9  Discussing smoking cessation chronic osteomyelitis of his knee. Tuberculosis has not been
suggested by chest X-ray or other results so far.
Ask about current smoking status Your task is to explain the likely diagnosis to him and
This includes asking about whether other household members propose referral to your orthopaedic colleagues for further
smoke. treatment and possible bone biopsy and surgery.

Advise about the benefits of smoking cessation


These include physical, social, financial and cosmetic. Emphasise the Your examiners will warn you when 12 minutes have
health benefits of stopping rather than the risks of continuing elapsed. You have 14 minutes to communicate with the
smoking. Reinforce the patient’s own motivation for wanting to stop.
patient / subject followed by 1 minute of reflection. There
Ask about reasons for wanting to stop and will then follow 5 minutes of discussion with the examin-
assess motivation ers. Do not take the history again except for details that
Ask if the patient really wants to stop and if they would be will help in your discussion with the patient / subject. You
prepared to stop now or within the next few weeks. Find out about are not required to examine the patient/subject.
previous attempts to stop and what measures helped or hindered.
Ask if other household members are willing to stop, too. Patient / subject information
Discuss ways of assisting Mr Ash Khan is a 38-year-old asylum seeker. He has been
These could include setting a date and stopping completely, in the UK for 1 year with his wife and two children and
enlisting the help of family and friends, enlisting the help of health has found work in a restaurant. He was referred to clinic
promotion services and GP, nicotine replacement therapy (best with sweats, fevers and painful joints, although language
avoided in severe cardiovascular disorders) and bupropion differences have thwarted good history taking on previous
(amfebutamone). It helps many patients to consider themselves occasions. On one occasion there was an interpreter in
‘non-smokers’ rather than ‘ex-smokers’.
clinic but he is not here today. However, Mr Khan’s English
comprehension has improved steadily over the last year.
After extensive investigations he appears to have chronic
cardiovascular disease, chronic obstructive pulmonary
osteomyelitis affecting his knee. Tuberculosis is possible
disease, cancer of lung, mouth, throat, larynx, oesophagus,
but not likely on the basis of tests so far. Mr Khan simply
pancreas, bladder and cervix, peptic ulcer disease and
wants to be relieved of pain and has been taking a lot of
complications of pregnancy. Nicotine addiction is now
tablets from a local herbalist.
acknowledged as a treatable condition and there is sub-
stantial evidence of the effectiveness and cost effectiveness
of behavioural and pharmacological interventions. How to approach the case
Communication skills (conduct of interview,
Communication in special exploration and problem negotiation) and
circumstances ethics and law
1.  Introduction
Introduce yourself and confirm his identity. Inevitably if
CASE 4.9  CROSS-CULTURAL there are language differences a consultation has the
COMMUNICATION potential to take longer and ideally in practice (not in
PACES) you would allow more time for this.

Candidate information 2.  Remember to treat patients equally and do


Role not do very much differently!
When people communicate, there is a vast array of cultural
You are a doctor in the medical outpatient clinic.
influences. Cultural background, health beliefs and expec-
Please read this summary.
tations affect all health-care encounters with patients. These
Scenario differences may seem stark when there is an additional
language difficulty, but perhaps most important is to
remember that we are usually more similar to each other
Re: Mr Ash Khan, aged 38 years than we are different! The range of human beliefs, concerns
Mr Khan is an asylum seeker. He has been in the UK for 1 year and expectations that we must be aware of with any patient
with his family and was referred to your clinic with sweats, fevers
needs to be considered, and perhaps with heightened
and painful joints, although language differences have thwarted
some elements of history taking on previous occasions. On one awareness when treating people from different cultures.
occasion you had an interpreter in clinic but he is not here today. Beware of the tendency to stereotype people – to assume
However, Mr Khan’s English comprehension has improved steadily they have characteristics because they appear to belong to a
over the last year. After extensive investigations he appears to have certain group. When stereotyped, people can become

635
Station |4| Communication skills and ethics

victims of prejudice – opinions, often unfavourable, formed


without good reason. This can jeopardise good care. Box 4.10  Potential problems when using
interpreters
3.  Establish as far as possible the patient’s
• Assuming the patient cannot understand any of what is being
concerns and give non-verbal reassurance said – beware of assumptions and seek to find out what a
This patient might simply want to have his pain relieved, patient can and cannot understand
and you should indicate that this too is a concern of yours, • Failure to address both the patient and the interpreter
but that you wish to treat the underlying cause. • Failure to communicate simply and in small chunks
• Incorrect understanding or transmission of information
4.  Frame your suggested management plan • Avoidance of delicate topics
The medical steps are fairly clear – continuing to relieve pain, • Domination of the patient by the interpreter
and urgent referral to orthopaedic colleagues. Sometimes • Failure to allow enough time (at least a doubling of time is
religious reasons mandate an approach to management. often required)
During Ramadan, for example, Muslim patients may not eat
anything, including tablets, during daylight hours and often
eat a large meal after dark. Type 2 diabetes is common in
Asian people and if Muslim there may be important reper- commonly to describe people of a shared national, ethnic
cussions on the style of management chosen. or regional origin, but there is a dynamic social context
for every individual’s culture that embraces age, gender,
5.  Keep explanations clear and simple,   education, socioeconomic background, language, family,
avoiding jargon occupation, religion and so forth.
The same principles of good communication apply when What potential problems may arise when
dealing with patients from different cultures. In other
words, you should try to do more of the same, rather than using interpreters?
treat such patients differently. In this case, as well as These are outlined in Box 4.10.
explaining the way forward with respect to input by the
knee specialists, you might need to counter the herbal
remedy beliefs. You might also need to explore his work
further, particularly if tuberculosis is still in question! CASE 4.10  COMMUNICATING
WITH ANGRY PATIENTS
6.  Repeat important information OR RELATIVES
This can be particularly important when communicating
with people whose first language is not English.
Candidate information
7.  Respond to patient cues
Role
Throughout, be alert to his reactions or ask what he 
thinks, encouraging good ideas whilst gently countering You are a doctor on the medical ward.
bad ones. Please read this summary.

8.  Confirm understanding and acceptance Scenario


It would be important to ask him to recount what he
understands to be the main points of the discussion. Re: Mr Charlie Buchanan, aged 69 years
Mr Buchanan has a left hilar mass on his chest X-ray,
9.  Encourage feedback and invite questions discovered during the course of investigations for pneumonia.
Is there anything you would like me to go over again or are The computed tomography scan suggests a potentially operable
there any questions you would like to ask me? bronchial carcinoma, with lymphoma the main differential
diagnosis. Although recovered from his pneumonia, and now
10.  Agree a way forward and ensure follow-up feeling well, he was kept as an inpatient because the respiratory
team had a slot for bronchoscopy at the end of the week.
arrangements are in place However, you have now been informed it has been postponed
Ensure that there are arrangements for treatment and because of a full list of urgent bronchoscopy appointments. You are
follow-up. also aware that the respiratory team is thin on the ground next
week because of a conference and have been informed by the
Discussion respiratory specialist registrar that it will be the week after next.
He advises you to send Mr Buchanan home in the meantime. Mr
What do understand by the term culture? Buchanan is angry.
Your tasks are to listen to his concerns and discuss possible
It refers to the shared beliefs, values and attitudes that solutions.
guide the behaviour of a social group. It is applied most

636
Case 4.10  Communicating with angry patients or relatives

Your examiners will warn you when 12 minutes have emotion such as fear, guilt or uncertainty. Discovering the
elapsed. You have 14 minutes to communicate with the underlying emotion is far more likely to achieve resolu-
patient / subject followed by 1 minute of reflection. There tion than taking anger at face value. Since minor disagree-
will then follow 5 minutes of discussion with the examin- ment can explode unpredictably into more serious
ers. Do not take the history again except for details that confrontation, dialogue should, from the start, work
will help in your discussion with the patient / subject. You towards de-escalation and resolution.
are not required to examine the patient / subject.
4.  Acknowledge the concerns
Patient / subject information Legitimising rather than confronting his understandable
concerns will help bridge the gap between his initial anger
Mr Buchanan is a 69-year-old man with possible lung and your remit to help:
cancer discovered on chest X-ray during the course of
investigations for pneumonia. The computed tomography Obviously you are very upset by this. I understand.
scan suggests a potentially operable tumour. Although I can understand fully why you are upset. I am very concerned,
recovered from pneumonia, and now feeling well, he was too, that you are having to wait.
kept in hospital because the respiratory team had arranged
his bronchoscopy at the end of the week. However, the 5.  Explore the emotions and concerns
respiratory specialist nurse advised him that this has now
In this case fear of cancer spreading and opening of the
been postponed and that it might be best if he went home
old wounds of his wife’s experiences will doubtless be
as the respiratory team will not be around next week. Mr
forefront in his mind:
Harris does not think he should be told ‘what is best for
him.’ Mr Buchanan lives alone and his son and daughter- I agree that we must sort this out as quickly as possible. I will
in-law are going on holiday next week. He has a history do everything I can to help. It will help me if I understand
of anxiety and depression and feels that going home what you fear most if the investigations are delayed.
without all of this being resolved and without support will
be hard. His wife died from oesophageal cancer 2 years 6.  Work towards resolution, weaving in the facts
ago and he recalls similar delays with her investigations. Steer away from areas of conflict – comments from other
He is worried that each day missed is a day when the staff that may have upset him, for example. Good com-
cancer could be spreading. He has always been an avid munication and finding common ground are the keys to
supporter of the NHS but his support is now beginning to successful resolution. Focus on constructive ways forward.
waver and with this news he feels angry. He is particularly
upset that the respiratory team are going to be ‘sunning 7.  Try to ameliorate concerns that can honestly
themselves’ whilst he waits for vital tests. be ameliorated
Give clear advice and honest professional opinions about
How to approach the case the implications of delay. Delays in investigations are
Communication skills (conduct of interview, unhappily part of everyday experience in the NHS, and 
not always with as devastating implications as patients
exploration and problem negotiation) and imagine.
ethics and law
1.  Introduce yourself 8.  Never criticise colleagues
Introduce yourself and confirm the name / identity of the Avoid comments that might incriminate colleagues. It
patient. would not help to focus on the fact that the respiratory
team is largely away next week, as educational events are
inevitable and essential components of medicine and you
2.  Make it clear that you want to help
are not responsible for how another team has organised
Show from the outset that you are here to try to help. itself or at liberty to comment. Criticism of any colleague
Remain polite. The nature of medical training equips most or department or service is always counterproductive. Most
doctors well with the skills of negotiation. Some patients NHS employees are stretched and working together is
simply do not have these skills. People are capable of paramount.
acting out of character and appearing aggressive when
upset. His primary motivation is likely to be fear or dis- 9.  Encourage feedback and invite questions
tress and he is depending on you to help. Do not take any
I appreciate we’ve covered rather a lot there. Is there anything
criticisms of the system personally. you would like me to go over again or are there any
questions you would like to ask me?
3.  Remember to deal with emotions before facts
and that anger is not usually anger 10.  Agree a way forward
Always try to calm emotions before dealing with any facts Assure him that you will find out exactly when his test is
and remember that anger is usually secondary to another going to be.

637
Station |4| Communication skills and ethics

Scenario
Box 4.11  Handling threatening, abusive and
violent behaviour
Re: Mr George Tilner, aged 63 years
• Unhappily abuse by patients to staff is increasingly Ms Tilner is the sister of a 63-year-old man, Mr George Tilner,
common, often threatening words but sometimes physical or who has just died in the emergency department. He had advanced
even sexual. non-Hodgkin’s lymphoma that was no longer responsive to
• Doctors must avoid the temptation to take the law into their chemotherapy, and had recently received palliative radiotherapy. He
own hands, and first and foremost appreciate that there are was admitted 2 hours earlier from a peripheral hospital with sepsis
often medical reasons for this (confusion due to infection or and peri-arrest but your rapid perusal of his case notes showed
metabolic disturbance, for example) and such patients cannot that he had spinal and renal metastases and you judge that a
be left without treatment. decision to resuscitate would not be appropriate if he had a
• If threats are towards a particular staff member they should no cardiac or respiratory arrest despite fluids and antibiotics. He was
longer be involved in that patient’s care. an accountant and his last wish was that he completed his clients’
• Sedation may be considered if violence is a symptom work and that his briefcase be brought with him in the ambulance.
of illness and it is necessary to prevent injury to patient You know that his sister is in the relative’s room, and is very upset.
or staff. Your personal feeling is that an advance decision should have been
made to manage any deterioration at the peripheral hospital.
• Restraining measures are occasionally necessary, but may Your tasks are to explain to her that he has died, and manage
inflame or worsen a situation and cot-sides are frankly the emotional situation.
dangerous.
• Advice from a psychiatrist may be sought, especially if
anti-psychotic medication is needed.
Your examiners will warn you when 12 minutes have
• Security should be called immediately if a patient is physically
violent. But removal by the police is not acceptable if there are elapsed. You have 14 minutes to communicate with the
medical reasons underlying the behaviour. patient / subject followed by 1 minute of reflection. There
• Empathy and practical support might need to be given to will then follow 5 minutes of discussion with the examin-
nursing and other staff. ers. Do not take the history again except for details that
will help in your discussion with the patient / subject. You
are not required to examine the patient / subject.

Patient / subject information
Discussion You are the sister of Mr George Tilner, a 63-year-old man
admitted to the emergency department 2 hours ago from
Would you encourage him to make a formal a peripheral hospital. He has advanced non-Hodgkin’s
complaint? lymphoma that was no longer responsive to chemother-
He is perfectly at liberty to complain, but more imperative apy, and had recently received palliative radiotherapy. You
is to find a mutual way forward than potentially drive the have just arrived at the hospital, having received a call
divide further by encouraging a complaint. However, if he from nursing staff at the peripheral hospital to say your
requests help in making a complaint you should direct brother was very unwell with infection and was being
him to the local complaints procedure. If you feel that transferred to the emergency department. He was an
there may be a system failure a Clinical Incident report accountant and his last wish was that he completed his
could be forwarded to Trust Risk Management. clients’ work. Although you knew he was very unwell, you
are still awaiting news from the doctor about his condi-
What do you do if a patient is threatening, tion. You hope the doctors will do all that they can to save
his life, but your sister-in-law is a district nurse and has
abusive or violent? gradually persuaded your over recent weeks that in the
Strategies are outlined in Box 4.11. event of a deterioration from which recovery seems remote
it might be best to ‘let him go’ peacefully. You are about
to speak to the doctor.

CASE 4.11  COMMUNICATING How to approach the case


WITH UPSET OR DISTRESSED
RELATIVES Communication skills (conduct of interview,
exploration and problem negotiation) and
ethics and law
Candidate information
1.  Preparation and scene setting
Role Introduce yourself and confirm the identity of the relative.
You are a doctor on the acute medical unit. Ideally, you would ensure that a member of the nursing
Please read this summary. staff is there, and talk in a setting where interruptions are

638
Case 4.12  Discharge against medical advice

very unlikely to occur. Sit at the same level as the relative, 9.  Check present information needs
and use a calm tone.
While it is generally good practice to encourage feedback
and invite questions, this is one situation where asking 
2.  Give vital information early a relative if she has further questions can sound a little
Do not give a detailed account of events up to his death, pressurising if handled as a direct question. It might 
other than making it clear that he was very unwell. be preferable to give an assurance that you can answer
questions now but are available to talk further at any time
Ms Tilner, your brother was very unwell – comatose – when he later.
arrived, and I am very sorry to tell you that he has passed away.

While bad news communicated to patients should be 10.  Make clear what support is available
delivered gently with attention to cues from the patient, I’ll leave you with sister / staff nurse for now, but I am here, and
communicating the death of a patient to a relative should my team are here, at any time.
be equally gentle but not delayed, with attention to cues
guiding subsequent discussion. Discussion
3.  Deal with emotions before facts How might you detach yourself from
She is unlikely to take in much more, if any, information
distressed relatives?
immediately. Allow her time to exhibit emotions. It can sometimes be difficult to detach yourself from rela-
tives who are very distressed or upset, although largely
4.  Acknowledge distress and support ventilation there is a discernible point when all that is to be said has
of feelings been said and relatives are not ready or able to ask more
questions. You should always leave relatives with a point
Above all, people need to know that you understand, and of contact, usually a nurse, and express your willingness
you can do so non-verbally, practically (e.g. by offering to come back at any time if there is anything you might
tissues) or by acknowledging that emotions are under- help with or answer.
standable (legitimising). Remember that at this point
patients are too preoccupied to assimilate further
information.
CASE 4.12  DISCHARGE AGAINST
5.  Respond to patient cues MEDICAL ADVICE
Throughout, be alert to any cues.

6.  Gently explain what happened Candidate information


Usually, distressed relatives say what they need to say, and Role
you are there to support, acknowledge and provide what-
You are a doctor on the medical ward.
ever additional comfort seems intuitive.
Please read this summary.
I know that he was very ill. And I know he wouldn’t have
wanted to suffer. His wife, my sister-in-law, knew that he Scenario
didn’t have very long. It’s just such a shock. But he wouldn’t
have wanted things to drag on and be in pain, I know that.
Re: Miss Sarah Walker, aged 23 years
It might be right to say, at this point: Miss Walker was admitted 12 days ago with high fevers,
subsequently confirmed as acute bacterial endocarditis. She has
I know it may not be of any great comfort now, but I think it is been on intravenous antibiotics four times daily with advice from
important you know that I do not believe that he was in any microbiology. She was taking intravenous heroin until the date of
pain … or even aware of things. admission and with assistance from the drug and alcohol advisory
service is now on an appropriate methadone replacement regimen.
Keep all explanation clear and straightforward. She now wishes to discharge herself from hospital. She says that
she feels better. She spends considerable time off the ward
7.  Do not raise personal concerns smoking, and the nursing staff have expressed concerns about
possible ongoing drug misuse, as her partner meets her outside the
Your feeling that acute admission might have been avoided hospital. Your medical plan is to keep her in hospital until her
with advance planning is not helpful at this time. Focus inflammatory markers are much improved, with repeat
on supporting her through what has happened. echocardiography. There remains the possibility of cardiothoracic
surgery being needed if her valve decompensates.
8.  Identify patient support systems Your tasks are to explore her reasons for wanting to
self-discharge, discuss the medical reasons for her staying and find
Try to establish who, if anyone, is going to be available to alternative solutions if she cannot be persuaded to stay.
be with her.

639
Station |4| Communication skills and ethics

Your examiners will warn you when 12 minutes  4.  Explain the medical reasons for wanting the
have elapsed. You have 14 minutes to communicate  patient to stay
with the patient / subject followed by 1 minute of reflec-
tion. There will then follow 5 minutes of discussion with This is essential for her to reach an informed decision
the examiners. Do not take the history again except for because she might be unaware that the micro-organisms
details that will help in your discussion with the must be fully eradicated (and that this usually requires a
patient / subject. You are not required to examine the few weeks of intravenous antibiotics) otherwise the
patient / subject. problem will recur, with worsening damage to the valve.
Explain also that the diseased valve must be reassessed
after treatment. Confirm her knowledge that intravenous
Patient / subject information drug use is a high-risk practice for developing
Miss Sarah Walker is a 23-year-old woman admitted 12 endocarditis.
days ago with high fevers, subsequently confirmed as acute
bacterial endocarditis. She has been on intravenous anti- 5.  Aim to address the patient’s concerns
biotics four times daily. She was taking intravenous heroin
It would be wise to ask the social worker to review the
until the date of admission and with assistance from the
welfare of the children. This could be presented in a non-
drug and alcohol advisory service is now on a methadone
judgemental way to her partner. He would have a right to
replacement regimen but does not feel the dose adequate
look after his children if he is the father and he is fit to do
because she still feels psychological withdrawal. She is very
so but if he is not the father, or if there are concerns as 
keen to try to stop heroin altogether. She has two young
to his fitness to do so, then social workers could 
children at home, both currently being looked after by 
become involved on the presumption of assistance but in
her partner. She has little confidence in him doing so
parallel explore the children’s safety. It might also be 
because he was the person who introduced her to drugs
possible for the children to visit hospital more or for 
and continues to keep unreliable hours at home. She 
her to be allowed home for periods of time between 
is meeting him regularly outside the hospital to try to
antibiotic treatment, although not wise to send her into
negotiate his agreeing to her sister, whom she trusts, 
the community with an intravenous cannula. Addressing
caring for the children. She now wishes to discharge
her other concerns could also lessen the impact of
herself from hospital because of her concerns. She spends
hospitalisation.
considerable time off the ward smoking, but feels the
attitudes of the nursing staff are prejudiced against this,
while she feels it is currently her only enjoyment. Venous 6.  Accept that the patient may self-discharge
access is poor and although she has ‘had enough’ of treat- It is, ultimately, a competent, informed patient’s accepta-
ment she might be persuaded to continue if she knows her ble choice to self-discharge if she or he wishes.
children are safe.

7.  If self-discharge seems inevitable, try to reach


How to approach the case
the best compromise
Communication skills (conduct of interview, This might include discussing with the microbiologist the
exploration and problem negotiation) and possibility of less frequent intravenous antibiotics deliv-
ethics and law ered by a district nurse and / or oral antibiotics and early
follow-up in clinic.
1.  Introduction
Introduce yourself and confirm her identity. 8.  Confirm understanding and acceptance
Confirm that she is fully informed to make her 
2.  Make it clear you want to help
decision.
It is crucial to start with the premise that she may have
fully understandable reasons for wanting to leave hospital,
and show that you are prepared to listen to these. 9.  Encourage feedback and invite questions
I appreciate we’ve covered rather a lot there. Is there anything
3.  Explore concerns and reasons for wanting to you would like me to go over again or are there any
questions you would like to ask me?
self-discharge
Patients’ stated reasons for wanting to self-discharge may
mask deeper or more personal reasons. Fears for family
10.  Agree a way forward and ensure follow-up
members are often high on the list. Frequently a list of arrangements are in place
factors conspires to make the hospital unpalatable – Ensure that she signs a self-discharge form taking legal
uncomfortable investigations and treatments, poor com- responsibility for her own care, document all discussions
munication, noisy wards, other patients, disturbed sleep, clearly and inform your consultant and her general prac-
unpalatable food …. titioner of arrangements.

640
Case 4.13  Delayed discharge

Discussion many weeks. An interim bed is available and has been


suggested by the discharge team in an alternative nursing
Is a discharge against medical advice form a home, but your father died in that nursing home and you
legal necessity? do not want her to go there. You feel she would receive
better care in hospital until the favoured placement
It is not legally necessary, although it may enhance future becomes avail­able, but would be amenable to considering
evidence to obtain signed declaration by the patient that alternatives that did not carry sad and emotive memories.
discharge is against advice. It is important to document You hope the doctor will agree to this and feel your mother
that the patient had capacity and was fully informed and deserves such care.
understood the medical reasons for staying.
How to approach the case
CASE 4.13  DELAYED DISCHARGE Communication skills (conduct of interview,
exploration and problem negotiation) and
Candidate information ethics and law
1.  Introduction
Role
Introduce yourself and confirm the name / identity of the
You are a doctor on an elderly care ward. relative.
Please read this summary.
2.  Make it clear that you want to help
Scenario
Establish from the outset that you want to work with her
to find the best possible solution that caters for the
Re: Mrs Anna Clare, aged 93 years optimum care her mother deserves. The fastest way to dis-
Mrs Clare was admitted to your ward 1 month ago with solve rapport and thwart a way forward is to give the
numerous medical problems – inoperable peripheral arterial impression that you are there to tell her what to do.
occlusive disease, anaemia due to aspirin-induced gastritis,
dizziness induced by antihypertensives and back pain due to
osteoporosis. She has a past history of recurrent strokes and may 3.  Start by listening and agreeing
have had a further one whilst in hospital. She is now very unsteady Ask her if she understands exactly what has happened to
even with a walking frame and assistance and unsafe to go home. her mother during her lengthy hospital admission, and
She has poor memory. She accepts she may need nursing home offer to summarise it. Then state ground where you are
care. Her daughter, Mrs Kenton, is keen on this but wants her to likely to both agree:
stay in hospital until the ‘right one comes up.’ An interim place is
available in a home that is not the daughter’s favoured choice. That’s right. We are, of course, very pleased that she has
Your task is to discuss potential discharge solutions with the recovered somewhat from the problems that brought her into
daughter. hospital. But there have been significant problems, as I know
you are very much aware, some of which are unlikely to go
away and some of which could even get worse over time. So
Your examiners will warn you when 12 minutes have it is the opinion of our nursing staff, our therapists, ourselves,
elapsed. You have 14 minutes to communicate with the and most importantly of course, herself and yourself, that she
patient / subject followed by 1 minute of reflection. There would be safer in the longer term in a place where full-time
care could be provided.
will then follow 5 minutes of discussion with the examin-
ers. Do not take the history again except for details that
will help in your discussion with the patient / subject. You 4.  Explore ideas, concerns and expectations
are not required to examine the patient / subject. Explore any specific fears she has about particular nursing
homes. There is a clear reason she does not want to
Patient / subject information proceed with one nursing home and that is very reason-
able to accept.
You are Mrs Kenton, daughter of Mrs Anna Clare who is
aged 93. Mrs Clare was admitted to hospital 2 weeks ago I understand. I am sorry that that one was suggested to you.
with numerous medical problems (previous strokes, poor
memory, poor circulation, anaemia to aspirin, dizziness 5.  State a suggested management plan
due to anti­hypertensives and osteoporosis). The major I understand that you have a favoured nursing home. We would
issue is that she is no longer safe from the point of view of course support her getting a place there. What may be less
of either her mobility or her ability to be at home and has easy is to get it quickly, and if that is the case then we need
accepted long-term residential care. You have inspected to think very carefully about finding a way forward that is
numerous nursing homes on the advice of the social work acceptable to her and which provides for her needs.
department and have a very definite favourite where a bed She may say that hospital is surely the best place for her
will not be available for an indeterminate time, possibly mother until such time.

641
Station |4| Communication skills and ethics

Hospital has certainly been the best place for her, whilst she has outcome measures where a unit already functions well.
been so unwell. But an acute hospital setting is not ICPs are more useful in setting up a new unit from scratch.
necessarily the best place at this stage. Because our nursing Clerking proformas for particular diseases such as stroke,
staff and therapists are so necessarily occupied with their however, may be useful.
acutely unwell patients, they do not always have that much
time with less acutely unwell patients, who by their very
nature may be more stimulated to do things in a more Hospital trust managers are very keen on
streamlined setting. There are also risks to being in hospital discharge planning. Do you think you
that we often take for granted, but which should not be
ignored – the fact that a hospital bed in a ward of acutely
should be as concerned as your hospital
unwell patients carries by its very nature an increased risk of managers?
developing hospital-acquired infections. Discharge planning is a crucial part of modern hospital
practice, and often badly managed. Depending upon the
6.  Be alert to cues patient, it might include information to patients and
She is likely to show acceptance or disappointment in your carers, training of carers, pre-discharge home visits by the
suggestions. Look out for cues, and respond if necessary. multidisciplinary team (initially to assess the home and
subsequently to assess the patient in their home), pre-
7.  Do not criticise hospital managers discharge case conferences for patients with complex care
Managers generally want to help and must give patients packages, clear instructions of who to contact should there
choice until the multidisciplinary team confirms the need be problems, further community rehabilitation at home
for a specific transfer. Never tell patients or relatives you or in a day hospital and follow-up arrangements.
would like patients to stay in but managers will not allow
it. We all have a duty to discharge or transfer as soon as What is early supported discharge?
appropriate. Managers are often prepared to come and talk Early discharge of selected patients is cost effective and safe
with patients, explaining the number of acute coronary (shown by Langhorne and colleagues in a metaanalysis
syndromes waiting in emergency departments (there has published in the Lancet in 2005). It might include
been an appreciable increase in medical admissions since three to seven daily visits per week for up to a month by
the mid-1990s but an unparalleled and relentless sharp the multidisciplinary team (e.g. occupational therapy
upstroke, especially in the elderly, since autumn 2004 – assistant, physiotherapy assistant, health-care assistant).
with complex reasons but perhaps in part related to a
decrease in out-of-hours primary care) and assuring Is there a role for discharge teams?
patients that they will get the bed they want but that they
cannot wait for it in an acute hospital bed. What they Increasing pressure on ward staff with acute admissions
cannot do is put a patient in an exiting ambulance against within a target-driven health service, and in the context of
their will (assault) or evict (not legally tested and brave changing community resources and a thrust towards
the Trust that tries!). There will always be the difficult or patient choice, has led to difficulties in finding time for
resistant 1% of patients or relatives (but with the best of optimum discharge planning. Discharge teams assist when
intentions and at an emotional time). complex discharge is likely. Such patients might be elderly,
need rehabilitation, have social problems including home-
8.  Encourage feedback and invite questions lessness, or have palliative care needs. Concerns might also
be raised by the family. Discharge teams help patients
I appreciate we’ve covered rather a lot there. Is there anything
through the system by ensuring maximum utilisation of
you would like me to go over again or are there any
questions you would like to ask me? community hospital beds, identifying rehabilitation needs,
ensuring timely paperwork, working with, for example,
9.  Confirm understanding and acceptance social work and palliative care, and generally acting as a
resource for discharge-related dilemmas. Patients will gen-
Make sure that she is generally accepting of your sugges- erally be discharged to one of three settings, as follows.
tions so far.
Home
10.  Agree a way forward
A good social history is vital, and discharge teams may
Agree, for example, to speak to the hospital discharge team work to integrate this information, for example by using
and discuss appropriate interim alternatives. a booklet at the end of the bed for the patient and family
to fill out, along with:
Discussion • Assessing the realism of the patient’s expectations
What is an integrated care pathway (ICP)? • Ensuring early referral to occupational therapy,
physiotherapy or social work
ICPs aim to map a patient’s journey from admission to • Prompting multidisciplinary discussion
discharge or beyond and have been used for many condi- • Prompting specialist assessment of care needs
tions including stroke but have not been shown to improve (SACN) and care packages

642
Case 4.14  Cancer – potentially curable

• Identifying community resources such as nurse multidisciplinary team assessment of breathing, feeding
specialists, the voluntary sector, the housing and diet, gut function, bladder, toileting, dressing, mobil-
department and interim placement. ity and transfer, stair assessment, skin, pain, sensation
(hearing, vision), communication, comprehension, sleep,
Rehabilitation or other interim settings memory, depression and anxiety, initiative and involve-
ment, relationships, past roles and physical behaviours.
Discharge teams coordinate rehabilitation or other com-
Also considered are requirements for equipment.
munity beds for patients who might benefit from rehabili-
tation or for patients with chronic diseases, who have What is meant by the term continuing care?
palliative care needs or who are awaiting care packages or
assessment for entry into long-term care. This refers to a situation where, following a thorough
assessment of needs, a person’s overall health needs are
Care home (residential or nursing, but the term judged to be so great that the NHS will manage and pay for
care home has been adopted since 2003) all the care they need. An NHS professional supervises the
agreed care plan, which can be in any setting, for example
Ideally the patient, family and the multidisciplinary team a person’s own home, a hospice, care home or hospital.
should agree on this. Once medically ready for discharge
and the SACN has been given to the social work depart-
ment, a patient’s discharge is officially delayed, and there
then arises:
Breaking bad news
• Pressure on the patient and family to find a care
home (or accept interim placement) based on advice CASE 4.14  CANCER – POTENTIALLY
from social workers as to which may be appropriate
based on the SACN (if a nursing home is needed CURABLE
then the nursing home staff also assess the patient)
• Pressure on the social work department, whose Candidate information
reimbursement starts dwindling for each day lost,
and pressure on the hospital because an acute bed is Role
blocked.
You are a doctor on the medical ward.
What is meant by delayed transfer Please read this summary.
of care (DTOC)? Scenario
DTOC occurs when a patient is ready for discharge from
an acute bed (clinical decision, multidisciplinary team Re: Mr James Oakley, aged 58 years
decision, safe to do so) and there is no medical reason for Mr Oakley is a previously well man on your ward with recent
the delay. There are various categories, depending on the night sweats, weight loss and abdominal discomfort. He works as
reason for DTOC, such as lack of availability of onward an information technology consultant and has been under a lot of
placement, delayed funding, delay in arranging domicili- work stress recently, travelling a lot and under pressure to meet
ary packages or delay in community equipment. It incurs sales targets. He attributed his symptoms to this but his general
bills immediately to social services. The conflicting practitioner admitted him with high fever and marked
mantras imposed on the NHS of expanding patient choice splenomegaly. He was initially treated for possible atypical
pneumonia but his full blood picture suggests possible chronic
(which often drives unrealistic expectations) and targets
myeloid leukaemia. The haematology team plan to perform a bone
for acute care (with 4-hour movement through emergency marrow aspirate and trephine but he wants to know the diagnosis
departments but prompt discharge if there is not an acute you suspect.
problem) mean that doctors and managers must work Your task is to discuss the possible diagnosis, addressing his
together towards the same goal. concerns.

What is meant by the term


Your examiners will warn you when 12 minutes have
intermediate care? elapsed. You have 14 minutes to communicate with the
This refers to care outside the acute setting that occupies patient/subject followed by 1 minute of reflection. There
a transition period to another setting. It includes rehabili- will then follow 5 minutes of discussion with the examin-
tation after an acute event, responses to an overt crisis and ers. Do not take the history again except for details that
care whilst long-term care is being considered or assessed. will help in your discussion with the patient / subject. You
are not required to examine the patient / subject.
How is a patient assessed for entry into
long-term care? Patient / subject information
Individual trusts have developed processes assessing  Mr James Oakley is a previously well 58-year-old man 
not just the medical diagnosis and treatment but with a history of recent night sweats and abdominal 

643
Station |4| Communication skills and ethics

discomfort. He works as an information technology con- I believe you put your symptoms down to stresses at work. Did
sultant and has been under a lot of work stress recently, any other possibilities cross your mind?
travelling a lot and under pressure to meet sales targets. Have you thought about any other possibilities for the cause of
He attributed his symptoms to this but his general practi- your symptoms?
tioner (GP) admitted him with high fever and marked
splenomegaly. He was initially treated for pneumonia but Such a question both explores ideas and concerns and
his blood tests are apparently not so straightforward. The prepares for more serious news. He might elaborate his
haematology doctors plan to perform a bone marrow aspi- concerns about the possibilities. Non-verbal skills such as
ration but he wants to know why, and what diagnosis his silence, active listening and simple words such as yes, and
doctors suspect. He is married with two children and is then and hmm, and repeating and reflecting back informa-
keen to get back to work soon. tion you are told can be very effective.

So you had been worried about something like that?


How to approach the case
He might indicate immediately that he has an idea of
Communication skills (conduct of interview, what might be wrong:
exploration and problem negotiation) and
I’m worried it might be cancer.
ethics and law
If he already suspects a diagnosis then it is possible to
1.  Preparation and scene setting
gently confirm rather than break bad news at this stage.
‘Bad news’ is any information that drastically changes a
patient’s view of their future for the worse. The way in 3.  Establish what the patient wants to know
which it is communicated affects their perception of the
A very useful step if handled well. Possible questions are:
situation and, crucially, their attitude to living with an
illness and the ability to adjust. Many candidates are natu- You said you wanted me to be honest and open with you. Are
rally adept at creating rapport and building a feeling of you the sort of person who likes to know everything about his
trust with patients. Important components of preparing condition?
include: Would you like me to explain the problem as we understand it
• Having the time and privacy to talk without so far?
interruptions We can’t be certain of what is wrong from the tests so far. But
• Being clear about the diagnosis, problem or test are you the sort of person who likes to know all the
results so far and suitable ways forward possibilities?
• The presence (not in PACES) of a supportive How much information would you like me to give you about
co-worker such as a named nurse or ward sister your diagnosis and treatment?
• The presence (not in PACES) of a relative or close
person if a patient wishes Patients tend to declare outright or indicate that they 
• Sitting close enough for good eye contact and do not want to talk about their disease (in which case it
detection of non-verbal cues; sitting at the same level may be appropriate to ask if they would like you to 
as the patient rather than above (or worse, standing) explain things in more detail to a relative) or they want to
is less threatening to an already apprehensive patient know more.
• Opening with a neutral statement:
How are you feeling today?
4.  Give a warning shot
You and I know that you have had these pains in your tummy When a patient is apparently unaware of potential bad
for some time now. news, the skill is in conveying information without pro-
voking overwhelming distress or pushing them into
• Active listening, being attentive to what you hear denial:
and showing concern and empathy through eye
contact, head nodding and other natural body I’m afraid that things look a little more complicated than we
language. had thought / hoped.
In short, your patient should feel that this discussion is I’m afraid it doesn’t look like a straightforward infection.
the most important thing on your agenda, and not that
you are distracted. The next thing to do is wait! Allow time for the warning
shot to sink in. He can signal that no further information
2.  Establish what the patient knows already is wanted:
Establishing previous experience and knowledge is crucial. Just tell me what we have to do next.
The rule is to ask before telling with such questions as:
or that more information is wanted:
Can you tell me what you understand about your illness?
What have you been told about things so far? What do you mean not straightforward?

644
Case 4.14  Cancer – potentially curable

5.  Break bad news gently 8.  Check present information needs
When the pathologist looked at the blood samples he found Is there anything else you want to ask me about?
some abnormal cells. Do you want to tell me anything more?

Again, he can signal ‘enough’: Research shows that too much information can cause as
big a problem as too little.
What do you need to do next?
9.  Identify patient support systems
or ‘go on’:
Who is at home with you?
What do you mean by abnormal? I need to know if you mean What about family / friends / others?
what I think you do?
10.  Make clear what support is available and
At this point be as honest and informative as possible:
what is going to happen
There is a possibility that they might be cancerous, a form of Never remove all hope, however bad a diagnosis, giving
blood cancer or leukaemia. absolute assurance of what can be done and what should
be done next – further investigations or treatments,
Bad news should always be broken gently but not be support of hospital staff, GP, nursing staff, support groups
unduly delayed. It should be given in clear, simple, small and so forth and follow-up arrangements (making it clear
pieces and patients should be given plenty of opportunity that the patient can be seen at any time before this if there
to respond or question at any stage. Important informa- are questions or concerns).
tion should be repeated and patient understanding might
need to be checked, remembering that patients are likely
to experience a heavy impact and that repeating the word
Discussion
‘cancer’ is not necessary unless it is very clear that it has How should bad news be broken?
not been understood. Such fear words are best used only
when a diagnosis is established beyond doubt or is a There are many suggested approaches on how best to
strong possibility. break bad news. The truth is that there are many accept-
able ways to do it, and many unacceptable ways. One
6.  Acknowledge distress and support   guide or framework for breaking bad news is to use the
SPIKES acronym:
ventilation of feelings
Above all, people need to know that you understand, not
• Setting: Point 1 above
that you feel sorry for them. You can do this non-verbally,
• Perception: Point 2 above
practically (e.g. by offering tissues) or by acknowledging
• Invitation: Point 3 above
that emotions are understandable (legitimising):
• Knowledge: Points 4 and 5 above
• Empathy: Points 6–8 above
It’s normal to be upset. I understand that this is very hard. • Strategy and summary: Points 7–10 above.

Never rush in with immediate optimism or reassurance


(I’m sorry to tell you it’s cancer but the good news is / let me Box 4.12  Mistakes when breaking bad news
tell you what we can do . . .). Always remember that at this
• Avoiding it in the first place and expecting that someone else
point patients are too preoccupied to assimilate further will do it
information. Research shows that subduing or not allow-
• Not scene setting appropriately, e.g. not enough time or not in
ing ventilation of feelings and concerns thwarts recall of privacy
what you tell patients next. • Not establishing background knowledge
• Not establishing how much the patient wants to know
7.  Identify and prioritise concerns • Giving the bad news too quickly, rather than gently
It may be appropriate, after a while, to ask: • Hesitating too long before getting to the bad news, e.g. a
junior emergency department doctor goes to speak to anxious
What are the particular things you are thinking about? parents about the accident their child has had; the doctor
How does this news make you feel? explains what happened in detail, what was done in the
resuscitation area and only after considerable time tells them
It is always important to elicit all concerns before giving that the outcome of their efforts to resuscitate was
advice or information. When concerns are not explored or unsuccessful
patient needs not established patients are more likely to • Not acknowledging the patient’s distress
remain preoccupied with their concerns, fail to assimilate • Not exploring the patient’s concerns
what is said, perceive information as inadequate and • Not exploring the patient’s information needs
develop anxiety or depression that will hinder the control • Being dishonest, e.g. ‘We think you’ll be all right.’
of future symptoms such as pain.

645
Station |4| Communication skills and ethics

Of course no single formula guarantees effective and weeks ago following a transfusion to correct anaemia. The
sensitive delivery of bad news, but appreciation that it is a urgent outpatient tests were performed to look for a reason
conversation in which each exchange directs the next step for anaemia and were upper gastrointestinal endoscopy
– based on evolving knowledge and feedback and your own and an abdominal computed tomography (CT) scan.
intuition – is key. This is why the more experienced, senior Colonoscopy was not performed, jointly agreed by her and
members of the team should ideally break bad news. the doctor when in hospital on account of her frailty and
it being against her wishes. Mrs Paget has mild hearing
Can you think of some common mistakes difficulty but has previously been well and is not taking
when breaking bad news? any regular medications. Cognition is normal but she is
functionally limited by arthritis and dependent on her
These are listed in Box 4.12. family to assist her with dressing and meals. She has not
really considered possible diagnoses but feels that ‘at her
time of life’ she can accept any bad news.
CASE 4.15  CANCER – PROBABLY
INCURABLE How to approach the case
Communication skills (conduct of interview,
Candidate information exploration and problem negotiation) and
Role ethics and law
You are a doctor in the medical outpatient clinic. 1.  Preparation and scene setting
Please read this summary. Introduce yourself, ensuring that she can hear all that is
said. You may need to speak up. Ensure that she and her
Scenario family do not sense that you are pressured for time. Sit
forward, remembering that your first duty is to her, aiming
for good eye contact and active listening. Ask if she agrees
Re: Mrs Jean Paget, aged 86 years to your speaking openly with her and her family:
Mrs Paget is a previously well woman with mild hearing
difficulty, in clinic for the results of recent investigations for iron May I take it that you are happy for us to speak openly in front
deficiency anaemia. She was discharged from hospital 3 weeks ago of your family?
following a transfusion to correct her anaemia. Subsequent urgent
outpatient investigations comprised an oesophagogastroduodeno­ 2.  Establish what the patient knows already
scopy (OGD) and an abdominal computed tomography (CT) scan.
Colonoscopy was not performed, jointly agreed by patient and At this point, it is appropriate to seek her knowledge of
doctor on account of her being physically frail and it being against the reasons for investigation so far:
her wishes. The OGD showed an isolated, large gastric ulcer for
which she has started a proton pump inhibitor and preliminary Could I ask, first of all, what you understand about the reasons
histology confirms malignancy but not the type. The CT scan shows for your recent tests?
two large cystic masses in the pelvis, probably arising from an
ovary. The radiology report suggests that these may be malignant, It may be helpful to summarise:
but you have not had an opportunity to discuss the scan in any
more detail with the radiologist. Mrs Paget’s daughter and You will recall that we recently found you to be anaemic – your
son-in-law are present, supportive family members with whom Mrs blood count to be low – and that one of our concerns was
Paget lives. that there might be a reason for this in the tummy, giving rise
Your task is to break the news to Mrs Paget. to a slow leakage of blood internally. That is why we agreed
we have a look into the tummy with the endoscopy, together
with a scan.
Your examiners will warn you when 12 minutes have
elapsed. You have 14 minutes to communicate with the 3.  Establish what the patient wants to know
patient/subject followed by 1 minute of reflection. There The rule is to ask before telling, but not to delay breaking
will then follow 5 minutes of discussion with the examin- bad news. In this situation it may be appropriate to say:
ers. Do not take the history again except for details that
will help in your discussion with the patient / subject. You We have the preliminary results of those tests, and I take it you
would want me to be frank with you about everything that
are not required to examine the patient / subject.
we know.

Patient / subject information 4.  Give a warning shot


Mrs Jean Paget is an 86-year-old woman who lives with There was an ulcer in the stomach … and, in addition, two
her daughter and son-in-law, supportive family members lumps in the pelvis were seen on the scan … and the early
accompanying her in clinic today. She has come for results signs are that these findings may not be nice.
of outpatient tests. She was discharged from hospital 3 Wait for a few seconds for this to register!

646
Case 4.15  Cancer – probably incurable

5.  Break bad news gently 9.  Identify patient support systems
The key here is to continue explaining results clearly and Find out a little about how things are at home.
slowly, taking account of her reaction as each piece of
information is related: 10.  Make clear what support is available and
The biopsy – when they took a small piece of tissue from the what is going to happen
stomach – suggests that the problem may be a growth. Agree to meet again in the very near future to consider the
best way forward.
6.  Acknowledge distress and support ventilation
of feelings Discussion
At this point there may be little value and further distress
in giving more details unless she asks for more informa- What information would you need before
tion. Now respond to reactions with appropriate gestures seeing her again?
(it may be reasonable to touch on the forearm, for example, A review of the CT scan with the radiologist, the final
during the above explanation). histology report and possibly a CA125 blood test.
7.  Identify and prioritise concerns
The radiologist, at a second view, thinks
Be open to any questions that she or her family might have.
the pelvic masses could be simple cysts
8.  Check present information needs and are likely to be unrelated to the gastric
At the end of this, she might ask what needs to be done. malignancy. The histopathology of the
You do not have enough information to suggest a definite latter, surprisingly, reports a high-grade
course of action, but you can hypothesise: non-Hodgkin’s B-cell lymphoma rather than
We are still waiting for the final pieces of information from the carcinoma. When you see her again, will
biopsy. And I would also like to see the pictures of the scan you attempt to persuade her to have
myself and discuss these in more detail with the radiologist. treatment?
But if this is what we suspect, then you and I will need to
discuss the way forward. It seems likely that the lumps in the You would, of course, discuss the results and reach a
pelvis are connected to what we have found in the stomach, shared management plan. You might suggest that the only
but even if they are not, then I do not think it likely that an way to be sure about the pelvic lumps would be to obtain
operation on the stomach would be the answer. An operation a biopsy, and this would require further hospital interven-
is, I think, unlikely to be possible and even if it were, it would tion. But that this does not escape the fact that there is,
not be without considerable risk. There is a possibility that
involving the stomach, a form of growth called lymphoma
we might hold things at bay for a period of time with some
other form of treatment – drug treatment. But the other that is not treatable by operation. There may be a possibil-
consideration that would be for you and your family to think ity of treatment involving chemotherapy or radiotherapy,
about is that, if you are having no symptoms, it may be but to know the absolute answer to this one would need
reasonable to leave things alone. to obtain more information and seek advice from a spe-
cialist who treats lymphoma.
She might then ask:
And what would happen then, doctor? She asks if she will die without treatment.
How might you respond?
You would need to make it very clear that you would
have a plan to support her: It is sometimes worth exploring why patients ask this
question. It might seem obvious, but it is important to
We would talk with your doctor (general practitioner) and let know what patients are thinking and saying:
him / her know of the situation in full, so that treatment can
I can answer that. But could I firstly ask you why you’ve asked
be given for any symptoms that arise. But in the first place, I
me that?
think it would be sensible for us to meet again in a week or
so, by which time we should have the final results of the might help you understand her main concerns or fears.
biopsy and I should have had an opportunity to review your She might tell you that her husband had chemotherapy
scan in more detail with the radiologist.
and that she does not want any treatment if it is not going
In other words, it is often desirable to discuss the diag- to make her better or cure her and ask:
nosis of cancer at the first appointment, but defer any
Could we do anything else, doctor?
definite decisions about the way forward (especially if
results are still pending) until a second appointment (or to which you could tell her:
subsequent ward round for inpatients), by which time
patient and family will have had time to discuss the impli- We could make sure you were as comfortable as possible and
cations amongst themselves. try to let you spend as much time as possible at home.

647
Station |4| Communication skills and ethics

She says that she does not wish more meticillin-resistant Staphylococcus aureus has been cultured
hospital tests or treatments. She says ‘When and is receiving intravenous antibiotics. Her doctors were
concerned as to why she developed pulmonary emboli
my time has come, my time has come; I’d and a computed tomography scan of her thorax, abdomen
rather take my chances than have any more and pelvis was arranged. Mrs Broadley is a pragmatic
tests or treatment.’ Would you accept this? woman who likes to know the facts, is aware that there is
If this is her informed choice, yes. a suspicious lump that can be felt in her abdomen and
that the scan was to look for evidence of cancer. She is
about to speak to the doctor who has the results.

CASE 4.16  CANCER – PATIENT NOT How to approach the case


FIT FOR ACTIVE TREATMENT
Communication skills (conduct of interview,
exploration and problem negotiation) and
Candidate information ethics and law
Role 1.  Preparation and scene setting
You are a doctor on the medical ward. This case combines confirming bad news to a patient who
Please read this summary. already knows it is a possibility with making a manage-
ment plan. The problems are complex, and the discussion
Scenario should focus on possibilities rather than making absolute
decisions now. In real life discussion might be with her
alone or with relatives if that is her wish, and ideally in
Re: Mrs Rose Broadley, aged 79 years
Mrs Broadley has been under your team’s care for almost a the presence of her nurse. It seems very likely she has
week, and is receiving anticoagulation for two large pulmonary colorectal cancer with lymph node spread and that this
emboli diagnosed on admission. She has been non-specifically explains her thromboembolic tendency. The challenge is
unwell for the past 2 months, feeling tired and losing a little that you do not have a tissue diagnosis, and the need for
weight and admission to hospital was arranged when she anticoagulation and the sepsis make further investigation
developed worsening breathlessness. Unfortunately, she also has a (colonoscopy, biopsy) and potential treatments (curative
swollen left thigh and groin with an ulcerated patch from which surgery or surgery to debulk tumour and reduce the likeli-
meticillin-resistant Staphylococcus aureus has been cultured and is hood of obstruction and / or chemotherapy) difficult.
receiving intravenous antimicrobials. Inflammatory markers have
These facts need to be included in your discussion.
been high, perhaps not surprisingly, but because of the systemic
symptoms and the absence of any clear precipitating cause for her
pulmonary emboli, a computed tomography scan of thorax, 2.  Establish what the patient knows already
abdomen and pelvis was arranged that revealed a large caecal You know that she understands what has been happening,
mass and para-aortic lymphadenopathy. The diagnosis is almost but it is still helpful to summarise:
certainly cancer. Mrs Broadley asked earlier in the week if the
reason for the scan was to look for cancer, which you confirmed. As you know, it has been a difficult sequence of events. You have
Your task is to explain results, and discuss possible ways been feeling unwell for some time, and when you first came
forward. into hospital we suspected, and confirmed, a blood clot on the
lung. This was compounded, as I know you are all too well
aware, by the infection in the upper part of the leg. And you
Your examiners will warn you when 12 minutes have know, as we have discussed, that we were rather worried that
there might be some underlying reason for all of this
elapsed. You have 14 minutes to communicate with the
happening to you … hence, the scan.
patient / subject followed by 1 minute of reflection. There
will then follow 5 minutes of discussion with the examin- She will probably acknowledge all of this. Whilst vital
ers. Do not take the history again except for details that not to delay giving the results of the scan, a preamble like
will help in your discussion with the patient / subject. You this not only leads seamlessly into breaking the bad news
are not required to examine the patient / subject. but also shows that you are fully aware of her case and this
is crucial to maintaining her confidence.
Patient / subject information
Mrs Rose Broadley is a 79-year-old woman who has been  3.  Establish what the patient wants to know
in hospital for almost a week, and is receiving anticoagula- Here you may assume that she wants to know
tion for two large pulmonary emboli diagnosed on  everything.
admission. She has been tired and losing weight for 2
months and was admitted to hospital when she developed 4.  Give a warning shot
worsening breathlessness. Unfortunately, she also has a The scan does give us a reason for all of this. It does show an
swollen left thigh and groin with ulceration from which abnormality, and one that may not be nice.

648
Case 4.16  Cancer – patient not fit for active treatment

5.  Break bad news gently decide that, even if surgery were possible, you would not wish
it. And simply accept that things are likely to progress.
It shows a lump, within the bowel – within the lower part of
the bowel – that is of considerable size. We would fully understand if you were to say ‘if you’re not
sure it’s going to make it go away, I don’t want to go
She will probably register understanding. It may be through the pain of an operation’. On the other hand, if you
appropriate to confirm, as patients cannot start to ventilate were to say ‘I know it’s risky but if it’s the only way to
possibly make things go away…’ we would do our best to
feelings until you do:
bring everything, and everyone, together to make that
It is very likely to be a growth. happen.
One thing that you should be aware of is that, if surgery cannot
Occasionally, if you choose to use the word growth remove it completely, it might be possible to remove some of
rather than cancer, patients will rephrase this by asking it and that without surgery there is a possibility that things
Cancer? – to which it is appropriate to confirm that single could, at some stage, become obstructed.
word. It may then be appropriate to say that there are also We will respect whatever you decide / We will do our best
lumps in glands adjacent to the bowel. whatever you decide. It’s not an easy decision to make, I
know.
6.  Acknowledge distress and support   She may want to know what would happen without
ventilation of feelings further treatment.
Now give her time to come to terms with this news.
We would simply keep a very close eye on things, and do
whatever we can to ease / alleviate the situation.
7.  Identify and prioritise concerns
As a pragmatic woman who has already suspected serious 9.  Identify patient support systems
possibilities, she will probably want to know what needs
to be done. The following is an outline of areas your dis- Find out a little more, if you do not already know, about
cussion should explore. Speak clearly and not too quickly her home circumstances.
and be ready to address questions or clarify information
throughout. 10.  Make clear what support is available and
what is going to happen
There are a number of things to consider. The first is that, What I would suggest, if I may, is that we think about this a
although it seems very likely that this is what we suspect it is, little more and talk again. Meantime, if you agree, we will ask
the way to be certain would be too look directly into the the surgeon to give an opinion on surgery. And we are here
bowel with a flexible telescope with a light on the end – a at any time, before we next meet, if there is anything you
test called colonoscopy – and that would enable us to take a want to discuss, or if you would wish us to sit down with you
needle sample from it. and your family … I am here, other members of my team are
The problem is that the blood-thinning medication you here, and ward sister is here.
are taking makes it a little tricky, though by no means
impossible.
Discussion
We strongly suspect that the colonoscopy would show a
growth, and a nasty one. The reason that we would do it An elderly woman has liver metastases from
would be to consider what to do about it, including surgery or
other forms of treatment. an unknown primary site. She is frail,
I would suggest that we do not make any decisions now deemed medically unfit to undergo
but simply say that ultimately we have to decide – and in more than palliative treatment, and
the last analysis you have to decide – what is the right thing wishes to know ‘just what I need to
to do.
know, doctor, but no more’. You judge her
8.  Check present information needs mental capacity to be borderline. What
She might want to know more about possible might you tell her?
treatments: She could be told that the news is not good; that there is
trouble in the liver that is not nice (to use the word growth
We might be in a position to consider an operation. And what or cancer would very much depend upon feedback from
we would like to do, if you agree, is to ask one of our
her); that a lot of tests could be done to try to find out
specialist surgical doctors to see you and talk though what
they think possible. An operation would not be without where it came from, but that there would not seem to be
considerable risk, because of the blood clots and because of any ground gained by subjecting her to uncomfortable
the infection, and because of the infection it is not something tests; that the fact is the growth is here, and that it seems
we could consider immediately. best now to try to get her home, perhaps with a period of
An operation, depending upon what was found, might carry a convalescence, with the support of family, and keep a close
possibility of getting rid of it, but also the possibility of not eye on things, treating any symptoms such as pain if and
getting rid of it completely. On the other hand, you might when they arise.

649
Station |4| Communication skills and ethics

An 88-year-old man has been losing weight. examiners. Do not take the history again except for details
He smokes and has a recent Horner’s that will help in your discussion with the patient / subject.
You are not required to examine the patient / subject.
syndrome and a Pancoast’s tumour clinically.
He was informed that this is worrying. The Patient / subject information
chest X-ray now confirms it. What would
Mrs Amy March is a 28-year-old teacher who presented to
you tell him? the eye clinic 6 months ago with blurred vision in her 
Using the 10-step framework for breaking bad news, he right eye, diagnosed as optic neuritis. A second episode
might be told: occurred 2 months ago and she was referred to the neurol-
ogy clinic where it was suggested to her, when she asked,
Remember how we discussed that we were very worried about
that multiple sclerosis was a possibility. She is at the clinic
things. The X-ray confirms our worries. What we are seeing
may not be nice. today for results of a magnetic resonance (MR) scan to
look for evidence of multiple sclerosis. She is very con-
If he does not indicate that he has heard enough: cerned about what a diagnosis of multiple sclerosis might
mean if she wished to become pregnant, and the future
We must wait until we know exactly what we are dealing with, implications of this disease.
but there is a high possibility of a growth.

When he is ready, he will want direction, but may  How to approach the case
not know what to say. He may say something as open
ended as:
Communication skills (conduct of interview,
exploration and problem negotiation) and
Not good, is it? ethics and law
You must not leave him without support and clear sense 1.  Preparation and scene setting
of your ownership in helping him: Introduce yourself, and ask how she has been since the
No, but it is here. And what we must now do is decide how we last appointment.
are going to deal with what is here.
2.  Establish what the patient knows already
Establishing previous experience and knowledge is crucial,
not least exactly what she was told about the possibility
CASE 4.17  CHRONIC DISEASE of multiple sclerosis and whether she had still been think-
ing about it.
Candidate information
3.  Establish what the patient wants  
Role to know
You are a doctor in the neurology clinic. Establish what has gone through her mind about multiple
Please read this summary. sclerosis, and if it engendered any particular worries. Here
you may assume that she wants to know everything.
Scenario
4.  Give a warning shot
Re: Mrs Amy March, aged 28 years Tell her that the scan does confirm the thoughts that the
Mrs March was referred to the neurology clinic 2 months ago neurologist had last time.
with blurred vision in her right eye, subsequently diagnosed as a
second episode of optic neuritis; she had had a similar episode 6 5.  Break bad news gently
months previously and attended the eye clinic. During her last visit,
it was suggested by the neurologist, when she asked, that her Explain the results of the MR scan, that it does suggest
symptoms could be due to multiple sclerosis. Magnetic resonance multiple sclerosis and allow her time to take this on 
imaging arranged after that appointment and performed last week board.
has been reported as showing multiple demyelinating plaques in
the brainstem and periventricular areas consistent with multiple 6.  Acknowledge distress and support  
sclerosis.
Your task is to explain the diagnosis.
ventilation of feelings
Watch for her reactions.

Your examiners will warn you when 12 minutes have 7.  Identify and prioritise concerns
elapsed. You have 14 minutes to communicate with the When she appears ready, explore particular concerns. Tell
patient / subject followed by 1 minute of reflection. There her that people with multiple sclerosis can have very
will then follow 5 minutes of discussion with the healthy pregnancies, indeed that sometimes multiple

650
Case 4.18  Discussing an acutely terminal situation with relatives

sclerosis tends to improve during pregnancy, although Scenario


symptoms can flare up after delivery and so close monitor-
ing is needed from the obstetrician and neurologist. Reas-
Re: Mr Frank Wentworth, aged 88 years
sure her that not all patients develop disabling disease,
Mr Wentworth is an 88-year-old man admitted to hospital 2
and that it can vary very much from one patient to another hours ago with a thoracic aneurysm rupture (‘mediastinal
but that most patients continue to lead normal lives. catastrophe’) diagnosed clinically and with a radiologically wide
Explain that it tends to relapse and remit, but that some mediastinum on chest X-ray. You wondered about a computed
patients go for many years between relapses. Explain that tomography scan but your consultant feels that the diagnosis is
her specialist will discuss what, if any, treatments might beyond significant doubt, that there is no alternative, reversible
be desirable at this stage. explanation for his presentation and that a scan would not alter
management. A team decision has been reached to keep him
8.  Check present information needs comfortable. You have just come back to review him, and his
daughter is now with him. You have not spoken with her before
Do no overwhelm her with information; at this stage, but it is clear to her that her father is very unwell and it appears to
largely respond to her concerns: you that he is deteriorating quickly because he is less alert than 2
hours ago, with gradually falling blood pressure. You offer to go to
Is there anything else you want to ask me about? the ward sister’s office to discuss her father’s case.
Do you want to tell me anything more? Your task is to discuss with her what has happened and what
the outcome is likely to be, responding to her distress.
9.  Identify patient support systems
Establish who is at home with her and if her symptoms
have interfered with her work at all. Your examiners will warn you when 12 minutes have
elapsed. You have 14 minutes to communicate with the
10.  Make clear what support is available and patient / subject followed by 1 minute of reflection. There
what is going to happen will then follow 5 minutes of discussion with the examin-
Explain that you will speak with the neurology consultant ers. Do not take the history again except for details that
about any treatment and ensure that things are kept under will help in your discussion with the patient / subject. You
review. are not required to examine the patient / subject.

Patient / subject information
Discussion
You are the daughter of Mr Frank Wentworth, an 88-year-
Would you allow a patient in denial old man admitted to hospital 2 hours ago with a ruptured
of their illness to continue in such denial, thoracic aneurysm. You are a schoolteacher, phoned at
or attempt to change this? work by a nurse at the hospital to say that your father was
very unwell and that you should come in. You have just
Occasionally, it is necessary to challenge denial because it arrived. You do not know anything else, but have been 
hinders their ability to deal with important unfinished at the bedside for a few minutes and can see that your
business or their treatment. It is sometimes possible  father is clearly critically unwell and barely conscious. 
to confront patients gently by highlighting the inconsist- You are not really sure what an aneurysm is but fear the
encies, such as a declared belief they are getting better in worst, although you do not think this means imminent
the face of increasing disability. But where a patient is in decline. Your husband is also on the way to hospital. 
persistent denial, it may be because they are unable to Your father has previously been very healthy, playing 
tolerate the pain of reality and leaving them in that defence golf until last year. He has been widowed for 2 years 
mode until further opportunities during the illness may but has remained independent on his own. You have 
be best. been invited to the ward sister’s office to talk to the doctor.
You are extremely worried and will be very distressed to
hear the worst but need to know exactly what the doctor
thinks.
CASE 4.18  DISCUSSING AN
ACUTELY TERMINAL SITUATION How to approach the case
WITH RELATIVES
Communication skills (conduct of interview,
exploration and problem negotiation) and
Candidate information ethics and law
Role 1.  Preparation and scene setting
You are a doctor on the acute medical unit. Introduce yourself, and sit close enough to allow good eye
Please read this summary. contact and at her level.

651
Station |4| Communication skills and ethics

2.  Establish what the relative knows already 8.  Check present information needs
Ask what she knows so far but do not labour it; she will How long do you think he has got?
want to know what is going on straight away. These hours are critical.

3.  Establish what the relative wants to know One concern that it is always helpful to anticipate and
mention is about pain and distress.
This step in the breaking bad news 10-step sequence
should be bypassed if it is clear that she is waiting in dis- What we can do is make certain that, whatever else, he
tressed anticipation of what she needs to know. is not in any pain or distress. As he becomes less
conscious it is very unlikely he will feel any pain, not
4.  Give a warning shot at this stage. But if we had any reason to suspect
discomfort at any stage we could treat that with
Simply saying: medication, with morphia.
(I’m afraid) the news is not good

may be enough to start. 9.  Identify support systems


You may wish to check that her husband is on his way,
5.  Break bad news gently and whether or not she has other relatives who should be
Explain what has happened, clearly and calmly. So that aware of the situation.
there is little doubt of the critical position, one way to
conclude the explanation would be to say:
10.  Make clear what support is available and
There may not be an easy way out of this … there may not be what is going to happen
a way out of this.
Assure her that the doctors and nursing staff are available
pausing between both statements, the first telling her at any time to speak to her again.
gently that he may not recover, the second building on the
first to inform her that you may have run out of options
to save his life. Discussion
6.  Acknowledge distress and support ventilation Patients with incurable but more
of feelings chronic conditions often ask about
She will be very distressed and now is the time to let your how much time is left. How might
comments sink in, and not the time to give more you respond?
information. Clearly it depends upon the situation. It is important to
acknowledge that you cannot be certain but patients then
7.  Identify and prioritise concerns often ask you to guess. It may be appropriate, if realistic,
Every case must be treated on its own merit. Sometimes  to say something such as ‘Well, the way things have been
it is appropriate to ask how a patient feels about bad  going it might be just months’, but while it is important
news that has been broken. Here, it would be inappropriate to be as honest as possible about prognosis it is important
to question her thoughts, which are obvious, and you not to give specific time frames. Patients sometimes ask
should simply continue to acknowledge her distress, about specific dates like whether they will be here for
respond to questions and speak honestly, clearly and with Christmas. You might say ‘I would hope so’, but ask if
understanding. there is any reason they mention Christmas in particular.
She may ask: It might be, for example, that a relative died at Christmas
and the patient is worried that it will be particularly hard
What can you do? on other family members.
A suitable reply would be:

We can only watch and wait and see if the bleeding stops. It is Why is it important to respond
all that we can do. There is simply no way of getting to where
the bleeding is. He may be dying.
to questions about prognosis?
Right now? Patients might otherwise be misled about their outlook
Yes.
and might not use their remaining time to deal with
important practical and emotional unfinished business
It can be difficult for relatives to accept such abrupt news with loved ones. This will also make the bereavement
like this. But it is vital to be straightforward and honest. process more difficult for loved ones and increase psychi-
atric morbidity from, for example, major depressive
Things are absolutely critical. disorder.

652
Case 4.19  Legal points in confidentiality

2.  Establish rapport


Confidentiality, consent and capacity
There is potential conflict between her right to confiden-
tiality and the rights of her husband (and baby) to infor-
mation and possible treatment in the event that he (they)
CASE 4.19  LEGAL POINTS IN was (were) infected. From the outset, you may be balanc-
CONFIDENTIALITY ing her confidentiality against potential risks to others. But
avoid giving her ‘ultimatums’ at this stage. It is vital not to
‘rush in’. Your first duty is to her, analysing her perspective
Candidate information and how you may best help. Explain this to her, and assure
Role her you are there to help.

You are a doctor on the medical ward. 3.  Explore patient understanding  
Please read this summary.
and concerns
Scenario Ensure she is aware of the problem and try to explore her
beliefs, concerns and expectations about the problem.
Re: Mrs Marcia Smith, aged 30 years
Mrs Smith, a Thai woman who speaks good English, was admitted 4.  Attempt to discover the patient’s reasons  
to the ward with abdominal discomfort and fatigue, now confirmed to for not wanting to disclose
be due to hepatitis B infection. She also has inguinal lymphadenopathy
and you have sought her consent for a human immunodeficiency virus Is she scared about being diagnosed with HIV? If so, why?
test. She has lived in the UK for 6 months with her English husband Is she scared for herself, or for the effect it will have on
whom she met and married a year ago. She refuses the test. She also her family or marriage? Perhaps she knows that she 
tells you not to tell her husband her diagnosis. has been exposed to HIV in the past and has always feared
Your tasks are to explore her reasoning, and discuss the this sort of predicament. Is it possible that she has been
importance of the test and possible consequences of refusal. diagnosed with HIV in the past but has been living in
denial?
Your examiners will warn you when 12 minutes have
elapsed. You have 14 minutes to communicate with the 5.  Share your concerns and desired
patient / subject followed by 1 minute of reflection. There management plan
will then follow 5 minutes of discussion with the examin-
Your first concern is that she may be infected with HIV in
ers. Do not take the history again except for details that
addition to hepatitis B. Try to let her see the importance
will help in your discussion with the patient / subject. You
of excluding or confirming the diagnosis. A supportive
are not required to examine the patient / subject.
approach is essential, letting her see the potential advan-
Patient / subject information tages – the HIV test may be negative; were the test positive,
it would allow earlier institution of treatment and testing
Mrs Marcia Smith, a 30-year-old Thai woman who speaks of contacts (who could well be negative). Much better
good English, was recently admitted to hospital with treatments are available for HIV than her friend would
abdominal discomfort and fatigue, now confirmed to be have received. Denial will not make the problem go away.
due to hepatitis B infection. She also has lymph node She will know this, and her decision to avoid testing and
enlargement in her groin and her doctors have asked if she telling is almost certainly motivated by fear.
might consent to a test for human immunodeficiency virus
(HIV). She has lived in the UK for 6 months with her 6.  Respond to patient cues
English husband, whom she met and married a year ago. Continuously look for non-verbal as well as verbal 
She is very afraid. A friend of hers died 2 years ago from cues.
HIV in Thailand, although sustained treatment had not
been available. She is also very afraid for her 2-month-old
7.  Discuss possible consequences of refusal
baby, and her husband. She does not want the doctors 
to perform the test, nor tell her husband of her current In a non-threatening way, explain that you have a dilemma
diagnosis. of interests. Whilst your first concern is for her, you are
very concerned that in the event that she does have a posi-
How to approach the case tive test there are wider implications.

Communication skills (conduct of interview, 8.  Confirm understanding


exploration and problem negotiation) and Ensure that she fully understands that your first duty is to
ethics and law her and not her husband, that you will not tell her
husband, but that for the best steps to be taken for her
1.  Introduction health and the potential health of her family she needs to
Introduce yourself and confirm her identity. tell him.

653
Station |4| Communication skills and ethics

9.  Allow time to think about what has been


Box 4.13  Circumstances in which confidential
discussed and invite questions information might be disclosed
Allow her time to think about what has been discussed
and to ask questions. She may have initially decided that Circumstances in which confidential information
the worst outcome would be for her husband to know that must be disclosed
she has hepatitis or HIV. Your task has been to identify • When legally required by a Court Order
other outcomes for her to consider. • When a statutory duty to statutory regulatory bodies, as in
communicable disease notification and the reporting of births,
10.  Agree a way forward deaths, abortions and work-related accidents
• In cases of national security such as terrorism or major crime
If she does not immediately agree, it is likely that she prevention or in assisting in the solving of a major crime
needs more time to contemplate what you have discussed (although you may not disclose detailed information to police
and you should at least attempt to end the discussion with officers about any matter).
a plan to talk again. Circumstances in which confidential information
may, at a doctor’s discretion, be disclosed
Discussion • When a third party is deemed to be at risk of harm, e.g. at risk
of contracting a serious infectious disease
Why is confidentiality important? • In the public interest, e.g. informing the Driver and Vehicle
Licensing Agency in the case of a patient with seizures known
Confidentiality is central to the trust that exists in the to continue to drive illegally
relationship between doctor and patient. This has been • Through sharing information with the health-care team.
enshrined in doctors’ codes of professional conduct When disclosure of relevant information between health-care
including the Hippocratic Oath, the Declaration of Geneva team members is clearly required for investigation or treatment to
and General Medical Council (GMC) Guidelines. Infor- which a patient has agreed, the patient’s explicit consent may not
be required. Such sharing of information is not generally viewed by
mation that a doctor learns from a patient ‘belongs’ to that
the law as breaching confidentiality. Examples include passing
patient. From a legal point of view a patient’s case records information to medical secretaries for typing and writing
do not belong to that patient, but from an ethical point information on request forms for investigations.
of view patients should have the right to determine in
Timing of disclosure
general who has access to information they provide.
Breaching confidentiality should usually be the ‘last resort’. The patient
Respect for patient autonomy supports confidentiality and
should be given an explanation as to why disclosure is intended, and
opposes paternalistic attitudes, common in the past, in the doctor must be able to justify the decision. Before disclosing, a
which doctors ‘knew what was in a patient’s best doctor might sensibly seek advice from colleagues, professional bodies
interests’. such as the British Medical Association, and their medical defence
organisation.
What is your legal commitment
to respect confidentiality? What are the possible consequences
The general obligation is to keep patient information con- of breach of confidentiality?
fidential. This obligation is not absolute; the law may These include loss of trust and breakdown in doctor–
allow or obligate breach of confidentiality, but any breach patient relationship, disciplinary action by the GMC,
should only be to the relevant person or authority. The investigation of serious professional misconduct by the
question of whether or not breach of confidentiality is GMC and civil legal action.
legal is often a matter of balancing harm to the patient if
confidentiality is breached against harm to others if it is Does information remain confidential
not. The equation is seldom simple and the law sees a after death?
strong public interest in individual doctor–patient confi- Yes, and information cannot be released without consent
dentiality being maintained. of the patient’s executor or a close relative fully informed
GMC guidelines do not have the force of law but are of the consequences of disclosure.
taken seriously by the Courts.
Breach of confidentiality does not occur if a patient is
not identifiable or if a patient has consented to disclosure CASE 4.20  BREACHING
of information. CONFIDENTIALITY WHEN A THIRD
Doctors have a duty to take reasonable precautions to PARTY MAY BE AT RISK
prevent confidential information from falling into the
wrong hands.
Candidate information
Under what circumstances might Role
confidential information be disclosed? You are a doctor on the renal ward.
These are outlined in Box 4.13. Please read this summary.

654
Case 4.20  Breaching confidentiality when a third party may be at risk

Scenario and treatments has he had so far? Who has been supervis-
ing his care? He may feel that these are irrelevant questions
and that you do not need this information. Explain that
Re: Mr Joe Green, aged 25 years
Mr Green is a builder about to undergo a renal biopsy for
it may be important in building a better understanding of
suspected immunoglobulin A nephropathy. He tells you that he is his kidney problem and that a renal biopsy can be danger-
hepatitis C positive but instructs you not to share this information ous if liver function is impaired.
with anyone else.
Your task is to discuss this with him and decide whether or not 4.  Attempt to discover the patient’s reasons for
the biopsy should proceed. not wanting to disclose
Is he scared about his girlfriend finding out because of the
Your examiners will warn you when 12 minutes have impact on their relationship or are there other reasons?
elapsed. You have 14 minutes to communicate with the You might explore why he has told you of his diagnosis
patient / subject followed by 1 minute of reflection. There but instructed that others not be informed. Exploring his
will then follow 5 minutes of discussion with the examin- fears may give you the answer.
ers. Do not take the history again except for details that
will help in your discussion with the patient / subject. You 5.  Share your concerns and desired
are not required to examine the patient / subject. management plan
Your first concern is that he has a condition that could put
Patient / subject information colleagues at risk. Although all patients should be deemed
Mr Joe Green is a 25-year-old builder who has been diag- potential sources of risk when performing invasive proce-
nosed with probable glomerulonephritis following an dures knowledge of the condition is important both for
episode of macroscopic haematuria. He has been admitted taking extra vigilance with procedures and in his overall
to hospital for further tests including a renal biopsy. He has assessment. Certain forms of glomerulonephritis are more
previously been well but has sporadically taken intrave- common in hepatitis C infection or with HIV (his last test
nous drugs and has had multiple sexually transmitted was 2 years ago). A further concern is for his girlfriend.
diseases. He was tested for hepatitis C at the genitourinary Explain that it would be important for the medical team
medicine clinic 2 years ago and the test was positive involved in the procedure and in his management to know
(human immunodeficiency virus (HIV) being negative) that he has hepatitis C, but that the information would go
but has not required any specific treatment. He was due for no further than those involved in his immediate care.
follow-up blood tests last year but did not attend. He was Explain that it would be sensible to obtain up-to-date
afraid of possible progression. His girlfriend works as a serology for hepatitis B and C and HIV to build up a better
nurse at your hospital. She does not know he is hepatitis C picture of his condition, institute early treatment if there
positive and he is afraid that if this information is recorded is evidence of active disease, and reassure if results are
she could conceivably have access to it. He will tell her unchanged. Explain why his girlfriend should know.
about his hepatitis but now is ‘not the right time.’ He felt
he should tell the doctor in case it affects his biopsy but 6.  Respond to patient cues
does not want anyone else to know. Continuously look for non-verbal as well as verbal cues.

How to approach the case 7.  Discuss possible consequences of refusal


Communication skills (conduct of interview, If he still refuses disclosure, explain that you may need to
talk to colleagues on astrictly ‘need to know’ basis. Discus-
exploration and problem negotiation) and sion with your medical defence organisation may be
ethics and law appropriate.
1.  Introduction 8.  Confirm understanding
Introduce yourself and confirm his identity.
Ensure that he understands the reasons for his hepatitis
2.  Establish rapport being known, not least that he may be offered best
treatment.
There is potential conflict between his right to confidenti-
ality and the potential risk to colleagues, especially the 9.  Allow time to think about what has been
doctor performing the biopsy and those performing phle-
botomy. Start by reassuring him that you will try to help.
discussed and invite questions
Allow him time to think about what has been discussed
3.  Explore patient understanding and concerns and to ask questions.
It is always best to gather information before reaching
conclusions and deciding on a course of action. Ask more 10.  Agree a way forward
about his hepatitis C. How does he know that he has The examiners are aware that such a discussion is difficult.
hepatitis C? How did he contract it? What investigations They are not looking for a ‘model answer’ but checking

655
Station |4| Communication skills and ethics

your awareness of the range of issues provoked and ability


to handle these sensitively and professionally. They would CASE 4.21  BREACHING
be more concerned if you found such a case clear cut and CONFIDENTIALITY IN THE
chose one of two extremes – agreeing to the patient’s
instructions or informing him that you intended to breach
PUBLIC INTEREST
confidentiality without having had careful discussion
around it. If he remains firm in his decision and refuses Candidate information
to allow disclosure, you may conclude that failure to
inform some of your colleagues will place them at serious Role
risk. It is then possible to breach confidentiality on the
You are a doctor in the neurology clinic.
grounds of risk to others and on a strictly ‘need to know’
Please read this summary.
basis. You should inform him of what you propose to do.
Scenario
Discussion
Does his girlfriend have a right to know? Re: Ms Holly Long, aged 22 years
Ms Long is a previously well student brought into hospital 6
A patient who has decided not to tell contacts at risk must weeks ago after a road traffic accident. She lost consciousness at
recognise that this decision may have serious outcomes for the wheel and drove into a wall. She has had a normal ambulatory
these contacts and that breach of confidentiality would be electrocardiogram and echocardiogram. She says she lost
a potential ultimate step. Persuading him to share that consciousness only transiently after she took a fizzy drink giving
knowledge is preferable, and other possible routes could rise to severe heartburn. To build in evidence that this was likely to
be to involve his general practitioner or his genitourinary be situational syncope rather than seizure activity with an aura you
medicine specialist, ideally with his consent. feel she also warrants magnetic resonance brain imaging and an
electroencephalogram. She is here for the results of her heart tests
and is keen to drive again.
A 32-year-old man has presented Your task is to talk with her.
to your general medical clinic with weight
loss and admits to a history of intravenous
drug misuse and having multiple sexual
partners. He is now living with his Your examiners will warn you when 12 minutes have
elapsed. You have 14 minutes to communicate with the
girlfriend, who is pregnant. You mention patient / subject followed by 1 minute of reflection. There
the possibility of HIV infection and he will then follow 5 minutes of discussion with the examin-
admits to having suspected this but adds ers. Do not take the history again except for details that
‘if I had AIDS I would want to kill myself, will help in your discussion with the patient / subject. You
doctor’. He also says that he feels angry. are not required to examine the patient / subject.
How might you respond?
Patient / subject information
It takes time to come to terms with the possibility of a
serious diagnosis such as cancer or HIV infection, and very Ms Holly Long is a previously well 22-year-old student
often anger is a secondary emotion, the primary one fear. brought into hospital 6 weeks ago after a road traffic acci-
At this stage you are not talking to him about having HIV dent. She lost consciousness at the wheel after a gulp of a
infection, but about the importance of testing for it. Again, fizzy drink with immediate heartburn and drove into a
you should attempt to explore his fears, and find out why wall and has had subsequent tests to exclude a heart
he would want to kill himself. It may be that to him HIV problem. She was told she could not drive until after the
is a ‘death penalty’ and that he does not fully understand results. She is here for the results. She has a partner at a
its natural history and current treatments. It may be appro- different university 100 miles away and desperately wants
priate to offer him a referral to trained HIV counsellors for to be able to drive to visit him at weekends. She feels well
further discussion of the implications of the test and its and has decided to drive ‘whatever the doctor says’ and
possible result. His partner and the fetus would be at risk will tell him so.
and would warrant screening for HIV and the hepatitis B
and C viruses if this patient were positive. There are also How to approach the case
implications for the obstetric team. The difficulty is that
this patient has not yet been tested. Although he has not Communication skills (conduct of interview,
refused to be tested, he has threatened self-harm should exploration and problem negotiation) and
the test be positive. Hopefully, exploring his views and ethics and law
reaching a shared understanding of the problem may per-
suade him of the value of being tested and sharing the 1.  Introduction
problem with his partner. Introduce yourself and confirm her identity.

656
Case 4.21  Breaching confidentiality in the public interest

2.  Establish rapport individual or society outweigh the patient’s interests in


keeping that information confidential. Where there is a
Your first duty is to her, analysing her perspective and how
serious risk to patients or others, disclosures may be justi-
you may best help. Show this from the start.
fied even where patients have been asked to agree to dis-
closure but have withheld consent. You should inform
3.  Explain the results
patients that disclosure will be made, where practical to
Explain that the test results were reassuring. Explain that, do so. Ultimately, the public interest can be determined
whilst this is so, the tests do not exclude a seizure. Whilst only by the Courts, and the General Medical Council may
physicians might not treat a first seizure, driving regula- also require you to justify your actions.
tions do not permit driving within 12 months of unex-
plained loss of consciousness.
What is the driving advice
4.  Explore patient understanding and concerns in medical conditions?
Ensure she understands this and explore any difficulties it A person who drives with a medical condition has auton-
might pose. omy and a right to confidentiality. Yet breach of this con-
fidentiality may be more important in the greater interests
5.  Attempt to discover the patient’s reasons for of benefit to society. The Driver and Vehicle Licensing
not wanting to adhere to advice Agency (DVLA) publishes a booklet on driving rules in
medical conditions. A summary of rules for more common
Show that you appreciate her concerns about not being
conditions is given below.
able to drive to visit her partner.

6.  Explain the risks and consider alternatives Seizures


Reiterate the dangerous accident she had, threatening both Drivers with class 1 licences (non-heavy goods or public
her own life and that of others. Are there alternatives to vehicles) must inform the DVLA after a first seizure. They
her, such as public transport? Ensure that she is aware it must be seizure free for 12 months before they may drive,
is only for a period of time. on or off medication. The DVLA may apply discretion and
allow resumption of driving after a clearly provoked
7.  Respond to patient cues seizure (e.g. head injury); alcohol withdrawal does not
qualify as provoked. The same 12-month restriction is
Continuously look for non-verbal as well as verbal cues.
invoked after any subsequent seizure. The restriction is for
3 years if seizures continue but are purely nocturnal.
8.  Discuss possible consequences of refusal Patients should not drive for 6 months following any anti-
In a non-threatening way, explain that you have a dilemma epileptic drug (AED) dose reduction, since there is a 40%
of interests. Whilst your first concern is for her, you are very risk of seizures occurring with drug withdrawal. This
concerned that, were she to continue driving, she might be should be borne in mind with inpatients receiving AEDs;
at risk personally and legally, and be a risk to others. if a seizure occurs through inadequate prescribing then
DVLA restrictions are fully imposed.
9.  Allow time to think about what has been People with non-class-1 licences may not drive until
discussed and invite questions seizure free for 10 years.
Allow her time to think about what has been discussed
and to ask questions. She may have initially decided that Syncope
the worst outcome would be for her not to be able to drive The rules following syncope are outlined in Case 2.41
for 6 months. She may now see that taking your advice is, (Syncope).
on balance, best.

10.  Agree a way forward Cardiac conditions


Remember that patients have a right to a second opinion, The rules depend on the condition. An acute coronary
although it may be appropriate to advise patients that some syndrome imposes a 4-week restriction (reduced to 1 week
decisions are likely to be uniform amongst doctors, particu- if successful percutaneous coronary intervention), and
larly where bound by a professional or legal framework. patients need not inform the DVLA.

Discussion Stroke / transient ischaemic attacks


When may disclosure of confidential A 4-week restriction applies if there are not considerable
problems with weakness, vision or cognition. Visual fields
information be made in the public interest? should be at least 120° in the horizontal plane with no
Personal information may be disclosed in the public inter- greater than 20° loss in up or down gaze. Formal field
est without a patient’s consent where the benefits to an testing is needed if recovery is in doubt, unless the field is

657
Station |4| Communication skills and ethics

Scenario
Box 4.14  Disclosure of confidential information
to the Driver and Vehicle Licensing
Re: Mr Peter Price, aged 34 years
Agency (DVLA)
Mr Price has muscular dystrophy and has been admitted to
• The DVLA is legally responsible for deciding if a person is your ward with lobar pneumonia. He is stable but has a number of
medically unfit to drive and should know of a condition likely, signs associated with a high risk (reduced consciousness,
now or in future, to affect safety. hypotension and leucopenia). His carer, who is not a relative, asks
• Patients should understand that their condition may affect the how he is. You also would value some background information
safety of themselves and others. about his home circumstances, quality of life and any prior
expressed wishes in the event of this sort of situation.
• Patients have a legal duty to inform the DVLA about their Your task is to speak to the carer.
condition.
• If a patient refuses to accept the diagnosis or its safety
implications, you should suggest a second opinion, assist the
patient in obtaining it and advise them to refrain from driving Your examiners will warn you when 12 minutes have
until it has been sought. elapsed. You have 14 minutes to communicate with the
• If a patient continues to drive when medically unfit, you should patient / subject followed by 1 minute of reflection. There
make every reasonable effort to dissuade them, which may will then follow 5 minutes of discussion with the examin-
include informing a next of kin. ers. Do not take the history again except for details that
• If this fails, you may disclose relevant medical information, in will help in your discussion with the patient / subject. You
confidence, to the DVLA’s medical adviser, informing the patient are not required to examine the patient / subject.
that you intend to do this.
Patient / subject information
You are a carer of Mr Peter Price, a 34-year-old man with
clearly too compromised for driving, as in the case of a muscular dystrophy. He has just been admitted to hospital
hemianopia or quadrantanopia. with lobar pneumonia. His condition is unstable. You
have cared for Mr Price for 4 years in his own home, but
despite physical disabilities, notably his being unable to
Dementia move his limbs, his general health has been very good.
The rules in dementia are outlined in Case 2.44 (Mild You are very anxious that he is now so unwell.
cognitive impairment and dementia). Where a patient is
deemed unsafe to drive by a doctor and may not have How to approach the case
insight, memory or capacity to inform the DVLA, nor 
next of kin to do so, a doctor may inform the DVLA Communication skills (conduct of interview,
immediately. exploration and problem negotiation) and
ethics and law
Other
1.  Introduction
People with a variety of other medical conditions (e.g.
Introduce yourself and establish the carer’s identity.
diabetes on insulin, Parkinson’s disease) must inform the
DVLA of their condition. I’m sure you will understand that out of respect for (patient’s
name), I need to be sure to whom I’m speaking.
When should you disclose confidential
information to the DVLA? 2.  Acknowledge a valued contribution
Your primary responsibility is to the patient, whose
This is outlined in Box 4.14. medical details remain confidential. It is very easy to forget
this when a patient is unable to speak. It is equally impor-
tant, however, to respect the concerns of a third party and
the valued information they can often provide. Show
CASE 4.22  CONFIDENTIALITY WHEN respect for the carer’s expertise (and experience with this
TALKING WITH RELATIVES AND particular patient), and acknowledge the interdependence
between patient and carer. The case records may be a valu-
OTHER THIRD PARTIES
able source of information and even demonstrate the
patient’s willingness for information to be shared with the
Candidate information carer.

Role 3.  Explain a little


You are a doctor on the medical ward. State, in broad terms, that the situation is not good. Listen
Please read this summary. to responses and look for cues. It is remarkable how

658
Case 4.22  Confidentiality when talking with relatives and other third parties

worried relatives and carers end up doing much of the informed that their information may be used for such
talking and by disclosing very little you can often open  purposes and have not objected.
the door to a lot of information from them. Frequently
the relative or carer knows, or has suspected, a lot more Under what circumstances must express
than you may think. consent be sought to disclose information
to third parties?
4.  Ask a little
Additional questions may open the door much wider! Express consent (Case 4.23), verbal or written, is needed
for disclosure of information to a third party under most
other circumstances, including to a nearest relative.
5.  Explain in more depth, directed by responses
You may then be in a position to speak more freely about What type of consent is needed when
the condition, again respecting your first duty to the
patient and making sure anything you say is strictly infor-
disclosing information to third parties such
mation that will be of use to the carer in his or her role, as employers, police, lawyers and insurance
and ultimately beneficial to the patient. companies?
Verbal consent from patients is usually sufficient when
6.  Involve the carer in the desired   giving information to relatives, but written consent should
management plan be obtained when communicating with employers and in
Involve the carer as far as you can in making decisions, any legal matters. Patients must be aware of the purpose
without compromising the autonomy of the patient. of disclosure, the obligation a doctor has to the third 
Asking ‘What do you feel he would want us to do?’ is not party and that this may include disclosure of personal
asking the carer to make a decision but inviting his or her information.
valued insights to help guide your own decision making.
Can information about a patient be
7.  Respond to cues disclosed for education and research
Continually look for non-verbal as well as verbal cues. without consent?
Information about patients can be useful for such pur-
8.  Consider other issues poses as education, research, audit, administration, moni-
It may or may not be appropriate, depending upon how toring, epidemiology and public-health surveillance.
well you feel the carer knows him, to establish whether Express consent is generally preferable for disclosure,
the carer is aware of any prior wishes regarding resuscita- whether or not it is judged that patients can be identified
tion in the event of a cardiopulmonary arrest. from the disclosure and even though disclosure is 
unlikely to confer personal consequences. Anonymised
9.  Allow time to think about what has been data without patient consent may be acceptable for educa-
discussed and invite questions tion and audit, and in research where it is not practicable
Allow the carer time to think about what has been dis- to contact patients a research ethics committee should
cussed and to ask questions. decide whether the likely benefits of research outweigh the
loss of confidentiality. Express consent should be sought
when publishing material such as case histories or photo-
10.  Agree a way forward
graphs of a patient, whether or not that patient is clearly
Assure him or her that you will do all that you can to get identifiable.
the patient better, and that now you have established the
carer’s involvement you would be happy to update him or
An 80-year-old man has just recovered
her at any time on the patient’s progress.
from a stroke. He lives independently
and is to be discharged from your ward
Discussion later this week. One of the nurses on your
Under which circumstances may implied ward asks if you will speak to his daughter.
consent be sufficient for disclosure of You are asked by a woman over the phone,
confidential information to third parties? ‘What has happened to my father?’ Would
Implied consent (Case 4.23) is appropriate for sharing
you tell her?
information in the health-care team, provided that wishes You would establish the caller’s identity. Clearly, if you
are respected if a patient objects to a particular person or have met her on the ward, recognise her voice, and know
agency being made aware and this would not put others that her father has always been happy to share his medical
at risk, or when disclosing information for clinical audit, problems with her, you would be less guarded. If you had
including identifiable information if patients have been no prior knowledge that he consents to divulging of

659
Station |4| Communication skills and ethics

information, you should explain why you could not Your examiners will warn you when 12 minutes have
without his permission. elapsed. You have 14 minutes to communicate with the
patient / subject followed by 1 minute of reflection. There
The daughter now requests that will then follow 5 minutes of discussion with the examin-
ers. Do not take the history again except for details that
you not tell him anything more
will help in your discussion with the patient / subject. You
about his condition without her are not required to examine the patient / subject.
prior permission – ‘he is a worrier’. Would
you agree to this? Patient / subject information
You should not be willing to withhold information from Ms Kate Morland, a 23-year-old nurse, was admitted to the
him based solely on his daughter’s wishes. You should emergency medical admissions unit last night with the
explain that he has a right to know what has happened worst headache she has ever experienced. She is a normally
(respecting his autonomy) and to make his own decisions fit and well young woman, who works at this hospital. She
based on that information. You could reassure her that sustained a minor whiplash injury 3 days ago in a minor
you would not force information on him, explaining only road traffic accident. She is very scared. She has never felt
as much as he appeared to want to know. You might also this unwell. She is afraid that colleagues have witnessed her
suggest that discussion by phone is not ideal and offer to tears and distress. She has been told that a lumbar puncture
speak to her if she comes in to hospital to visit her father. is necessary but is afraid of further investigations involving
You should avoid stating legal rights in these sorts of situ- needles and does not understand why she needs to stay in
ations. Relatives never warm to being told ‘I’m within my hospital because her brain scan was normal.
rights not to tell you’. Phrases like ‘I share your concerns’,
‘I understand how concerned you must be’, or ‘I appreciate
How to approach the case
how you must feel’ offer empathy and show that you are
listening. Communication skills (conduct of interview,
exploration and problem negotiation) and
What are Caldicott Guardians? ethics and law
Caldicott Guardians are senior people in NHS, local
authority social care, and partner organisations, responsi-
1.  Introduction
ble for protecting the confidentiality of patient informa- She is very apprehensive, probably a cumulative effect of
tion and enabling appropriate information sharing. recent personal stresses, being admitted to her own work-
place and fear of needles. Patients’ fears are multiplied
significantly if they feel, as they often do, uncertain about
what is happening to them, what is planned and why.
CASE 4.23  CONSENT FOR Establish an aura of calm. Explain that, before you do
anything else, you would like to recap on the reasons for
INVESTIGATION OR TREATMENT the scan, the desirability of doing one further test – the
lumbar puncture – and to talk this through with her so
Candidate information that she can decide whether or not to go ahead. Make it
very clear that in the last analysis it is her decision.
Role
2.  Explain the situation so far
You are a doctor on the acute medical unit.
Please read this summary. Reassure her that the test results so far are good and that
nothing bad, such as a brain tumour, has been seen.
Scenario Explain the main unresolved concern.

The main concern in this situation is the possibility of a small


bleed, known as a subarachnoid haemorrhage. This is usually
detected by the scan, but a small number of cases, perhaps
Re: Ms Kate Morland, aged 23 years
around 10%, may not be picked up. Although it is generally
Ms Morland was admitted last night with a severe occipital
small bleeds that are not picked up, the concern is that
headache of sudden onset. She is a normally fit and well young
subarachnoid bleeds arise from a weakness in the wall of a
woman, who works at your hospital as a nurse. She sustained a
blood vessel, and if not detected can recur with much more
minor whiplash injury 3 days previously in a minor road traffic
serious consequences. If detected, there are possible
accident. Her headache came on suddenly at a party. It was the
treatments to correct the underlying cause.
worst headache she has ever experienced and reduced her to tears.
Her computed tomography brain scan is normal.
Your tasks are to explain the reasons for performing a lumbar 3.  Explain the best way forward
puncture (to exclude subarachnoid haemorrhage) and obtain her The only way to detect a smaller bleed is with a test called a
consent. lumbar puncture, these days a relatively straightforward test
without too much discomfort.

660
Case 4.23  Consent for investigation or treatment

4.  Establish previous experience and be alert to What are the necessary requirements for
ideas and concerns valid consent?
Before explaining the test in more detail, establish any Consent rests on the principle of respect for patient auton-
specific ideas or concerns. Many patients have heard exag- omy. For it to be legally valid three conditions must be
gerated stories about lumbar punctures and big needles. met (Box 4.15).
Alternatively, she might have personal knowledge of
someone close to her who had a difficult lumbar puncture What is battery?
for a serious disease.
A procedure performed without consent may be grounds
5.  Explain the nature of the investigation   for battery. Generally, if a person touches another without
consent, this constitutes battery, for which damages may
or treatment be awarded. Unlike negligence (Case 4.40), proof of harm
Explain that a lumbar puncture involves, with local anaes- is not necessary.
thetic, a small needle inserted in to the lower part of the
back to draw off spinal fluid. In the case of a subarachnoid What is meant by implied
bleed, this should be blood stained. She will need to lie and express consent?
curled up on her side during the procedure and remain in
bed for around 4 hours afterwards. Types of consent are discussed in Box 4.16.

6.  Explain the risks and benefits of the


investigation or treatment Box 4.15  Valid consent
Complications include headache (common) and much
more rarely serious complications such as infection. Requirements for valid consent
1. The patient should be properly informed.
7.  Explain any possible alternatives 2. The patient should understand that information and be
competent (legally, have capacity) to give consent.
In this case the alternative is to do nothing, because sub-
3. Consent must be given voluntarily, without coercion.
arachnoid haemorrhage may be unlikely. Less optimal
A written record of the conversation is necessary.
tests in patients at higher suspicion of subarachnoid
haemorrhage who abjectly decline lumbar puncture might ‘Informed consent’
include magnetic resonance cerebral angiography, A patient needs a certain amount of information for consent to be
although this might only detect larger aneurysms. legally valid. He or she must be aware of:
• The nature of the intervention; avoidance of battery requires
8.  Show respect for autonomy that the doctor explains, in broad terms, the nature of the
In respecting her right to decide, you must be sure that  procedure
she has been informed and understands that the test  • The risks and benefits, and alternative options or treatments;
this relates to the avoidance of negligence. In general, a doctor
is ordinarily safe and straightforward and that the 
is not negligent if he or she has acted in accordance with the
possible diagnosis, albeit unlikely, is potentially  practice accepted as proper by a responsible body of doctors
life threatening. Without telling a patient what must  skilled in that art (i.e. what a responsible body of medical
be done, it is crucial to explain that in your opinion a opinion would disclose in the same or similar circumstances).
particular course of action seems to be in that patient’s This is termed the Bolam test after the relevant legal case.
best interests. However, the standard of information provision by doctors is
evolving in English law, mainly as a result of General Medical
9.  Confirm understanding and invite questions Council guidelines, and is drawing nearer to the prudent-
patient test widely used in North America. According to this
Allow her time to assimilate all of this information, test, doctors should provide the amount of information that a
confirm that she understands what you have explained ‘prudent patient’ would want.
and invite any questions.
What most patients would want to know
10.  Seek permission to proceed For an investigation or treatment most patients would want to
know:
Use an open question:
• The possible or likely diagnosis, and its prognosis with or
How does this sound? without treatment
• The purpose of the proposed investigation or treatment
(furthering diagnosis, excluding serious diagnoses, improving
Discussion prognosis)
• Any common or serious complications or side effects of the
What is consent? proposed investigation or treatment, and of the possibility of
Agreement to an action based on knowledge of what that proposed treatment failing to produce the desired aim.
action involves and its likely consequences.

661
Station |4| Communication skills and ethics

Box 4.16  Types of consent


• The purpose of any proposed investigation or
treatment and what it will involve
Implied consent • The potential benefits, risks and burdens, and the
likelihood of success, for each option; this should
Because touching a patient without consent can constitute battery,
consent is needed even when, for example, taking a pulse or include information, if available, about whether the
examining the chest. Yet doctors seldom obtain consent, either benefits or risks are affected by which organisation
expressed orally or in writing, for such routine tasks, and in law or doctor is chosen to provide care
patients could not successfully sue doctors most of the time • Whether a proposed investigation or treatment is
because the Courts accept the concept of implied consent. If a part of a research programme or is an innovative
patient offers their wrist while the doctor takes the pulse, consent treatment designed specifically for their benefit
is implied by their behaviour. However, the fact that a patient is in • The people who will be mainly responsible for and
hospital for examination does not constitute consent to involved in their care, what their roles are, and to
examination, investigation or treatment.
what extent students may be involved
Express consent • Their right to refuse to take part in teaching or
Express consent, explicitly given (usually verbal but sometimes research
written), for investigation or treatment is preferable when: • Their right to seek a second opinion
• The investigation or treatment is complex or involves significant • Any bills they will have to pay
risks or side effects • Any conflicts of interest that the doctor or
• Clinical care is not the primary purpose of the investigation organisation may have
• The investigation or treatment forms part of research • Any treatments believed to have greater potential
• There may be significant personal consequences for the patient. benefit for the patient than those the doctor or
Express consent may be oral or written, the latter a legal organisation can offer.
requirement in some circumstances, e.g. organ donation, and These matters should be explored with patients, with
desirable where interventions are complex, there may be significant concerns listened to, views sought and respected, and that
consequences on employment, social or personal life, clinical care is
information is understood checked. Questions should be
not the primary purpose of the intervention, or in research.
answered honestly and, as far as practical, as fully as
wished.

What information should be shared with a How should information be shared?


patient in order to obtain consent? The way information is communicated is as important as
The exchange of information between doctor and patient the information itself. It should be easy to understand and
is central to good decision making. How much informa- given with due consideration.
tion is shared with patients will vary, depending on their
individual circumstances. The approach to discussion
should be tailored according to: What would you do if a patient asks for
• Their needs, wishes and priorities decisions to be made on their behalf?
• Their level of knowledge about, and understanding If a patient asks you to make decisions on their behalf or
of, their condition, prognosis and the treatment wants to leave decisions to a relative, partner or friend, you
options should explain that it is important that they understand
• The nature of their condition the options open to them, and what the treatment will
• The complexity of the treatment involve. If they do not want this information, you should
• The nature and level of risk associated with the try to find out why. If the patient still does not want to
investigation or treatment. know in detail about their condition or the treatment, you
Assumptions should not be made about: should respect their wishes as far as possible. But you must
explain the importance of providing at least the basic
• The information a patient might want or need information they need in order to give valid consent to a
• The clinical or other factors a patient might consider proposed investigation or treatment. If the patient insists
significant
that they do not want even this basic information, you
• A patient’s level of knowledge or understanding of must explain the potential consequences of carrying out
what is proposed.
an investigation or treatment if their consent may be open
Patients must be given the information they want or to subsequent legal challenge. You must record the fact
need about: that the patient has declined relevant information and
• The diagnosis and prognosis who they asked to make the decision about treatment. You
• Any uncertainties about the diagnosis or prognosis, must also make it clear that they can change their mind
including options for further investigations and have more information at any time. Apart from cir-
• Options for treating or managing the condition, cumstances in which a patient refuses information, you
including the option not to treat should not withhold information necessary for making

662
Case 4.24  Consent and capacity

decisions, unless you believe that giving it would cause the Scenario
patient serious harm.
Re: Mr Ashley Wills, aged 77 years
Can a patient demand a treatment? Mr Wills has been on your ward for 4 weeks following a left
A patient with capacity may refuse a particular treatment total anterior circulation infarct with dense right-sided hemiparesis,
but cannot demand a particular treatment that the doctor expressive dysphasia and a degree of receptive dysphasia. Over the
does not consider to be in that patient’s best interests. past 4 weeks recovery has been minimal and his swallow, as
assessed by the speech and language therapists, remains unsafe. At
least one confirmed episode of aspiration has occurred. He has
What exceptions are there to informed pulled out his nasogastric tube on numerous occasions and feeding
consent? has been erratic because it is difficult to replace. His serum albumin
(although not reliably related to nutritional status) has fallen to
Exceptions are where a patient is judged not to have the 26 g / l. Your team feel that a percutaneous endoscopic gastrostomy
capacity to give informed consent. (PEG) tube is in his best interests to support adequate nutrition
and optimise potential recovery. It is increasingly likely that he will
How might doctors be at risk of committing require long-term nursing care. You cannot be certain that Mr Wills
battery when obtaining consent from a understands or retains information.
Your tasks are to discuss with Mrs Wills, your patient’s wife,
competent patient? why a PEG tube is being considered and obtain her consent.
By failing to give sufficient information about the nature
of the procedure.

How might doctors be at risk of negligence Your examiners will warn you when 12 minutes have
elapsed. You have 14 minutes to communicate with the
when obtaining consent from a competent
patient / subject followed by 1 minute of reflection. There
patient? will then follow 5 minutes of discussion with the examin-
By failing to give sufficient information about common ers. Do not take the history again except for details that
and rare serious side effects, benefits and possible will help in your discussion with the patient / subject. You
alternatives. are not required to examine the patient / subject.

A 65-year-old man has diabetic nephropathy Patient / subject information


as evidenced by microalbuminuria. You wish Your husband, Mr Ashley Wills, is a 77-year-old man who
to start an angiotensin-converting enzyme has been on the stroke unit for 4 weeks following a large
(ACE) inhibitor. What type of consent would stroke with dense right-sided weakness and loss of speech.
you obtain? His comprehension of anything he is told is variable, and
you cannot always be sure of what he understands. Over
Rights are so basic yet so easily ignored. This is especially the past 4 weeks recovery has been minimal and his
true of the right to consent to treatment or refuse it. In swallow remains unsafe. He is sometimes agitated, pulling
theory, consent should always be obtained for any treat- out his nasogastric feeding tube on numerous occasions
ment. In practice, there is often an implicit understanding and his feeding has been erratic. You can see that his
that consent for examination, investigation or treatment nutritional state is not optimal and the doctors have told
is implied. Although such assumptions seldom result in you that this is confirmed on blood tests. The doctors feel
problems, legal justification for them is not inevitable. You that a percutaneous endoscopic gastrostomy (PEG), a
should obtain express consent, explaining why you feel feeding tube passed directly into the stomach, would
the ACE inhibitor would be beneficial (knowledge of support adequate nutrition and optimise potential recov-
disease and its treatment), important side effects (knowl- ery, but you have also been told that he is unlikely to get
edge of drug), any alternatives, and the risks or benefits home again. Your husband has always feared ‘ending up
and likely outcomes of these. in a nursing home’ and you are not sure that he would
want further intervention.

CASE 4.24  CONSENT AND How to approach the case


CAPACITY Communication skills (conduct of interview,
exploration and problem negotiation) and
Candidate information ethics and law
Role 1.  Introduce yourself and establish the  
You are a doctor on the stroke unit. relative’s identity
Please read this summary. Introduce yourself and confirm that she is Mr Wills’ wife.

663
Station |4| Communication skills and ethics

2.  Establish background knowledge a PEG tube unless decline seems inevitable even with
nutrition, in which case doctors would not consider it.
Find out what she understands about her husband’s con-
dition and the problems faced (rehabilitation challenges,
nutrition compromised) to get an idea of where to start 8.  Relative agrees or relative disagrees with
your explanation. what you see as in patient’s best interests
Most relatives agree to PEG feeding if it is clinically indi-
3.  Explain the problem cated. In those uncommon circumstances where there is a
Explain that the stroke has led to swallowing problems, major differing view between relatives and medical staff
preventing adequate nutrition and putting him at risk  you should consider two possibilities. If you can be con-
of further episodes of aspiration. Explain that he has vinced that information provided by his wife confirms his
pulled out the nasogastric tube on numerous occasions advance refusal of consent to a feeding tube, this should
and that it has proved challenging to replace and that  be respected. If you cannot be convinced of this, and feel
in any event this is not an ideal long-term solution to that these are exclusively his wife’s wishes then you should
meeting his nutritional needs. Explain that with time  resolve to seek further advice from senior colleagues and,
and without adequate nutrition her husband will inevita- ultimately, you may need to contemplate a course of
bly decline. action that is at odds with a relative’s wishes but that is
seen as in that patient’s best interests by the medical team;
such a course of action should generally be followed
4.  Explain possible solutions gently, allowing relatives time to question the issue them-
Explain that one option is a PEG tube, and explain that selves and see things from your perspective.
this involves insertion of a small feeding tube directly into
the stomach through the abdominal wall. This can be 9.  Confirm understanding and invite questions
performed in various ways but the most common method
Be clear that she understands the issues raised, and invite
is via endoscopic guidance, under local anaesthetic and
questions.
with a degree of sedation. Explain the potentially serious
complications (infection, perforation, death). PEG tubes
carry a modest but significant mortality as well as signifi-
10.  Agree a way forward
cant morbidity. A likely way forward will be to request PEG insertion, with
the understanding that this decision might be revised if he
5.  Be alert to cues were to deteriorate in the next few days but proceed if he
remains stable.
You may perceive how she feels about your proposed strat-
egies from non-verbal cues.
Discussion
6.  Ask how the relative feels, and how she What are the key aspects, based on case
thinks the patient would feel about this law, of capacity to give or withhold
Sometimes patients’ relatives are reluctant to give a view consent?
because they feel it is too much responsibility for a loved
one to take. More often, relatives have concerns. They For a patient to give valid consent to a procedure or treat-
simply want what is best for their loved one but find it ment, she or he must be competent or, in legal terms, have
hard to make decisions because there seem to be risks with the capacity to do so. The key aspects of capacity are listed
every choice. Reassure her that you will always try to do in Box 4.17.
what is in the patient’s best interests, but that it is impor-
tant to know if to her knowledge her husband might have
had any strong wishes one way or the other. Box 4.17  The key aspects of capacity (the ‘test’
of capacity)
7.  Attempt to address concerns
Each of the following must apply for a person to have capacity:
Relatives struggle, understandably, to come to terms with 1. The ability to understand information relevant to the decision
major changes such as the likelihood of nursing home (proposed interventions such as investigations or treatments,
dependency. However, in coming to terms with this there alternative options, the intervention’s possible benefits and
are many smaller hurdles or decisions to be negotiated risks, and the consequences of non-treatment); the person
along the way. Apart from the general wishes of most should also of course believe that information to be true
people not to be ill or to face nursing care or mental inca- 2. The ability to retain that information (for how long depends
pacity, most will not have thought in detail and discussed upon what the information is)
specific wishes like PEGs and even with valid advance 3. The ability to weigh up the information to reach a decision
directives the devil is often in the detail. When faced with 4. The ability to communicate a decision.
a choice between nutrition to optimise whatever recovery
is possible and certain decline, most relatives will agree to

664
Case 4.24  Consent and capacity

Incapacity must be proven. All adults are assumed by Does the Mental Capacity Act aid
law to have capacity unless proven otherwise. Further, a determination of best interests?
person may not be globally competent or incompetent. It
is ‘function or decision specific’. For example, a patient It provides a best interests checklist (Box 4.20).
may be capable of declining surgery but incapable of ‘Best interests’ can be particularly emotive in end of life
judging their safety at home. Capacity may also fluctuate care. Case law suggests that the ‘intolerability’ of treatment
over time. Even where capacity may be limited, a doctor is not the sole test of whether treatment is in a patient’s
has a duty to give an account in simple terms to a patient best interests. The term ‘best interests’ encompasses
of what is being proposed. According to the Mental  medical, emotional and all other factors relevant to the
Capacity Act (below) a person lacks capacity if because of patient’s welfare. A patient’s best interests may be inter-
‘an impairment of, or a disturbance in the functioning of, preted as meaning that a patient should not be subjected
the brain and mind’ (which can be permanent or tempo- to more treatment than is necessary to allow them to die
rary) he or she is unable to fulfil any of the four criteria peacefully and with dignity.
in Box 4.17.
Can incapacity be inferred from a particular
Can any doctor (or health-care professional) medical illness or diagnosis?
judge a patient’s capacity? No. Alzheimer’s disease or any other dementia, for
It is the personal responsibility of any doctor or health- example, does not necessarily imply incapacity.
care professional proposing an investigation or treatment
to establish whether a patient has the capacity to give valid What do you know about legal
consent. Where incapacity is being judged it is wise to seek representation for incapacitated adults?
agreement from colleagues and other members of the
team involved in the patient’s care, and where there is any Social services can provide agency or appointeeship for han-
doubt to seek the opinion of a psychiatrist. dling the financial affairs of an incapacitated adult.
Doctors may sign letters confirming mental incapacity;
What do you know of the legal necessity to psychiatrists may become involved where mental state
treat incapacitated patients? assessment is challenging.

The concept of necessity is that not only is a doctor able Power of attorney (PA)
to give treatment to an incapacitated patient when it is
clearly in that patient’s best interests, but it is also common PA gives someone the legal right to act on behalf of
law to do so. This still only applies to treatment aimed to another. PA is only applicable for people who can under-
improve or prevent deterioration in health. If the patient stand and not for people with incapacity (compare with
is known to have prior objections to all or some parts  proxy decision making in Scotland). A solicitor, for
of a treatment, doctors are not justified in proceeding, example, may act as PA to sell your house while you are
even in emergency situations. If incapacity is temporary on holiday. PA formerly lapsed if a person became inca-
(e.g. anaesthesia, intoxication, unconsciousness), doctors pacitated but in 1985 a special type of PA known as endur-
should not proceed beyond what is essential to preserve ing power of attorney (EPA) changed this. PA can be revoked.
life or prevent deterioration in health. Sometimes PA representatives can try to take PA too far.
PA does not give the right to make medical decisions,
How may doctors be guided when making which remain up to the patient, or, if the patient is inca-
pacitated, the doctor acting in the patient’s best interests,
decisions for a patient without capacity?
except in those circumstances where lasting power of attor-
Doctors, not relatives, are responsible for making medical ney (LPA), which replaced EPA, has been invoked allowing
decisions about incapacitated patients. Any procedures for proxy decision making under the Mental Capacity Act.
requiring a consent form should be consented to by
doctors, not a next of kin. Guidance when making deci- Court of Protection / Court Appointed Deputies
sions for patients without capacity is given in Box 4.18.
Court of Protection (CoP) (now a system of Court
What are the key principles and innovations Appointed Deputies) applies to circumstances in which
EPA (now LPA) has not been given and a patient has
of the English Mental Capacity Act 2005? become incapacitated. It is concerned mostly with prop-
The Mental Capacity Act provides a statutory framework erty, and requires medical evidence. CoP applications can
to empower and protect vulnerable people who are not be made by a solicitor (usual) or the state (Director of
able to make their own decisions. It makes it clear who Social Services). Solicitors then request details as to why a
can take decisions, in which situations, and how they doctor feels that a patient is incapacitated. Many doctors
should go about this. It is underpinned by five key princi- (e.g. care of the elderly physicians) complete the forms
ples (Box 4.19). where incapacity is clear (brief, layperson’s language is
Key innovations of the Mental Capacity Act are outlined acceptable, e.g. poor memory, deluded); psychiatrists are
in Table 4.2. often involved where incapacity is not so clear. The Court

665
Station |4| Communication skills and ethics

Box 4.18  Guidance when making decisions for a patient without capacity

Best interests England and Wales neither welfare attorneys nor deputies can
Doctors may act in what they see as a patient’s best interests. The demand treatment that is clinically inappropriate.
doctrine of necessity underpinning the treatment of people who
lack capacity requires that there must be a necessity to act and that
Those close to a patient
such action must be in the best interests of the person concerned. Where a patient has not made an advance decision or has a legal
This is a grey area; traditionally it has been felt that best interests proxy, treatment decisions rest with the most senior clinician in
will be served when a doctor acts in accordance with accepted charge of the patient’s care. Decisions must be made on the basis
medical opinion (Bolam test). When more than one option seems of the patient’s best interests. In order to assess best interests, the
reasonably in a patient’s best interests (including non-treatment) views of those close to the patient should be sought, unless this is
that which least restricts that patient’s future choices should be impossible, to determine any previously expressed wishes and what
chosen. However, the Mental Capacity Act now more clearly the patient would be likely to consider of benefit. In reaching a
stipulates how best interests should be determined. decision, in England and Wales**, the Mental Capacity Act requires
that best-interests decisions must include seeking the views of
Advance decisions anyone named by the patient as someone to be consulted, anyone
Doctors may be guided by any expressed wishes or advance engaged in caring for the person or anyone interested in the
decisions made at a time when the patient had capacity. Losing patient’s welfare. Under the Act, all health-care personnel must act
capacity, such as losing consciousness, does not negate previous in the best interests of a patient who lacks capacity. In these
wishes and a valid and applicable advance directive (Case 4.28) circumstances, it should be made clear to those close to the patient
overrides best interests if these are discrepant. that their role is not to take decisions on behalf of the patient, but
to help the health-care team to make an appropriate decision in the
Legal proxy patient’s best interests.
If a patient has a legal proxy – a welfare attorney or guardian
– this person must be consulted about cardiopulmonary Independent mental capacity advocate
resuscitation decisions. In England and Wales* the Mental Capacity In England and Wales the Mental Capacity Act requires an
Act allows people over 18 years of age who have capacity to make independent mental capacity advocate (IMCA) to be consulted
a lasting power of attorney (LPA), appointing a welfare attorney to about all decisions about ‘serious medical treatment’ where patients
make health and personal welfare decisions on their behalf once lack capacity and have nobody to speak on their behalf. The
such capacity is lost. Before relying on the authority of this person, definition of serious medical treatment includes circumstances
the health-care team must be satisfied that the patient lacks where ‘what is proposed would be likely to involve serious
capacity to make the decision, a statement has been included in consequence for the patient’. An IMCA does not have the power to
the LPA specifically authorising the welfare attorney to make make decisions but must be consulted as part of the determination
decisions relating to life-prolonging treatment, the LPA has been of the patient’s best interests. The rights of an incapacitated patient
registered with the Office of the Public Guardian and the decision to such principles as non-discrimination, confidentiality, liberty and
being made by the attorney is in the patent’s best interests. In dignity are of course identical to those of a patient with capacity.

*In Scotland the Adults with Incapacity (Scotland) Act 2000 allows people over 16 years of age, who have capacity, to appoint a welfare
attorney to make decisions about medical treatment once that capacity is lost. The Sheriff may, on application, appoint a welfare guardian
with similar powers. Before relying on the authority of a welfare attorney or guardian, the health-care team must be satisfied that the
patient lacks capacity to make the decision (the terms of a power of attorney may state how incapacity is to be determined), the welfare
attorney or guardian has the specific power to consent to treatment (a register of valid proxy decision makers is held by the Public
Guardian), the decision being made by the attorney would benefit the patient and the attorney has taken account of the patient’s past and
present wishes as far as they can be ascertained. In Scotland, welfare attorneys and guardians cannot demand treatment that is clinically
inappropriate. In Northern Ireland there is currently no provision for anybody to make decisions on behalf of patients who lack capacity,
although those close to patients should be consulted where a best-interests decision is being made by the clinician in charge of the patient’s
care.
**In Scotland the Adults with Incapacity (Scotland) Act requires doctors to take account, so far as is reasonable and practicable, of the views
of patients’ nearest relatives and carers. In Northern Ireland, where there is no statutory provision for decision making for patients who lack
capacity, it is nonetheless good practice to discuss decision making with those close to the patient in order to determine what would be in
the best interests of the patient.

appoints a receiver (e.g. relative, accountant) who acts for putting on the gas, or outside on to the road. If she lacks
the incapacitated person. CoP is not necessarily perma- insight, she may ask to go home and simply reject all of
nent because it depends more on behaviour than your concerns. Your responsibility is for her safety. It may
prognosis. be impossible to put in care at home at night and if she
It is common to look after patients in hospital who are has progressive dementia without acute psychosis or
vulnerable and lack insight and capacity, for example the depression transferring her to psychiatric care is only shift-
elderly woman who lives alone in her own house, has no ing the problem. She will probably need gentle ushering
close relatives and who wanders at night around the house to a community hospital bed unless direct care-home

666
Case 4.24  Consent and capacity

Box 4.19  The five key principles of the mental Box 4.20  Determining best interests
capacity act
The person making a decision must:
1. People are assumed to have capacity unless proven • Consider when / if the person is likely to regain capacity
otherwise. • Encourage patient participation (e.g. repeatedly pulling out a
2. Before deciding that someone lacks capacity, all steps nasogastric tube may be indicative of wishes)
should be taken to enhance his or her decision-making • Consider past and present feelings, previous beliefs and values
abilities. and other relevant factors
3. Someone cannot be said to lack capacity simply because he or • Consult others (account should be taken, where practicable and
she is making what might be seen as an eccentric or unwise appropriate, of the views of at least one of: anyone named by
decision. the person to be consulted on matters of the kind; anyone
4. A person’s best interests should always be taken into account engaged in caring for the person or interested in their welfare;
in making a decision on his or her behalf. any donee with lasting power of attorney (LPA) granted by the
5. The least restrictive option (of basic rights and freedoms) person; any Court-appointed deputy). For people who lack
should always be used. capacity and who lack a spokesperson (no relative, friend, carer,
LPA or deputee) an independent consultee arrangement is
possible.

Table 4.2  Key innovations of the Mental Capacity Act

Key innovation What it means


Definition of incapacity A clear test of incapacity (see Box 4.17)
Designated ‘proxy’ decision makers Lasting power of A person who still has capacity may donate an LPA to one or more people.
may act on behalf of someone who attorney (LPA) This is similar to enduring power of attorney (EPA) which preceded it, but
lacks capacity under two allows for the donor’s (that person giving power of attorney) care, including
mechanisms medical care, to be decided by the donee (that person given power of
attorney) although an LPA cannot refuse life-sustaining treatment unless
expressively given this power and is bound by advance directives (whilst
these forms of proxy may become increasingly common, it is likely that most
patients will continue to have no such provision)
Court-appointed The Court can appoint a deputy or deputies who can make welfare, health
deputies and financial decisions on behalf of a person who lacks capacity but are not
able to refuse consent to life-sustaining treatment. They can only be
appointed if the Court cannot make a one-off decision to resolve issues
Development of two public bodies A new Court of Has jurisdiction relating to the whole Act and final arbiter for capacity
to support the statutory framework Protection matters
designed around the needs of
those who lack capacity A new Public Guardian Registering authority for LPAs and deputies

Greater emphasis on decisions Applies both to doctors and to any proxy decision makers
being made in the patient’s best
interests
Three further provisions to protect Independent mental Someone appointed to support a person who lacks capacity but has no one
vulnerable people capacity advocate to speak for them
Makes representations about feelings, beliefs and values and brings to
attention of decision maker relevant facts
Can challenge decision maker
Advance decisions to Statutory support of advance directives and decision making (does not apply
refuse treatment to any treatment a doctor considers necessary to sustain life unless strict
formalities have been complied with that include a decision in writing that is
signed and witnessed and that includes an express statement that the
decision stands ‘even if life is at risk’)
Criminal offence For ill treatment or neglect of a person who lacks capacity
Clear parameters for research Research may be lawful if approved by relevant bodies (e.g. ethics committees) and cannot be performed
as effectively on those with capacity
Carers or nominated third parties must be consulted and agree that patient would approve

667
Station |4| Communication skills and ethics

placement is available, with her general practitioner on


board. Although not urgent, ultimately her house may CASE 4.25  REFUSAL OF CONSENT
need to be sold and other affairs attended to. Therefore,
concurrently, assuming she has no LPA in a distant relative
or in her solicitor, social services should be contacted to Candidate information
start CoP proceedings. Role
You are a doctor on the medical ward.
Does the Mental Capacity Act allow Please read this summary.
detention of patients?
It is only permitted if it is reasonably believed to prevent Scenario
harm and measures must be proportionate to the likeli-
hood of serious harm. ‘Deprivation of liberty’ paperwork Re: Ms Eleanor Dashwood, aged 75 years
needs to be completed as part of the justification for  Ms Dashwood was admitted to hospital recently with
the decision, which may be a holding measure if specific pneumonia. During the course of investigations she was found to
sectioning under the Mental Health Act becomes have iron deficiency anaemia and subsequently a hard rectal mass
appropriate. was discovered, most probably a carcinoma. She is asymptomatic.
She is otherwise well and has no past medical history of note. She
has now made a good recovery from her pneumonia and is keen to
What do you know about safeguarding go home. She refuses any further investigations or treatments for
vulnerable adults in hospital? the likely tumour.
Your task is to establish that this is her informed decision.
Vulnerable adults include those who lack mental capacity,
those with cognitive impairment, those disabled mentally
or physically, those with sensory impairment, those in
institutions, those with no fixed abode and those socially Your examiners will warn you when 12 minutes have
isolated. elapsed. You have 14 minutes to communicate with the
Vulnerable adult risk management involves case-specific patient / subject followed by 1 minute of reflection. There
management, often a multi-agency approach, and involv- will then follow 5 minutes of discussion with the examin-
ing the patient as far as possible in decision making; where ers. Do not take the history again except for details that
this is not possible and there are no relatives or carers, or will help in your discussion with the patient / subject. You
if there are concerns about the motives of relatives or are not required to examine the patient / subject.
carers, an independent mental capacity advocate (IMCA)
may aid decision making in best interests, making more Patient / subject information
robust the traditional doctor ‘best interests’ decisions.
Ms Eleanor Dashwood is a previously well 75-year-old,
IMCAs may be helpful if there are major medical decisions
independent woman admitted to hospital with pneumonia.
or if care placement is being planned, and they have the
She is recovering from this but during the course of her stay
right to use the CoP if they have concerns about decisions
she was found to be anaemic and subsequently a rectal lump
being proposed.
was discovered, most probably a cancer. She told the doctors
Sadly, vulnerable adults are often subject to abuse, most
she had no symptoms but in fact has had constipation and
commonly by those ‘caring’ for them, often in institutions,
episodes of fresh rectal bleeding over the last 2 months. She
sometimes in their own home; carers may not be aware of
has not told anyone because her husband developed similar
the abuse, and abuse is not always intended; sometimes it
symptoms a few months before his bowel cancer was diag-
has become ‘the norm’ over many years, and sometimes
nosed. His cancer had spread to the liver at diagnosis and
the patient’s needs outgrow well-intentioned carers or
was incurable. He died despite a complex array of investiga-
institutions.
tions and treatments. She does not wish to ‘go down the
Signs of abuse may be physical (e.g. bruises, pressure
same route of hospitals and tests’ and would prefer to let
areas), behavioural (e.g. fear, avoidance), loss of financial
nature take its course, even if that means rapidly deteriorat-
assets or neglect (e.g. malnutrition, weight loss, dehydra-
ing like her husband. She will refuse any further investiga-
tion, unkempt appearance).
tions or treatments for the likely tumour.
The clinician’s role is an index of suspicion, accurate
record keeping, arranging any appropriate medical pho-
tography, multidisciplinary team liaison and referral to the How to approach the case
hospital social services team (HSST). The HSST coordi-
nates all subsequent investigation and referral to the adult Communication skills (conduct of interview,
protection (AP) team or the AP police who may determine exploration and problem negotiation) and
whether or not an offence has been committed; the AP ethics and law
police take over investigation of any offence, and it is
important for clinicians to share information on a need- 1.  Introduction
to-know basis. Introduce yourself and make it clear you are here to help.

668
Case 4.25  Refusal of consent

2.  Explain the situation so far 6.  Explain the risks and benefits of  
As I think you know, you had a rather nasty pneumonia the investigation or treatment and the
when you came in to hospital that has responded very consequences of not pursuing matters
well to antibiotics. But I think you also know that during
the course of our tests we found that you were anaemic The tests would be likely to tell us one of two things. Firstly,
– that your blood count was low. We also discovered a lump, they might show that this problem is very well localised, and
in the tail end, and this is very likely to be the reason for the if that is the case then very possibly curable. Secondly if, and
anaemia, very possibly a lump that may not be nice … a form there is always the possibility, things were not as confined as
of growth. we hoped, then we might well be able to predict future
problems and take action to lessen these. One of the more
She will be likely to acknowledge all of this and may  immediate problems that can sometimes arise is a blockage in
either tell you she has thought about this and decided she the bowel, which can be difficult, and far more difficult to
treat if it happens without our prior knowledge. There is, of
does not want anything done or she may be a little more
course, a possibility that the lump is not as worrying as we
open to your thoughts and ask what you feel ought to be had thought, and that the anaemia is due to something very
done. simple. It would be nice to know this.

3.  Explain the best way forward 7.  Explain any possible alternatives
The best way forward for her can only be established in She may ask what would happen next if she chose not to
conjunction with her. pursue tests.
That is for us to decide together. First of all, we would need to The alternative is simply to let you go home, perhaps asking
consider some further tests to try to establish accurately your doctor to keep a close eye on things, and arranging for
what it is. you to see us again at any stage if you were to change your
mind. Or we could see you here in clinic in a few weeks to
4.  Establish previous experience and be alert to have another talk about things. I would just say, however, that
beliefs and concerns you are a healthy lady in all other respects, and that makes it
much more likely that you would find the tests, and perhaps
Her husband’s experience is clearly pivotal in her decision. treatment, more straightforward.
Acknowledge this.
Ask if she has any relatives or people close to her 
I understand. I do understand. with whom she might wish to discuss options, or with
It may be that she imagines painful tests. It may be that whom she might wish you to talk. A common problem,
she imagines only chemotherapy, about which she has however, is that relatives may take a different view and
heard so many dreadful things, but would consider an exert pressure on the physician to act. The primary emotion
operation. There are many ‘maybes’. The point is that, is almost always desire for the best management of the
before respecting her wishes (autonomy) for no further patient. The physician may explain (if the patient is happy
tests, you must try to understand precisely what her posi- for such a discussion) that the issue has been fully dis-
tion is and help to inform her. Her decision may already cussed in terms of likely benefits and risks and outcomes,
have been made. But you should try to discover the reasons and that as a result of this information an informed deci-
she made it and whether or not she has sufficient informa- sion has been made by the patient. Occasionally, relatives
tion at her disposal to make an informed decision. She may feel that the patient is unable to make the best 
may choose not to receive any more information, but decision (‘she is old and does not know what is best’) to
ideally informed consent to no treatment should be which the physician must explain that there is no evidence
obtained and based upon discussion of the acceptability she or he does not have the capacity to make informed
and effectiveness of the various options. decisions.

5.  Explain the nature of the investigation   8.  Show respect for autonomy
or treatment We will do our best, whatever you decide.
I understand fully how you must feel. And I’ll say immediately
that we will respect any decision you make. I should also tell
9.  Confirm understanding and  
you that at this stage we are by no means certain of the invite questions
diagnosis and that tests would help us to be more certain Allow her time to assimilate all of this information,
– and to more certainly know whether this is a very different confirm that she understands what you have explained
situation from that of your husband. It may well be.
and invite any questions.
Were we to investigate, I would recommend a scan of the
tummy, and then very likely a biopsy of the lump – by asking 10.  Keep the door open
one of our surgical colleagues to take a sample of tissue from
the lump via a flexible telescope with a light on the end. We If she still refuses ongoing investigation, make it very clear
would then be in a much better position to know the best that you are happy to see her again. In this situation, where
way forward. she will have a lot to assimilate, it might be sensible to

669
Station |4| Communication skills and ethics

arrange follow-up in clinic anyway, if she agrees. She might


be more likely to miss out on potential treatments unless Box 4.21  Refusal to give consent
you set up possible avenues towards these. • Any adult with capacity, because of their right of autonomy or
self-determination, may refuse investigation or treatment for
Discussion reasons that are rational, irrational or for no reason.
• Legally, such refusal of treatment supersedes the sanctity of
If a young woman with Graves’ life.
thyrotoxicosis wanted to pursue • To treat in these circumstances would constitute trespass or
homeopathic treatment, even when the battery.
• Existing consent may be withdrawn at any time and new
endocrinologist has recommended refusal overrides prior consent.
carbimazole, would you agree? An example of refusal of treatment with a potentially
You may not have the facts in front of you, but feel she life-threatening outcome would be that of a patient who is a
‘needs’ more than homeopathic treatment. But it is vital Jehovah’s Witness refusing blood products for whom a blood
transfusion might be lifesaving. Respect for autonomy (self-
to understand why she feels the way she does and explore
determination) means that this refusal must be respected.
her reasons for choosing homeopathy. She may have expe- Notwithstanding this, if an individual chooses an option that not
rience of homeopathy for other conditions, she may have only contradicts what appears to be in his or her best interests and
been ready to consider carbimazole before learning of a that most people would choose, but also appears to contradict that
dangerous side effect (leucopenia), and she may have con- individual’s previously expressed attitudes, then you would be
sidered other forms of treatment. You should talk to her justified in questioning the individual’s capacity to make a valid
about Graves’ disease, its complications, the benefits and refusal in order to exclude a possible depressed or deluded state.
risks of treatment and the likely natural history without
treatment. You might tell her you are unaware of any 
effective homeopathic treatments. Is it possible, if she is
adamant that homeopathic treatment will help her, 
that she would consider conventional treatment as  CASE 4.26  DELIBERATE SELF-HARM
well? Failing this, might she accept switching to 
conventional treatment within a certain time limit if  Candidate information
her approach fails. You would of course inform your con-
sultant of any conversation and document your advice Role
carefully.
You are a doctor on the acute medical unit.
Please read this summary.
You have been seeing a 50-year-old
man with tuberculosis in the respiratory Scenario
clinic. He wishes to pursue homeopathic
treatment rather than continue with Re: Miss Jane Hughes, aged 22 years
antibiotics. Would you agree? Miss Hughes was admitted to the emergency medical
admissions unit earlier today after taking an overdose of
There comes a point where collusion with a patient is paracetamol. She reported taking 16 50 mg tablets 12 hours
wrong, clinically, ethically and legally. Here, you have not before admission, with the intention of committing suicide. Her
just to consider this man’s autonomy, but risks to other paracetamol level was below the treatment line and liver function
members of the public from his active tuberculosis. Rather tests are normal including the international normalised ratio. She
than approaching this dilemma in a dogmatic way, listen- is now keen to go home. The ward nurse is concerned that she
ing to the patient’s reasons, beliefs and concerns may may be at risk of further suicide attempts, and although a
avoid confrontation. Perhaps the importance of antibiotic psychiatry opinion has been requested this might not happen until
therapy has never been fully explained. Perhaps he has tomorrow.
heard that homeopathy can help a wide range of respira- Your tasks are to explore the reasons for her suicide attempt,
assess her current suicide risk and suggest a way forward.
tory conditions. Listening and simple explanation may be
enough to persuade him of his misconception. Perhaps he
feels that the antibiotics should have worked by now and
is concerned that he is still coughing. Perhaps he is frus-
trated that he cannot return to work and needs to vent his Your examiners will warn you when 12 minutes have
feelings. elapsed. You have 14 minutes to communicate with the
patient / subject followed by 1 minute of reflection. There
will then follow 5 minutes of discussion with the examin-
Can a patient refuse treatment, even to the
ers. Do not take the history again except for details that
point of death? will help in your discussion with the patient / subject. You
Refusal of consent is discussed in Box 4.21. are not required to examine the patient / subject.

670
Case 4.26  Deliberate self-harm

Patient / subject information Box 4.22  Assessment following


Miss Jane Hughes is a 22-year-old primary school teacher attempted suicide
with no medical history of note. She was admitted to
hospital this morning following an overdose of 16 para- Questions about the episode
cetamol tablets. She has recently moved to the area, having • Events preceding the act
finished her teacher training last year, and although she • Reasons for the act
has no history of psychiatric disease or suicide attempts • Suicide intent
herself in the past she has suffered recently ‘in silence’ • Any psychiatric disorders including previous suicide attempts
from the death of her father and taken to getting exces- • Family history
sively drunk with new friends in the area on week nights, • Coping resources and support network
and, increasingly, drinking alone. Last month one of her • Whether ready to accept help
class children died in a road traffic accident and she has
found herself ruminating over this a lot. She is ‘a bright Factors suggesting suicide intent
young thing’ according to friends, and ‘has everything • Act undertaken alone
going for her’ but suffers from low self-esteem, which she • Act timed that intervention unlikely
knows is irrational. She has never had any delusions or • Precautions to avoid discovery
hallucinations. She does not know why she took the over- • Active preparation for attempt
dose now, only that she told a friend who brought her to • Preparations made anticipating death
hospital. She just wants to go home and forget it hap- • Leaving a note
pened. She is about to see the doctor. • Failure to inform potential rescuers afterwards
• Admission of suicidal intent
How to approach the case Factors associated with an increased risk  
Communication skills (conduct of interview, of further suicide attempts
• Male
exploration and problem negotiation) and
• Under 19 or over 45 years of age
ethics and law • Unemployment
1.  Introduction • Separated, divorced, widowed or living alone
Introduce yourself and make it clear you are here to help. • Previous suicide attempts with admission to hospital
• Alcohol or drug problems
2.  Show empathy • Psychiatric disorder
• Chronic disease
Deliberate self-harm, unhappily, is an increasing problem
for emergency services and it is all too easy to assume that
‘just another overdose’ has arrived, ‘probably with no
intention of significant self-harm’. Suicide is an escalating 6.  Reassure that information given  
problem and no patient’s intentions should be underesti- is confidential
mated and every patient’s case should be considered on Give absolute reassurance of this.
its own unique merit. These cases are complex, and
warrant psychiatric appraisal. All doctors should be able 7.  Reassure that the psychiatrist will help  
to make an initial assessment of the situation and the risk.
Firstly, tell her that you are sorry she is in hospital and that
to identify problems
you understand things must have been very difficult to Explain that the reasons for seeking a psychiatrist’s help
have reached this stage. are that it is not your area of expertise and that you wish
to protect her from future self-harm by dealing with issues
3.  Establish ideas and concerns that hold her back so that she can get on with life.
There are clearly background issues, which she may or may
8.  Explain possible alternatives
not allude to.
She may ask what would happen if she chose not to wait
4.  Assess patient following a suicide attempt to see a psychiatrist. This depends on your assessment of
her risk. Options include trying to arrange for more imme-
All doctors should be able to perform an initial psycho­ diate psychiatric assessment by a liaison psychiatrist or
social assessment following attempted suicide (Box 4.22). member of the liaison psychiatric team or to seek tele-
phone advice from a psychiatrist as to whether it is felt
5.  Explain what should be done next that she could be allowed home with early review. If risk
Reassure her that you are satisfied medically but that you is sufficiently worrying, you should be honest with her and
are concerned for her overall welfare. explain that you have a duty of care to ensure that she does

671
Station |4| Communication skills and ethics

not leave until she has seen a psychiatrist. Application of Which aspects of the Mental Health Act are
the Mental Health Act should be a last resort. relevant to general medicine?
9.  Invite further questions The Mental Health Act 1983 permits compulsory deten-
Is there anything you would like me to go over again or are there
tion and / or treatment of patients with a mental illness
any questions you would like to ask me? and / or mental impairment of a nature and / or degree that
requires inpatient treatment against their wishes. If a
10.  Agree a way forward patient needs to be in hospital because of a risk to self or
others he or she may be detained or brought into hospital
The examiners are aware that such a discussion is difficult.
if appropriate people agree. But the Act does not allow
They are not looking for a ‘model answer’ but checking
treatment of mentally impaired patients for physical prob-
your awareness of the range of issues provoked and ability
lems against their will, even if the physical problem is a
to handle these sensitively and professionally.
result of deliberate self-harm such as self-poisoning.
Therefore psychiatrists, often asked to ‘section’ such a
Discussion patient for compulsory treatment, cannot do so, although
What would you do if a patient wants to treatment may be given under common law. The Act does,
however, allow treatment where a physical condition
leave hospital before being assessed by a
causes the mental condition, as in an organic psychosis.
mental health professional? Relevant sections of the Mental Health Act are listed in
Doctors have a duty of care that includes protecting Table 4.3. Although there is a different legal system in
patients as best as possible from ongoing risk. If a patient Scotland, the principles are the same.
refuses to stay for mental health assessment following an The Mental Health Act was revised in 2008, and a key
episode of deliberate self-harm, you should if possible change is a wider definition of mental disorder, encom-
make your own assessment of risk (see Box 4.22). If you passing any disorder or disability of the mind.
are still concerned, you should try to persuade the patient
to stay and if they still refuse you may detain them under
common law pending formal psychiatric assessment. If
you are satisfied that risk is low, you may allow the patient Table 4.3  Sections of the Mental Health Act
to be discharged with appropriate information to the
general practitioner (ideally after telephone approval from Section Provision
and perhaps follow-up with a psychiatrist).
2 Allows assessment and / or treatment for up to
What does common law allow in the matter 28 days
Requires two appropriately qualified doctors and
of detention or treatment of patients? a social worker
It allows doctors to act in a patient’s best interests in emer- 3 Allows extension for up to 6 months (may follow
gency situations where consent cannot be given (patient 2)
unconscious or lacks capacity). It allows detention pending
4 Allows patient to be brought to hospital in an
assessment or treatment against a patient’s will if in that emergency
patient’s best interests (by saving life or to ensure improve- Requires a doctor and social worker
ment or prevent deterioration of physical or mental
health). It should be documented that you are treating in 5 (2) Allows any registered (i.e. not Foundation Year 1)
best interests under common law. hospital doctor to detain an inpatient under the
nominated hospital consultant if psychiatric
assessment likely to be delayed
Can you detain or treat against a patient’s Patient may be detained for 72 hours pending
will if they have capacity? full Mental Health Act assessment
Initiated by a form H1 that is submitted to the
Detention or treatment against a patient’s will under
local Mental Health Act administration team
common law or under the Mental Health Act is not pos- Does not allow treatment of any kind, which
sible if a patient has capacity. To do so may constitute a must be under common law if a patient does not
criminal offence. Where there are doubts about capacity consent (herewith a ‘best interests’ decision
(which may be function specific rather than global and based on presumed current incapacity allows
may change over time) and a patient is at risk it is usually attempted treatment)
better to detain and treat than not do so. In the case of 5 (4) Allows a nurse to detain for 6 hours pending the
deliberate self-harm, patients frequently do have capacity arrival of a doctor to detain under 5 (2)
but you may equally be concerned that capacity is tempo-
rarily lost by a psychiatric illness such as depression and 136 Allows police to bring a patient to an accident
and emergency department, where doctors may
if your assessment deems a patient to be at serious risk
decide to informally assess or arrange for a
then you are unlikely to be criticised. Detention and treat- Section 2 or 3
ment may be given under common law.

672
Case 4.27  End of life and palliative care

An elderly woman takes a lethal ingestion Patient / subject information


with the intention of suicide. She declines You are the son of Mrs Earnshaw, an 85-year-old woman
intervention. She is assessed by the admitted 3 days ago following an attempt to pass a stent
psychiatric team who feel that she has into her oesophagus. You are aware that the procedure was
capacity to make this decision. Would you to relieve symptoms, but not really sure if that means an
accept this? operation to cure things will be possible at a later date.
She was admitted because of drowsiness following the
At first sight it may seem that any patient attempting stent procedure (an intended day case) and since then she
suicide must lack capacity because of depressed mood but has become more breathless. You are very concerned that
this is by no means necessarily so. Some patients do feel she is much worse than before she was admitted, and
that life is so miserable for intractable reasons and see no wonder what is happening to her.
hope, such that, notwithstanding possible depression,
they have the capacity to decide they no longer wish to How to approach the case
live. While every opportunity must be given to allow the
patient to change their mind, and it must be clear that the Communication skills (conduct of interview,
patient fully understands the implications of the decision, exploration and problem negotiation) and
if a psychiatrist has confirmed capacity and declared that
the decision should be respected, the patient’s autonomy
ethics and law
should be respected. 1.  Introduction
Introduce yourself and establish that he is Mrs Earnshaw’s
son.
End-of-life issues
2.  Establish what the relative knows
Firstly, establish a rapport with a relative you have not met
CASE 4.27  END OF LIFE AND before, establishing what he knows, appreciating that rela-
PALLIATIVE CARE tives may either have been told little or have been afraid
to ask for more information and may still have false expec-
tations about the future.
Candidate information
What have you been told so far about her condition?
Role
You are a doctor on the medical ward. 3.  Give a warning shot
Please read this summary. This is largely a breaking bad news scenario, and before
explaining her condition you should say that she is very
Scenario unwell. A simple statement, followed by a pause, is enough.
Your mother’s condition is not good.
Re: Mrs Sylvia Earnshaw, aged 85 years
Mrs Earnshaw was admitted to the ward 3 days ago following 4.  Explain the situation
an attempt to pass an oesophageal stent. A decision that curative Relatives sometimes then volunteer that they are aware of
surgery would not be possible for her oesophageal cancer was the overall prognosis, but that things have deteriorated
reached on a previous admission. She was admitted because of more quickly than anticipated, but are sometimes very
drowsiness following the procedure (an intended day case) and the surprised. Be prepared for any scenario.
procedure was not wholly successful, unable to negotiate the distal
end of the tumour, and a repeat attempt was not ruled out. Your mother has developed a rather nasty chest infection,
However, since admission she has deteriorated with aspiration almost certainly because of the growth in the oesophagus
pneumonia, thus far not responding to antibiotics and is – the gullet – that means that she has been at very high risk
progressively drowsy. of aspirating or regurgitating food and fluids and saliva down
Your task is to discuss the care now appropriate for Mrs into her lungs. The procedure, as I know you know, was tricky
Earnshaw with her son. and the gastroenterologist found it impossible to pass the
tube beyond the growth.

5.  Acknowledge distress and allow ventilation  


Your examiners will warn you when 12 minutes have
elapsed. You have 14 minutes to communicate with the of feelings
patient/subject followed by 1 minute of reflection. There Wait for his response before explaining more.
will then follow 5 minutes of discussion with the examin-
ers. Do not take the history again except for details that 6.  Identify and prioritise concerns
will help in your discussion with the patient / subject. You He will probably want to know what that means in terms
are not required to examine the patient / subject. of overall prognosis now.

673
Station |4| Communication skills and ethics

The fact that the growth was difficult to bypass is of course a What is meant by EoL care?
real concern. I know the gastroenterologists had hoped that
if they could alleviate the obstruction it might enable her to How people die will remain in the lasting
eat and drink for a time. And I know that they have not memory of relatives, carers, and the health and
discounted the prospect of having a further attempt. But
clearly, as more complications arise, it becomes harder to see social care staff who have cared for the dying
a good way forward. The growth, as you can appreciate, person and so it is important that all staff
has not been one amenable to removal, or cure, and any recognise their responsibility to provide the best
treatments have therefore been directed to keeping symptoms
at bay for as long as possible. possible care at the end of life.
(DOH 2008)
7.  Explain what is likely to happen
EoL care helps all those with advanced, progressive, incur-
Explain that you will continue to treat the infection able illness, including dementia, to live as well as possible,
actively, but there are inevitably limits. Explain that if it until they die. It enables the supportive and palliative care
becomes inevitable that she is not going to get better, it needs of both patient and family to be identified and met
might be appropriate to switch treatment entirely to con- throughout the last phase of life, including the last days
trolling pain and breathlessness. You might feel that that of life and care after death. This includes management of
stage has already been reached, but at least allow him pain and other symptoms and provision of psychological,
some time to come to terms with the sudden change in social, spiritual and practical support.
his mother’s condition. It may be appropriate to discuss EoL care generally applies to all patients who have a
continuation of intravenous fluids for the time being. prognosis of less than 1 year. Patients with dementia are an
exception as the time scale to reach EoL can be much
8.  Identify support networks
longer. Therefore it is important to involve patients with
It is human to want to discover a little more about the dementia in decisions regarding EoL care whilst they still
practical implications to him such as what other family have the cognitive ability to make choices.
members may be nearby, how far it is for him and other The National End of Life Strategy recognises that quality
family to visit and so forth. and choice in EoL care are far from equitable and sets 
out an ambitious, far-reaching agenda to address this,
9.  Invite questions including:
Always do this.
• Promotion of public awareness and reducing taboos
10.  Conclude with assurances about death and dying
• High-quality, integrated care across all settings
Always conclude with an assurance that you will do all • Identification of people approaching the EoL to be
that you can to keep her comfortable and always be happy strengthened
to discuss the situation with him or any member of the • All people approaching the EoL and their carers to
family. have their needs assessed and a care plan which
records their preferences, including where they
Discussion would wish to die
What is meant by end of life (EoL)? • Ensuring that care is well coordinated and can be
accessed rapidly 24/7, aiming to avoid unnecessary
Patients approaching the end of their life need high- emergency admissions to hospital
quality treatment and care that supports them to live as • Ensuring that health and social care staff provide
well as possible until they die, and to die with dignity. sensitive and culturally and spiritually responsive
Providing treatment and care towards the EoL will often care during the last days of life and after death
involve decisions that are clinically complex and emotion- • Ensuring health and social care organisations provide
ally distressing, and may involve ethical dilemmas and readily available information on all local services
uncertainties about the law. Patients, according to General • Recognising the needs of carers and families
Medical Council (GMC) guidance on EoL care, are • EoL care training.
‘approaching the end of life’ when they are likely to die
within the next 12 months but this includes patients
whose death is imminent (expected within a few hours or
What do you understand by the term
days) and those with: ‘palliative care’?
• Advanced, progressive, incurable conditions This is the holistic care of patients with advanced, progres-
• General frailty and coexisting conditions that mean sive, incurable illness, focused on the management of a
they are expected to die within 12 months patient’s pain and other distressing symptoms and the
• Existing conditions if they are at risk of dying from a provision of psychological, social and spiritual support to
sudden acute crisis in their condition patients and their families. Palliative care is not dependent
• Life-threatening acute conditions caused by sudden on diagnosis or prognosis, and can be provided at any
catastrophic events. stage of a patient’s illness, not only in the last few days of

674
Case 4.27  End of life and palliative care

life. The objective is to support patients to live as well as Maximising capacity to make decisions
possible until they die and to die with dignity.
If a patient’s capacity to make a decision may be impaired,
they must be provided with all appropriate help and
What principles govern EoL care? support to maximise their ability to understand, retain,
use or weigh up the information needed to make that
The choices and priorities of the individual are central to
decision or communicate their wishes. Detailed guidance
all EoL care planning and delivery. Care planning must
about maximising and assessing a patient’s capacity may
meet the wishes, beliefs and priorities of the individual
be found elsewhere in this book in the sections on consent
patient. The patient is central to the care plan, and they
and capacity.
and / or their carers should be involved from the outset.
The care plan should be regularly reviewed as the patient
has the right to change their plan at any time.
Important principles, outlined in GMC guidance on EoL Overall benefit
care, are as follows. If an adult patient lacks capacity to decide, decisions made
on that patient’s behalf must be based on whether treat-
Equalities and human rights ment would be of overall benefit to the patient and which
option (including the option not to treat) would be least
Patients who are approaching the end of their life  restrictive of the patient’s future choices. Those close to the
must have equal quality of care, and they and those close patient who lacks capacity should be consulted to help
to them receive dignity, respect and compassion. The reach a view. The term ‘overall benefit’ describes the ethical
Human Rights Act 1998 incorporates into domestic law basis on which decisions are made about treatment and
the bulk of the rights set out in the European Convention care for adult patients who lack capacity. It involves an
on Human Rights (ECHR). Health professionals must be assessment of the appropriateness of treatment and care
able to demonstrate that decisions are compatible with options that encompasses not only the potential clinical
human rights set out in the Articles of the Convention. benefits, burdens and risks of those options, but also non-
The rights in the ECHR that are most relevant to decisions clinical factors such as the patient’s personal circum-
about treatment and care towards the end of a patient’s stances, wishes, beliefs and values. GMC guidance on
life are: overall benefit is consistent with the legal requirement to
• Article 2: The right to life and positive duty on consider whether treatment ‘benefits’ a patient (Scotland)
public authorities to protect life or is in the patient’s ‘best interests’ (England, Wales and
• Article 3: The right to be free from inhuman and Northern Ireland), and applies principles set out in the
degrading treatment Mental Capacity Act 2005 and the Adults with Incapacity
• Article 5: The right to security of the person (Scotland) Act 2000.
• Article 8: The right to respect for private and family
life
• Article 9: The right to freedom of thought, What sorts of ethical and legal issues arise
conscience and religion
in EoL decision making?
• Article 10: The right to freedom of expression, which
includes the right to hold opinions and receive The most challenging decisions in this area are generally
information about withdrawing or not starting a treatment when it 
• Article 14: The right to be free from discrimination has the potential to prolong the patient’s life. This may
in the enjoyment of these other rights. involve treatments such as antibiotics for life-threatening
infection or renal dialysis, cardiopulmonary resuscita-
Presumption in favour of prolonging life tion, or ‘clinically assisted nutrition and hydration’ and
mechanical ventilation. There are many ethical and legal
Decisions concerning potentially life-prolonging treat- considerations, which is why a multidisciplinary approach
ment must not be motivated by a desire to bring about with emphasis on communication and individually 
the patient’s death, and must start from a presumption in tailored management is so important. Here are some
favour of prolonging life. This presumption will normally examples:
require all reasonable steps to be taken to prolong a
patient’s life. However, there is no absolute obligation to • Advance decision making (Case 4.28)
prolong life irrespective of the consequences for the • Cardiopulmonary resuscitation (Cases 4.29 and
patient, and irrespective of the patient’s views, if they are 4.30)
known or can be found out. • Withholding and withdrawing treatment (Cases 4.32
and 4.33)
• Clinically assisted nutrition and hydration (Case
Presumption of capacity 4.34)
The presumption is that every adult patient has the capac- • Respect for autonomy and choice
ity to make decisions about their care and treatment. Lack • The Mental Capacity Act 2005 and lasting power of
of capacity must not be assumed. attorney (LPA) (Case 4.24).

675
Station |4| Communication skills and ethics

Do you know of any decision-making effective care and continuity in its delivery to meet the
models in relation to EoL care? patient’s needs. Patients whose death from their current
condition is a foreseeable possibility are likely to want 
GMC guidance on EoL care incorporates decision-  the opportunity to decide what arrangements should 
making models for patients with or lacking capacity  be made to manage the final stages of their illness. This
(Box 4.23). could include having access to palliative care, and attend-
ing to any personal and other matters they consider
What measures in the acute hospital setting important. If a patient has a condition that will impair
help ensure good EoL care? their capacity as it progresses, they should be encouraged
to think about what they might want should this happen.
• Recognising and diagnosing EoL Advance care plans should be fully recorded and need 
• Advance care planning (ACP) to be reviewed and updated as the patient’s situation or
• Information, explanation and discussion with views change.
patient and family
• Treatment escalation plan (TEP) and Do not attempt
resuscitation (DNAR) documentation TEP and resuscitation decision  
• Preferred priorities of care (PPC) documentation patient-held record
• Advance decisions to refuse treatment (ADRT) These should be discussed and documented. Resuscitation
and LPA decision-making is covered elsewhere. Degrees or limits of
• The hospital e-discharge summary proposed treatment escalation should be discussed with
• The End of Life Electronic register (Electronic patients or relatives, and creating TEP forms without doing
Palliative Care Co-ordination System e.g. ADASTRA), so is a leading cause of complaint to hospital trusts. Whilst
a list of all EoL patients including their TEP/DNAR consent is not legally needed for withholding or limiting
status and their PPC a treatment, this is a hugely contentious area and best
• Assessment and reassessment of the situation and practice calls for discussion with patients or those close to
changing needs patients.
• Liverpool Care Pathway or equivalent
Preferred priorities of care (PPC) document
Recognising and diagnosing EoL
What are the patient’s feelings on their disease, what 
Commonly used tools include: concerns do they have, what goals are they looking to
• The surprise question: ‘Would you be surprised reach, do they understand their illness and its prognosis,
if this patient were to die in the next 6–12   and do they have particular care preferences, now and in
months?’ the future?
• Performance status PPC is a document that individuals hold and take with
• The patient’s choice to not have further treatment them. It enables them to write down their thoughts and
• Symptoms and clinical observations: weight loss of choices. It facilitates continuity of care. It is not legally
10% or more in 6 months; serum albumin < 25 g / l; binding. You must consider the Mental Capacity Act
rapid decline (2005), and any person making a decision regarding
• Would my patient now benefit from supportive and care / treatment of a patient lacking capacity has to take
palliative care? into account any statement of wishes and preferences
• More than three admissions in the last year for made previously.
exacerbations of chronic illness.
Some EoL prognostic indicators in certain conditions Advance decision to refuse treatment (ADRT)
are given in Box 4.24. This is legally binding and comes into effect when an
individual loses mental capacity. It can only be used to
Advance care planning (ACP) refuse treatment and applies to refusal of specific treat-
ACP is the process of discussing the type of treatment and ments such as intravenous antibiotics and assisted feeding.
care that a patient would or would not wish to receive in It cannot be used to refuse basic comfort care.
the event that they lose capacity to decide or are unable
to express a preference. It seeks to create a record of a Lasting power of attorney (LPA) – health
patient’s wishes and values, preferences and decisions, to
This is a strong legal document held by the patient. An
ensure that care is planned and delivered in a way that
appointed person has power of attorney to deal with the
meets their needs and involves and meets the needs of
ill person’s health and welfare decisions.
those close to the patient.
As treatment and care towards the EoL are delivered by
multidisciplinary teams often working across local health, The EoL register
social care and voluntary sector services, planning ahead This gives rapid information on the patient’s illness, 
as much as possible helps ensure timely access to safe, needs and wishes when critical intervention is being

676
Case 4.27  End of life and palliative care

Box 4.23  Decision-making models for patients with or lacking capacity

Patients with capacity to make a decision must take account of the proxy’s views (as someone close to
• The doctor and patient make an assessment of the patient’s the patient) in the process of reaching a decision.
condition, taking into account the patient’s medical history, • Where there is no legal proxy with authority, and the doctor is
views, experience and knowledge. responsible for making the decision, the doctor must consult
• The doctor uses specialist knowledge and experience and with members of the health-care team and those close to the
clinical judgement, and the patient’s views and understanding patient (as far as it is practical and appropriate) before reaching
of their condition, to identify which investigations or treatments a decision. The doctor will explain the issues, seek information
are clinically appropriate and likely to result in overall benefit. about the patient’s circumstances and seek views about the
The doctor explains the options, setting out potential benefits, patient’s wishes, preferences, feelings, beliefs and values. The
burdens and risks of each. The doctor may recommend a doctor may explore which options those consulted might see as
particular option, but must not put pressure on the patient to providing overall benefit for the patient, but the role of those
accept advice. consulted is to advise about the patient’s known or likely
wishes, views and beliefs and they must not be given the
• The patient weighs up the potential benefits, burdens and risks impression that they are being asked to make the decision. The
of the various options as well as any non-clinical issues relevant doctor must take the views of those consulted into account in
to them, and decides whether to accept any of the options. considering which option would be least restrictive of the
They have the right to accept or refuse an option for a reason patient’s future choices and in making the final decision about
that may seem irrational to the doctor or for no reason at all. which option is of overall benefit to the patient.
• If the patient asks for a treatment that the doctor considers • In England and Wales, if there is no legal proxy, close relative or
clinically inappropriate, the doctor should discuss the issues other person who is willing or able to support or represent the
with the patient and explore the reasons for their request. If, patient and the decision involves serious medical treatment, the
after discussion, the doctor still considers that the treatment is doctor must approach their employing or contracting organisation
inappropriate, they do not have to provide it. They should about appointing an independent mental capacity advocate
explain their reasons and any other options available, including (IMCA), as required by the Mental Capacity Act 2005. The IMCA
the option to seek a second opinion or legal representation. will have authority to make enquiries about the patient and
contribute to the decision by representing the patient’s interests,
Patients lacking capacity to make a decision but cannot make a decision on behalf of the patient.
• The doctor, with the patient (if able to contribute) and the • If a legal proxy or other person involved in the decision making
patient’s carer, makes an assessment of the patient’s condition asks for a treatment to be provided which the doctor considers
taking into account the patient’s medical history and the would not be clinically appropriate and of overall benefit to the
patient’s and carer’s knowledge and experience of the condition. patient, the doctor should explain the basis for this view and
• The doctor uses specialist knowledge, experience and clinical explore the reasons for the request. If after discussion the
judgement, together with any evidence about the patient’s doctor still considers that the treatment would not be clinically
views (including advance statements, decisions or directives), to appropriate and of overall benefit, they are not obliged to
identify which investigations or treatments are clinically provide it. However, as well as explaining the reasons for their
appropriate and are likely to result in overall benefit for the decision, the doctor should explain to the person asking for the
patient. treatment the options available to them, including the option of
• If the patient has made an advance decision or directive seeking a second opinion and applying to the appropriate Court
refusing a particular treatment, the doctor must make a for an independent ruling.
judgement about its validity and its applicability to the current Legal proxy refers to a person with legal authority to make
circumstances. If the doctor concludes that the decision or certain decisions on behalf of another adult. Legal proxies include: a
directive is legally binding, it must be followed in relation to person holding a lasting power of attorney (England and Wales) or
that treatment. Otherwise it should be taken into account as welfare power of attorney (Scotland), a Court-appointed deputy
information about the patient’s previous wishes. (England and Wales) or a Court-appointed guardian or intervener
• If a legal proxy has been appointed to make health-care (Scotland). Northern Ireland currently has no provision for
decisions for the patient, the doctor explains the options to the appointing legal proxies with power to make health-care decisions.
legal proxy (as they would do for a patient with capacity), Powers of attorney must be registered with the Office of the Public
setting out the benefits, burdens and risks of each option. The Guardian in England, Wales and Scotland. ‘Carer’ refers to the
doctor may recommend a particular option that they believe person supporting the patient and representing their interests in the
would provide overall benefit. The legal proxy weighs up these consultation about their health and what might be needed in terms
considerations and any non-clinical issues relevant to the of any investigations, treatment or care. The term ‘those close to the
patient’s treatment and care, and, considering which option patient’ means anyone nominated by the patient, close relatives,
would be least restrictive of the patient’s future choices, makes partners and close friends, paid or unpaid carers outside the
the decision about which option will be of overall benefit. The health-care team and independent advocates. It may include
doctor should offer support to the legal proxy in making the attorneys for property and financial affairs and other legal proxies,
decision, but must not pressurise them to accept a particular in some circumstances.
recommendation. Who it is appropriate and practical to consult will depend on,
• As well as advising the legal proxy, the doctor must involve for example: a patient’s previous request; what reasonable steps
members of the health-care team and those close to the patient can be taken to consult within the time available before a decision
as far as it is practical and appropriate. If the legal proxy does must be made; and any duty to consult or prioritise specific people
not have the power to make a particular decision, the doctor set out in relevant capacity laws or codes.

677
Station |4| Communication skills and ethics

setting and across settings if patients are transferred) and


Box 4.24  End-of-life (EoL) indicators place of death. It is used in the last days of life when death
Cancer is the expected outcome of this episode of care. It docu-
ments the process accurately and consistently (using a
• Disease progression not amenable to treatment single record that replaces all notes / care plans and is used
• Involvement of major organs by all staff). It is endorsed in the End of Life Strategy.
• Worsening performance status The LCP comprises three sections – initial assessment
• Worsening functional ability and diagnosing dying, ongoing assessment, and care after
death. The multiprofessional team must agree that the
Neurology
patient is dying and that two of the following apply: bed-
• Motor neurone disease an EoL prognosis from diagnosis bound; semi-comatose; only able to take sips of fluids; no
• Parkinson’s disease when drug treatment no longer effective longer able to take tablets.
• Multiple sclerosis very difficult but dyspnoea, worsening pain, The LCP is not incompatible with treatment of condi-
medical complications tions that may relieve suffering such as pneumonia, or
Chronic obstructive pulmonary disease delivery of parenteral fluids, particularly where withdrawal
of the latter at LCP induction may produce emotive con-
• Forced expiratory volume < 30% predicted
cerns. It may of course be discontinued if a patient
• Long-term oxygen
improves (around 10% of patients will live despite being
• Right-heart failure
on the LCP). The LCP is not a tool to hasten or stall death,
• > 6 weeks of systemic steroids in the last year but to provide comfort and dignity in situations where
• Heart failure death is likely within a week. Goals of care with the LCP
• Dyspnoea at rest include:
• Failure to respond to drug therapy
• Renal function declining
• Communication with patient and family
• Current medication assessed and non-essentials
Renal disease discontinued
• Conservative treatment, not dialysis • As required subcutaneous drugs written up according
to protocol
• Chronic kidney disease stage 5
• Symptomatic of renal failure • Comfort measures
• Psychological insight / religious needs
Dementia • Ongoing care and support over the days needed
• An inability to perform activities of daily living • Care after death.
• You would not be surprised if the patient died within the next Typical subcutaneous drugs include diamorphine 2.5–
6–12 months 5 mg for pain, hyoscine 400–1200 μg for noisy secretions,
• 10% weight loss in the previous 6 months without other midazolam 2.5–5 mg for agitation or restlessness and 
causes levomepromazine 6.25 mg for nausea and vomiting; 
• Recurrent infections and pneumonia these may be extended in doses over 24 hours in a syringe
• Double incontinence driver, if, for example, two or more boluses of medica-
• No consistently meaningful verbal communications tion are required. Discussing resuscitation and LCP deci-
sions with the patient, or, where incapable, relatives, is
imperative.

Which drugs may be used to alleviate pain


considered. It is an easily accessible electronic Web-based
in modern palliative care?
register with information for out-of-hours services. 
Most patients are put on the system by their general prac- Some are listed in Table 4.4.
titioner. It allows existing choices to be implemented. It
can prevent unwanted hospital admission. It is accessible What is the doctrine of double effect (DDE)?
on edit or view basis by staff involved in the patient’s care.
The DDE attempts to distinguish between harms intended
TEP / DNAR status and any PPC, ADRT and LPA may be
and harms foreseen but not intended. The DDE raises
entered. It is voluntary for the patient and needs the
concern that it may protect dangerous practice. The DDE
patient’s consent.
states that an action (such as an increase in opioid dose)
that the professional foresees may shorten life – but does
Liverpool Care Pathway (LCP)   not intend to have that result – is justified provided that
for the dying person the intention is to benefit the patient and not shorten their
The LCP (one of numerous for the dying) ensures equita- life. In the classic interpretation, a patient is in pain, a
ble management and care of patients who are dying irre- higher dose of opioid is given to relieve that pain, but at
spective of diagnosis, situation (designed to be used in any the same time in the belief that this may shorten life. The

678
Case 4.27  End of life and palliative care

Table 4.4  Drugs to alleviate pain in palliative care


Routine use Opioids such as morphine or diamorphine are commonly used as long-acting regularly prescribed preparations (e.g.
Zomorph) and short-acting preparations for breakthrough pain (e.g. Oramorph)
Renal disease Oral
If the eGFR is < 30, renally excreted opioids such as codeine, morphine and diamorphine should be avoided except in
very small doses. Oxycodone is 90% excreted by the liver. Buprenorphine, fentanyl and alfentanil are 100% excreted by
the liver. An analgesic step-ladder in renal impairment is:
1. Paracetamol (50% normal dose)
2. Tramadol (50% normal dose) or buprenorphine patch
3. Oxycodone (normal dose if eGFR > 30, 50% normal dose if eGFR 10–30) – immediate release Oxynorm and
modified release Oxycontin
If the eGFR is < 10, a fentanyl patch may be suitable, with immediate release buccal fentanyl for breakthrough pain.
The latter is immediate, rather than older breakthrough drugs which could take 30 minutes to act, and available as a
sublingual lozenge (e.g. Effentura) rather than the outmoded lollipop. Fentanyl may be fatal in opioid-naive patients in
whom it is better to judge opioid requirements first of all with shorter acting drugs
Subcutaneous
Alfentanil may be used for immediate release (0.5–1 mg), or in a subcutaneous syringe driver
Syringe driver
A 24-hour syringe driver to alleviate symptoms in dying patients could include:
• Alfentanil 250 µg (instead of morphine)
• Glycopyrronium 200 µg (instead of hyoscine)
• Midazolam 2.5 mg (not adjusted for renal impairment)
• Haloperidol 0.5–1 mg (instead of levomepromazine)
Buprenorphine This is generally used for those with renal disease or those with compliance or swallowing problems. It is available as 5,
10 or 15 µg/h Butrans patches over a week or as 35 µg Transtec patches; greater requirements than this may warrant
switching to fentanyl
Pregabalin Pregabalin is now first line for neuropathic pain, as is amitriptyline, but the latter has many side effects. Pregabalin is
best started in low dose (25–50 mg). Serotonergic syndrome is a potential complication
Lidocaine 5% lidocaine patches are licensed for post-herpetic neuralgia and said not to provide deep skin penetration but, in
practice, seem to help some patients with localised pain from, for example, bone metastases, and are relatively harmless
Non-steroidal anti- These have fallen from favour somewhat because of thrombotic risk but remain very useful in palliative care
inflammatory drugs and
cyclo-oxygenase-2 inhibitors
Tapentadol This is a novel synthetic opioid with some morphine agonistic action and substantial norepinephrine (noradrenaline)
reuptake inhibition. It acts on spinal cord pain pathways; 50 mg is equivalent to 10 mg oxycodone, 20 mg morphine
and 200 mg tramadol
Opioid-induced constipation Methylnaltrexone blocks opioid receptors peripherally in the gut but not centrally and is highly effective in opioid-
induced constipation
eGFR, estimated glomerular filtration rate.

Association for Palliative Medicine (APM) has provided used in line with accepted palliative care practice. The APM
position statements on the DDE and use of sedation at  believes that the DDE is unnecessary to justify the use of
the EoL: dosing regimens necessary to manage pain or distress in
all but the most exceptional circumstances.
The DDE states that the risk of a potential, All medication used in palliative care, including sedative
known (foreseen), unintended consequence or medication, is aimed at the relief of specific symptoms.
side effect of treatment is justified only if all the Medication that is sedating in its effect should be used
only if the symptom cannot be relieved with more specific
following criteria are met: the intended effect is interventions. Rarely, patients may experience distress
good in itself; the clinician’s intention is solely when symptoms cannot be controlled even after exhaus-
to produce the good effect; the intervention is tive attempts with specific interventions. In these circum-
proportionate to the situation; the good effect is stances, some patients may require sedating medication to
not achieved through the bad effect. diminish awareness of their suffering. If medication is
sedating in its effect, the dose should be monitored to
There is a misconception that morphine-related and ensure that it is the minimum required to relieve the
sedative drugs bring about death more quickly, and that patient’s distress. Medication used in this way does not
doctors both know this and in some way condone their shorten life. Sedation in palliative care is thus sedation
use with the double effect. The APM refutes this claim. It while the patient dies and is not sedating the patient to
knows of no credible evidence to suggest that a patient’s death. Morphine and related drugs are vital painkillers but
life is shortened either by opioids or by sedatives when are wholly unsuitable for use as sedation.

679
Station |4| Communication skills and ethics

Consider a patient dying from a Have you heard of the Assisted Dying for
malignant brain tumour who develops the Terminally Ill Bill and, if so, what ethical
pneumonia and respiratory failure. A issues does it raise?
ventilator may prolong life but may also This concerns deliberate acts to end life, not assisting in
delay an inevitable death. What courses of the natural dying process. The Bill was the subject of a
action are there? House of Lords Select Committee Report. The idea of
deliberately ending life brings intense ethical questions,
Broadly, there are five options at the EoL (Box 4.25). notably concerning morality, the integrity of the medical
profession and social implications of the effects of society
Is allowing a person to die different from permitting direct killing, and so weakening the prohibi-
killing that person? tion against killing, which currently protects us all. Pallia-
Many people believe that there is a moral difference tive care is concerned with enabling patients with advanced
between killing someone and allowing that person to die life-threatening conditions to live with the best possible
but Rachels (1986) showed that the distinction is not quality of life until death. Clinical experience and research
necessarily clear, describing two cases. In the first, person suggest that most requests for euthanasia or physician-
A stands to gain a large inheritance if his 6-year-old cousin assisted suicide arise because of poor symptom control,
dies and sneaks into the child’s bathroom and drowns the depression, poor social and family support and loss of
child in a way that will look like an accident. In the autonomy. Palliative care focuses on improving these
second, person B also stands to gain and plans to drown aspects of a patient’s life and many of us believe that the
the child but, on approaching the cousin, the child slips Assisted Dying Bill and attempts to legalise physician-
and hits his head, falling face down in the bath; person B assisted suicide are ethically unsound, fail to appreciate
watches the child die. A kills; B allows to die; but there the nature and scope of palliative care and undermine the
seems no moral difference here. The question of a moral progress made in the care of the dying in recent years.
difference becomes more relevant when applied to with-
holding or withdrawing life-prolonging treatments. The One of your patients, a dying man with
central idea is that if a doctor kills a patient the doctor end-stage renal failure, diabetes and
causes the patient’s death, but if a doctor allows a patient peripheral gangrene, has a hypoglycaemic
to die the patient’s death is the result of disease or ‘nature attack. Would you treat him?
taking its course’. Thus, options 4 and 5 in Box 4.25 seem
worse than 2 and 3, which are accepted medical practices. This would depend upon any prior wishes he may have
Option 1 is now uncommon. Options 4 and 5 are illegal, made. He may have told his doctors to ‘let him go’ if he
and arguments against euthanasia include palliative care takes any turn for the worse. He may, alternatively, have
obviating its need, concerns about manipulation or declined treatment for his renal failure and requested a
exploitation, and ‘slippery slope’ effects. Although emo- comfortable death. A scenario such as hypoglycaemia may
tionally it may be easier to withhold than to withdraw not have been anticipated or explicitly discussed. A
life-prolonging treatment, the British Medical Association common response to a potentially fatal change in condi-
and GMC have indicated that there are no legal, or neces- tion that has not been previously discussed is to treat the
sarily moral, differences between the two. current episode and later discuss with the patient what they
would wish were it to recur. This said, because hypoglycae-
mia, unlike other potentially fatal changes in his condi-
tion, is caused by the medication and easily reversed, there
Box 4.25  End-of-life options may be legal risks in not treating, whatever the patient says.

1. The sanctity of life view – to ventilate and therefore prolong A woman has breast cancer with cerebral
life whatever else may apply. He would probably die from
progression of his tumour. metastases. She asks you how many of her
2. To withhold life-prolonging treatment – to withhold ventilation, anticonvulsant pills she would need to take
for example. He would probably die from respiratory failure. to end her life. How might you respond?
This has sometimes in the past, but very contentiously, been
considered ‘passive euthanasia’. We sometimes hasten death in clinical practice. The crucial
3. To withdraw life-prolonging treatment – to ventilate then distinction is intention, which may be to hasten death or
withdraw from ventilation. He would probably die rapidly on relieve suffering. Doctors may set out to relieve pain and
withdrawal of ventilation. This has sometimes in the past, but suffering but see that life may be shortened. Foreseeing is
very contentiously, been considered ‘passive euthanasia’. not necessarily the same as intending. This has been tested
4. Assisted suicide – this is illegal. in law and held to be permissible and in keeping with the
5. Active euthanasia (performing an action that results in a duties of a doctor. It is one aspect of the DDE, which
patient’s death) – killing a patient for any reason is normally makes a distinction between harms that are intended and
murder. harms that are foreseen but not intended. Telling the
patient how many pills she would require to kill herself

680
Case 4.28  Advance decision making

could be seen as assisting suicide and a criminal act. Your non-invasive ventilation, with apparently good effect. You
communication skills should endeavour to elicit key have mixed feelings. On the one hand you are pleased to
aspects of her suffering as she sees them and explore alter- see your husband less distressed but on the other are con-
native ways of relieving them. cerned about his prior wishes being ignored. You wonder
if the advance directive has any validity.

How to approach the case


CASE 4.28  ADVANCE DECISION
MAKING Communication skills (conduct of interview,
exploration and problem negotiation) and
Candidate information ethics and law
1.  Introduction
Role
Introduce yourself and establish that she is Dr Winter’s
You are a doctor on the medical ward. wife.
Please read this summary.
2.  Establish rapport and be alert to cues
Scenario Firstly, establish a rapport with a relative you have not met
before, establishing what she knows, and if she does not
Re: Dr Max Winter, aged 79 years volunteer the fact it may become clear that she is a profes-
Dr Winter is a retired microbiologist with chronic obstructive sional colleague.
pulmonary disease (COPD). Four months ago he prepared an
advance directive in conjunction with his family. This stated that in
the event of worsening respiratory problems requiring admission to
3.  Seek further information about the patient
hospital he would not wish cardiopulmonary resuscitation or any Relatives are highly valuable resources of information. She
form of ventilation including non-invasive ventilation. He was will know intimately the nature of his condition and his
admitted to hospital a few minutes ago with an acute exacerbation wishes in the event of a precipitous acute episode of illness.
of COPD and marked hypoxia. He is confused, probably a result of
his hypoxia, and now says that he will accept any treatment to 4.  Be sure that you understand the legal
make him better, although this is at odds with his advance
directive. The emergency team have ignored his advance directive principles of advance decision making
and just instituted non-invasive ventilation, with apparently good Advance decision-making legal principles are discussed
effect. His wife wishes to discuss the advance directive with you. below.
Your task is to discuss his management with his wife, taking
into account the issue of the advance directive. 5.  Explain the situation
Be honest about what has happened. In this case there 
Your examiners will warn you when 12 minutes have is a possibility that he is temporarily incapacitated by
elapsed. You have 14 minutes to communicate with the hypoxia and the previous advance directive, made when
patient / subject followed by 1 minute of reflection. There he had capacity, means there is a risk that the doctors
will then follow 5 minutes of discussion with the examin- applying non-invasive ventilation are committing battery.
ers. Do not take the history again except for details that However, it seems very reasonable to have erred on the
will help in your discussion with the patient / subject. You side of the possibility of his having changed his mind.
are not required to examine the patient / subject. Where there is uncertainty it is always preferable to make
the choice that preserves future choice as much as possible.
Patient / subject information His wife will probably fully accept this, provided you are
open with her and any Court (it would be very unlikely
You are a retired general practitioner. Your husband, Dr to go that far) would almost certainly see it that way, too!
Max Winter, is a 79-year-old retired microbiologist with
chronic obstructive pulmonary disease (COPD). Four 6.  Have a plan
months ago he prepared an advance directive in conjunc-
tion with his family. This stated that in the event of worsen- The best way forward is likely to be to continue to treat
ing respiratory problems requiring admission to hospital non-invasively, stopping short of invasive ventilation,
he would not wish cardiopulmonary resuscitation or any until her husband is able to competently confirm his
form of ventilation including non-invasive ventilation. He wishes about any subsequent institution of non-invasive
was admitted to hospital a few minutes ago with an acute ventilation.
exacerbation of COPD and marked hypoxia. He is con-
7.  Ensure that your suggestions do not conflict
fused, probably a result of his hypoxia, and now says that
he will accept any treatment to make him better, although with any ideas, concerns and expectations
this is at odds with his advance directive. The emergency Most importantly, ensure that she understands that where
team have ignored his advance directive and just instituted there was doubt the emergency team had little choice but

681
Station |4| Communication skills and ethics

to act in what they saw as being his current wishes or in lose capacity, and made clear that, although future deci-
his best interests but that you will be very sure to ascertain sions cannot be bound by request for a treatment, the
his wishes as soon as he is able to communicate these. request will be given weight by those making the decision.
A patient’s previous request, when benefits, burdens and
8.  Seek acceptance of this plan risks are finely balanced, will usually be the deciding
Highlight that the decision was ultimately that of the factor.
medical team, but that in future you will ensure that any
unequivocal wishes will be confirmed and respected. How might you act on advance refusals
of treatment?
9.  Invite questions Some patients worry that towards the end of their life they
Make it clear that you are happy to answer questions now may be given medical treatments that they do not want.
or later. They may want to make their wishes clear about particular
treatments in circumstances that might arise in their future
10.  Conclude with assurances care. When discussing any proposed advance refusal it
should be explained how such refusals would be taken
Always conclude with an assurance that you will do your
into account if capacity is lost.
best to keep her husband comfortable and accord with his
wishes as soon as these can be confirmed.
Binding advance refusals
Discussion An advance decision to refuse treatment (ADRT) states
what a patient does not want to happen to them and must
What is an advance statement? relate to a specific treatment and a specific circumstance.
It comes into force when the patient loses capacity to
This is a statement of a patient’s views about how they
consent to or refuse treatment. It is legally binding in
would or would not wish to be treated if they became
England and Wales (and potentially binding in Scotland
unable to make or communicate decisions for themselves.
and Northern Ireland) if constituted according to the con-
It can be a general statement about, for example, wishes
ditions in the Mental Capacity Act and if it meets the
regarding place of residence, religious and cultural beliefs,
specific clinical circumstances. An ADRT for life-sustaining/
and other personal values and preferences, as well as about
life-prolonging treatment requires more rigorous legal cri-
medical treatment and care. Whilst not legally binding,
teria, and must be in writing, signed and witnessed and
such statements must inform best interests decisions.
state that it should be enforced even if life is at risk. There
are restrictions on an ADRT, for example that basic nursing
What is an advance decision or care cannot be refused.
advance directive?
This is a statement of a patient’s wish to refuse a particular Non-binding advance refusals
type of medical treatment or care if they become unable Written and verbal advance refusals of treatment that are
to make or communicate decisions for themselves. They not legally binding should be taken into account as evi-
are called advance decisions in England and Wales, and dence of the person’s wishes when assessing whether a
advance directives in Scotland. In England and Wales, particular treatment would be of overall benefit.
advance decisions are covered by the Mental Capacity Act
2005. In Scotland and Northern Ireland, advance direc- How might you assess the validity and
tives are not covered by statute but it is likely that they are
binding under common law.
applicability of advance refusals?
Assessment of validity and applicability is outlined in 
How might you act on advance requests Box 4.26.
for treatment?
An elderly woman with dementia, living
When planning ahead, some patients worry that they 
will be unreasonably denied certain treatments towards
alone but with carers, fractures the neck
the end of their life, and so may wish to make an advance of her femur. She is admitted to hospital
request for those treatments. They may want a treatment with an abbreviated mental test score
that has some prospects of prolonging their life, even if  (AMTS) of 4 / 10. Her son says she would
it has significant burdens and risks. When responding  not wish any treatment. Would you agree
to a request for future treatment the reasons for the
request, and the degree of importance the patient attaches
to his request?
to it, should be explored. It should be explained how  Regarding her autonomy, the patient is probably not com-
decisions about the overall benefit of the treatment would petent and her AMTS is likely to represent a pot pourri of
be influenced by the patient’s current wishes if they  background cognitive impairment, pain and possible

682
Case 4.29  Resuscitation status decision making – discussion with patient

Box 4.26  Assessing the validity and applicability of advance refusals

Validity Applicability
The main considerations are that: The following considerations apply across the UK:
• The patient was an adult when the decision was made (16 • Whether the decision is clearly applicable to the patient’s
years old or over in Scotland, 18 years old or over in England, current circumstances, clinical situation and the particular
Wales and Northern Ireland) treatment or treatments about which a decision is needed
• The patient had capacity to make the decision at the time it • Whether the decision specifies particular circumstances in which
was made (UK wide) the refusal of treatment should not apply
• The patient was not subject to undue influence in making the • How long ago the decision was made and whether it has been
decision (UK wide) reviewed or updated (this may also be a factor in assessing
• The patient made the decision on the basis of adequate validity)
information about the implications of their choice (UK wide) • Whether there are reasonable grounds for believing that
• If the decision relates to treatment that may prolong life it must circumstances exist which the patient did not anticipate and
be in writing, signed and witnessed, and include a statement which would have affected their decision if anticipated; for
that it is to apply even if the patient’s life is at stake (England example, any relevant clinical developments or changes in the
and Wales only) patient’s personal circumstances since the decision was made.
• The decision has not been withdrawn by the patient (UK wide) If there is doubt or disagreement about the validity or
• The patient has not appointed an attorney, since the decision applicability of an advance refusal of treatment, further enquiries (if
was made to make such decisions on their behalf (England, time permits) and a ruling from the Court may be sought. In an
Wales and Scotland) emergency, if there is no time to investigate further, the
presumption should be in favour of providing treatment, if it has a
• More recent actions or decisions of the patient are clearly
realistic chance of prolonging life, improving the patient’s condition,
inconsistent with the terms of their earlier decision, or in some
or managing their symptoms.
way indicate they may have changed their mind.

analgesia. On the principle of justice, most patients are


more likely to do well with repair of their fractured neck CASE 4.29  RESUSCITATION STATUS
of femurs and the benefits of surgery may outweigh the DECISION MAKING – DISCUSSION
risks. It may be desirable to go ahead with treatment, with
efforts to explain the rationale and get her family ‘on WITH PATIENT
board’.
Candidate information
A young woman with a history of asthma,
necessitating multiple admissions to ITU, Role
and depression is admitted with a further You are a doctor on the respiratory ward.
exacerbation of asthma requiring immediate Please read this summary.
ITU support. A handwritten letter in her Scenario
notes addressed to her chest physician
states that she wishes no treatment in the Re: Mr Robert Churchill, aged 75 years
event of future collapse. Should she be Mr Churchill has severe chronic obstructive pulmonary disease.
admitted to ITU? He has a home oxygen cylinder. For the last 3 years he has been
confined to his house, with twice-daily carers to assist with
The answer is almost certainly yes. The validity of the bathing, meals and housework. Breathlessness is his main
advance decision may be in question for numerous limitation. Over the last year he has been admitted to hospital five
reasons. It might have been written while she was depressed times with increasing frequency and with exacerbations of
and her capacity might have been in question. The details breathlessness, sometimes precipitated by infection. Investigations
may be sparse rather than specific. There may not be evi- for pulmonary emboli have been negative. He was readmitted
dence that it was written by her. In case law there is a yesterday with breathlessness and responded to initial non-invasive
potential risk of battery if she receives treatment against ventilation from which he has been successfully weaned. However,
his resuscitation status has not been documented.
the instructions of a valid advance directive but not treat- Your task is to discuss his resuscitation status with him. You
ing puts her doctors at risk of manslaughter and because may assume that you were involved in his care yesterday and on
of the uncertainty the correct course of action is almost previous admissions and that he knows who you are.
certainly to treat because of the uncertainty.

683
Station |4| Communication skills and ethics

Your examiners will warn you when 12 minutes have had hoped. Someone with a lung condition as bad as yours
elapsed. You have 14 minutes to communicate with the may suddenly take a turn for the worse. I know you know
patient / subject followed by 1 minute of reflection. There this, having used the breathing apparatus last night and on
will then follow 5 minutes of discussion with the examin- other occasions in the past. We will of course do all that we
can to improve the situation for you this time. But there is
ers. Do not take the history again except for details that
always the possibility, as with any unwell patient, that things
will help in your discussion with the patient / subject. You could go very wrong: that the heart could stop, or the
are not required to examine the patient / subject. breathing stop, altogether. Have you ever thought about what
your wishes might be in that situation?
Patient / subject information
Mr Robert Churchill is a 75-year-old man who has severe 5.  Many patients immediately understand  
chronic obstructive pulmonary disease. He has a home all of this, and pre-empt further discussion  
oxygen cylinder. For the last 3 years he has been confined by declaring clearly pre-considered wishes.  
to his house, where he lives alone, having been widowed Some patients wish to go further and may  
for 5 years. He has twice-daily carers to assist with bathing,
ask about resuscitation
meals and housework. Breathlessness is his main limita-
tion. Over the last year he has been admitted to hospital If things were to go horribly wrong, and your heart or
breathing were to stop, would you wish us to try to re-start
five times with increasing frequency and with exacerba-
things with heart-starting machines or breathing tubes and
tions of his breathlessness. He was readmitted yesterday so forth?
with breathlessness and required a tight breathing mask
to improve respiratory distress. He is better today but 6.  Be prepared to deal with emotions  
knows things are not going to get any better in the long
run. He has two children, but they live a long distance before facts if you sense the discussion  
away. He has considered the future and ‘doesn’t want to is causing distress
go on indefinitely like this’ but the issue of whether or not I can see that you find this difficult. We could talk about it later,
he should be resuscitated in the event of a cardiac or res- if you prefer, or, if you wish, with members of your family here
piratory arrest has never been discussed. (if you know there are family members) or some patients
simply prefer that we do what we think is the very best for
them.
How to approach the case
Communication skills (conduct of interview, 7.  Many patients prefer to leave the decision to
exploration and problem negotiation) and their doctors
ethics and law I would leave it in your hands, doctor. Do as you think best.

1.  Introduction, setting and rapport 8.  Some patients might ask what you think
A discussion about resuscitation should be between a Well, we would have great concerns that in the event of things
patient and ideally a doctor who has already created a getting to that stage, trying to rescue the situation would be
rapport with that patient, even if brief. The setting should in vain. It simply would not work.
ideally be quiet, and without distractions. It is not inappropriate to tell patients or relatives that
you think resuscitation extremely unlikely to be successful
2.  Ensure the patient has enough information or to be futile or that invasive ventilation would be very
about their condition unlikely to bring recovery. Many patients do not appreci-
A resuscitation discussion should only come once he has ate, until it is explained, that resuscitation and the poten-
a full grasp of his condition and its likely prognosis even tial consequences of temporary revival without meaningful
with treatment. recovery or longer term survival might simply be increas-
ing the distress of an inevitable natural death, prolonging
3.  Then come directly to the reason   death rather than sustaining life, or increasing the likeli-
hood of ‘two deaths’.
for the discussion
Mr Churchill, there is one thing I should discuss with you that
relates to what we might do in the event – not that I’m
9.  Confirm patient understanding and explore
expecting it – but in the event of things going very wrong any other concerns
(with your health) on this admission to hospital. Ensure that he is clear about the course of action, be pre-
pared to explore any other concerns and ask if he would
4.  Pace the explanation slowly and carefully, and like anything clarified.
in words the patient will understand, allowing
him to assimilate what you are saying 10.  Conclude with assurances
You have been in hospital rather a lot over the last year, and Always conclude with an assurance that, if a decision is
now you’re back in hospital sooner than you and your doctors made not to resuscitate, you and your team will continue

684
Case 4.29  Resuscitation status decision making – discussion with patient

to do all that you can to actively treat his condition and care. Any decision about whether or not to attempt CPR
to relieve symptoms. Make it clear, where appropriate, that must be readily accessible to all health professionals who
a decision can be reviewed and altered at any time. may need to know it. The person who makes a CPR deci-
sion is responsible for ensuring that the decision is com-
Meanwhile, we will do all that we can to get you better (up to municated effectively to other relevant health professionals.
the brink). Decisions about CPR must be reviewed regularly and espe-
cially whenever changes occur in the patient’s condition
Discussion or in the patient’s expressed wishes.

Why is it important to consider Do you know of any CPR decision-making


cardiopulmonary resuscitation (CPR) decision frameworks?
making in advance care planning?
A CPR decision-making framework is shown in Table.4.5.
The primary goal of health care is to benefit patients, by
restoring or maintaining their health as far as possible, What is the ‘presumption in favour of CPR’
thereby maximising benefit and minimising harm. If treat-
ment fails, or ceases to benefit, or if an adult with capacity
when there is no DNACPR decision?
has refused treatment, that treatment is no longer justified. There is always a presumption for protecting life and
Prolonging a patient’s life usually provides a health benefit attempting CPR in unknown situations.
to that patient. Nevertheless, it is not appropriate to
prolong life at all costs with no regard to quality or to the Emergencies
potential burdens of treatment. The decision to use any Emergencies can arise when there is no time to make a
treatment should be based on the balance of burdens, proper assessment of the patient’s condition and the likely
risks and benefits, and that principle applies as much CPR outcome of CPR, when no previous DNACPR decision is
as to any other treatment. in place. If no explicit decision has been made in advance
CPR has a reasonable success rate in some circum- about CPR and the express wishes of the patient are
stances but the survival rate after cardiorespiratory arrest unknown and cannot be ascertained, there should be a
and CPR is generally relatively low. After CPR for cardiores- presumption in favour of CPR. In such emergencies there
piratory arrest that occurs in hospital, the chances of  will rarely be time to make a proper assessment of the
surviving to hospital discharge are at best about 15–20%. patient’s condition and the likely outcome of CPR and so
Where cardiac arrest occurs out of hospital, the survival  attempting CPR will usually be appropriate.
rate is lower, at best 5–10%. The probability of success
depends on factors including cause, how soon CPR is
If further information comes to light
started, and equipment and staff available to deliver it.
Attempting CPR carries a risk of significant adverse effects There may be situations in which CPR is commenced on
such as rib or sternal fractures, hepatic or splenic rupture, this basis, but during CPR further information comes to
or prolonged treatment in an intensive care unit, possibly light that makes continued CPR inappropriate. That infor-
including prolonged artificial ventilation. There is also a mation may consist of a DNACPR order or a valid and
risk that the patient will be left with hypoxic brain damage applicable advance decision refusing CPR in the current
and resulting disability. And if CPR is unsuccessful, it may circumstances, or may consist of clinical information indi-
mean that the patient dies in an undignified and traumatic cating that CPR will not be successful. In such circum-
manner. stances, continued attempted resuscitation would be
It is not uncommon for difficult decisions about CPR inappropriate.
to arise in respect of patients for whom it may be possible
to re-start the heart after cardiac arrest but for whom Certainty of sufficient information to judge that
admission to an intensive care unit for continued organ CPR will not be successful
support would be clinically inappropriate because they There will also be some patients for whom attempting
would be unlikely to survive that admission. CPR is clearly inappropriate, for example a patient in 
the final stages of a terminal illness where death is immi-
Do not attempt CPR (DNACPR) decisions nent and unavoidable and CPR would not be successful,
If cardiac or respiratory arrest is an expected part of the but for whom no formal DNACPR decision has been
dying process and CPR will not be successful, an advance made.
decision not to attempt CPR will help to ensure that the
patient dies in a dignified manner. These management What is the approach to decision making
plans are called Do not attempt CPR (DNACPR) orders,
or Do not attempt resuscitation (DNAR) or Allow natural
when CPR will not be successful?
death (AND) decisions. The overall clinical responsibility There are circumstances in which CPR will not to be suc-
for decisions about CPR, including DNAR decisions, rests cessful or will be unlikely to be successful and, if success-
with the most senior clinician in charge of the patient’s ful, likely to prolong or increase suffering.

685
Station |4| Communication skills and ethics

Table 4.5  Cardiopulmonary resuscitation (CPR) decision-making framework


Is cardiac or respiratory arrest a clear No If there is no reason to believe that the patient is likely to have a cardiac or respiratory
possibility in the circumstances of the → arrest it is not necessary to initiate discussion with the patient (or those close to a
patient? patient who lack capacity) about CPR. If, however, the patient wishes to discuss CPR
this should be respected.
Yes ↓ 
Is there a realistic chance that CPR No When a decision not to attempt CPR is made on these clear clinical grounds, it is not
could be successful? → appropriate to ask the patient’s wishes about CPR, but careful consideration should be
given as to whether to inform the patient of the Do not attempt resuscitation (DNAR)
decision.
Where the patient lacks capacity and has a legal proxy, this person should be informed
of the decision not to attempt CPR and the reasons for it.
If a second opinion is requested, this should be respected wherever possible.
Yes ↓ 
Does the patient lack capacity and have Yes If a patient has made an advance decision refusing CPR, and the criteria for
an advance decision refusing CPR or a → applicability and validity are met, this must be respected. If legal proxy has been
legal proxy with relevant authority? appointed they should be consulted.
No ↓
Are the potential risks and burdens of Yes When there is only a very small chance of success, and there are questions about
CPR considered to be greater than the → whether the burdens outweigh the benefits of attempting CPR, the involvement of the
likely benefits of CPR? patient (or, if the patient lacks mental capacity, those close to the patient) in making
the decision is crucial. When adult patients have mental capacity their own view should
guide decision making.
No ↓
CPR should be attempted unless the
patient has capacity and states that
they would not want CPR attempted.

CPR will not be successful template of appropriate evidence-based multidisciplinary


care for the last days of life and prompts clinicians to con-
In some cases, the decision not to attempt CPR is a
sider and document the patient’s CPR status.
straightforward clinical decision. If it is believed that CPR
will not re-start the heart and maintain breathing, it Communication
should not be offered or attempted. However, the patient’s
When the clinical decision is that CPR should not be
individual circumstances and the most up-to-date guid-
attempted because it will not be successful, and the patient
ance must be considered carefully before such a decision
has not expressed a wish to discuss CPR, it is not necessary
is made. The responsibility for making the decision rests
or appropriate to initiate discussion with the patient.
with the most senior clinician, although they may delegate
Careful consideration should be given as to whether or not
the task to another person who is competent. Wherever
to inform the patient of the decision. Although patients
possible, a decision should be agreed with the whole
should be helped to understand the severity of their condi-
health-care team. If there is genuine doubt or disagree-
tion, whether they should be informed explicitly of a
ment a further senior clinical opinion should be sought.
DNACPR decision will depend on the individual circum-
stances. In most cases a patient should be informed, but for
CPR unlikely to be successful and likely to some, for example those who know that they are approach-
prolong or increase suffering ing the end of their life, information about interventions
When a patient is in the final stages of an incurable illness that would not be clinically successful will be unnecessarily
and death is expected within a few days, CPR is very burdensome. Others indicate by their actions and involve-
unlikely to be clinically successful. In some cases it may ment in decision making that they want detailed informa-
prolong or increase suffering and subject the patient to a tion. Therefore an assessment should be made of how
traumatic and undignified death. In these circumstances, much information the individual patient (or, if the patient
most patients want a natural death without unnecessary lacks capacity, those close to them) wants to know. If con-
interventions that most consider to be undignified. Earlier cluded that the patient does not wish to know about or
discussions with patients about their general care and treat- discuss a DNACPR decision, their agreement should be
ment aims may have addressed this issue. The Liverpool sought to share with those close to them, with carers and
Care of the Dying Pathway provides a comprehensive with others, information they may need in order to support

686
Case 4.29  Resuscitation status decision making – discussion with patient

the patient’s treatment and care. If a patient lacks capacity, successful CPR would be acceptable to the patient, should
any legal proxy and others close to the patient should be be discussed.
informed of the DNACPR decision and the reasons for it. A decision that CPR will not be attempted because the
A decision not to attempt CPR applies only to CPR. It must burdens outweigh the benefits should be made only after
be made clear to patients, people close to patients and careful consideration of all relevant factors, discussion
members of the health-care team that it does not apply to with the patient, or those close to patients who lack capac-
any other aspect of treatment. ity, and these include the factors in Box 4.27.
Some patients may wish CPR when there is only a small
What is the way forward if a patient chance of success, in spite of the risk of distressing clinical
requests CPR in situations where it will not and other outcomes. If it is the considered judgement that
CPR would not be clinically appropriate for the patient,
be successful? the patient should have accurate information about the
Neither patients nor those close to them can demand nature of possible CPR interventions, and the length of
treatment that is clinically inappropriate. If the health-care survival and level of recovery realistically expected if suc-
team believes that CPR will not re-start the heart and cessfully resuscitated. The reasons for their request should
breathing, sensitive efforts should be made to provide a be explored, with an attempt to reach agreement; for
realistic view of the procedure without causing undue example, limited CPR interventions could be agreed in
alarm. If, ultimately, the patient or those close to the some cases. When the benefits, burdens and risks are finely
patient do not accept a DNAR decision in these circum- balanced, the patient’s request will usually be the deciding
stances a second opinion should be offered. Although a factor. If, after discussion, CPR is still considered not clini-
patient does not have a legal right to demand any treat- cally appropriate, provision for it is not obliged. The
ment, including resuscitation, and the general require- reasons and any other options should be explained,
ment is for discussion rather than agreement, and although including seeking a second opinion.
it might seem medico-legally more attractive to concord Some patients may, despite potentially distressing
with such a patient’s wishes, the ethical dilemma is that it adverse effects, have specific reasons for wanting to try to
may not seem in their best interests and resources could delay death, even if this is only for a very short period of
be put to the test in the unfortunate scenario of a concur- time. If such a wish is expressed, accurate information
rent cardiac arrest elsewhere for the team. In practice, for- must be provided about the likelihood and length of sur-
tunately, patients and doctors reach the same view the vast vival that might realistically be expected, and about the
majority of the time, but if a patient explicitly requests potential risks and effects of attempted CPR.
resuscitation and the medical view is that it would not be Restriction of CPR to treatment of ‘shockable’ rhythms
of benefit then many doctors would choose to accord with only is advocated by some clinicians in some specific clini-
that patient’s wishes and make a sensible judgement about cal settings. Any such decision must be thought through
how far to take that resuscitation attempt at the time it clearly on the basis of the balance of risks, burdens and
happens, limiting any burdens and harms that may arise benefits to the individual patient and should be discussed
from resuscitation. with the patient (or those close to patients who lack
capacity).
What is the approach to decision making
when CPR may be successful?
These decisions are made by weighing up the potential
benefits and burdens resulting from CPR.
Box 4.27  Factors to consider in weighing up the
Patients who have capacity benefits and burdens of cardiopulmonary
If CPR may be successful, the benefits of prolonging life resuscitation (CPR) where CPR may be successful
must be weighed against the potential burdens and risks.
• The likely clinical outcome, including the likelihood of
This is not solely a clinical decision. Discussion must be
successfully re-starting the patient’s heart and breathing for a
sensitive and, if the patient is prepared to engage, informa- sustained period, and the level of recovery that can realistically
tive of the burdens and risks, including likely clinical and be expected after successful CPR
other outcomes if CPR is successful. This should include • The patient’s known or ascertainable wishes, including
sensitive explanation of the extent to which other inten- information about previously expressed views, feelings, beliefs
sive treatments and procedures may not be seen as clini- and values
cally appropriate after successful CPR. For example, in • The patient’s human rights, including the right to life and the
some cases, prolonged support for multi-organ failure in right to be free from degrading treatment
an intensive care unit may not be clinically appropriate • The likelihood of the patient experiencing severe unmanageable
even though the patient’s heart has been re-started. Any pain or suffering
doubts the health-care team may have about whether the • The level of awareness the patient may have of their existence
burdens and risks of CPR would outweigh the benefits, and surroundings
including whether the level of recovery expected after

687
Station |4| Communication skills and ethics

Patients who lack capacity Scenario


The approaches are discussed in Case 4.30 (Resuscitation
status decision making – discussion with relative). Mr Chris Ryder, aged 55 years
Mr Ryder has advanced Huntington’s disease. For the last year
he has required full-time care in a nursing unit with specialised
What is the ethical and legal stance on psychiatric skills because of behavioural disturbance. He is not
refusal of CPR by adults with capacity? competent to make decisions for himself. He has been admitted
with aspiration pneumonia and has worsening acute renal failure
Adults with capacity have the right to refuse any medical
due to sepsis. Haemodialysis may be the only way to manage his
treatment, even if that refusal results in their death. Where renal failure in the short term, but in the context of his comorbidity
health-care teams believe that CPR may be successful in you have to decide if this is appropriate. His sister is here to talk
re-starting a patient’s heart and breathing for a sustained with you. She is not genetically affected.
period, discussion should take place with the patient to Your task is to discuss the appropriateness of haemodialysis
determine their views and wishes regarding CPR. If a and intensive care, along with resuscitation status, with his sister.
patient decides that they do not wish to have CPR
attempted, this should be documented carefully and steps Your examiners will warn you when 12 minutes have
should be taken to ensure that this is communicated to elapsed. You have 14 minutes to communicate with the
those who need to know. Patients are not obliged to justify patient / subject followed by 1 minute of reflection. There
their decisions, but health professionals usually wish to will then follow 5 minutes of discussion with the examin-
discuss the implications of a refusal of treatment with ers. Do not take the history again except for details that
patients in order to ensure that the decision is based on will help in your discussion with the patient / subject. You
accurate information and not on any misunderstanding. are not required to examine the patient / subject.

Advance decisions refusing CPR Patient / subject information


CPR must not be attempted if it is contrary to valid and
You are the sister of Mr Chris Ryder, a 55-year-old man with
applicable advance decisions made when patients had
advanced Huntington’s disease. For the last year he has
capacity (Case 4.28 Advance decision making).
required full-time care in a nursing unit with specialised
psychiatric skills on account of behavioural disturbance.
When might it be appropriate to He is not competent to make decisions for himself. You
temporarily suspend a DNACPR decision? have lived with the knowledge of his inevitable decline for
Uncommonly, some patients for whom a DNACPR deci- many years and anticipated situations from which he may
sion has been established may develop cardiac or respira- not recover. You do not carry any risk of developing the
tory arrest from a readily reversible cause such as choking, disease yourself. He has now been admitted with aspira-
induction of anaesthesia, anaphylaxis or blocked trache- tion pneumonia and has severe renal failure caused by his
ostomy tube. In such situations CPR would be appropri- infection. Your brother did not make a written advance
ate, while the reversible cause is treated, unless the patient directive about his future wishes, but your feeling is that he
has specifically refused intervention in these circum- would not wish to be kept alive if there were no possibility
stances. It may also be appropriate to temporarily suspend of long-term cure of the underlying cause. You are a trained
a DNACPR decision during some procedures if the proce- nurse and feel that intensive-care transfer, if it came to that,
dure itself could precipitate a cardiopulmonary arrest – for would not be in his best interests.
example, cardiac catheterisation, pacemaker insertion, or
surgical operations. If a patient wishes an advance decision How to approach the case
refusing CPR to remain valid during a procedure or treat- Communication skills (conduct of interview,
ment that significantly increases the risks of the procedure
exploration and problem negotiation) and
it would be reasonable not to proceed.
ethics and law
1.  Introduction
Introduce yourself and establish that she is Mr Ryder’s
CASE 4.30  RESUSCITATION STATUS sister.
DECISION MAKING – DISCUSSION
2.  Establish what the relative knows
WITH RELATIVE
Firstly, establish a rapport with a relative you have not met
before, establishing what she knows, and what she thinks,
Candidate information before moving to a gentle explanation of where you are in
terms of her brother’s medical condition and possible
Role ways forward:
You are a doctor in the acute medical unit. What have you been told so far about his condition since
Please read this summary. admission to hospital?

688
Case 4.30  Resuscitation status decision making – discussion with relative

3.  Seek further information about the patient


Box 4.28  Cardiopulmonary resuscitation
Relatives are highly valuable sources of information. She (CPR) decision making in adults who
may know intimately the duration and nature of his
lack capacity
decline, his care needs and, in a chronic condition such as
this, may well have anticipated what his wishes might be Advance decision or directive
in the event of a precipitous acute episode of illness, and A valid and applicable advance decision or directive should be
established her own views. respected (Case 4.28).
4.  Do not forget empathy Legal proxy
No matter how pragmatic she may seem about his illness, If a patient has a legal proxy, this person must be consulted about
remember that even expected declines often still strike CPR decisions. The rules governing the role and validity of the legal
harshly. Remain empathetic and pace your explanations proxy are exactly the same for CPR decision making as any other
slowly and carefully, and in words relatives will treatment decision and are detailed in Case 4.24 (Consent and
capacity).
understand.
Those close to the patient
5.  Explain the situation Where a patient has not made an advance decision or appointed
Explain that his pneumonia is in itself treatable, but that it a legal proxy, the treatment decision rests with the most senior
has provoked kidney failure that may or may not be recov- clinician in charge of the patient’s care. Where CPR may re-start
erable. Explain that pneumonia is a more likely event in the patient’s heart and breathing for a sustained period, the
patients with advancing chronic conditions like Hunting- decision as to whether CPR is appropriate must be made on the
ton’s disease but that the kidney problem is now the most basis of the patient’s best interests. In order to assess best
interests, the views of those close to the patient should be
threatening. Explain that it is being managed with careful
sought, unless this is impossible, to determine any previously
fluid replacement and monitoring of blood chemistry. expressed wishes and what level or chance of recovery the patient
would be likely to consider of benefit. In reaching a decision
6.  Approach the issue of more invasive in England and Wales the Mental Capacity Act requires that
interventions best-interests decisions must include seeking the views of anyone
The next, very difficult question that I know you know I’m named by the patient as someone to be consulted, or anyone
coming to is, if things were to deteriorate sharply, what we engaged in caring for the person or interested in the patient’s
might do. We are treating as energetically, as vigorously, as welfare. Under the Act, all health-care personnel must act in
we can, with fluids and antibiotics and by correcting the the best interests of a patient who lacks capacity. In these
abnormalities in the blood which arise from the kidney failure. circumstances, it should be made clear to those close to the patient
And we hope that things may improve. But he is at serious that their role is not to take decisions on behalf of the patient, but
risk, and there is no getting away from that. to help the health-care team to make an appropriate decision in
the patient’s best interests.
7.  Ensure that your explanation does not conflict Independent mental capacity  
with any ideas, concerns and expectations advocate (IMCA)
Most importantly, ensure that she understands that his life In England and Wales the Mental Capacity Act requires an IMCA
is threatened and if she makes it clear that she does and that to be consulted about all decisions about ‘serious medical
she does not think that he would wish for more complex treatment’ where patients lack capacity and have nobody to
interventions it is appropriate to reinforce her views: speak on their behalf. The definition of serious medical treatment
includes circumstances where ‘what is proposed would be likely to
I would have to agree. And my strong feeling and advice to you involve serious consequence for the patient’. It can be argued that
is that if there were further deterioration / a disaster, I would a decision not to attempt CPR because it will not work will not
not be inclined to go down the route of dialysis or ventilators have ‘serious consequences’ for the patient, because the patient
and intensive care … because I just do not think he would will die with or without attempting CPR. For this reason, an IMCA
come through it. And the trouble is that even if he were to does not need to be called when it is clear to the medical team
come through it all, without a clear way forward beyond all of that CPR would not be successful. Where there is genuine doubt
this, without seeing him getting better, I think it would be about whether or not CPR would have a realistic chance of
putting him through unnecessary distress. success, or if a DNACPR decision is being considered on the
balance of benefits and burdens, in order to comply with the law
8.  Many relatives worry that the decision is an IMCA must be involved. If a DNACPR decision is needed when
theirs and theirs alone an IMCA is not available (for example at night or at a weekend),
the decision should be made, recorded in the health record and
It may be important to highlight that the decision is ulti- discussed with an IMCA at the first opportunity. An IMCA does
mately yours, but that her views of what his wishes might not have the power to make a decision about CPR but must be
be are highly valuable. consulted as part of the determination of the patient’s best
interests.
9.  Confirm understanding and invite questions
Make it clear that you are happy to answer questions now
or later.

689
Station |4| Communication skills and ethics

10.  Conclude with assurances Patient / subject information


Always conclude with an assurance that if a decision is Mrs Angela Elliot is a 62-year-old woman who has
made not to resuscitate, you and your team will continue advanced multiple sclerosis. She is fully dependent upon
to do all that you can to actively treat his condition and carers for transfer from her wheelchair to her bed, and
to relieve symptoms. Make it clear, where appropriate, that unable to feed herself. She can swallow food, albeit very
a decision can be reviewed and altered at any time. carefully prepared and of the right consistency, but swal-
But we will do all that we can, keeping a very close eye and lowing is further impaired each time she is unwell. Percu-
treating all that is treatable here on the ward. And we are taneous gastrostomy feeding has been considered and is
here, if there are any questions at any time. acceptable to her. Unfortunately, she has been readmitted
to hospital with her fourth episode of pneumonia in 
Discussion 12 months. On this occasion admission to the intensive
therapy unit (ITU) was considered, but she ‘made it
What do you know about cardiopulmonary through the night’ with oxygen, chest physiotherapy and
resuscitation decision making for adults antibiotics. She is slowly improving, and no longer breath-
who lack capacity? less. She has great concerns about a ventilator because she
does not want to be ‘kept alive indefinitely on an artificial
The possibilities are described in Box 4.28 and more detail machine’. Her daughter died after spending 4 months in
on treating adults who lack capacity is in Case 4.24 an ITU following a road traffic accident. She has no other
(Consent and capacity). children, and is widowed. She would like to discuss
whether ITU would really benefit her in future if this
happens again. She has a fear of being ventilated only to
CASE 4.31  APPROPRIATENESS OF die at the end of months of distress.
INTENSIVE THERAPY UNIT TRANSFER
How to approach the case
Candidate information Communication skills (conduct of interview,
exploration and problem negotiation) and
Role ethics and law
You are a doctor on the medical ward. 1.  Introduction, setting and rapport
Please read this summary.
Introduce yourself and state that you are glad to see her
Scenario feeling a little better. Make it clear that you will do every-
thing you can to continue to make her feel comfortable,
and treat the chest infection.
Re: Mrs Angela Elliot, aged 62 years
Mrs Elliot has advanced multiple sclerosis. She is immobile, 2.  Acknowledge how unwell the patient has been
dependent upon carers for transfer from her wheelchair to bed,
and unable to feed herself. Bulbar function is significantly impaired
Mention that you know how unwell she has been, and
whenever she has intercurrent infection, its recovery following that, while you feel she should continue to improve on
episodes is incomplete and she can only just tolerate a strict diet. this occasion, she came very close to needing ventilatory
Percutaneous gastrostomy feeding was being considered. support.
Unfortunately, she has been readmitted to hospital with her fourth
episode of pneumonia in 12 months. On this occasion admission to 3.  Explore the patient’s concerns
the intensive therapy unit (ITU) was considered for the first time, Mention that the nursing staff have told you she had some
but because of a lack of ITU beds she was observed carefully by concerns at being treated in intensive care. Explain that
the ITU outreach team and ‘made it through the night’ with
while you are always happy to consider what seems the
high-flow oxygen, suction and antibiotics. She is slowly improving,
and no longer breathless. The nursing team have questioned best treatment at the time, it would be important to know
whether she should be considered for ITU next time she has if she has any strong feelings one way or the other about
pneumonia, as Mrs Elliot has apparently expressed concerns about any specific measures, including intensive care.
‘going on a ventilator’ and would like to speak to a doctor.
Your task is to address her concerns. 4.  Address the patient’s concerns
Patients with long-term conditions have often given
Your examiners will warn you when 12 minutes have thought to what their wishes might be in the event of hypo-
elapsed. You have 14 minutes to communicate with the thetical clinical circumstances. While some patients would
patient / subject followed by 1 minute of reflection. There ‘want everything done’, many are very clear about circum-
will then follow 5 minutes of discussion with the examin- stances they would not wish to occur, and this is very likely
ers. Do not take the history again except for details that to include being artificially ventilated when there is no
will help in your discussion with the patient / subject. You prospect of recovery (doctors would be unlikely to offer
are not required to examine the patient / subject. such a treatment in any event). The difficulty is that

690
Case 4.31  Appropriateness of intensive therapy unit transfer

medicine is seldom clear-cut, and the prospect of recovery unassailable to physicians outside it, but studies have
is often a case of best judgement rather than absolute cer- shown that the often ‘poor prognostic’ pessimism of ITU
tainty one way or the other. It is reasonable, provided the physicians is not always thoroughly founded. For example,
patient wishes it, to consider ventilation when there is a being old, being immunosuppressed or having malig-
reversible acute cause for respiratory failure such as infec- nancy have all been shown to have good outcomes with
tion, but the balance of favouring to ventilate is progres- ITU where appropriate patient selection occurs. It does,
sively tempered as worsening of the underlying condition however, seem clear that poor candidates for ITU generally
thwarts the likelihood of successful weaning from ventila- are those with a combination of severe illness, a poor level
tion, and ultimately ventilation is tilted out of favour. of pre-morbid function (e.g. chair- or bed-bound) and
Patients may choose not to be ventilated even when the multi-organ failure. However, it can be difficult to make
balance seems to be in their favour, provided this is an physiological and ethical judgements at the same time and
informed choice. Occasionally, patients will feel guilty early, collaborative decision making between physicians
about refusing an option that may prolong their life, espe- and intensivists is to be encouraged.
cially if they have close relatives. Ventilation may carry the
prospect of recovery from future infective exacerbations, Is invasive ventilation contraindicated
but the patient will judge if the extent of that recovery is in COPD?
something she wishes to be brought back to. ITU would, of
course, decline admission when recovery does not seem Invasive ventilation is more likely to be appropriate where
likely. Increasingly, non-invasive ventilation has been an there is a reversible cause and an acceptable quality of life
alternative to invasive ventilation and can be applied or habitual level of activity. Each case must be assessed
outside ITU, but its best evidence is in the setting of chronic individually. COPD is not a blanket reason not to ventilate
obstructive pulmonary disease (COPD) exacerbations. because of fears about weaning. You should get an idea,
for example, of how much oxygen is required at home or
5.  Discuss resuscitation status what is meant exactly by ‘housebound’. Invasive ventila-
It is often, but not inevitably, appropriate to discuss resus- tion may be appropriate for a remediable acute cause, if
citation status. Such discussion is usually a logical and there is no significant other organ failure, for a first episode
natural extension if you have reached a decision about ITU of respiratory failure or if the patient has made an informed
being inappropriate, but should be discussed sensitively. wish for it to be attempted, whereas it would be inappro-
priate for a patient with end-stage COPD and a high pre-
6.  Ensure that the patient is fully informed morbid arterial pCO2.
If a decision is to be made not to admit to ITU in future,
especially if there is still a prospect of ITU benefit out- How does age affect ITU outcome?
weighing risk, ensure that this is made with adequate Around 25% of ITU admissions are of patients over 75
understanding of the alternatives and the consequences, years of age, and this is likely to increase. Surgical patients
and formed without coercion or undue influence. are more likely to need admission than medical, usually
related to sepsis and other postoperative complications.
7.  Agree a plan Around 80% of patients over 80 years of age who are
Agree to respect her informed choice. ventilated with sepsis do not survive. Other adverse factors
include poor cognition, decreased consciousness, recent
8.  Confirm understanding stroke, limited activities of daily living, poor nutrition and
Ensure that she is clear about the course of action and be unplanned admission. Despite this, age is a risk factor for
prepared to explore any other concerns. poor outcome (7%) by only around one-tenth that of the
underlying physiology (73%) in the acute physiology and
9.  Invite questions chronic health evaluation (APACHE) III scoring system.
Ask if there is anything she would like clarified. Physiological problems should be much bigger determi-
nants of admission to ITU than age; single-organ reversible
10.  Conclude with assurances disease even in very old people who were previously
Always conclude with an assurance that, if a decision is healthy (another important determinant of outcome) cer-
made not to admit to ITU in future, you and your team tainly deserves consideration of ITU involvement.
will continue to do all that you can to actively treat her
condition and to relieve symptoms. What complications can follow a spell
in ITU?
Discussion Post-critical illness morbidity is increasingly recognised,
Which patients fare better in ITU, particularly in centres operating post-ITU clinics, and this
includes neurocognitive dysfunction, neuromuscular dys-
and which worse? function, disordered taste, a wide variety of neuropathies
The role of ITU is largely in support whilst primary therapy including peripheral and entrapment neuropathy, trache-
works. There is often a sense of ITU being inaccessible or ostomy site complications, and depression.

691
Station |4| Communication skills and ethics

How to approach the case


CASE 4.32  WITHHOLDING AND
WITHDRAWING LIFE-PROLONGING Communication skills (conduct of interview,
TREATMENTS – ANTIBIOTICS AND exploration and problem negotiation) and
DRUGS ethics and law
1.  Introduction
Candidate information Introduce yourself, establish his identity and acknowledge
that it is a difficult time.
Role
You are a doctor on an elderly care ward.
2.  Acknowledge the value of the relative
Please read this summary. As his wife is not able to speak for herself, his presence is
highly valuable in determining what her wishes might have
Scenario been. Acknowledging him as her closest relative, it may be
helpful to ask if there are other family members nearby
who might also wish to be involved in discussions.
Re: Mrs Eliza Bennett, aged 83 years
Mrs Eliza Bennett has been on your ward for 3 weeks with
unresolving pneumonia. She has significant comorbidity. She has
3.  Reassure the relative that any decisions  
had type 2 diabetes for many years, latterly requiring insulin, with will be carefully considered and that the ‘burden
associated chronic kidney disease and advanced diabetic of responsibility’ for difficult decisions will not
maculopathy. She was admitted with right lower lobe pneumonia,
which has failed to respond to two courses of intravenous
rest with him
antibiotics. The microbiologist has suggested a third regimen, but Reassure him that decisions will consider what is best for
the possibility of non-infective underlying causes, including cancer, his wife, and that his views will be very helpful. Explain
is there, although not established, and she has been too unwell for that you are very concerned that treatments do not seem
complex investigations. She has steadily declined, with a falling to be working as you had hoped, and that you now must
serum albumin and increasing peripheral oedema. She is now too contemplate what are reasonable interventions and what
drowsy to engage in discussion. Intravenous access is now not
may not be reasonable interventions and not in his wife’s
possible peripherally, and your house officer called you to consider
a central venous catheter. Mr Bennett is visiting his wife. best interests. Explain that decisions will very much take
Your task is to discuss her treatment options with Mr Bennett, account of any views he has but that decisions will be
her husband. made together. Remember that your consultant is there to
help with difficult decisions such as this, and you should
make difficult decisions with more senior team members.
Your examiners will warn you when 12 minutes have
elapsed. You have 14 minutes to communicate with the 4.  Explain the problem
patient / subject followed by 1 minute of reflection. There
This should start with a summary of the background,
will then follow 5 minutes of discussion with the examin-
explaining that his wife’s pneumonia has not responded.
ers. Do not take the history again except for details that
Acknowledge that there is uncertainty as to why this is so.
will help in your discussion with the patient / subject. You
It is likely that her diabetes (hyperglycaemia, small-vessel
are not required to examine the patient / subject.
disease, renal failure) is impairing the fight against infec-
tion. It may be that there is an underlying reason (includ-
Patient / subject information ing a growth) predisposing to infection, but his wife has
Your wife, Mrs Eliza Bennett, is an 83-year-old woman been too unwell for more complicated investigations.
who has been on the ward for 3 weeks with pneumonia Explain that, as time goes on, the prospects of recovery
that is steadily getting worse. She has been unwell, in your from the pneumonia diminish, particularly as other 
view, for the last year. She has longstanding diabetes, things start to go wrong (drowsiness, hypoalbuminaemia,
requiring insulin as well as tablets, with associated kidney oedema). Explain that all you have been able to do recently
problems and visual impairment. She used to be very is continue with antibiotics, attempt to correct any abnor-
active and has been very depressed for the last year, virtu- malities in the blood chemistry and watch and wait, trying
ally housebound by weakness in her legs and poor vision. to ensure that she is comfortable. Explain that the new
To you, and you think to her, this is the last straw. She is problem is that it is no longer possible to secure peripheral
not getting better, her legs are swelling up and you have intravenous access for delivery of antibiotics.
seen her becoming more confused and drowsy since
admission. She has always said that she would not want 5.  Explain your view
to go on if her diabetes had got the better of her and would Explain that the next step could be to consider intravenous
not want to be dependent upon carers or nurses. The access via a catheter inserted into a larger vein in the neck,
doctors are concerned that she has not responded to anti- but that this is more invasive. Acknowledge that this is not
biotics and they appear to be running out of options. without complications, and not always comfortable to

692
Case 4.33  Withholding and withdrawing life-prolonging treatments

insert, but is an effective way of ensuring that treatment Must you provide treatment to a patient
continues, although you cannot say if treatment will work. who demands it but which you do not think
It is unlikely that she is able to take tablets at the moment,
but if she were to improve and show the ability to take
is in that patient’s best interests?
things orally, then oral antibiotics could be considered. If a patient asks for a treatment that the doctor considers not
clinically appropriate, the doctor is not obliged to provide
6.  Explore what the relative feels the patient it, but should offer to arrange for a second opinion.
would have wanted How might you be guided in making
He may request, based on his knowledge of his wife’s decisions about limitation of treatment
values and views, that this would be a step too far.
for patients without capacity?
7.  Explore any concerns the relative may have Any valid advance refusal of treatment must be respected.
A common concern is that the patient may suffer or be in Without this, assessment of the benefits, burdens and risks 
pain. When a patient is not responding to treatment, it is and overall acceptability of treatment must be made on a
crucial to reassure relatives that one thing you can do is patient’s behalf by the doctor, taking into account what is
ensure their comfort. known of the patient and information from those closest to the
patient. Best-interests decisions and the use of valid advance
8.  Consider and justify, or plan to change, any decisions or legal proxy or information from third parties are
described in more detail in Case 4.24 (Consent and capacity).
apparent discrepancies such as delivering some Prolonging life will usually be in a patient’s best inter-
medications but not others ests if treatment is not excessively burdensome or dispro-
If a decision is made to withdraw antibiotics, the issue  portionate to expected benefits.
of continuing insulin may need to be discussed. If she But not continuing or starting treatment is in a patient’s
cannot take tablets, and is already insulin dependent, then best interests when there is no net benefit. Life has a
withdrawal of insulin could lead to rapid spiralling of natural end and doctors should not strive to prolong the
hyperglycaemia and possible death from hyperglycaemic dying process. Case law suggests that life-prolonging treat-
coma. This could be avoided with subcutaneous insulin ment can lawfully be withheld or withdrawn from a
and this may still be a reasonable intervention while patient who lacks capacity when starting or continuing
nature confirms its likely course without antibiotics. If and treatment is not in their best interests, and that there is no
when it becomes certain that she is dying then it may be obligation to give treatment that is futile or burdensome.
appropriate to withdraw insulin in favour of purely symp- When reaching a view on whether a particular treatment
tomatic palliative measures. would be more burdensome than beneficial, assessments
of the likely quality of life for the patient with or without
9.  Confirm understanding and invite questions that treatment may be one of the appropriate considera-
Make it clear that you are happy to answer further ques- tions. Where it is decided that treatment is not in a patient’s
tions now or at any time. best interests, there is no ethical or legal reason to provide
it and thus no need to distinguish withdrawal of treatment
10.  Give strong reassurance about continuing from not starting treatment.
Where a patient lacks capacity and there is uncertainty
with care about appropriateness of treatment, treatment that may be
Make it clear that the team will continue to care for his of some benefit should be started until clearer assessment
wife with very close attention to what is reasonably treat- is made. This is particularly important in emergencies
able and to alleviation of symptoms. Tell him that you are where there may be doubt about the severity of a condi-
happy to answer questions at any time. tion or the benefit of a treatment.

Discussion CASE 4.33  WITHHOLDING AND


Do you know of any guidance on WITHDRAWING LIFE-PROLONGING
withholding and withdrawing life- TREATMENTS – CLINICALLY
prolonging treatments? ASSISTED NUTRITION AND
The 2010 General Medical Council (GMC) guideline on HYDRATION
end-of-life care supersedes previous guidance on with-
holding and withdrawing life-prolonging treatment and
covers greater ground, providing decision-making models  Candidate information
(Case 4.27 End of life and palliative care) for patients 
with or without capacity in all areas of end-of-life care Role
including withholding and withdrawing life-prolonging You are a doctor on the stroke unit.
treatments. Please read this summary.

693
Station |4| Communication skills and ethics

Scenario father. Although relatives often wish strong account be


taken of their views, they may also need to be reassured
that ultimately the ‘burden of responsibility’ for decisions
Re: Mr Harold Hunt, aged 86 years
Mr Hunt was admitted to your ward 3 days ago following a left
rests with the medical team. She will wish to express the
total anterior circulation stroke. He remains unconscious and the perceived values and preferences of her father but may well
consensus opinion of all members of the health-care team is that feel uncomfortable if she feels that a life and death deci-
his prognosis is very poor. His daughter feels that he should be ‘left sion about a family member is hers.
in peace’. He is on ‘a drip’ but one of the nurses mentioned the
possibility of a ‘feeding tube’ to her. She is now not sure if he is 4.  Explain the reasons for and against clinically
being starved.
Your task is to discuss the issue of clinically assisted nutrition assisted nutrition and hydration
and hydration with his daughter. Explain that where a patient cannot take fluids or food
orally, an assessment is always made of the requirements
Your examiners will warn you when 12 minutes have for hydration and nutrition, and the appropriateness of
elapsed. You have 14 minutes to communicate with the instituting these. Explain that in this situation intravenous
patient / subject followed by 1 minute of reflection. There fluids were started initially because it could not be deter-
will then follow 5 minutes of discussion with the examin- mined whether her father would recover and, if so, how
ers. Do not take the history again except for details that long this would take, but that supportive fluids are neces-
will help in your discussion with the patient / subject. You sary early on if there is to be some recovery. Explain that
are not required to examine the patient / subject. artificial nutrition is less immediate, but should be started
via a tube passed through the nose directly into the
Patient / subject information stomach (nasogastric tube) within the first few days in
Mr Harold Hunt, your father, is an 86-year-old man who patients showing signs of recovery. Acknowledge that it is
was admitted to the ward 3 days ago following a very large a very sensitive issue and not one with absolutely right or
stroke. He remains unconscious and the consensus opinion wrong answers, but that the generally accepted view is not
of all members of the health-care team is that his prognosis to institute artificial nutrition unless a patient is showing
is very poor. You feel that he should be ‘left in peace’. He had signs of recovery. Specifically, a patient should be con-
a very active life in the army, but since being widowed 3 scious and able to sit up and ideally aware of the reasons
years ago became rather withdrawn and always said he for the tube.
would want ‘to have all of his faculties’. He has been on ‘a
drip’ since admission but one of the nurses mentioned the 5.  Explain your view
possibility of a ‘feeding tube’. You do not see the value of It is often best, particularly where you feel relatives are
feeding your father if he is going to die, but are worried that showing agreement, to suggest what you think might be
if he is being starved it may be uncomfortable for him. You appropriate, provided this is not dogmatic and you dem-
wonder if the drip should continue, and if so whether it onstrate willingness to be fluid in your decisions depend-
might be delaying an inevitable death. ing upon feedback. You might consider artificial nutrition
if there were a spontaneous change for the better, but now
How to approach the case it is entirely reasonable to watch and wait.
Communication skills (conduct of interview, 6.  Explore what the relative feels the patient
exploration and problem negotiation) and would have wanted
ethics and law She will probably agree with withholding nutrition.
1.  Introduction
Introduce yourself, establish her identity and acknowledge 7.  Explore any concerns the relative may have
that it is a difficult time. She may need to be reassured that there is no evidence
that withholding nutrition causes distress by ‘starvation’.
2.  Acknowledge the value of the relative She may not see the logic of maintaining intravenous
As her father is not able to speak for himself, her presence fluids when nutrition is being withheld.
is highly valuable in determining what his wishes might
have been. It is always useful to establish if there are other 8.  Consider and justify or plan to change any
people who might wish to be involved in discussions. apparent discrepancies such as delivering
3.  Reassure the relative that any decisions   hydration but withholding nutrition
will be carefully considered and that the   The British Medical Association (BMA) and General
Medical Council (GMC) have stated that withdrawing 
‘burden of responsibility’ for difficult decisions
a life-prolonging treatment may not be ethically 
will not rest with her distinguishable from withholding it, although the former
Reassure her that decisions will always consider her father’s can seem more difficult. Whilst intravenous or subcutane-
best interests first, aided by her better knowledge of her ous fluids could be withdrawn, there are reasons

694
Case 4.33  Withholding and withdrawing life-prolonging treatments

for continuing their provision, although these cannot be How might you decide if clinically assisted
evidence based. In this case 3 days may seem too soon to nutrition or hydration should be provided?
withdraw a relatively non-invasive treatment, especially if
access for fluids is not causing distress. Unlike nutrition, Providing nutrition and hydration by tube or drip may
fluids pose little risk. Nasogastric feeding may provoke provide symptom relief, or prolong or improve quality of
aspiration and may be uncomfortable and is not suitable life, but may also present problems, and decision-making
for unconscious or agitated patients or for patients who models should be used to help determine whether these
cannot sit up. Percutaneous endoscopic gastrostomy should be provided (Boxes 4.23 and 4.29).
(PEG) feeding is not suitable unless patients demonstrate
potential for longer term survival. Hydration may avert
Should clinically assisted nutrition
distress, although there is no convincing evidence for this or hydration should be provided in
and indeed the Liverpool Care Pathway for palliation spe- advanced dementia?
cifically excludes hydration and nutrition. An increasingly common scenario is advanced dementia,
when poor oral intake is a result of irreversible brain failure
9.  Confirm understanding and invite questions resulting in loss of motivation for intake with apathy, and
Make it clear that you are happy to answer further ques- sometimes to an extent a weak swallow. Here it is usually
tions now or at any time. appropriate to exclude reversible factors, including depres-
sion, and encourage oral intake with all possible means,
10.  Give strong reassurance about continuing which can be time intensive. If clear that the patient is
declining primarily because of the dementia and that the
with care
poor oral intake is a consequence of this, the benefits of
Explain that needs and symptoms may change, and that clinically assisted hydration and nutrition become less and
he will be carefully assessed regularly. less persuasive and are not without considerable burdens
and risks. Ultimately, the ethical distinction between
Discussion allowing a patient to decline or die primarily from dehy-
dration (usually unethical) or accepting decline and death
When might the question of clinically primarily from an underlying irreversible disease in which
assisted nutrition or hydration be dehydration is a secondary inevitable factor (which may be
considered? ethical) is an important one. Furthermore, in advanced
dementia the risks of artificial feeding are higher than the
The offer of food and drink by mouth is part of basic care benefits, invariably, and the point is that food and fluid are
(as is the offer of washing and pain relief) and must always not withheld but always offered orally.
be offered to patients able to swallow without serious risk
of choking or aspirating. Food and drink can be refused A 68-year-old man with advanced Lewy
by patients at the time it is offered, but an advance refusal body dementia is admitted with an unsafe
of food and drink has no force. If a patient is not receiving swallow and multiple failed nasogastric
adequate nutrition or hydration by mouth, even with
support, an assessment of their condition and their indi-
feeding attempts because he pulls out the
vidual requirements must be made. Their needs for nutri- tubes. He is receiving intravenous fluid. His
tion and hydration must be considered separately with essential medical therapy is warfarin for a
consideration of what forms of clinically assisted nutrition metallic heart valve. What are the options?
or hydration may be required.
Percutaneous gastrostomy feeding would probably not be
appropriate in this situation. Blood tests are appropriate to
What is meant by clinically assisted guide a life-saving treatment (warfarin) until a definitive
nutrition or hydration? decision for no further active management has been made.
Clinically assisted nutrition includes intravenous feeding, The option to comfort feed may seem the most appropriate
and feeding by nasogastric tube and by PEG and radio­ to medical staff, but his family’s views on his perceived
logically inserted gastrostomy feeding tubes through the wishes may help. Withdrawal of hydration before discus-
abdominal wall. All these means of providing nutrition sion would not seem correct immediately, particularly
also provide fluids necessary to keep patients hydrated. while his family are coming to terms with a difficult situa-
Clinically assisted hydration can also be provided by intra- tion. His family are likely to be strongly in agreement that
venous or subcutaneous infusion of fluids through a ‘drip’. no further attempts should be made to feed via a nasogas-
The terms ‘clinically assisted nutrition’ and ‘clinically tric tube. It may be decided that if no future can be envis-
assisted hydration’ do not refer to help given to patients aged beyond this admission to hospital, no further
to eat or drink, for example by spoon feeding – these are intravenous or subcutaneous access for hydration would be
generally considered part of nursing care. The terms are appropriate once the current cannula ‘tissues’. A decision
replacing the term ‘artificial hydration’ and ‘artificial should then also be made as to whether or not to give anti-
nutrition’. biotics in the event of aspiration. Assurance of preservation

695
Station |4| Communication skills and ethics

Box 4.29  Deciding if clinically assisted nutrition or hydration should be provided

If clinically assisted nutrition or hydration is necessary to keep a clinically assisted nutrition or hydration would be of overall benefit
patient alive, the duty of care will normally require the doctor to should be made using the decision model in Case 4.27 (End of life
provide it, if a patient with capacity wishes to receive it. But the and palliative care) with the same clinical considerations as for
evidence about the benefits, burdens and risks of these techniques patients who have capacity.
as patients approach the end of life is not straightforward and
clear. This can lead to concerns that patients who are unconscious Adult patients who lack capacity and are not expected to
or semi-conscious may be experiencing distressing symptoms and die within hours or days
complications, or otherwise be suffering either because their needs Clinically assisted nutrition or hydration must be provided if it
for nutrition or hydration are not being met or because attempts to would be of overall benefit, taking into account the patient’s beliefs
meet their perceived needs for nutrition or hydration may be and values, any previous request for nutrition or hydration by tube
causing them avoidable suffering. Clinically assisted nutrition or or drip and any other views he or she has previously expressed
hydration may be withheld or withdrawn if the patient does not about care. The patient’s request must be given weight and, when
wish to receive it, or if the patient is dying and the care goals the benefits, burdens and risks are finely balanced, will usually be
change to palliative care and relief of suffering, or if the patient the deciding factor. If judged that the provision of clinically assisted
lacks capacity to decide and it is considered that providing clinically nutrition or hydration would not be of overall benefit to the patient,
assisted nutrition or hydration would not be in their best interests. it may be concluded that treatment should not be started or should
Nutrition and hydration provided by tube or drip are regarded be withdrawn. This view should be explained to the patient, if
in law as medical treatment and should be treated in the same way appropriate, and those close to the patient, and the patient’s
as other medical interventions. Nonetheless, some people see interests must have been thoroughly considered including where
nutrition and hydration, whether taken orally or by tube or drip, as appropriate steps to get a second opinion from a senior clinician
part of basic nurture for the patient that should almost always be with experience of the condition but not directly involved in the
provided. For this reason it is especially important to listen to and patient’s care. If consensus is reached that clinically assisted
consider the views of the patient and of those close to them nutrition or hydration would not be of overall benefit treatment is
(including their cultural and religious views) and explain the issues, withdrawn or not started, and the patient must be kept comfortable
including the benefits, burdens and risks of providing clinically and monitored for any change with preparedness to reassess the
assisted nutrition and hydration. All should understand that when benefits, burdens and risks of providing clinically assisted nutrition
clinically assisted nutrition or hydration would be of overall benefit or hydration in light of changes. If clinically assisted nutrition or
it will always be offered, and that if a decision is taken not to hydration is started or re-instated after a later assessment, and it is
provide clinically assisted nutrition or hydration the patient will subsequently concluded that it would not be of overall benefit to
continue to receive high-quality care, with any symptoms continue a further second opinion should be sought.
addressed.
Adult patients who lack capacity and are expected to die
Patients who have capacity within hours or days
If it is considered that a patient is not receiving adequate nutrition
If a patient is expected to die within hours or days, and it is
or hydration by mouth, the decision model in Case 4.27 should be
considered that the burdens of providing clinically assisted nutrition
followed. Nutrition and hydration should be considered separately
or hydration outweigh the benefits they are likely to bring, it will not
and considered clinically appropriate because, for example, they
usually be appropriate to start or continue treatment (it is important
would provide symptom relief or are likely to prolong life. Benefits,
to be aware that not eating and drinking is often a part of the dying
burdens and risks should be explained. If clinically assisted nutrition
process and should be distinguished from dying as a result of not
or hydration is not considered clinically appropriate the patient’s
receiving food or fluid). If a patient has previously requested that
condition should be monitored and the benefits, burdens and risks
nutrition or hydration be provided until death, or those close to the
reassessed as the condition changes. If a patient asks for nutrition
patient are sure that this is what the patient wanted, the patient’s
or hydration by tube or drip, the reasons for their request should be
wishes must be given weight and, when the benefits, burdens and
explored, giving weight to the patient’s wishes and values. When
risks are finely balanced, will usually be the deciding factor.
the benefits, burdens and risks are finely balanced, the patient’s
request will usually be the deciding factor. However, if after Patients in a persistent vegetative state (PVS) or
discussion the treatment is still not considered clinically appropriate, similar condition
it does not have to be provided, but the reasons and other options
should be explained, including the option to seek a second opinion. If considering withdrawing nutrition or hydration from a patient in a
PVS or a condition closely resembling PVS, the courts in England,
Adult patients who lack capacity Wales and Northern Ireland currently require that they be
If a patient lacks capacity and cannot eat or drink enough to meet approached for a ruling. The courts in Scotland have not specified
their nutrition or hydration needs, assessment of whether providing such a requirement, but legal advice should be sought.

of his dignity should be given, with a shift in care to comfort He shows signs of improvement
and suppression of distress. Warfarin should be withdrawn with comfort feeding but has
as blood tests cannot be obtained, and the possibility of choking episodes with comfort feeding.
sudden valve obstruction accepted as a potential mode of
death unless he is not likely to die from other causes quickly
What would you do now?
in which case subcutaneous heparin could be administered There are two, at first glance conflicting, options but each
after discussion with his family. one depends upon the type of improvement. If he is

696
Case 4.34  Percutaneous endoscopic gastrostomy feeding

showing signs of returning to his pre-admission self, then Patient / subject information


it may well be appropriate to revise decisions, and this
could include stopping comfort feeding, administering Mr James Hawkins, your husband, is an 82-year-old man
artificial hydration and considering percutaneous gastros- admitted to hospital a week ago with aspiration pneumo-
tomy feeding, although there are enormous concerns nia. On admission the prognosis seemed poor and 
about the risk–benefit ratio of the latter in dementia. If, resuscitation in the event of cardiac or respiratory arrest
however, he shows the day-to-day minor undulation that was not considered appropriate. He has epilepsy (on 
is true of many hospital patients, then it is vital for his treatment since childhood) and Parkinson’s disease (on
family not to be given false hope or to burden the patient treatment for 8 years). A year ago he had a stroke after
or family with swinging decisions in the context of an which he spent 6 weeks in a rehabilitation hospital before
inevitable end. returning home. You are his main carer, and he requires
help with washing, dressing, meals and moving around
the house. He is confined to a wheelchair much of the
time, but sometimes can walk with a stick. Following anti-
CASE 4.34  PERCUTANEOUS biotic treatment he has recovered somewhat, but cannot
ENDOSCOPIC GASTROSTOMY swallow. The speech and language therapist believes he
might never safely swallow again. Unfortunately, he has
FEEDING not tolerated numerous attempts to pass a nasogastric
feeding tube and the speech and language therapist has
Candidate information suggested that a percutaneous endoscopic gastrostomy
(PEG) feeding tube would enable him to receive nutrition.
Role Unfortunately, he is unable to participate in such a discus-
sion because he has been too cognitively impaired for the
You are a doctor on the medical ward.
last year. You fear that he will ultimately starve without a
Please read this summary.
tube and will not receive his usual medications, and wish
to discuss the issues with the doctor.
Scenario
How to approach the case
Re: Mr James Hawkins, aged 82 years
Mr Hawkins was admitted to your ward a week ago with Communication skills (conduct of interview,
aspiration pneumonia. On admission the prognosis seemed poor exploration and problem negotiation) and
and resuscitation in the event of cardiac or respiratory arrest was
not considered appropriate. He has idiopathic epilepsy (since
ethics and law
childhood, and currently treated with phenytoin, carbamazepine 1.  Introduction
and sodium valproate) and Parkinson’s disease, on treatment for 8
years. A year ago he had a stroke after which he spent 6 weeks in Introduce yourself, confirm her identity and say that you
a rehabilitation hospital before returning to his wife at home. She would like to respond to her concerns about feeding and
is his main carer, and he requires help with washing, dressing, discuss possible ways to tackle this.
meals and mobility around the house. Following antibiotic
treatment he has recovered to the point of cardiorespiratory 2.  Explain the current situation as you see it
stability, but cannot swallow. The speech and language therapist
assessing him believes he might never safely swallow because this Explain that, despite some improvement, the position
was significantly impaired before admission and the balance has remains very serious. Explain the reasons for considering
now been tipped. Unfortunately, Mr Hawkins has not tolerated a PEG tube, in this case delivery of medications and
numerous attempts to pass a nasogastric feeding tube and the administration of feeding.
speech and language therapist has suggested that a percutaneous
endoscopic gastrostomy (PEG) would enable him to receive 3.  Explain the main issues relating to  
nutrition. Unfortunately, Mr Hawkins is unable to participate in
such a discussion because he has been too cognitively impaired for
PEG tubes
the last year. His wife fears that he will starve without a PEG and Discuss the issues relevant to PEG tube insertion in more
wishes to discuss the issues with you. detail. Explain that it might not be a means of improving
Your task is to discuss the options with her and help her reach or prolonging life. Explain there is no good evidence that
a decision. it would do so where there are problems with memory
and understanding. There would be a small but significant
Your examiners will warn you when 12 minutes have risk of complications including peritonitis (around 2%)
elapsed. You have 14 minutes to communicate with the and an increased risk of aspiration, which in this case is
patient / subject followed by 1 minute of reflection. There particularly relevant.
will then follow 5 minutes of discussion with the examin-
ers. Do not take the history again except for details that 4.  Try to establish the patient’s wishes
will help in your discussion with the patient / subject. You Ask if she and her husband have ever discussed what his
are not required to examine the patient / subject. wishes might be if such a scenario ever arose.

697
Station |4| Communication skills and ethics

5.  Explain alternatives instigation is favoured in malnourished patients. Methods


for enteral feeding are nasogastric tube (NGT) feeding,
Explain that the alternative is to administer medications
PEG or less commonly percutaneous endoscopic jejunos-
via alternative routes (phenytoin intravenously, diazepam
tomy feeding.
rectally), but that not all drugs can be delivered, and for
practical purposes there would be no feeding and so inevi-
tably there would be a starvation factor. Point out that
At day 2 the SLT team feel his swallow
fluids can be given intravenously or subcutaneously (a is still unsafe and NGT feeding is
small needle under the skin of the abdomen). suggested. What are the complications of
NGT insertion?
6.  Be alert to cues
These include discomfort, epistaxis, oesophageal stenosis,
Do not simply deliver your spiel without being aware of
aspiration and complications from enteral feeds.
any reactions she may have.

7.  Explore her concerns At day 10 the SLT team feel his swallow has
Understand that it is a heart-rending decision. Ask if she shown little sign of improvement. A
has any particular feelings about things at this stage. gastrostomy tube is recommended. What do
Explain that a decision does not have to be made now. you know about gastrostomy feeding?
Patients’ and relatives’ expectations from PEG tube feeding
A gastrostomy tube is considered when nutritional 
are often improved nutrition, prevention of aspiration,
intake is likely to be impaired for more than a few weeks
extension of life, improvement of pressure sores, comfort,
or when a NGT is not tolerated or is contraindicated. PEG
strength and help in overcoming an acute illness. The
feeding is an effective way of providing enteral feeding 
evidence is that rarely do patients achieve these expecta-
to patients who have functionally normal gastrointestinal
tions; in 70% there may be no improvement and there is
tracts but who cannot meet their nutritional needs 
no prevention of aspiration; in the frail elderly there is a
because of inadequate oral intake. The procedure places a
30-day mortality of 22% and a 1-year mortality of 50%.
feeding tube directly into the stomach via a small incision
8.  Respond to questions through the abdominal wall and utilises a powerful light
source from an endoscope and insufflation of air to allow
She might ask for your advice. You might say that putting
positioning.
everything together, without seeing a clear way forward to
long-term improvement, it may not be in his best interests
What are the benefits
to ‘go there’. If that is what she decides then assure her that
there is no evidence that he would suffer from things like of gastrostomy feeding?
hunger pains, not at this stage. Malnutrition determines disease outcomes because it
affects every system in the body, leading to both physical
9.  Confirm understanding and agree a way and psychological disability. PEG feeding aims to improve
forward nutritional status and reduces mortality, length of hospital
Make sure that she has enough information and either stay and complications in carefully selected patients who
agree a way forward or allow her more time to think things are likely to be or later become nutritionally depleted for
over. longer than 4–6 weeks. Studies have shown clear benefits
of PEG feeding after stroke (in terms of improving nutri-
10.  Reassure that decisions are not irreversible tional status and reducing mortality) and in patients with
Make it clear that no decision that she or the medical team oropharyngeal cancer (in terms of improving nutritional
makes is irreversible and of course if the situation were to status). When compared with other methods of enteral
change then any decision could be reviewed. nutrition, such as NGT feeding, gastrostomy feeding
caused less discomfort and had lower rates of complica-
Discussion tions such as bleeding, blockage and dislodgment of the
tube, and possibly lower rates of reflux and aspiration
A 68-year-old man has a left total anterior although these may still occur.
circulation stroke with right hemiparesis
and dysphagia. His swallow is deemed Which patients may be considered for
unsafe by the speech and language therapy gastrostomy feeding?
(SLT) team. His family want to know if you Although studies have shown benefits for PEG feeding in
will be feeding him to keep his strength up. stroke and oropharyngeal cancer, the appropriateness of
When is enteral feeding recommended and gastrostomy insertion in other patient subgroups is contro-
versial. Other conditions for which patients are commonly
what methods are there? referred for PEG tube insertion include motor neurone
Enteral feeding in adult hospital patients is indicated if disease, multiple sclerosis, Parkinson’s disease, dementia,
feeding is likely to be delayed more than 5–7 days. Earlier head injury, intensive care patients, oropharyngeal cancer

698
Case 4.34  Percutaneous endoscopic gastrostomy feeding

and oesophageal cancer. The National Confidential Enquiry patients with dementia is an emotive and controversial
into Patient Outcome and Death (NCEPOD) undertook question, compounded by the fact that patients often lack
the largest study in the UK of mortality after PEG insertion capacity. A Cochrane review showed no evidence of
and found a 6% mortality in a cohort of 16 648 patients. Of increased survival, reduced pressure ulcers or improved
those who died, 43% died within 1 week of PEG insertion, quality of life, nutritional status, function, behaviour or
and in 19% of patients PEG insertion was thought to have psychiatric symptoms of dementia in patients with
been futile. Each patient must be considered according to advanced dementia fed with gastrostomy tubes. No large
his or her individual needs. prospective studies have examined outcomes of PEG
feeding in patients with dementia. A retrospective study of
What are the contraindications 361 patients found that patients with dementia who had
to gastrostomy? a PEG inserted had higher mortality than other patient
subgroups (54% 30-day mortality and 90% at 1 year).
Active coagulopathies and thrombocytopenia must be cor- These findings have been reproduced by other investiga-
rected. Anything that precludes endoscopy, such as haemo- tors, who found that eating problems occurred in 85.8%
dynamic compromise, sepsis or a perforated viscus, is an of patients with dementia before death, which suggests
absolute contraindication. that difficulties with feeding are an end-stage problem.
Guidelines on dementia highlight the importance of
What are the complications quality of life in advanced dementia and support the role
of gastrostomy feeding? of palliative care in these patients from diagnosis until
death. Best practice in these patients could be to encourage
These may be immediate, early or late: eating and drinking by mouth for as long as tolerated, to
• Immediate (< 72 hours) – endoscopy related, use good feeding techniques, to alter the consistencies of
haemorrhage or perforation, aspiration, oversedation food, and to promote good mouth care. When disease
• Early – ileus, pneumoperitoneum, wound infection, progression is such that the patient no longer wants to eat
wound bleeding, trauma or drink, then rather than inserting a gastrostomy tube
• Delayed – gastric outlet obstruction, buried bumper end-of-life care pathways might be considered.
syndrome, dislodged tube, peritonitis, peristomal
leakage or infection, skin or gastric ulceration, What is re-feeding syndrome?
blocked tube, tube degradation, gastric fistula after
removal of tube, granulation around site of insertion. Re-feeding syndrome refers to potentially fatal shifts in
fluids and electrolytes that may occur in malnourished
Overly granulated stoma sites are common. Treating the patients receiving rapid enteral or parenteral nutrition.
cause, such as gastric leakage, infection or poor position, The syndrome is due to metabolic and hormonal changes.
may be more appropriate than the non-evidence-based The hallmark is hypophosphataemia, but the syndrome is
measures to treat. Blockage is usually secondary to drugs or complex and may include hypokalaemia, hypomagnesae-
feed. It can sometimes be removed by massaging the tube, mia, thiamine deficiency and changes in sodium and fluid
or a push–pull method using a syringe on the end, or balance, changes in calcium, and changes in glucose, fat
enzyme preparations or fizzy drinks may be delivered into and protein metabolism.
the tube. Inadvertent removal occasionally occurs, and
delay in recognising may result in stoma closure; a urinary
catheter may be used as a holding measure to prevent Normal glucose metabolism
closure. Feed-related peritonitis is possible after reinsertion Glycolysis normally converts glucose to pyruvate via a
of a gastrostomy tube. When uncertainty exists about the series of oxidation reactions and enzymes including pyru-
position of a replacement tube, water-soluble contrast can vate kinase to produce ATP / energy (Fig. 4.1). Excess
determine position before feeding is re-started. The ‘buried glucose is converted to glycogen for storage via glycogen-
bumper’ syndrome is rare but serious – the internal bumper esis. During strenuous exercise glycogenolysis and glycoly-
migrates from the gastric wall towards the skin, anywhere sis produce more energy, but if the rate of these processes
along the PEG tract, as a consequence of excessive tension exceeds oxygen delivery then anaerobic glycolytic path-
between the internal and external bumper. Symptoms may ways are used, which generate lactate.
include pain on feeding, retrograde leakage of feed onto
skin, and rarely gastric perforation. Starvation
In early starvation the body switches from carbohydrate to
What is the role of gastrostomy feeding protein and fat metabolism. The glucose needed to
in dementia? produce energy from glycolysis is generated from two
There is insufficient evidence to support PEG feeding in mechanisms; both of these gluconeogenesis mechanisms
dementia and other neurodegenerative diseases. Patients occur chiefly in the liver (Fig. 4.1):
with advanced dementia develop feeding problems from • Lipolysis, which converts triglycerides in adipose
lack of motivation and sometimes impaired swallow. tissue to fatty acids and glycerol, the latter
Whether or not to use percutaneous gastrostomies to feed convertible to glucose

699
Station |4| Communication skills and ethics

ATP/energy

Pyruvate

Glycolysis
Normal energy production Glucose

Excess glucose stored


Glycogenesis as glycogen

Utilises carbon
from AAs
Early starvation Protein (muscle) Amino acids (AAs) Glucose

Urea via Oxidation via Gluconeogenesis


urea cycle TCA cycle in liver

Glycerol

Triglycerides Acetyl CoA


Acetone
(adipose tissue) HMG-CoA
Ketotic in liver
pathway
Fatty acids Ketone bodies Acteoacetate

β-hydroxybutyrate

Prolonged Diabetic
starvation ketoacidosic
Ketotic pathway used Ketotic pathway used early because,
after gluconeogenesis despite hyperglycaemia, glucose
pathways exhausted cannot be incorporated intracellularly
because of insulin deficiency

Figure 4.1  Glucose metabolism, starvation and diabetic ketoacidosis. ATP, adenosine triphosphate; CoA, coenzyme A; HMG, 3-hydroxy-3-
methylglutaryl; TCA, tricarboxylic acid cycle..

• Protein catabolism, in which protein, largely from normal since these electrolytes are mainly in the intracel-
muscle, is degraded to amino acids and the carbon lular compartment, which contracts in starvation, and
from amino acids is subsequently used in glucose renal excretion is reduced.
synthesis; starvation is a highly catabolic state, and
urea is produced reflecting this. Diabetic ketoacidosis (DKA) similarities  
In prolonged starvation, metabolic and hormonal to starvation
changes aim to prevent muscle and protein breakdown, The ketotic pathway in DKA is the same as that used in
and the liver reduces its rate of gluconeogenesis to preserve starvation (Fig. 4.1), but it occurs earlier and is more
muscle. Now, the ketotic pathway (Fig. 4.1) in which fatty severe. In starvation insulin levels fall in response to the
acids are used to produce ketone bodies, an alternative lack of food and glucose. In DKA insulin levels are low
source of fuel for most cells, is prominent. The brain, owing to insufficient production leading to hyperglycae-
generally unhappy to use ketones for energy, may be mia; however, intracellular glucose levels are very low
forced to do so in more desperate starvation states. During because without insulin glucose cannot be taken up by
prolonged starvation, some electrolytes such as phosphate cells, and alternatives to glycolysis for energy production
are severely depleted, although serum levels may remain must be sought immediately.

700
Case 4.35  Vegetative state

Re-feeding over 12 hours. Where magnesium is moderately depleted


(0.5–0.7 mmol / l), intravenous replacement should be
Glycaemia in re-feeding increases insulin and decreases
0.5 mmol / kg / day over 24 hours then 0.25 mmol / kg / day
glucagon secretion, stimulating glycogen, fat and protein
for 5 days; where magnesium is severely deplete
synthesis, a process requiring phosphate, magnesium and
(< 0.5 mmol / l), intravenous replacement should be
thiamine, and driving potassium and glucose into cells
24 mmol over 6 hours then as for moderately depleted.
through an ATPase sodium–potassium pump; phosphate
and magnesium are also taken up into cells. The clinical
manifestations of re-feeding syndrome result from these
depleted electrolytes and the sudden increase in metabolic CASE 4.35  VEGETATIVE STATE
rate.

Clinical manifestations of re-feeding syndrome Candidate information


These are outlined in Table 4.6.
Role
How may re-feeding syndrome be You are a doctor on the medical ward.
prevented and managed? Please read this summary.

An individualised re-feeding regimen should be  Scenario


determined by a dietitian to prevent re-feeding syndrome
(Box 4.30).
Where phosphate is moderately (0.3–0.6 mmol / l) or Re: Mr Roger Hunt, aged 19 years
severely (< 0.3 mmol / l) depleted, an intravenous infusion Mr Hunt is a 19-year-old patient with type 1 diabetes who
of 9 mmol or 18 mmol, respectively, should be delivered sustained severe cortical brain damage from an episode of
prolonged hypoglycaemia while out drinking with friends at New
Year. He breathes spontaneously but has been on your ward for 4
Table 4.6  Clinical manifestations of re-feeding syndrome months and not shown any signs of interacting with his
environment. He receives percutaneous gastrostomy feeding. He
has had two episodes of aspiration pneumonia. His father is
Deficiency Clinical manifestations convinced he will get better one day, but is concerned that the
doctors will label his son as being in a persistent vegetative state
Phosphate Widespread cellular dysfunction of most
and seeks your reassurance that you will not stop actively treating
physiological processes
future infections or providing artificial hydration and nutrition.
Potassium Cardiac arrhythmias Your task is to address his father’s concerns.
Magnesium Cardiac and neuromuscular instability
Thiamine Wernicke’s encephalopathy (ocular abnormalities, Your examiners will warn you when 12 minutes have
ataxia, confusion, hypothermia, coma) elapsed. You have 14 minutes to communicate with the
Korsakoff’s syndrome patient / subject followed by 1 minute of reflection. There
Sodium and Fluid overload (rapid introduction of glucose will then follow 5 minutes of discussion with the examin-
water after starvation reduces urine output) ers. Do not take the history again except for details that
will help in your discussion with the patient / subject. You
are not required to examine the patient / subject.

Box 4.30  Prevention of re-feeding sydrome Patient / subject information


• Check potassium, calcium, phosphate and magnesium Your son, Roger Hunt, is 19 years old and has type 1 dia-
• Prior to feeding administer thiamine 200–300 mg daily orally, betes. Four months ago he sustained severe brain damage
one or two high-potency vitamin B tablets thrice daily (or
from an episode of prolonged hypoglycaemia while out
intravenous vitamin B), and multivitamin or trace element
supplements daily drinking with friends. He breathes spontaneously but,
• Start feeding slowly (no more than 50% of energy
according to the doctors, has not shown any signs of inter-
requirements to start with, 0.0418 MJ/kg / day, slowly acting with his environment. However, you are certain that
increasing over 4–7 days) he occasionally smiles and even seems to shed tears in
• Ensure adequate hydration your and your wife’s company. He receives percutaneous
• Supplement and / or correct any levels of potassium gastrostomy feeding. He has had two episodes of pneumo-
(2–4 mmol / kg / day), phosphate (0.3–0.6 mmol / kg / day), nia. You are convinced he will get better one day, but are
calcium and magnesium (0.2 mmol / kg / day intravenously or very concerned that the doctors will label him as being in
0.4 mmol / kg / day orally) and monitor and amend treatment a persistent vegetative state. You want reassurance that the
for these electrolytes for 2 weeks doctors will not stop actively treating future infections or
providing artificial hydration and nutrition.

701
Station |4| Communication skills and ethics

How to approach the case 8.  Concede uncertainty


Despite this, concede that at this stage there are no certain-
Communication skills (conduct of interview, ties; that alone is often a heart-rending thing to wrestle
exploration and problem negotiation) and with.
ethics and law
9.  Invite questions
1.  Introduction
Make sure there are no areas that have not been discussed
Introduce yourself and establish that he is Mr Hunt’s
and express that you are willing to address any issues at
father.
any time with him.
2.  Establish what the relative knows 10.  Conclude with assurances
Firstly, establish what he knows about his son’s condition, Reiterate that at this stage a diagnosis of persistent vegeta-
so that you have a basis for addressing his concerns. tive state has not and cannot be made, and that you will
continue to actively treat his son’s condition and any
3.  Elicit ideas, concerns and expectations complications.
Explore his ideas, concerns and expectations, actively
listening. Discussion
4.  Address ideas, concerns and expectations What is the vegetative state?
Start with an immediate reassurance that there are no The vegetative state is a complex neurological condition
plans to withdraw care for his son. Note his remark about in which patients appear awake but show no sign of aware-
a persistent vegetative state. Explain that at this stage in his ness of themselves or their environment. It may develop
son’s illness there would be every attempt to vigorously suddenly (as a consequence of traumatic or non-traumatic
treat any infection, manage his diabetes carefully and of brain injury, such as hypoxia, infection or haemorrhage)
course provide full hydration and nutrition in conjunction or gradually (in the course of a neurodegenerative disor-
with appropriate colleagues. Explain that certain move- der, such as Alzheimer’s disease). It is uncommon, and
ments and facial gestures are not uncommon in this situ- appears perplexing because there is apparent dissociation
ation, and that while you do not have the expertise to be between the two cardinal elements of consciousness:
sure that they are not signs of awareness, your understand- awareness and wakefulness. Guidance produced by the
ing is that they could very well be involuntary reflex move- Royal College of Physicians in 2003 defines it as ‘a clinical
ments. The point is that you do not want to unequivocally condition of unawareness of self and environment in
remove hope and yet you want to truthfully reinforce what which the patient breathes spontaneously, has a stable
seems likely. circulation, and shows cycles of eye closure and opening
which may simulate sleep and waking.’ Thus three main
5.  Be alert to cues clinical features define the vegetative state (Box 4.31).
Throughout your explanation, be alert and ready to Since the brain damage is cortical and the brainstem is
address any cues suggesting concern or disagreement. preserved, awareness and consciousness are lost but breath-
Above all, be empathetic and at no point challenge his ing and circulation are spontaneous and support in an
desire to see his son get better, even though you may be intensive care unit is not usually required. Patients may
less certain of its likelihood. experience sleep–wake cycles or be in a state of chronic
wakefulness. Wakefulness is determined by nuclei in the
6.  Elaborate on the meaning of the term
persistent vegetative state
Box 4.31  The three features of the
Offer to explain more about a persistent vegetative state.
vegetative state
In particular explain that a diagnosis cannot be made this
early on in the illness and can only be made later by 1. Cycles of eye opening and closing, giving the appearance of
certain specialists. sleep–wake cycles (whether this reflects the circadian rhythms
is unclear)
7.  Be honest about the prognosis as it   2. Complete lack of awareness of the self or the environment
appears now 3. Complete or partial preservation of hypothalamic and brainstem
autonomic functions
At the same time, do not reinforce what you feel to be Royal College of Physicians guidelines consider a vegetative
excessive hope. Explain that you and the medical and state persistent when longer than a month and permanent when
nursing team would have hoped that more definite signs longer than 6 months for non-traumatic brain injuries (3 months in
of improvement would have started to develop by now, US guidelines) and 1 year for traumatic brain injuries. Persistent
and that as time goes on the prospects of the recovery that and permanent vegetative states are often abbreviated to ‘PVS’.
you would all wish for become less easy to envisage.

702
Case 4.35  Vegetative state

hypothalamus that coordinate the circadian rhythm and is state (that is, they show signs of wakefulness) within 
a prerequisite for consciousness and awareness, but in the 4 weeks.
event that higher centres (the cerebral cortex) are severely
damaged or destroyed the wakefulness is without con- How is coma measured?
sciousness or awareness. Apparent awareness in this state
The depth of coma is measured by the Glasgow Coma
of wakefulness, including opening of eyes, movements,
Score (Table 4.8).
swallowing and even emotional outbursts, are automatic
reflexes. Spontaneous eye opening indicates intact brain-
What is the minimally conscious state?
stem arousal mechanisms but not necessarily awareness.
The minimally conscious state is a condition in which
How does the vegetative state differ patients appear not only to be wakeful (like the vegetative
from other disorders of consciousness state) but also to exhibit inconsistent (fluctuating) but
reproducible signs of awareness (unlike the vegetative
such as coma? state). It may be a transient state and precede recovery of
Experts suggest that the vegetative state should be seen as communicative function or it may last indefinitely.
part of a continuous spectrum of disorders of conscious-
ness, in which someone’s wakefulness and / or awareness What is locked-in syndrome?
are impaired after severe brain injury (Table 4.7).
Locked-in syndrome (or pseudocoma), although not a
disorder of consciousness, may be confused with vegeta-
What is coma? tive state. Patients are both awake and aware, yet entirely
Coma is a condition of unresponsiveness in which patients unable to produce any motor output or they have an
lie with eyes closed, do not respond to attempts to arouse, extremely limited repertoire of behaviours (usually vertical
and show no evidence of awareness of self or surround- eye movement or blinking). Numerous anatomical and
ings. Patients lack not only signs of awareness (similar to aetiological types are possible, but a classic locked-in syn-
vegetative state) but also wakefulness (unlike vegetative drome occurs with basilar territory infarction sparing res-
state) regardless of how intensely they are stimulated. piratory drive centres but damaging long tracts and cranial
Patients typically either recover or progress to a vegetative nerve nuclei in the brainstem bilaterally. The cortex is

Table 4.7  Spectrum of disorders of consciousness

Condition Definition Main clinical characteristics Consciousness Purposeful


motor
behaviour

Awareness Sleep–wake
cycles
Coma Unarousable state Absence of eye opening (even with intense No No No
of unresponsiveness stimulation)
No evidence of awareness of self or environment
Condition protracted for more than 1 hour
Vegetative Wakefulness Presence of eye opening and closing No Yes No
state accompanied by the Absence of any reproducible purposeful behaviour
absence of any sign including: (a) response to sensory stimulation; (b)
of awareness awareness of self or environment; (c) language
comprehension or expression
Minimally Reproducible signs Presence of eye opening and closing Partial, Yes Inconsistent
conscious of awareness Presence of inconsistent but reproducible purposeful fluctuating
state behaviour including (any of): (a) non-reflexive response
to sensory stimulation; (b) awareness of the self or the
environment; (c) language comprehension or
expression
Lack of functional communication or object use
Locked-in Impairment in the Presence of eye-coded communication (limited to eye Yes Yes Yes
syndrome production of movements, depending on lesion)
voluntary motor Preserved awareness
behaviour Complete or partial inability to produce motor
behaviour

703
Station |4| Communication skills and ethics

(b) sustained, reproducible, purposeful, or voluntary


Table 4.8  Glasgow Coma Score
response to visual, olfactory, auditory, tactile or noxious
Eye opening stimuli; and (c) comprehension of language or expression.
• Spontaneously 4
If evidence of these exists, the patient is considered to be
(minimally) aware. Examinations should be conducted at
• To speech 3 different times of the day because patients who are mini-
• To pain 2 mally conscious as opposed to vegetative may have alter-
nating periods of awareness and unawareness as well as
• No response 1 circadian oscillations in levels of wakefulness.
Best verbal response
• Orientated 5
Why do we provide life-sustaining
treatments to patients in a
• Disorientated 4
vegetative state?
• Inappropriate words 3
There are well-documented cases of recovery from a veg-
• Incomprehensible sounds 2 etative state, which is why it has been referred to as a
• No response 1 persistent rather than permanent vegetative state. This
said, it usually is permanent because of the nature of the
Best motor response underlying damage. Important considerations in the
• Obeys verbal commands 6 future care of a patient in a vegetative state are summarised
in Box 4.32.
• Localises painful stimuli 5
• Withdrawal to pain 4 Does misdiagnosis of the vegetative
• Flexion to pain (decorticate posturing) 3 state occur?
• Extension to pain (decerebrate posturing) 2 Current clinical methods of diagnosis are limited, evi-
• No response 1
denced by a high rate (about 40%) of misdiagnosis (that
is, patients who are aware are considered to be uncon-
scious). The main causes of misdiagnosis are associated
with a patient’s disability (such as blindness), confusion
unimpaired but cannot deliver any of its actions. Patients in terminology, and lack of experience of a relatively rare
can breathe and think normally but can perform few if any condition. Furthermore, standard behavioural assess-
actions. ments cannot distinguish an aware (minimally conscious)
but completely immobile patient from a non-aware
Why are coma and the vegetative state not patient (vegetative state).
the same as brain death?
In brain death there is loss of brainstem activity. Brainstem Is there a place for brain imaging as a
reflexes are absent and spontaneous breathing and circula- diagnostic tool?
tion cannot occur and can only be maintained artificially. In such behaviourally non-responsive patients, functional
Only irreversible coma with absent brainstem reflexes neuroimaging methods (such as magnetic resonance
indicates brain death. imaging or electroencephalography) can detect residual
cognition and awareness and can even establish two-way
What affects prognosis in patients with a communication, without requiring any behavioural
vegetative state? output from patients. The Multi-Society Task Force on 
Three major factors affect the prognosis: time spent in the PVS suggests, however, that such tests, although recognised
vegetative state, age and type of brain injury; traumatic as providing useful information when used in conjunction
brain injuries are associated with better outcomes at 1 year
than non-traumatic injuries.

How is the vegetative state diagnosed? Box 4.32  Considerations in the future care of a
patient in a persistent vegetative state
No tool exists for quantifying the extent of consciousness.
Differentiating between awareness and non-awareness • Establishing the diagnosis
ultimately relies on a principle that someone is conscious • Deciding on its permanence
if they can indicate so. The diagnosis of the vegetative state • The presence of any valid advance decision
is based on a detailed history and careful (but subjective) • Deciding whether or not to withdraw life-prolonging
observation of the patient’s spontaneous and elicited treatments, which even in the presence of an advance decision
behaviour. Clinical assessments involve repeated examina- often has to be decided in the Courts
tions for evidence of: (a) awareness of self or environment;

704
Case 4.36  Brainstem death

with clinical evaluation, are believed to be unable,  How to approach the case
alone, to either confirm the diagnosis of vegetative state
or predict the potential for recovery of awareness. Current Communication skills (conduct of interview,
guidelines should be modified to embrace functional exploration and problem negotiation) and
neuroimaging. ethics and law
1.  Introduction
CASE 4.36  BRAINSTEM DEATH Introduce yourself and establish that she is Mrs Daly’s
daughter.

Candidate information 2.  Establish what the relative knows


Ask her what she knows about her mother’s condition.
Role
You are a doctor on the medical ward. 3.  Explain the situation
Please read this summary. Explain that there have been no signs of recovery of con-
sciousness and that this has been the major concern.
Scenario
4.  Explain brainstem death
Re: Mrs Anne Daly, aged 58 years Explain sensitively and empathetically that, as a result of
Mrs Daly is a previously well 58-year-old woman admitted to the injury, she has sustained irreversible brain damage and
hospital 2 days ago with collapse. She was intubated on arrival to this has just been confirmed by the specialists. Explain that
hospital but subsequent brain imaging confirms that she has had a she will not wake up and that this means that she has died
spontaneous subarachnoid haemorrhage. She has not received
as a result of her injuries. Point out the ventilator is sus-
sedation since initial ventilation and shows no signs of recovery.
Two consultants have confirmed brainstem death. She was taining her breathing and that when turned off her heart
previously well, and normotensive. Her daughter, Mrs Watts, would stop soon afterwards.
is in the waiting room, aware that brainstem testing is being
carried out although not completely aware of the implications 5.  Acknowledge distress and allow ventilation  
of this. of feelings
Your task is to discuss the results with her daughter.
Wait for her response before giving any more details.

6.  Explain what will happen now


Your examiners will warn you when 12 minutes have Explain that the medical team has to make the decision to
elapsed. You have 14 minutes to communicate with the turn off the ventilator, that it will not cause her mother
patient / subject followed by 1 minute of reflection. There any distress, and that she need not feel any responsibility
will then follow 5 minutes of discussion with the examin- for this decision.
ers. Do not take the history again except for details that
will help in your discussion with the patient / subject. You 7.  Discuss organ donation
are not required to examine the patient / subject. Thank her for being able to mention this at this time.
Assure her that as it is her mother’s wish you will arrange
Patient / subject information for the transplant team to assess her mother for suitability,
and until such time as any organ retrieval were to take
You are the daughter of Mrs Daly, a previously active place ventilation would be continued.
58-year-old woman. She was admitted to hospital 2 days
ago following a fall whilst walking her dogs on the moor. 8.  Identify support networks
She sustained a significant head injury. She was artifi-
It is human to want to discover a little more about the
cially ventilated on arrival to hospital and subsequent 
practical implications to her such as what other family
brain scans showed that she had sustained a large sub-
members may be nearby and need to know, and even a
arachnoid haemorrhage. She has subsequently shown no
little about the dogs.
signs of recovery. She was previously well, and normoten-
sive. You are aware that brainstem testing is being carried
9.  Invite questions
out, and although not completely aware of the implica-
tions of this sense that the news will be bad and that your Ask if she has any immediate questions.
mother is not going to recover. You are surprised that she
is so desperately unwell, however, because her heart 10.  Ensure that the relative has a point of contact
appears good on the monitor and she looks well but Ideally there would also be a nurse present to attend to
asleep. She carried a donor card, and you will tell the her now but be prepared to meet her again later in the day
doctor this. or week if she has further questions.

705
Station |4| Communication skills and ethics

Discussion the oculovestibular (caloric) response test the VOR, but


the latter is more sensitive.
How might brainstem function Major brainstem pathology is unlikely with normal
be assessed? VORs. An absent horizontal VOR with an intact vertical
VOR may indicate pontine damage. If both are absent
Assessing brainstem function is vital to managing coma, there may be wider brainstem pathology or a metabolic
raised intracranial pressure, brainstem strokes and con- disturbance, the latter more likely if pupils react. A very
firming brain death. small number of drugs depress brainstem function. VOR
Pupils assess midbrain integrity. Pupil size and reactivity testing may be difficult if there are dysconjugate eye move-
assess third nerve function through the superior colliculus ments (when brainstem dysfunction is likely) or if there
and connections to the Edinger–Westphal nucleus (mid- are fast saccades (when brainstem dysfunction is unlikely).
brain) and efferent parasympathetic outflow.
The corneal reflex assesses the fifth and seventh nerves,
Which brainstem syndromes arise from
connected in the pons.
Resting eye position might suggest asymmetric brain- brain shift?
stem dysfunction, dysconjugate position a disorder of the There are four important syndromes (Box 4.33).
third, fourth or sixth nerves or their nuclei (third and
fourth in midbrain and sixth in pons). Spontaneous sac-
cadic (fast) horizontal and vertical eye movements suggest Box 4.33  Brainstem syndromes arising from
that the brainstem mechanism for generating this is intact brain shift
and there is no need to test for oculocephalic or oculoves-
tibular responses. Horizontal saccades rely on an intact Central herniation
paramedian pontine reticular formation (pons), third A supratentorial mass displaces the diencephalons (cerebral
nerve nucleus, sixth nerve nucleus and median longitudi- peduncles, thalamic areas, thalamus and related structures between
nal fasciculus connecting these. Vertical saccades rely on the cerebral hemispheres and upper brainstem) through the
the dorsal midbrain. tentorium to compress the upper midbrain, then the pons, then the
Swallowing requires intact glossopharyngeal and vagus medulla. Signs such as unilateral hemiplegia, together with
paratonia (gegenhalten) and extensor plantars (diencephalic stage),
nerves and connections including the swallowing centre
precede diminished alertness and Cheynes–Stokes breathing. The
in the reticular formation of the medulla. pupils are small, perhaps because of hypothalamic sympathetic
Respiratory pattern is sometimes useful in localisation. dysfunction, but reactive. There is decorticate (flexor) posturing to
Apneustic breathing (prolonged inspiration followed by a pain. Progression from midbrain to upper pontine dysfunction
period of apnoea) and cluster breathing (closely grouped causes temperature fluctuation, unreactive pupils in mid-position,
respirations followed by a period of apnoea) imply loss of vertical eye movements (doll’s head manoeuvre),
pontine damage. Ataxic breathing (chaotic) and gasping diminishing vestibulo-ocular reflexes (VORs) and apneustic or
breathing (gasps followed by variable apnoea) occur when cluster breathing or central hyperventilation. Decerebrate posturing
the medullary respiratory centre is damaged and are pre- to pain develops. Progression to lower pontine and upper
medullary damage causes chaotic breathing and absent VORs. Tone
cursors to respiratory arrest. Cheynes–Stokes breathing
is flaccid. Progression to medullary damage is terminal. Heart rate
(often used to describe a range of brainstem damage  may fall and blood pressure rise (Cushing response), breathing is
patterns of breathing) can be due to cortical damage, but chaotic or gasping, and pupils become dilated and fixed.
is mostly due to cardiovascular or respiratory disease.
Slow, shallow breathing may occur when drugs depress Lateral (uncal) herniation
medullary function. Central neurogenic hyperventilation Lesions in the lateral middle fossa or temporal lobe push the
(rapid, deep continuous breathing at a rate of around  medial edge of the uncus and hippocampal gyrus over the free
25 cycles / min) is not localising but suggests deepening lateral edge of the tentorium. The first sign is a unilateral dilating
coma and worsening prognosis. pupil as the third nerve is compressed, followed by a third-nerve
palsy. Midbrain compression with diminished alertness and coma
Long-tract signs can occur and may be crossed with
and the sequence as for central herniation can follow, without an
cranial signs because of decussation. initial diencephalic stage and decorticate responses.
What are oculocephalic and False localisation
oculovestibular responses? Expanding supratentorial lesions may distort structures and
Passive head rotation stimulates vestibular and neck recep- produce signs due to traction from afar, often affecting cranial
nerves V to VIIII.
tors which in comatose patients with intact brainstems
leads to reflexive slow conjugate eye movements mediated Tonsillar herniation
by the vestibulo-ocular reflex (VOR) pathway in the brain- Subtentorial lesions may cause herniation of the cerebellar tonsils
stem in the direction opposite to head rotation. Ice water through the foramen magnum to compress the pons and midbrain
irrigation of the ear switches off this pathway, leading to directly. There may also be upward compression, giving rise to
the unopposed contribution of the contralateral vestibular supratentorial effects, but the rostrocaudal sequence of damage
system, and the eyes deviate towards the ice. Both the typical of central herniation is lacking.
oculocephalic (doll’s head manoeuvre) response and 

706
Case 4.37  Discussing live organ donation

Your examiners will warn you when 12 minutes have


Table 4.9  Brain death criteria
elapsed. You have 14 minutes to communicate with the
Preconditions Irreversible brain damage, e.g. head injury, brain patient / subject followed by 1 minute of reflection. There
haemorrhage, anoxia will then follow 5 minutes of discussion with the examin-
Apnoeic coma ers. Do not take the history again except for details that
Absence of reversible factors such as drugs or will help in your discussion with the patient / subject. You
metabolic or temperature disturbance are not required to examine the patient / subject.
All brainstem Pupils fixed and unresponsive to bright light (not
reflexes necessarily dilated) Patient / subject information
absent Absent corneal reflexes
Absent vestibulo-ocular reflexes You are the sister of Stephen Wells, a 45-year-old solicitor
No motor responses within the cranial nerve who has chronic kidney disease from diabetes mellitus
distribution to stimulation of any somatic area and kidney damage as a child. He currently receives
No reflex response to touching the pharynx (gag haemodialysis. He is on the transplant list. You are with
reflex) or a suction catheter in the trachea him today because you have both discussed the idea of
(cough reflex) live organ transplantation, having read of its success. Your
Apnoea No respiratory movements without the ventilator concerns are the risks to yourself and your brother, but you
would prefer your brother to have your kidney than one
from an unknown source, and feel his body would not
reject an organ from a relative.
How is brain death determined?
There is irreversible loss of capacity for consciousness and How to approach the case
capacity to breathe. Both of these are lost without the
Communication skills (conduct of interview,
stem. Because it is possible to maintain circulation and
ventilation with ventilators in patients with irreversible exploration and problem negotiation) and
brain damage without brainstem function, criteria for ethics and law
brain death were developed (Table 4.9). Testing should be
1.  Introduction
by two experienced clinicians, at least one a consultant.
Introduce yourself. Both potential recipient and donor
Is there a legal definition of death? should be present (in PACES only one subject but the
principles are the same).
There is no legal definition of death. Death is usually
defined by irreversible loss of capacity for consciousness 2.  Establish background knowledge
together with irreversible loss of capacity to breathe due
Find out what they know about organ donation.
to cessation of brainstem function. Brainstem death there-
fore equates to death of an individual. 3.  Explore ideas, concerns and expectations
Explore their understanding of the advantages of live
organ donation. Explain that there are very clear advan-
CASE 4.37  DISCUSSING LIVE tages but that the complications of transplantation,
ORGAN DONATION including rejection, remain a risk.

4.  Explain the principles of live organ donation


Candidate information Explain briefly there would be a dual hospital stay for the
operation but that the surgical procedure is not your area of
Role expertise and you would arrange a meeting with the trans-
You are a doctor in the renal clinic. plant surgeon were they to decide to go down this route.
Please read this summary.
5.  Respond to ideas, concerns and expectations
Scenario Explain that matching is a potential problem but that it
needs not be perfect nor, indeed, need it be a relative. Her
Re: Mr Stephen Wells, aged 45 years being a sibling does not remove the possibility of rejection
Mr Wells, a solicitor, has chronic kidney disease due to diabetes and he will still need to take immunosuppression with its
mellitus on a background of poor renal reserve caused by damage attendant risks.
from sepsis as a child. He currently receives haemodialysis. He is on
the transplant list. His sister is with him today because they have 6.  Explain potential benefits to recipient  
both discussed the idea of live organ transplantation, having read and donor
of its success. Her concerns are the risks to her and her brother.
Your task is to discuss live organ transplantation. Explain that outcome is better with donation from a live
than a deceased person. While on the waiting list there is

707
Station |4| Communication skills and ethics

no guarantee of when a suitable organ will be available donate. Shortage of organs is doubtless in part the result
and his health might deteriorate in the meantime; live of the understandable reluctance of staff to approach rela-
organ donation would remove this problem. There may tives for consent after a patient dies.
be significant psychological satisfaction in donating to a
relative. How might the supply of organ donors
be increased?
7.  Explain potential risks to recipient and donor
The situation could be improved by educating the public
Risks to the donor include operative and perioperative to leave evidence of their wishes, reducing the need to
risks (major surgery requiring around a week in hospital broach the subject with grieving relatives. The default posi-
and some weeks off work), and having a single kidney tion of a population opting in to donate is mooted by
(although compatible with a normal life). Risks to the governments.
recipient are immediate surgical complications and
rejection. Does the Human Tissue Act improve the
8.  Other matters number of organ donors?
There may be insurance issues for the donor. Currently, even if the deceased carries a donor card, a rela-
tive’s objections can prevent transplantation. The Act
9.  Invite questions ensures that the patient’s recorded wishes, or the decision
I appreciate we’ve covered rather a lot there. Is there anything of a representative nominated by the patient, must be fol-
you would like me to go over again or are there any lowed. Failing this, consent from a person in a ‘qualifying
questions you would like to ask me? relationship’ is necessary (there is a hierarchical list). The
Act unambiguously outlaws all commerce in human
Be honest in your explanations and prepared to admit bodies and body parts.
uncertainty (with assurance you will seek the answer) if
there are questions you cannot answer. Can dying patients who are potential
donors be ‘kept alive’ in intensive care
10.  Explain what would happen next while consent is being sought for organ
Explain that, if they decide to take things further,  donation?
formal assessment by the transplant team would be
needed. Donor ventilatory and circulatory support is essential
before heart transplantation and desirable for other organs.
Discussion Transplantation procedures can only occur when brain-
stem death has been determined, and this should be by a
How is organ transplantation governed team separate to that caring for the patient. ‘Elective venti-
in the UK? lation’, the concept of sustaining the life of a dying patient
for the sole purpose of transplantation, appears to be
Following the retained organs controversy (Case 4.38), the illegal, but the Act authorises methods to preserve organs
Human Tissue Act in 2004 introduced new legislation and of deceased individuals while consent is being sought.
regulation for all human material, whether from living or
deceased people. It applies to England, Wales and North- Are there advantages of live organ
ern Ireland and the Scottish Assembly passed similar leg-
islation in 2006. The Human Tissue Authority (HTA) is donation?
the body that ensures that human tissue is used safely and Carefully selected living related donors are more immu-
ethically, and with proper consent. The HTA regulates the nologically compatible and the chances of graft rejection
removal, storage, use and disposal of human bodies, are reduced.
organs and tissues for purposes such as research, trans-
plantation, and education and training, and gives approval What ethical criteria should be met by live
for organ and bone marrow donations from living people. organ donors?
It was created by the Human Tissue Act 2004, which
replaced the Human Tissue Act 1961, the Anatomy Act These are listed in Box 4.34.
1984 and the Human Organ Transplants Act 1989. The
Unrelated Transplant Regulatory Authority (ULTRA) and Is there a place for unrelated live
the post of HM Inspector of Anatomy were abolished and transplants?
their functions transferred to the HTA. A donation may be obtained from a non-genetically-
related person provided no payment is involved. Histori-
Is there a shortage of organ donors? cally, live unrelated donors of kidneys were seldom
There is a worldwide shortage of cadaveric donor organs, considered because there was no greater chance of graft
and a living related donor is more likely to provide the survival, but modern immunosuppression means that less
best histocompatibility but more likely to feel pressure to well-matched grafts can now survive.

708
Case 4.38  Requesting an autopsy (post mortem)

Box 4.34  Criteria for live organ donors Box 4.35  Common reasons for requesting
an autopsy
• The risk to the donor must be low
• The donor must give full informed consent • The cause of death was not clear
• The consent must be given freely and without coercion or • Whilst the cause of death may have seemed clear, some
pressure features of the disease remain unusual
• The donor must understand that he or she may withdraw • The disease was rare and an autopsy could shed new light on
consent at any time before the procedure it
• The offer of the organ must be without any inducements, • The information from the autopsy might help the management
including financial of other patients (in some cases family members) who suffer
• There must be a good chance of a successful outcome for the from the same condition
recipient

not expected her to die so suddenly. The doctor is about to


CASE 4.38  REQUESTING AN ask your permission for an autopsy. You are not happy for
an autopsy, feeling that if cancer was likely there is no
AUTOPSY (POST MORTEM) reason to further explore. You are also worried an autopsy
could delay funeral arrangements and cause disfigurement.
Candidate information After consideration, you will agree to an autopsy.

Role How to approach the case


You are a doctor on the medical ward.
Communication skills (conduct of interview,
Please read this summary.
exploration and problem negotiation) and
Scenario ethics and law
1.  Introduction
Re: Mrs Jane Ayre, aged 76 years Introduce yourself and confirm her identity.
Mrs Ayre was a 76-year-old woman who died yesterday on
your ward. She had been admitted with weight loss, vomiting and 2.  Acknowledge that it is a difficult time
hypotension. She was awaiting further investigation after an
abdominal computed tomography scan showed bilateral adrenal Tell her how sorry you are that her mother has died and
masses. The concern had been whether she might have adrenal that staff members were saddened by the news. Pause to
metastases and secondary hypoadrenalism. Her daughter is here, allow her to express her emotions. Asking for permission
and your consultant has asked you to seek permission for an to carry out an autopsy at a time when relatives may be
autopsy to clarify cause of death, but is happy for a death distressed is not always easy and not a task that should be
certificate to be issued if she does not consent. delegated to the most junior member of the team. Before
Your task is to request permission for an autopsy.
approaching relatives, it is important to establish if the
patient had made any prior wishes.

3.  Retrace the relevant background


Your examiners will warn you when 12 minutes have Explain the medical team’s thoughts about what might
elapsed. You have 14 minutes to communicate with the have been wrong with the patient and why.
patient / subject followed by 1 minute of reflection. There
will then follow 5 minutes of discussion with the examin- 4.  Explain the reasons for requesting an autopsy
ers. Do not take the history again except for details that Common reasons for requesting an autopsy are listed in
will help in your discussion with the patient / subject. You Box 4.35.
are not required to examine the patient / subject. Additional benefits include education and training,
research and audit.
Patient / subject information
Your mother, Mrs Ayre, died yesterday on the hospital 5.  Explain the autopsy consent form
ward, aged 79. The cause of death was unclear but very Explain that she will need to sign a consent form. This
possibly cancer. She had been admitted from home follow- includes an option for relatives to request a limited
ing 1 week of vomiting and dizziness. She had been losing autopsy, with separate sections for permission for organ
weight for a few months prior to this. She and you had retention, permission for education and training and per-
been told that cancer was a possibility although you had mission for the use of retained tissue in research.

709
Station |4| Communication skills and ethics

6.  Explore concerns What is the role of the Human Tissue


She may have concerns about further examination when Authority?
all she wants, and she feels her mother wants, is to be left It is to regulate the removal, storage, use and disposal of
in peace. human bodies, organs and tissue from the living and
deceased.
7.  Be prepared to discuss arrangements  
about the body What are the implications for hospital
Explain that an autopsy will not (usually) delay funeral autopsies?
arrangements. Relatives sometimes ask if the body can be Most hospital trusts have guidelines for autopsy requests.
viewed after autopsy and might need to be reassured that Most autopsies in hospital are hospital requested and per-
in hospital autopsies standard incisions are used that are formed by a hospital pathologist, and for these relatives
not visible when the body is gowned and that the body must grant permission.
can be viewed after autopsy. Consent forms usually cover the option for relatives to
request a limited autopsy, and separate sections for per-
8.  Invite questions mission for organ retention (usually brain or lungs), per-
I appreciate we’ve covered rather a lot there. Is mission for education and training (often a clause stating
there anything you would like me to go over that information may be used for such purposes unless
again or are there any questions you would like to ask me? relatives disagree) and permission for the use of retained
tissue in research.
9.  Explain what would happen next Disposal of organs retained can either be by the pathol-
Explain that nothing needs to be done by the daughter,  ogy department or returned to the funeral director at the
that the hospital would arrange everything and that  family’s request.
the funeral director would liaise with the hospital  There is concern that hospital autopsies will decline in
directly. number, and training will suffer, although the legislation
acknowledges that people expect clear information and
10.  Seek consent or accept refusal partnership with the medical profession. Hospital trusts
must therefore produce clear procedures and consent
And thank her for discussing this. forms and adequate training in gaining consent from
relatives.
Discussion
In which circumstances might the
What changes have occurred in recent coroner (in Scotland Procurator Fiscal)
years in the law regarding retention wish to perform an autopsy?
of human tissue and organs? While the statutory duty to report a death to the coroner
The public inquiries at Bristol Royal Infirmary and Alder resides with the registrar of births and deaths, doctors
Hey Children’s Hospital found organ retention without should be aware of the circumstances in which a registrar
the knowledge or consent of relatives to be common prac- is required to report a death and should report a death
tice. The Chief Medical Officer subsequently arranged a themselves in those circumstances (Box 4.36).
consensus that revealed 105 000 retained organs, body
parts, stillbirths and fetuses within English hospital trusts
and the Retained Organs Commission was set up to Box 4.36  Deaths that should be reported
oversee the return, donation for research or disposal of to the coroner
retained material according to the wishes of relatives who • Deaths that are sudden or unexplained
enquired, and to advise on future legislation.
• Deaths where the deceased had not been attended by a
The Human Tissue Act makes it a criminal offence to medical practitioner for their last illness and within 14 days
remove, retain or use tissue or organs without consent and before death
covers all material containing human cells removed  • Deaths not due, or entirely due, to natural causes, e.g. accidents,
from the living or dead (gametes or embryos are covered deaths occurring after an operation, occupational disease, lack
by separate legislation). Thus no research or teaching is of care, self-neglect, suicide and self-harm, violence
now possible without consent. Use for clinical or  • Deaths involving septicaemia if originating from injury or
public health audit, education or research using ano- medical intervention
nymised material does not require consent if the material • Dead on arrival to hospital
comes from the living, but will always be needed if from • Deaths in police custody or prison or following admission from
deceased persons. The Act is not be retrospective, but the these
Human Tissue Authority, the regulatory body for the Act, • Deaths where the cause is natural but unknown and the doctor
issues codes of practice for dealing with material already cannot issue a death certificate
retained.

710
Case 4.39  Critical incident

Does a coroner’s autopsy need consent? Your examiners will warn you when 12 minutes have
elapsed. You have 14 minutes to communicate with the
No. Relatives do not have the right to either grant or refuse
patient / subject followed by 1 minute of reflection. There
permission for an autopsy requested by the coroner, nor
will then follow 5 minutes of discussion with the examin-
any organ retention, although the Act will make it neces-
ers. Do not take the history again except for details that
sary for consent to be obtained for retention or use of
will help in your discussion with the patient / subject. You
material after investigations are concluded.
are not required to examine the patient / subject.

Patient / subject information
Clinical governance
Your mother, Mrs Jane Parker, is an 84-year-old woman
who was admitted to hospital 3 days ago with a bleeding
stomach ulcer. She had a stroke 2 years ago, leaving her
CASE 4.39  CRITICAL INCIDENT relatively immobile and she lives with you. She has con-
tinued to have ‘mini-strokes’ with effects on mobility and
Candidate information memory and you have been concerned about her quality
of life. At endoscopy the bleeding ulcer, which you
Role accepted as life threatening, was treated and she was
admitted to the highdependency unit (HDU) because of
You are a doctor on the acute medical unit.
very low blood pressure. You were very happy with the
Please read this summary.
admitting doctor’s (the candidate’s) communication to
you of events. Subsequently, your mother has had a stroke
Scenario and is now very unwell. You were not surprised by this,
although you were upset when another doctor, in the
HDU, told you that the admitting doctor had placed a
Re: Mrs Jane Parker, aged 84 years catheter incorrectly into the artery of her neck and that this
Mrs Parker was admitted 3 days ago with life-threatening probably caused the stroke. The consultant has subse-
haematemesis. She had a history of a stroke from which she had quently made you feel easier, explaining that the line was
made a partial recovery. You resuscitated her with blood. The placed incorrectly, but that it does not seem to have been
gastroenterologist requested central access for monitoring purposes the cause of the stroke but that the matter will be looked
while preparations were made for endoscopy. This was a difficult
into thoroughly. You believe the consultant, have no wish
procedure because her systolic blood pressure was 80 mmHg.
Eventually you obtained flashback of dark-red blood in the left to make a complaint because you have been very happy
internal jugular region and gained access. At endoscopy a bleeding with the care given to your mother over the years and
gastric ulcer was injected and she was admitted to the high- accept that she has been unwell for some time. However,
dependency unit (HDU). you would like to speak to the admitting doctor, both to
The gastroenterologist met you yesterday to let you know of a hear his view of events and to be further reassured that the
critical incident. The day after admission to the HDU Mrs Parker HDU doctor was ‘out of line’.
suffered a further stroke with dense left-sided weakness and a
computed tomography scan showed a very large right parietal
infarction, probably from watershed ischaemia. It was also noticed
How to approach the case
in the HDU that your central line was positioned in the left carotid Communication skills (conduct of interview,
artery. It had not been used, although for reasons not yet
established had not been removed despite a chest X-ray showing exploration and problem negotiation) and
malposition. Unhappily, the HDU doctor told Mrs Parker’s daughter ethics and law
that arterial catheterisation should never occur because it can
cause a stroke. You honestly cannot recall reviewing the chest 1.  Introduction and setting
X-ray. Show that you are willing to listen, explain and help in
The gastroenterologist is convinced that the stroke was whatever way you can. Angry or upset patients and rela-
caused by hypotension on a background of a cerebrovascular
disease, not least because the stroke occurred on the wrong
tives should never feel threatened.
side to implicate the catheter. He is also very irritated by the
careless remarks of the HDU doctor. However, he feels that the
2.  Listen to concerns
incident should be reported because although unlikely to be Allow her to say what she wants to, without interruption.
causative, and a well-recognised complication of the procedure, a Find out the exact nature of her concerns. More often than
system failure led to the potentially dangerous situation not, venting of emotions is what people want, in this case
of non-removal of the malpositioned line. He has spoken to concern and likely confusion created by the careless words
the daughter and given these views. However, the patient’s of a colleague.
daughter, with whom you spoke on admission, would also like to
talk to you. 3.  Acknowledge concerns
Your task is to talk to the daughter and respond to her
concerns. First and foremost, make her see that you understand her
concerns. Make it very clear that you and the consultant

711
Station |4| Communication skills and ethics

are concerned about the incident. Tell her that she has a 10.  Document everything fully and carefully
right to know exactly what happened.
The vast majority of critical incidents are errors not of one
person but of a system within which fallible people
4.  Apologise, if appropriate operate. Discuss the incident with your consultant, docu-
Where it is clear that a mistake has been made, say how ment everything clearly and depending upon the signifi-
sorry you are that the incident occurred. In this case, cance of the incident discuss it confidentially with your
however, you should not admit to the allegation of the medical defence organisation.
HDU doctor but you do want to concede that a recognised
complication of the procedure occurred. You may also Discussion
need to say how sorry you are that she has been upset and
confused by apparently conflicting comments. What do you understand by the term
clinical governance (CG)?
5.  Do not criticise colleagues but give your view
The full definition, originally from the government paper
Colleagues have the same rights to be consulted and given ‘A First Class Service 2000’, is ‘a framework through which
the opportunity to seek advice, and ill-considered remarks NHS organisations are accountable for continuously
could lead patients or relatives to misleading conclusions improving the quality of their services and safeguarding
and prejudice a colleague’s interests. Colleagues should high standards of care by creating an environment in
never be criticised openly, but this does not necessarily which excellence in clinical care will flourish’. In short, it
mean dismissing or excusing a patient’s or relative’s means striving for the best quality, minimising harm and
expressed concern or complaint. While you should not maximising benefit within budget and resources.
criticise the HDU doctor (no matter how you may feel CG was given impetus by health-care disasters like
about his careless remarks) it is appropriate to say you Shipman and the Bristol Enquiry. Key areas of CG are out-
agree with the comments made by the consultant, rein- lined in Box 4.37. Ideally, CG should be proactive, but it
forcing what you see as correct information. Overall, you often needs to react to complaints, claims and incidents.
need to let her see how sorry you are that her mother’s
condition has deteriorated, without focusing so much on What is a critical incident?
the HDU doctor’s view.
A critical incident is an event that gives rise to, or has the
6.  Explain how the incident occurred potential to produce, unexpected or unwanted effects
involving the safety of patients. It is a serious event that
Explain, as clearly as possible, how the incident occurred.
harmed or could have harmed and as such would be likely
Explain that your action was in good faith in the best
to give rise to public concern or criticism of the service
interests of her mother, who was very unwell, and that you
involved. Annually in the NHS there are known to be
were deeply concerned to learn that you had placed the
around 850 000 adverse events and 28 850 000 com-
catheter in the artery as soon as it became known to you.
plaints. This may be the tip of the iceberg. Trusts have
clinical risk management teams to aid investigation of
7.  Work with facts – do not speculate!
clinical incidents and many have their own legal depart-
Since you should work with facts, do not speculate as to ment and solicitors.
why the catheter was left incorrectly in situ until the facts
have been established. Should you report all incidents?
8.  Give an assurance of further action All incidents should be reported, but, because of the enor-
mous number of them, grading systems or risk matrices
Explain that normally mechanisms should be in place to
recognise such a complication (chest X-ray +  / − blood
gases). Tell her that the incident will be reported as a ‘criti- Box 4.37  Key areas of clinical governance
cal incident’ to the risk management team, who will inves-
tigate it further and decide what action might be taken to • Clinical risk management, e.g. incident reporting
avert such an incident in future (the use of Doppler-guided • Education and training, e.g. standardised outcomes and
central line placement is now considered best practice). assessments
• Patient and public involvement, e.g. expert patients, copying
letters to patients
9.  Invite questions and provide further
• Staffing, e.g. appropriate selection
information if needed • Effective communication and use of information, e.g. use of
Ask if there are any other specific concerns she has not computerised imaging and electronic patient records
mentioned. Hopefully she accepts your candid response • Audit, e.g. local, national
to the incident but if she wishes to make a formal written • Research, evidence-based practice and clinical effectiveness,
complaint she should be informed of to whom to write. e.g. use of National Institute for Health and Clinical Excellence
In any event, you should offer to be of further assistance guidelines
should she have further questions or concerns.

712
Case 4.39  Critical incident

have been introduced that incorporate both their likeli-


hood and their consequence, to give priority to the more Box 4.38  National Patient Safety Agency
serious. (NPSA) approach
Promoting a safety culture
What do you understand by the term
This includes enhancing awareness and building safety into daily
system error? work with such activities as induction, handovers and audit.
Seldom is one individual wholly responsible for a critical
Leading and supporting staff
incident. The role of risk management teams is to investi-
gate the system within which individuals operate to iden- This includes clinical leadership and supporting staff ideas.
tify gaps that expose it to errors. The above case illustrates Integration of risk management activity
such gaps (e.g. the use of Doppler imaging for inserting This includes working across teams, departments, trusts and the
central venous catheters, chest X-ray confirmation post- NHS, e.g. the tracking of blood for transfusion from source to
procedure and the question of who is responsible for patient. It embraces all information such as incidents, complaints,
checking it if a patient moves to a different team’s care). litigation and audit, and balances costs with levels of risk. It is not
Incidents happen because of failures in the organisation’s feasible, for example, for all surgical instruments to be single use.
management, local conditions (e.g. understaffing, tired- Risk management at the directorate level includes risk and medical
ness, under-resourced), individual unsafe acts (e.g. missing management, critical incidents and audit, and at the governance
out a step in a procedure because short of time or multi- committee level are a health and safety committee, evidence-based
tasking) and what has been called ‘the final defence barrier practice, infection control and so forth.
breach’ (e.g. in an adequately staffed procedural room, a Promotion of reporting
doctor may have omitted to check the date and label on This includes reporting of incidents, near misses and methods for
the local anaesthetic but a nurse might have noted it). For investigation and review (information, mapping of event, barriers to
these reasons alone, it is never acceptable to rush to blame. safety). Barriers to safety include discontinuity of care,
communication failure and the different layouts of each ward, and
What is the National Patient Safety barriers to reporting include fear of blame, no clear benefit, ‘not my
job’, no feedback or ‘a waste of time’. A non-punitive atmosphere
Agency (NPSA)? is essential to promote reporting.
Everyone makes mistakes. The questions are why are they
made repeatedly, how can we learn from them and can we Communication with patients and public
eliminate some of them. The NSPA takes a seven-themed Key principles are honesty, apology, explanation and thorough
approach to this (Box 4.38). investigation.
Learning and sharing lessons
How is a hospital managed? Information needs to be disseminated by, for example, email,
A Trust Board has overall responsibility for scrutinising written memos, policy documents and discussion.
trust policy, implementing national / government policies Implementation of solutions
and advising the Trust Executive. The Trust Board com-
A National Confidential Enquiry into Patient Outcomes and Deaths
prises a chief executive together with executive and non-
(NCEPOD), for example, under the NPSA, recommends more
executive directors. Board executives are usually ex-bankers consultant involvement in clinical care to recognise and manage
or lawyers or business people, and sometimes nurses. critical illness.
The Trust Executive is the hospital’s functioning 
body. It comprises clinical directorates and corporate
directorates. clinical work. Roles include ensuring clinical and opera-
Clinical directorates typically include medicine (usually tional performance, changing strategies within the trust in
enormous), general surgery, specialist surgery, trauma and line with national policies and working with primary care
orthopaedics, child and women’s health, critical care /   trusts. Much work is ‘fire fighting’ – handling complaints,
emergency medical services, diagnostic services and  approving study leave and so forth.
professional services. Each clinical directorate provides  Each corporate directorate comprises a director and pro-
services, e.g. the disciplines of general medicine and  vides certain services. Corporate directorates might typi-
specialist medicine within the Medicine Directorate, and cally include the Chief Executive, the Medical Director,
physiotherapy, occupational therapy, speech and language finance and information management and technology
therapy and dietetics within the Professional Services (IM&T), human resources (staffing, occupational health),
Directorate. Each clinical directorate comprises a Clinical operations (e.g. clinical directorate management, opera-
Director (CD), directorate manager, a lead nurse and a tional performance, business planning, emergency prepar-
directorate accountant. These are all responsible to the edness), facilities (e.g. estates, catering, laundry, parking,
Medical Director, who in turn is answerable to the Chief security, fire, bookshop), nursing and service improve-
Executive. These people meet regularly. The CD is usually ment (e.g. complaints handling) and planning.
a doctor, who relies very much on the directorate manager The flow of NHS money is from the Department of
as he or she usually struggles to combine CD work with Health, which feeds it to primary care organisations,

713
Station |4| Communication skills and ethics

which feed via the Planning Directorate to NHS trusts will then follow 5 minutes of discussion with the examin-
(linked to strategic health authorities), who set their own ers. Do not take the history again except for details that
budgets. will help in your discussion with the patient / subject. You
are not required to examine the patient / subject.
What do you know about the Access to
Health Records Act? Patient / subject information
This gives patients the general right to see their medical
You are the daughter of Mr Jeremy Finch, a 64-year-old
records, obtain copies thereof and have the records
man who has been told by his doctor this morning that
explained to them. It applies only to records after 1
the result of his lung biopsy is that he has lung cancer. You
November 1991. A doctor may deny access (it is not that
are understandably upset although you had been con-
the whole record should be withheld but only specific
cerned for some time that he had been coughing up blood
information within it) on the grounds that it is ‘likely to
and it took you some months to persuade your father to
cause serious harm to the physical or mental health of the
come to hospital. You are, however, very upset that yester-
patient or any other person, or could lead to the identifica-
day evening a junior doctor on night shift came to take
tion of another individual (other than the health profes-
blood from your father with you present and commented
sional) who has been involved in the care of the subject’.
that very often these turn out to be benign. You are con-
An application must be in writing and made by a patient,
cerned because this is not the first ‘disaster’. When your
person authorised by the patient, person appointed by the
father was first admitted he was told he had pneumonia
Court or an executor. When application for access is made
and given a dose of penicillin when it was documented
by an individual on behalf of a patient who is incompe-
very clearly in his general practitioner’s referral that he was
tent or deceased, no information can be given that the
allergic to penicillin. You feel that the hospital makes one
patient had considered confidential and the holder is not
mistake after another and want to complain and may
required to explain why any part of the record has been
consider suing the hospital for negligence.
withheld. Viewing of the records should be provided
within 21 days (40 days for records at least 40 days old).
A reasonable fee may be charged. How to approach the case
Communication skills (conduct of interview,
exploration and problem negotiation) and
CASE 4.40  MANAGING A
ethics and law
COMPLAINT AND THE QUESTION OF
NEGLIGENCE 1.  Introduction and setting
Introduce yourself and confirm her identity. Show that you
are willing to listen, explain and help in whatever way you
Candidate information can. Angry or upset patients and relatives should never feel
threatened.
Role
You are a doctor on the medical ward.
2.  Listen to concerns / complaint
Please read this summary.
Allow her to say what she wants to, without interruption.
Scenario Do not take any criticism personally. Find out the exact
nature of her complaint. More often than not, venting 
of feelings to a sincere and responsible listener is enough
Re: Mr Jeremy Finch, aged 64 years for patients or relatives. Progression to a complaint is less
Mr Finch is a 64-year-old man on your ward who last week likely if you can facilitate this.
had an endobronchial biopsy for a suspicious lung lesion. Yesterday
evening he was told by a member of staff (you are not yet sure
who), in the presence of his daughter, that the lesion might well be 3.  Acknowledge concerns
benign, and yet this morning you received the formal report Let her see that you understand her grievances, and are
suggesting it is an adenocarcinoma. You discussed this sensitively dedicated to responding to these appropriately.
and Mr Finch seemed to accept the diagnosis but his daughter
now wants to talk to you.
Your task is to address his daughter’s concerns. 4.  Apologise, if appropriate
Where it is clear that a mistake has been made, say how
sorry you are that this incident occurred. Unless it is very
Your examiners will warn you when 12 minutes have clear that a complaint is unreasonable, it is always best to
elapsed. You have 14 minutes to communicate with the offer a sincere apology, and, if unreasonable, to say you
patient / subject followed by 1 minute of reflection. There are sorry the person feels that way.

714
Case 4.40  Managing a complaint and the question of negligence

5.  Do not criticise colleagues but give your view altercation. The General Medical Council recommends
that patients and relatives have a right to expect prompt,
Focus on any problem in the system, not on any
open, constructive and honest responses to their concerns.
individual.
This will include an explanation of what has happened
and, where appropriate, an apology. Communication is
6.  Explain how the incident occurred the key to ‘damage limitation’, while withholding facts or
Explain, as clearly as possible (if you know), how the retreating behind a wall of silence can inflame concerns.
incident occurred. Clear documentation is the key to defending actions now
that may be questioned at any time in the future. Happily,
7.  Work with facts – do not speculate! gratitude is overwhelmingly more common than
It is inappropriate to speculate (or worse, ascribe blame) complaints.
when you do not have all the facts; these should be estab-
lished, if need be, by proper inquiry. Emphasise the clini- Must you cooperate with a complaint,
cal imperatives – to now seek advice from the respiratory even if you disagree with the person
and cancer specialist about the best way to treat her father’s making the complaint?
condition.
You must cooperate fully with a complaint or formal
inquiry, although any complaint made that demands
8.  Give an assurance of further action about  
more than an initial verbal response should be handled
the complaint by the complaints procedure. You must provide informa-
Tell her that you will share the information with your tion as to whom to write – the complaints manager / com-
consultant and report the incident as a ‘critical incident’ plaints department, through whom all complaints are
to the risk management team, who will investigate things managed.
further and decide what action might be taken to avert
such an incident in future. What are the purposes of NHS complaints
procedures?
9.  Invite questions and provide further
These are to resolve a patient’s complaints (e.g. by explana-
information if needed tion or apology) and to improve NHS services. Where
Hopefully she accepts your candid response to the inci- compensation is sought, this is no longer a complaint to
dent but if she wishes to make a formal written complaint be managed by the complaints department, but a claim,
she should be told to whom to write. In any event, you and the patient must seek legal representation.
should offer to be of further assistance should she have
any further questions or concerns. What are the general levels of complaints
procedures?
10.  Document everything fully and carefully
There are two levels to which a complaint might go (Box
The vast majority of critical incidents are errors not of one
4.39). Most are locally resolved, with a response within 
person but of a system within which fallible people
20 days.
operate. Discuss the incident with your consultant, docu-
ment everything clearly and depending upon the signifi-
cance of the incident discuss it confidentially with your
Can an NHS complaints procedure be used to
medical defence organisation. discipline a doctor or award compensation?
No.
Discussion
Why do people make a complaint?
Box 4.39  Levels of complaint
People complain to vent anger, seek changes, receive an
apology, feel better, blame someone else and stop the Local procedures
same thing happening again. Sometimes they complain • Attempt to resolve complaints by local staff and managers
seeking compensation. People complain about clinical
care, staff attitudes, outpatient delays, poor communica- NHS ombudsman
tion, discharge and aspects of non-clinical care. • Can investigate, if certain criteria are fulfilled, usually in matters
of maladministration
How might complaints be avoided • Ombudsman decides whether to investigate, conducts
or minimised? investigation and reports to the NHS body involved,
disseminating findings widely and implementing action
Knowing why people complain can help minimise com- needed
plaints. Every effort should be made to avoid dispute or

715
Station |4| Communication skills and ethics

What sorts of medical error occur?


Box 4.40  Conditions for negligence
Common errors in order of frequency are:
1.  The professional had a duty of care to the
• Failure or delay in diagnosis
patient
• Medication error
• Unsatisfactory management The law imposes an onus on a doctor only if a professional
relationship exists between the doctor and the patient. The doctor
• Failure of supervision
has particular knowledge and skills and the patient consults them,
• Complication of procedure
as when a patient consults their general practitioner or is cared for
• Injury. by a doctor in hospital. A duty of care does not exist for strangers
Common missed acute diagnoses (which may lead to a (‘Good Samaritan’ acts), although a doctor may become liable if he
negligence claim) are: or she proceeds to treat improperly or causes harm. It is usually
easy to establish whether a duty of care exists.
• Myocardial infarction in younger women
• Infective endocarditis (often because investigated 2.  The professional was in breach of the
through a care pathway, e.g. anaemia investigation) appropriate standard of care
• Subarachnoid haemorrhage (with reliance on A patient must show that the doctor did not fulfil his or her duty.
computed tomography imaging, which misses   Doctors have a broad range of duties including making appropriate
10%) diagnoses, giving appropriate treatments and ensuring that
• Meningitis with delayed antibiotics patients are fully informed so that consent is valid. However, it is
• Diabetic ketoacidosis well known that equally knowledgeable, skilled and experienced
• Hyponatraemia due to Addison’s disease doctors may tackle a case differently. Therefore the Courts have
used the ‘Bolam test’ to decide whether a duty of care has been
• Occult neoplasm
breached – that duty is not breached if a ‘responsible body of
• Alcohol-related Wernicke’s encephalopathy or
medical opinion would have acted in the same way in the same
subdural haematoma situation’. It is also accepted that experts differ in opinion (e.g.
• Asthma being diagnosed without a peak expiratory treating cough, sputum and fever with antibiotics or considering it
flow rate viral) and either may be acceptable. The test, however, must be that
• Pneumonia without early antibiotics other experts would have done the same, and not ‘that could have
• Pulmonary embolism and deep vein thrombosis happened to me, I could easily have found myself in that situation’;
• Medication error. the latter analogous to driving at 40 mph in a 30 mph zone.
3.  Harm resulted
What are the conditions for negligence?
Negligence cannot be considered if no damage was incurred, even
People often use the term professional negligence loosely if there was a breach of duty.
to mean bad clinical practice. The legal definition is much
more precise. Legal action for negligence is the patient’s
4.  The breach of duty caused the patient’s harm
claim for compensation for losses caused by the profes- The patient must show that the doctor’s mistake or failure or
sional. For a successful claim, all of the conditions in Box action caused the injury or other losses. This is the most difficult
condition to prove, as breach of duty and harm may both have
4.40 must be proved.
occurred, but that the former caused the latter does not necessarily
These four conditions may be summarised as ‘duty, follow.
deviance, damage and due to’.

Is there a time limit for suing for an act


of alleged negligence? Where clearly indefensible (e.g. wrong finger amputated),
it will attempt to settle out of court rather than proceed to
A patient must sue within 3 years of the act of alleged civil court proceedings. Doctors are therefore witnesses in
negligence. For a complaint the time limit is 6 months, proceedings and not personally liable for payment of legal
often extended to a year. costs or damages, but their involvement in a claim may be
considered as a separate issue by the trust’s consultants or
Are individual doctors or trusts sued? directorates.
Before 1990 a patient or relative who wished to make a
civil claim as a result of NHS care sued an individual, Are damages awarded in cases
named doctor or health-care professional and the defence of negligence punitive?
of the claim was handled by the relevant defence organisa-
Any damages awarded aim to meet losses incurred, but not
tion. Since then ‘Crown indemnity’ has been introduced,
to exceed them. In other words, punitive damages are not
and it is the trust that is sued and the case is investigated
awarded.
and defended by the hospital’s legal team, solicitors and
the NHS Litigation Authority. The NHS Litigation Author-
ity has developed as a powerful centralised legal body for
What are vicarious liabilities?
the NHS. It decides on the basis of the four ‘D’s if it thinks NHS employers may be covered for cases of negligence by
the case if defendable, which in most cases it will be. the Crown Indemnity Scheme. Patients may sue health

716
Case 4.41  Fitness to practise – poor performance in a colleague

professionals personally but, provided certain conditions Your examiners will warn you when 12 minutes have
are fulfilled, they may choose to seek compensation from elapsed. You have 14 minutes to communicate with the
the employer. The advantages of vicarious liability to patient / subject followed by 1 minute of reflection. There
patients is that more substantial claims for damages may will then follow 5 minutes of discussion with the examin-
be affordable to employers and, if action is brought many ers. Do not take the history again except for details that
years after an event, the health professional may be harder will help in your discussion with the patient / subject. You
to trace. are not required to examine the patient / subject.

What is an inquest? Patient / subject information


This is a fact-finding enquiry about a reported death, con- Dr Stephen Cotton has been an F1 doctor (house officer)
ducted by the coroner, who requests written and sometimes for 4 months. He found his first 3-month post enormously
verbal evidence from a variety of sources. It is not a trial. difficult because there was simply too much work, and he
There are no parties, and there is no indictment, prosecu- was constantly being bleeped with more of it. Other staff
tion or defence. Possible verdicts in order of decreasing seemed not to be pulling their weight. He felt out of his
likelihood include natural causes, accidental death, indus- depth, and he found the F2 doctor (senior house officer)
trial disease and unlawful killing. The verdict for a patient was never there to help or advise him. He felt he did not
who has died from complications following an operation know what he was doing much of the time, and so simply
may be accidental death because the operation was planned did what everyone – usually nursing staff – told him to do.
but the outcome was not. An open verdict is returned if This frustrated him, as he was a very good student and had
the coroner believes there is insufficient evidence to reach great hopes of being a good doctor. For the past month he
a decision. The coroner’s court is a public court and the has been in a different post where the senior doctor is very
press may attend, as do a hospital’s legal team. helpful, but he is still not sure that he is performing well.
He has heard colleagues in the hospital mess laughing
about him because of his inefficiency but he does not know
CASE 4.41  FITNESS TO PRACTISE how he could work any harder. His senior colleague is
– POOR PERFORMANCE IN A about to talk to him about his performance.
COLLEAGUE How to approach the case
Communication skills (conduct of interview,
Candidate information
exploration and problem negotiation) and
Role ethics and law
You are a doctor on the medical ward. 1.  Introduction and setting
Please read this summary.
Ensure you have time to talk with him, in a non-pressured
environment, and make it clear the discussion is
Scenario
confidential.

Re: Dr Stephen Cotton, aged 25 years 2.  Open diplomatically


Dr Cotton has been your F1 doctor (house officer) for 1 month, Telling someone they are no good does not open dialogue.
having worked on a different ward for 3 months beforehand. He
Tell him that you wanted to discuss his experiences of the
has a further 2 months with you. You think his medical knowledge
is sound and do not doubt his genuine intention to be a good post so far. Explain that it is often difficult to judge how
doctor because he clearly works very hard and stays late after work a relatively new doctor is feeling about the job, although
to complete tasks he has been unable to complete during the day. it has become apparent to you that he works exceptionally
However, his work is ineffective. You believe his main problem is hard and feels a sense of duty to respond immediately to
inability to prioritise tasks. He tends to respond to every request every request.
made of him by nursing and other staff immediately, although
dropping what he is doing in the process. If he is on your ward 3.  Make it clear you are there to offer
round, for example, and a nurse tells him a drug chart needs to be constructive help
written in the next bay, he will leave to rewrite the drug chart. He
seems to see every task as equally weighted. Furthermore, when he Explain that this is an impossible task for any doctor and
is on your ward rounds he does not appear to listen to instructions if there are strategies to make the burdens of the job seem
or advice because he seems too nervous about all of the jobs he easier, and the remaining 2 months as educationally valu-
has to do. You can cope with a weaker member of the team, but able as possible, you would like to try to identify these.
are aware that colleagues are increasingly frustrated by his
performance. Some laugh about it behind his back. You feel he 4.  Listen to the experiences of the poorly
could benefit from confidential support and advice. performing doctor
Your tasks are to explore his poor performance and offer
constructive advice. Listen supportively, as the last thing he needs is for every-
body to takes sides against him. Allow him to tell you how

717
Station |4| Communication skills and ethics

he sees the job, which might be very different from how care is considered to be at risk, these doctors should be
you or your colleagues see it. identified with a view to further training or re-education,
or other help as necessary. A plan for this should be con-
5.  Share the good points structed in a tailored way in conjunction with the consult-
Show understanding of how a house officer’s job is difficult ant in charge and sometimes the postgraduate deanery.
– doctors must adjust from the learning environment of Re-appraisal will be necessary.
medical school to the intensely practical and often mundane
tasks of clinical work; when emergencies happen you can What types of problem doctor can
feel out of your depth; support can sometimes seem lacking; you identify?
there always seems to be too much work and not enough The General Medical Council (GMC) identifies the broad
time; and beyond all of this all doctors, as all people, need categories of poor performance (incompetence), miscon-
personal time and a life outside work. Tell him that you are duct (bad behaviour) and problems of physical or mental
in no doubt that he is a very hardworking doctor. health (sickness), although each of these is of variable
significance and often these overlap. It may be preferable
6.  Be honest about where you think to consider doctors as being in difficulty, often with a
performance falters conspiring cluster of causes including personality type,
Explain that where you think this falters is in his desire to behavioural predisposition and life events.
do everything, and now. Medicine, more than many jobs,
requires many generic skills, and one of these is prioritisa- Is the problem usually of a doctor in
tion. Explain that this is not a comment on his medical difficulty or a difficult doctor?
knowledge, which is perfectly satisfactory. It may be either.
7.  Identify problems and possible solutions Is the problem usually clinical or
Explain that the transition from medical school to house behavioural?
officer means seeking and using the support of more
It is invariably behavioural. Complaints about doctors are
senior staff. Perhaps explain that on your next ward round
usually to do with problems with attitude, communica-
he should not leave unless a job is urgent, and that you
tion, decision making, team working and insight. Behav-
and he can decide this together. Offer to supervise his work
iours may be:
more closely, constructively correcting any problems and
regularly teaching on ward rounds. Offer to meet at the • Work based, e.g. lateness, absence, work backlog
end of shifts to see what work is remaining and, of this, • Performance, e.g. over- or underinvestigating, poor
what needs to be done now, what can wait and what decision making, poor record keeping, complaints
should be handed over. Ensure that he is taking all leave • Cognitive, e.g. memory, attention and learning
to which he is entitled. problems
• Language / cultural
8.  Invite further questions • Psychological/personal, e.g. irritability,
Invite him to discuss anything not covered. unpredictability, forgetfulness, high self-
criticism / perfectionism, arrogance, lack of insight,
9.  Agree a plan excess risk taking
• Social, e.g. isolation, withdrawal, poor personal
Make and agree a definite plan, using the list of identified interaction.
problems and possible solutions.
Stress is largely related to personality (degrees of open-
10.  Offer ongoing help ness, conscientiousness, extraversion, agreeableness, neu-
roticism), rather than work environment. The strongest
Ensure he knows you will be an ongoing source of confi-
predictors of success at work are emotional stability and
dential help.
resilience, conscientiousness and intelligence. A high degree
of ‘neuroticism’ is less favourable. All behaviours may of
Discussion course be strengths or weaknesses, e.g. diligence can be very
productive or create impossible demands. Further, positive
Do you have a duty to identify poorly behaviours may become destructive if they go too far, which
performing doctors? they may do under stress, e.g. enthusiasm becomes volatility,
Poorly performing doctors should be identified and sup- careful becomes too cautious, focused becomes passive
ported in a proper manner. There should be a sense of aggressive, confident becomes arrogant, charming becomes
responsibility for these doctors, even where performance manipulative, diligence becomes perfectionism.
is not considered abjectly dangerous because doctors are
just as susceptible to personal problems and work difficul-
What are the causes of poor performance?
ties as anyone else and need support rather than being left These are wide ranging, e.g. poor knowledge or skills, dif-
to languish in this and subsequent posts. Where patient ficulty putting theory into practice, arrogance or failure to

718
Case 4.41  Fitness to practise – poor performance in a colleague

know limitations (usually poor insight or overcompen-


sated insecurity), poor communication, poor teamwork, Box 4.41  Duties of a doctor
offhand attitudes, laziness and dishonesty (almost Patients must be able to trust doctors with their lives and health.
undoubtedly the worst). Often one problem suggests that To justify that trust you must show respect for human life and you
a cluster may be present. Interestingly, those who think must:
they are good tend to overestimate themselves, and those • Make the care of your patient your first concern
who think they are not so good, underestimate! Good • Protect and promote the health of patients and the public
doctors can be bad doctors, e.g. the perfectionist who • Provide a good standard of practice and care
cannot delegate, and in general there are five working • Keep your professional knowledge and skills up to date
styles and we tend to each have a couple – ‘hurry up’, ‘be • Recognise and work within the limits of your competence
strong’, ‘be perfect’, ‘try hard’ or ‘please people’. • Work with colleagues in the ways that best serve patients’
interests
What should be done when a doctor in • Treat patients as individuals and respect their dignity
difficulty is identified? • Treat patients politely and considerately
Early recognition is imperative. One of the worst repercus- • Respect patients’ right to confidentiality
sions of failure to take remediable action as soon as  • Work in partnership with patients
possible is a doctor who becomes alienated and loses • Listen to patients and respond to their concerns and
allegiance to their team or organisation. preferences
The principles with any poorly performing doctor are: • Give patients the information they want or need in a way they
can understand
• Ensure patient safety – always the first question to • Respect patients’ right to reach decisions with you about their
answer (training is always a balance between patient treatment and care
safety and learning opportunity). • Support patients in caring for themselves to improve and
• Let the doctor know; too commonly he or she is the maintain their health
last to know. • Be honest and open and act with integrity
• Be objective. • Act without delay if you have good reason to believe that you
• Discover the cause (trainee, team or external factors?) or a colleague may be putting patients at risk
– clarify concerns, identify contributing factors and • Never discriminate unfairly against patients or colleagues
assess the capacity to improve. • Never abuse your patients’ trust in you or the public’s trust in
• Decide what action needs taking. the profession
Action at your stage is to speak to your consultant.
Further action depends on the problem, ranging from
informal action to, at the other extreme, a formal hearing.
The people who might become involved to help with train- Box 4.42  Good medical practice
ees are the clinical and educational supervisor, the clinical
tutor, and if ongoing or not resolvable the postgraduate Domain 1: Knowledge, skills and performance
dean and support services in the deanery, often in conjunc- • Maintain your professional performance
tion with the occupational health service if there are health • Apply knowledge and experience to practice
issues contributing; sometimes the National Clinical Ass­ • Ensure that all documentation (including clinical records)
essment Service (NCAS), an organisation which supervises formally recording your work is clear, accurate and legible
doctors with performance or educational problems (often
communication problems), acts on a strictly need-to-know Domain 2: Safety and quality
basis with that doctor, retaining considerable confidential- • Contribute to and comply with systems to protect patients
ity for the doctor. The NCAS mainly assists consultants and • Respond to risks to safety
non-training grade doctors, largely aiming to support them • Protect patients and colleagues from any risk posed by your
through the problems and considering fitness for purpose, health
but it does get involved with some trainees.
Domain 3: Communication, partnership and
List some duties of a doctor teamwork
• Communicate effectively
The GMC advises that to maintain a good standard of • Work constructively with colleagues and delegate effectively
practice and show respect for human life doctors should
• Establish and maintain partnerships with patients
respect those duties listed in Box 4.41.
Domain 4: Maintaining trust
List some key components of good • Show respect for patients
medical practice • Treat patients and colleagues fairly and without discrimination
The 2012 GMC guidance on good medical practice involves • Act with honesty and integrity
four domains (Box 4.42), updated from seven in 2006 – • Openness with legal / disciplinary procedures
good clinical care, maintaining good medical practice,

719
Station |4| Communication skills and ethics

teaching and training, relationships with patients, working Patient / subject information


with colleagues, probity and health.
Dr Harry Winter is a competent and very confident doctor,
A colleague is making small clinical errors. so much so that he often feels his senior colleagues need to
re-train. He has read a lot, knows a lot and gets very frus-
Who should be made aware?
trated when senior colleagues waste time with what he sees
This starts internally by involving your consultant. The as meaningless interventions and drug doses that do not
colleague should be asked for his or her side of the story seem to make a difference. He wants to be an emergency
from the outset. Depending upon the level of concern, the physician. He hates his current post on the elderly care ward,
matter might be referred to the lead clinician of your where things are too slow. In fact, he has contempt for most
department, the clinical director, the medical director or elderly care physicians, perceiving that a lot of resources are
the chief executive. It is also possible to report concerns ‘wasted’ on older people who do not seem to get better.
anonymously to the National Patient Safety Agency, which Yesterday, an 84-year-old woman with a total anterior circu-
can conduct a non-judgemental investigation. lation stroke developed aspiration pneumonia and was very
distressed. While his senior colleague was assessing her he
asked Dr Winter to ensure that 5 mg of morphine was pre-
pared and administered. Dr Winter felt this dose stupidly
CASE 4.42  FITNESS TO PRACTISE low and had given much higher doses to young trauma
– MISCONDUCT IN A COLLEAGUE patients in the emergency department. Dr Winter decided to
give 15 mg and although the woman became extremely
drowsy she is still alive today and this, he feels, vindicates his
Candidate information decision. He only prescribed 5 mg, however, knowing his
Role senior colleague would not have approved. The nurse with
whom he prepared the morphine challenged him about the
You are a doctor on an elderly care ward. discrepancy between what he had asked for and what he had
Please read this summary. prescribed. Dr Winter does not feel it is a nurse’s place to
question a doctor but to appease her said that he discarded
Scenario 10 mg as it was subsequently not needed. This is not the first
time he has given bigger doses than recommended or
Re: Dr Harry Winter, aged 28 years requested, but he feels he is right. The senior doctor now
Dr Winter, your new junior colleague, is developing a reputation wishes to talk to Dr Winter about these episodes, and he will
as a rather arrogant doctor who wants to be an emergency have to admit the truth, albeit exposing his contemptuous
physician. He has just come from the intensive therapy unit (ITU), convictions, because the alternative is that he is perceived as
where his advice to colleagues was either that their referred taking the morphine himself, which he is not. In fact, he
patients were too sick to come to ITU (‘wouldn’t survive thinks drug addicts should be shot.
anyway’) or too well to warrant ITU and he seldom if ever took a
patient. He has just rotated to working on your elderly care ward. How to approach the case
One of your patients, an 84-year-old woman with a total anterior
circulation stroke, developed aspiration pneumonia and was very Communication skills (conduct of interview,
distressed. While you were assessing her you asked that Dr Winter
ensure that 5 mg of morphine be prepared and then administered exploration and problem negotiation) and
to relieve her distress. He administered the injection and wrote up ethics and law
5 mg on the drug chart. Later, the nurse told you that Dr Winter
had asked her for 15 mg. She is sure that she made up 15 mg 1.  Introduction and setting
with him and it is documented accordingly on the controlled drugs Ensure you have time to talk with him, in a non-pressured
record. You are aware that not infrequently larger doses of opiates environment, as much for your sake as his.
are prepared than are used and the remnant amounts discarded.
But this is not the first time the same issue has been noted with Dr 2.  Open diplomatically
Winter.
Your task is to discuss the incident with Dr Winter. Tell him that you wish to discuss the discrepancy between
the dose of morphine apparently administered and the
dose given. Make it clear that you are not being judgemen-
tal, and that there are of course numerous potential
reasons this could occur. You should be aware, however,
Your examiners will warn you when 12 minutes have that if he is lying about drug doses that will be a very
elapsed. You have 14 minutes to communicate with the serious issue of honesty and probity.
patient / subject followed by 1 minute of reflection. There
will then follow 5 minutes of discussion with the examin-
3.  Make it clear you are there to offer
ers. Do not take the history again except for details that constructive help
will help in your discussion with the patient / subject. You Explain that, whatever the motives, you would hope to be
are not required to examine the patient / subject. able to offer constructive help (it is of course more difficult

720
Case 4.42  Fitness to practise – misconduct in a colleague

to do this when a doctor with poor conduct does not seem 9.  Agree a plan
to appreciate this).
Make and agree to a definite plan, using the list of identi-
fied problems and possible solutions. Tell him that the
4.  Listen to the experiences of the doctor with matter is not one that you can keep to yourself, and you
poor conduct will need to also discuss it with your consultant.
Allow him to tell his story. Try to identify, if he admits to
the act, his reasoning behind it and ask him if he saw any 10.  Offer ongoing help
alternatives, such as discussing his thoughts with his Offer to be a source of advice and help during his attach-
senior colleagues first. ment to the elderly care ward.

5.  Share any good points Discussion


Acknowledge that he is generally a knowledgeable and
competent doctor. What types of professional misconduct do
we sometimes hear about in doctors?
6.  Be honest about where you think These include (not in any order of importance):
performance falters • Lack of probity or honesty
Explain that you feel his ethical decision making is leading • Lack of reliability and poor time keeping
him into territory that is dangerous to patients and risking • Acting under the influence of alcohol or drugs
his professional integrity; explain that these actions could • Inappropriate or criminal behaviour
have very serious repercussions, not least threatening his • Inappropriate use of NHS facilities
future career. • Failure to follow organisational policies
• Bullying
7.  Identify problems and possible solutions • Sexual or racial harassment
With a lesser issue you might advise him that you would • Neglect of or disregard for responsibility to patients
wish him to discuss all treatment decisions while attached • Breach of confidence
to the elderly care ward with you. Tell him that medical • Unprofessional attitudes to work.
decisions are always within a framework that puts patient
safety first. Here, you must inform your consultant. What are the possible outcomes of
challenges to a doctor’s actions?
8.  Invite further questions Possible outcomes are listed in Table 4.10. Your first duty
Invite him to question your suggestions. is to speak to your consultant if you have concerns about

Table 4.10  Possible outcomes of challenges to a doctor’s actions

Procedure Who or what Who Purpose Criteria Possible outcomes


instigates conducts
NHS complaints Patients or Hospital To resolve Explanation or apology from
relatives managers or complaints and managers
independent improve service
panel
Negligence claim Patients Civil court Compensation ‘Bolam test’ / prudent Financial compensation to
with a judge patient test patients
Employer’s disciplinary Reports from Trust Protection of Breach of employment Warning, suspension or
proceedings staff managers patients contract or law dismissal
Professional bodies Health Professional Assessing fitness Guidance such as GMC Admonition, registration with
such as GMC professionals committees to practise /  guidance conditions, suspension from
public protection medical register or erasure
from medical register
Criminal negligence Crown Criminal court Prosecution /  Gross negligence Absolute or conditional
Prosecution punishment discharge, suspended
Service imprisonment or imprisonment
Coroner’s inquest A reported Coroner To determine Verdict on cause of death
death cause of death
GMC, General Medical Council.

721
Station |4| Communication skills and ethics

a doctor. Depending upon the seriousness of the problem, Your examiners will warn you when 12 minutes have
others involved include the Medical Director, the Chief elapsed. You have 14 minutes to communicate with the
Executive, the deanery and the General Medical Council. patient / subject followed by 1 minute of reflection. There
The police may be involved if there is criminal action. In will then follow 5 minutes of discussion with the examin-
the event of improper use of internet sites, trust IT depart- ers. Do not take the history again except for details that
ments may not inform the individual but have direct links will help in your discussion with the patient / subject. You
to the police who will take it extremely seriously. Other are not required to examine the patient / subject.
people who may be involved in misconduct investigations
are outlined in Table 4.10. Depending upon the action, Patient / subject information
restriction of practice or exclusion may ensue before inves-
Dr Emma Wood has been an F1 doctor in medicine for 3
tigations start. It is not acceptable to apply different out-
months. She was an average student, performing well in
comes to doctors as to other members of staff. If dismissal
psychiatry, but finds being a doctor exceptionally difficult.
is appropriate for a porter, then it is for a doctor.
Initially she found it a culture shock to have to see so many
patients, and felt that most doctors did not spend enough
Should you accept a gift from a patient?
time talking to their patients. She was told by one consult-
The general answer is no. Gifts that should not politely be ant that this is the modern NHS, there is a service com-
returned, such as chocolates, might be shared with other mitment to see patients and it is better to ‘get around all
members of the team; money should never be accepted and patients missing irrelevances’ than to see one or two in
the patient might be advised to contact the Trust Patient depth. She found this infuriating and against what she
Advice Service if they wish to make a charitable donation. found as a student in psychiatry, when she would fre-
quently put in a whole afternoon talking with one patient.
Should you always see a patient if a junior She now feels contempt for physicians. She cannot be
asks you to? bothered working with the team, which she thinks just
Patient safety comes first and if there are concerns about runs around in circles, and feels she is ‘destined for better
a junior’s level of competence this should be handled things’. She has been drinking more and more alcohol in
second and your conduct in protecting patients will be recent months – she always drank a lot as a student – and
paramount. You should see that patient. is aware that she comes to work after one or two morning
drinks ‘to steady her frustration’ but does not think her
colleagues have noticed. In any event, she enjoys it, and
CASE 4.43  FITNESS TO PRACTISE has increasingly been feeling that alcohol is a good thing
for young people earning money and hates being criticised
– HEALTH PROBLEMS IN A by people who tell her differently. Her senior is about to
COLLEAGUE speak to her about his concerns, and she may eventually
see that there could be a problem and agree to seek advice.

Candidate information How to approach the case


Role Communication skills (conduct of interview,
You are a doctor in the emergency medical admissions exploration and problem negotiation) and
unit. ethics and law
Please read this summary.
1.  Introduction and setting
Scenario Ensure you have time to talk with her, in a non-pressured
environment, and make it clear that the discussion is
Re: Dr Emma Wood, aged 26 years confidential.
Dr Wood, your new F1 doctor, has worked with you for 3
months and is persistently late for work. When at work, her 2.  Ask if the sick doctor recognises the problem
performance is highly substandard. Notably, she is extremely slow, Tell her that you are concerned she may be drinking
being able to clerk in one or two patients on days she is ‘on take’. heavily. Ask if she feels there may be a problem.
Initially, you gave her the benefit of the doubt as she told you she
found work very stressful. She feels that doctors do not give 3.  Make it clear you are there to offer
enough time to patients, declaring that patients have ‘emotional
needs as well as physical’. But you now think this a thin excuse. constructive help
She arrives at work smelling of alcohol, and colleagues have told Explain that you are concerned and that you would like,
you how she is often at parties all night or in her room with and may need, to help or intervene if there is a problem.
dubious friends drinking until an hour before work. On two
occasions you have sent her home because she was intoxicated. 4.  Listen to the experiences of the doctor
Your tasks are to confirm that she has an alcohol problem and
find ways to manage the problem. Listen supportively, but be aware that she may not admit
to having a problem. She may not see that she does.

722
Case 4.44  Recruitment to a randomised controlled trial

must give an honest explanation to an appropriate person


Box 4.43  CAGE questions in your health-care organisation, such as your consultant
• Have you ever felt you ought to Cut down your drinking? or the Medical Director. If local systems do not or cannot
• Have people Annoyed you by criticising your drinking? resolve the issue, then a professional organisation such as
• Have you ever felt bad or Guilty about drinking?
the GMC may need to be informed of your concerns.
If a colleague smells of alcohol and it is difficult to judge
• Have you ever had a drink first thing in the morning, an Eye
opener? if he or she is drunk, and this is a first time, then it is more
likely a case of local help and discipline with close obser-
vation. But any professional who smells of alcohol cannot
see patients.
5.  Ask specific questions In recent years, gambling has also become a health-
related concern.
Ask the CAGE questions (Box 4.43). Ask if anyone else has
In all cases, important considerations are:
hinted that there may be a problem. Crucially, you are
exploring whether or not she has insight into the problem. • To alert the individual of your concerns
Explore the possibility of depression. • To ensure patient safety, e.g. is immediate exclusion
from work needed?
6.  Be specific about your concerns • To involve your consultant, who should act upwards
depending upon the level of concern – Medical
If she denies a problem, suggest the evidence. Explain you
Director, Chief Executive, GMC
are concerned that continuing in this way may put patients
at risk. Explain you are also very concerned about the pos- • The health of the individual, e.g. occupational health
department.
sible implications for her, including not adequately
making it through house year and, ultimately, a risk of not
obtaining General Medical Council (GMC) registration.
Explain that if patients are at risk she has a duty to seek CASE 4.44  RECRUITMENT TO A
help and if she does not then it is necessary for colleagues
to intervene. Explain that you are obliged to speak to her
RANDOMISED CONTROLLED TRIAL
consultant.
Candidate information
7.  Explore any other concerns
Invite her to discuss anything not covered. Role
You are a doctor on the acute medical unit.
8.  Discuss possible solutions Please read this summary.
These include support from you, and help from her regis-
tered general practitioner and/or the occupational health Scenario
service, and confidential services for sick doctors including
the British Medical Association, the Sick Doctors’ Trust Re: Mr John Walker, aged 54 years
and the Doctors’ Support Network. You are conducting a randomised controlled trial (RCT) with
your consultant on inpatient insomnia. You have designed the RCT
9.  Agree a plan to involve four groups: no treatment, an eye-shield alone, earplugs
alone, and an eye-shield plus earplugs. Patients will be randomised
Make and agree a definite plan.
to one group for one night only and report their sleep satisfaction
on the basis of a rating scale.
10.  Offer ongoing confidential help Your task is to ask Mr Walker, a patient admitted with
Ensure she knows that you will be an ongoing source of dehydration and acute renal impairment, if he might agree to
confidential help. participate in the trial and counsel him about it.

Discussion Your examiners will warn you when 12 minutes have


elapsed. You have 14 minutes to communicate with the
If you suspect a colleague’s sickness is patient / subject followed by 1 minute of reflection. There
putting patients at risk, how do you decide will then follow 5 minutes of discussion with the examin-
when ‘whistleblowing’ overrides the ers. Do not take the history again except for details that
confidentiality of the colleague? will help in your discussion with the patient / subject. You
are not required to examine the patient / subject.
You must protect patients from harm posed by another
health-care professional’s health, conduct or performance Patient / subject information
and this includes problems of alcohol or drug misuse.
Patient safety comes first. Serious concerns should provoke Mr Walker is a 54-year-old mathematician who has been
immediate action to establish if they are well founded. You admitted with acute renal impairment caused by

723
Station |4| Communication skills and ethics

gastroenteritis and dehydration. He is about to be asked if is needed for research, doctors must seek consent for its
he might consider participating in a treatment trial, and disclosure whenever practicable, anonymise data where
will be very interested to know all about it, not least about this will suffice and keep all disclosures to the minimum
the safety aspects and the ethics of it being conducted. necessary. If consent is not practicable, then the ethics
committee may need to decide whether the likely benefits
How to approach the case of the research outweigh the loss of confidentiality.

Communication skills (conduct of interview, 7.  Discuss consent


exploration and problem negotiation) and If he would like to proceed then you would wish to obtain
ethics and law consent. Sometimes it is preferable to give patients time to
think about things. On no account must a patient feel
1.  Introduce yourself and the topic for discussion coercion. They also have the right to withdraw from
Explain that research is vital in improving the care of research at any time without reason. Any additional infor-
patients and that you would like to invite your patient to mation given, other than that required to fulfil a doctor’s
participate in research and the reason why. Ask if he is obligation to provide informed consent, will depend very
happy to discuss your project. much upon the individual. It should be determined by
effective communication, by not making assumptions and
2.  Explain the nature of the study by asking about specific concerns. An information leaflet is
often very helpful before discussing research with patients.
Explain the nature of the research, its aims and an outline
of the method. For such a randomised controlled trial 8.  Consider conflict of interest
(RCT), the nature of the randomisation and any reasons
Any conflict of interest should be declared from the outset
for it should be explained, although by its very nature this
and research should not be motivated by financial, 
RCT cannot be a double-blind RCT in which neither
personal, political or other interest. Ethics approval is
patient nor researcher know whether patient is receiving
mandatory.
treatment or placebo.
9.  Confirm understanding and invite questions
3.  Discuss ethics approval I appreciate we’ve covered rather a lot there. Is there anything
Explain that ethics approval has been obtained and that you would like me to go over again or are there any
the research does not come before the best interests of a questions you would like to ask me?
patient and that there are legal rights and safeguards. Any
foreseeable risks must never outweigh potential benefits 10.  Seek consent or refusal
and developing new knowledge or treatment should never And do not question his decision if he refuses.
take precedence over a patient’s interests. An RCT is not
ethical if there is evidence that an intervention is danger- Discussion
ous and an RCT must be stopped if evidence emerges for
a benefit or risk before completion. Doctors involved in What do you understand by the term
research have an ethical duty to respect human life and evidence-based medicine (EBM)?
people’s autonomy (if that research is an experimental
study into the causes, treatment or prevention of disease, EBM refers to the conscientious, explicit and judicious
involving people or their tissues, organs or data; audit is application of current best evidence in making decisions
not experimental study). about the care of individual patients. Practising EBM must
integrate external evidence with clinician’s expertise and
patient’s beliefs, concerns, expectations, values and
4.  Explain potential benefits and risks informed choice. Evidence-based health care extends the
There may be potential benefits and risks. Explain these, application of principles of EBM to all professions involved
and any arrangement for dealing with adverse events. in health care.
Explain that the results are not predictable.
What do you understand by the term critical
5.  Explain data handling reading or critical appraisal?
Explain that you will record and report all results accu-
The ability to evaluate scientific literature, analyse data,
rately. Explain that you will aim to complete the research
and base management decisions on the evidence.
unless evidence emerged of a risk or no potential benefit
to participants. Information about outcomes will be
available.
What broad types of research are there?
Quantitative research
6.  Discuss confidentiality This is concerned with inferences about populations and
Explain that all information will fully respect your patient’s testing theories. A collection of numerical data is subject
right to confidentiality. Where data or patient information to statistical analysis to test hypotheses. Things that are

724
Case 4.44  Recruitment to a randomised controlled trial

measurable by quantitative studies include disease fre- Subjects are selected before the outcome of interest is
quency (incidence and prevalence), factors that modify risk observed. It is not strictly possible to determine causality
of disease, methods for detection and diagnosis of disease, from a cohort study (something which tobacco companies
costs and consequences of health states and effects of treat- have used in their defence in the face of overwhelming
ments and interventions (clinical trials). evidence) but strength of association may be used as a
marker of risk. An RCT is needed to prove causality, but may
Qualitative research not be ethical in the face of a strong association. Cohort
This is concerned with understanding the experiences of studies may be used to determine prognosis.
individuals and using this to build theories. It is guided
not by hypotheses, but by questions and issues and seeks Clinical trial
to describe culture, beliefs and attitudes and to understand This is a planned experiment designed to assess the efficacy
behaviour. It may generate hypotheses for testing using of a treatment in humans. The RCT is the most powerful
survey or intervention designs. It may utilise, for example, experimental design to obtain evidence of causation or the
case reports, interviews (structured and non-structured), impact of an intervention. Patients are randomly allocated
health diaries, audio recordings, focus groups or group or randomised into an experimental group to receive an
interviews. intervention and a control group to receive standard treat-
ment or placebo. These groups are followed up and com-
What types of study design are there? pared for differences in outcomes of interest and any
differences are tested for significance. The process of infer-
Study designs may be non-interventional (case reports,
ence from an RCT will typically result in significance tests
cross-sectional studies, case–control studies, cohort
(P values), estimates of risk or treatment effect and confi-
studies) or interventional, which measure ‘before’ and
dence intervals (below). Advantages of an RCT include the
‘after’ (RCTs, systematic reviews, meta-analyses). Study
rigorous evaluation of a single variable in a defined patient
designs are listed below in the order of their ascending
group, the prospective nature eliminating bias, the search
hierarchy of evidence.
to falsify rather than confirm its hypothesis and the facility
for meta-analysis at a later date. Disadvantages include time
Case report
and cost, hidden biases such as imperfect randomisation,
This reports a single patient’s story. A report on a series of funding sometimes by pharmaceutical companies, and the
patients with an outcome of interest is a case series. use sometimes of surrogate endpoints.

Cross-sectional (descriptive) study Systematic review and meta-analysis


A representative sample of people (‘subjects’) is studied at A systematic review is a summary of the medical literature
a single point in time. It may be used to determine the that uses explicit methods to perform a thorough literature
prevalence of a particular disease. Subjects are selected search and critical appraisal of individual valid studies and
without regard to the outcome of interest. that uses appropriate statistical techniques to combine
these studies. Meta-analyses pool data from numerous
Case–control (retrospective) study trials, and the Cochrane Collaboration is one body that
This involves identifying subjects with a particular condi- has standardised this approach.
tion of interest (cases) and comparing or ‘matching’ these
with subjects without the condition of interest (controls). What is a clinical practice guideline?
Retrospective data are collected from patient records to try
to identify a past exposure or potential causal agent of A systematically developed statement designed to assist
interest that discriminates between the two groups. It is clinicians and patients in appropriate decision making.
not possible to demonstrate causality from a case–control
study; the association of A with B does not prove that A Are there guidelines for critically
causes B. Subjects are selected based on the outcome of appraising papers?
interest. Checklists to assist critical appraisal have been developed
for many different types of study and may include a clearly
Cohort (prospective) study specified objective, a design appropriate for the objective, a
This is an observational study of subjects without the condi- definition of the target population and whether generalisa-
tion of interest; they are selected and then followed up to tion from the study to the target population is possible,
monitor the occurrence of the outcome. Two or more subject selection (how the subjects were selected for inclu-
groups (cohorts) are selected on the basis of a difference in sion), clearly defined treatments / interventions, whether ran-
exposure to a particular agent of interest (e.g. drug, potential domisation was used for allocation (including treatment
toxin, smoking) and followed up to see how many in each masking / blinding and if not whether this could affect
group develop a disease or other outcome. Cohort studies may outcomes), justification of the sample size, the outcome
take years. A famous cohort study was that by Sir Richard measures (primary and secondary, with statement of
Doll to investigate smoking as a risk factor for lung cancer. reliability and validity) and completeness (compliance,

725
Station |4| Communication skills and ethics

drop-outs or missing data). Checklists to assist critical What is the null hypothesis?
appraisal have been developed specifically for RCTs. One
That there is no real difference between intervention and
such is the Consolidated Standards of Reporting Trials
control (relative risk = 1 or absolute risk = 0). Trials start
(CONSORT) checklist.
with this presumption, and then set out to disprove it. In
other words, evidence is based on an innocent until
What is bias?
proven guilty basis, a rather tortuous approach that
Bias refers to deviation of results from the truth, or proc- requires a ‘P’ value to be < 0.05 to be beyond reasonable
esses leading to such deviation. Selection bias in a sample doubt. We can never prove the null hypothesis, but only
is an error caused by systematic differences between groups try very hard to reject it.
of subjects or between those selected and those who are
not. For example, volunteers may differ from non- What is a ‘P’ value?
volunteers in health terms (motivated, free time, well).
Selection should really be random. Information bias refers The P value relates to the confidence that a difference
to a flaw in measurement that results in a difference in the between intervention and control groups is not the result
quality of information between groups. There are many of chance alone. Its value is arbitrary, and by convention
types of information bias: < 0.05 (i.e. a < 0.05 or < 1 in 20 chance of the difference
being due to chance alone) is deemed statistically signifi-
• Interobserver bias resulting from different observers cant. Sometimes tiers are considered, < 0.01 being yet
classifying outcomes stronger evidence and < 0.001 being very strong evidence.
• Recall bias resulting from incomplete recall, worse in The smaller the P value, the less likely the difference is to
a case–control study be the result of chance alone and the more likely it is to
• Lead-time bias, important in measuring survival if a be significant (but only if the sample size is large enough).
new procedure has been introduced (i.e. is an A significant result suggests that the author reject the null
apparent increase in survival because of early hypothesis, and a non-significant result implies either no
intervention or a parallel diagnosis?) difference or too small a sample. The P value must always
• Performance bias, resulting from differences in care be read in the context of the sample size and confidence
provided apart from the intervention (e.g. a doctor intervals. Formerly, P values were often given alone when
may or may not counsel as well as prescribing reporting studies but, now, confidence intervals tend to be
antidepressants) given also. Importantly, statistical significance need not
• Exclusion bias, which implies systematic differences in necessarily imply practical, real or clinical significance.
trial withdrawals Furthermore, absence of evidence is not the same as evi-
• Detection bias, which implies a systematic difference dence of absence!
in outcome assessment.
What do you understand by the term
What information might you need
confidence interval (CI)?
to determine the sample size necessary
for a study? If we were to repeat a trial hundreds of times, we would not
get the same results each time but on average we would
Sample size should be calculated by a statistician before establish a level of difference between the two arms of the
embarking on a study. A larger sample may yield a more trial. In 90% of the trials the difference between the two
significant result. Sample sizes are determined from: arms would lie within certain broad limits; 95% of the
• An estimation of prevalence of the outcome of interest trials would lie within certain, even broader limits and
in the population being studied or the control group. 99% within broader limits still. The CI is the range within
• ‘Clinical intuition’ to decide the minimal clinical which we would expect the true value of a statistical measure to
difference or smallest change in outcome between lie. It is basically the degree of wobble in a result that is
treatment and control groups that would be deemed based on a small sample population (those in the study)
clinically relevant. rather than the entire population. The CI is usually accompa-
• The significance level, which refers to the probability nied by the percentage value for the level of confidence that
of a significant result, when in fact there is no the true value lies within this range. For example, a number
difference. The smaller the better, but it could never needed to treat (NNT; see Case 4.6) of 20 with a 95% CI of
be 0% unless we were to study the entire population. 15 to 25 implies that we are 95% confident that the true
By convention the significance level is 5% or 0.05, NNT is between 15 and 25. It is standard to have 95% 
represented as the ‘P’ value (below). CIs. CIs can be applied to any statistical test, e.g. NNT, 
• Power, which refers to the probability that a study of relative risk, absolute risk, odds ratio, or sensitivity. A 
a given size statistically detects a real difference. The P value might be 0.05, but if, for the sake of example, the
bigger the better, and the more likely to detect CI for that P value were 1.1 to 25 then the result could
smaller differences, but power could never be 100% be of almost no significance because this CI is wide and 
unless we were to study the entire population; 80% 1.1 is close to the line of no significance. A wide difference
or 90% are by convention the standards. in results (a wide CI), especially if close to or crossing 

726
Case 4.45  Genetic testing

the line of no significance, is less significant. A narrow  Scenario


difference in results (a narrow CI), distant from the line 
of no significance, is more significant. The line of no 
Mr Roger Molton, aged 58 years
significance is 1 for ratios (negatives are not possible), such Mr Molton was recently admitted to the neurology ward for
as relative risk or odds ratio, and 0 for absolutes, such as assessment of a movement disorder. Genetic testing has recently
absolute risk. confirmed Huntington’s disease and you are about to speak to his
son, who wants to know more about the condition, particularly
What is a type 1 error? whether or not it is inherited and whether or not he and his
siblings should be tested.
The intervention is shown to work but it does not! This is Your task is to explain the diagnosis and counsel him about
equivalent to the P value. testing for this condition.

What is a type 2 error?


Your examiners will warn you when 12 minutes have
The intervention is shown not to work but it does! This is elapsed. You have 14 minutes to communicate with the
more common than a type 1 error. patient / subject followed by 1 minute of reflection. There
will then follow 5 minutes of discussion with the examin-
What do you understand by the term ers. Do not take the history again except for details that
generalisation? will help in your discussion with the patient / subject. You
are not required to examine the patient / subject.
Making inferences from a clinical trial involves a process
of generalisation from the sample to the target population.
The extent to which this is appropriate is known as
Patient / subject information
generalisability. You are the son of Mr Roger Molton, a 58-year-old man
who has recently been diagnosed with Huntington’s
What do you understand by the term disease. You know a little about the effects of the disease
confounding factors? and have heard that it might be inherited. You wish to
discuss the likelihood of being affected and if there is a
Mixing of two or more factors so that their individual test that you could have. You currently have no children
effect upon an outcome cannot be separated. but are concerned about the possible risk of passing the
condition on to any children in future.
What do you know about intention
to treat (ITT) analysis? How to approach the case
Drop-outs or non-respondents can alter a sample size and Communication skills (conduct of interview,
conclusions significantly. ITT is an analysis in which the exploration and problem negotiation) and
results in the control and treatment groups are analysed
with respect to the numbers of patients entering the study ethics and law
(regardless of whether or not they received treatment or 1.  Introduction
completed participation in the RCT). By including for
Introduce yourself and establish his identity.
analysis all entrants to the study, ITT analysis avoids biases
because of a failure of concordance and admits side effects
of a treatment causing subjects to drop out. ITT analysis
2.  Establish area for discussion and background
provides a pragmatic estimate of the overall benefit of knowledge
intervention in the population studied. Discover his pre-existing knowledge of Huntington’s
disease.

3.  Establish any family history


Other communication, ethical and
Try to obtain a family tree and ask if he knows of anybody
legal scenarios in the family who has had similar symptoms.

4.  Explore ideas, concerns and expectations


CASE 4.45  GENETIC TESTING
Ask if he wishes to know anything else about the disease
or its implications.
Candidate information
5.  Respond to ideas, concerns and expectations
Role Explain that Huntington’s disease is caused by an abnor-
You are a doctor on the neurology ward. mal gene that is passed on to 50% of children. Explain
Please read this summary. that the disease may not become apparent until later life

727
Station |4| Communication skills and ethics

in affected offspring, but that there is a tendency for suc- 10.  Agree a way forward
cessive generations to be affected at an earlier age. Explain
This may well be for him to think about the above before
that offspring who do not carry the gene cannot ever be
contemplating the next steps.
affected or pass on the condition.
Discussion
6.  Advise about genetic testing
Explain that genetic testing can determine whether he What is Huntington’s disease?
carries the abnormal gene. Explain that false-positive and Huntington’s disease is an inherited neurodegenerative
false-negative tests do occur, but are very unusual. Point disease characterised by progressive motor, cognitive and
out that there are potential medical, family, occupational, psychiatric symptoms. Chorea and loss of balance are early
insurance and financial implications of a positive result. symptoms that patients notice, although families often
Point out that there is no current prevention or cure for notice cognitive or personality changes first. The disease is
those carrying the gene. Point out that for those carrying most common in people of northern European origin. The
the gene there is no means of predicting age of onset of mean age of onset of symptoms is 40 years, but juvenile
symptoms, severity of symptoms or rate of progression. onset (< 20 years) and older onset (> 70 years) forms are
well recognised. Although relatively uncommon, Hunting-
7.  Advise about counselling for genetic testing ton’s disease can be devastating for patients and their
All doctors should have a grasp of the principles and be families. People who are at risk of developing the disease
able to respond to immediate questions and concerns. because of a family history face difficult decisions about
However, counselling for genetic testing is a specialised genetic testing.
skill and counsellors should be specifically trained and What are the clinical features of
part of a multidisciplinary team with psychosocial support.
Having confirmed his understanding of Huntington’s
Huntington’s disease?
disease, explored and responded to his concerns, and The disease was originally named Huntington’s chorea
explained the nature of testing, advise that a decision after George Huntington, who wrote the first detailed
about the test is something he should not make now, but description in 1872. The name has changed to Hunting-
consider discussing with those close to him. Advise that ton’s disease to reflect the fact that chorea is not the only
he might discuss the implications of the test and the con- important manifestation. Huntington’s disease progresses
dition further at a regional genetic centre if he wished, and over 15–20 years and characteristic symptoms reflect a
in the company of someone close to him, but at the very triad of motor, cognitive and psychiatric manifestations.
least your neurology consultant should be involved in the • Motor symptoms can be divided into two categories:
process. A minimum of 1 month is desirable between added involuntary movements such as chorea and
initial pre-test information and testing and results should impaired voluntary movements, which cause limb
always be given in person by the counsellor. Post-test incoordination and impaired hand function. These
counselling is mandatory whatever the result. symptoms are worsened by loss of postural reflexes.
The pattern of symptoms tends to change over time,
8.  Consider legal aspects with chorea declining and dystonia, rigidity and
As with human immunodeficiency virus testing, testing for bradykinesia becoming more marked.
Huntington’s disease should never be performed without • Cognitive impairment includes slowing of thought
counselling and explicit informed consent. Counselling processing and deterioration of executive functions  
must include up-to-date information about the condition (high-level cognitive processes that control other aspects
and the implications of testing that allow a patient to make of cognitive function). Typically, patients report difficulty
an informed choice. The test is available only to those who with multitasking, concentration and short-term
have reached the age of maturity. Ownership of the test memory. Thinking style becomes more concrete and less
result is with the subject who requested it but ownership efficient, and the planning, initiation and organisation of
of the stored DNA is with the patient. DNA from another time, thoughts and activities become harder. People with
affected family member may be needed, who of course Huntington’s disease are often impulsive and develop
must give consent, but asking an affected family member psychomotor perseveration. Visuospatial perception can
who is unaware of or unwilling to acknowledge their symp- also deteriorate
toms may be considered an invasion of privacy. Care • Psychiatric symptoms include depression, obsessive–
should be taken when a test result may provide informa- compulsive disorders, anxiety, irritability, apathy,
tion about a third party who has not requested a test. Oral hypersexuality (uncommon) and psychosis
and written information should be provided by the team (uncommon). Suicide is not uncommon.
providing the testing service.
What is the genetic basis of
9.  Invite questions Huntington’s disease?
Ask if he has any other questions or concerns that have Huntington’s disease is a single gene disease with auto-
not been addressed. somal dominant inheritance. The genetic abnormality is

728
Case 4.46  HIV testing

an expanded CAG trinucleotide repeat within the hunting- How is Huntington’s disease managed?
tin (HTT) gene on chromosome 4, and it can be identified
The aim of treatment is to manage symptoms and improve
through genetic testing. The HTT gene encodes the protein
quality of life. No current treatments can slow disease
huntingtin, essential for normal neural development,
progression. The choice of drugs for symptom relief is
although its functions are incompletely understood. In
based largely on clinical experience rather than evidence.
Huntington’s disease the expanded HTT gene encodes a
Tetrabenazine has the best evidence of efficacy in Hunt-
mutant form of huntingtin protein, which leads to the
ington’s disease and has been shown to reduce chorea.
development of Huntington’s disease through many path-
Many non-drug measures are effective, within a multidis-
ogenic mechanisms. Offspring of an affected parent have
ciplinary approach. Planning for end-of-life care raises
a 50% chance of inheriting the genetic abnormality, and
several ethical problems often relating to how far medical
males and females are affected equally. Huntington’s
interventions should be pursued when patients no longer
disease does not skip generations.
have capacity to make their wishes known. Advanced deci-
sions to refuse treatment can be extremely helpful. As
How is genetic testing undertaken? Huntington’s disease progresses, it often becomes increas-
Genetic testing for Huntington’s disease is performed by ingly difficult to provide care at home, and a nursing home
measuring the CAG repeat length in the HTT gene. A ‘posi- may be needed. Insertion of a gastrostomy tube may be
tive’ test result refers to CAG lengths in the pathogenic appropriate in patients who are unable to maintain ade-
fully penetrant CAG repeat range of more than 39 repeats. quate nutrition, depending upon extent of disease and
Testing falls into two categories. pre-emptive information and wishes.
• Diagnostic testing is carried out to confirm (or
refute) the diagnosis in a patient with symptoms
suggestive of Huntington’s disease. It is a test for CASE 4.46  HIV TESTING
manifest disease and is most commonly undertaken
by neurologists. A positive diagnosis has numerous
implications for family members (especially children
Candidate information
and siblings) and for the patient. Role
• Predictive testing is carried out in a person who has
no symptoms of the disease, but who is at risk You are a doctor on the medical ward.
because of their family history. It determines whether Please read this summary.
that person carries the expanded HTT gene and will
Scenario
develop Huntington’s disease in the future. A
positive predictive test result indicates that they will
certainly develop Huntington’s disease at some point Re: Mr Julian Lyons, aged 36 years
if they do not die from something else. Mr Lyons is a 36-year-old ex-intravenous drug user, who has
abstained from drugs for 3 years. He has been admitted to your
ward with a respiratory illness, mild hypoxia and X-ray findings that
What ethical considerations surround would be consistent with Pneumocystis jiroveciI pneumonia. He has
genetic testing? also been losing weight. Your consultant has asked that you
discuss an a human immunodeficiency virus (HIV) test with him.
Confidentiality and consent Your tasks are to explain the possible diagnosis, approach the
possibility of HIV infection and counsel him for an HIV test.
Strict confidentiality is observed before, during and after
predictive testing. Written informed consent must be
obtained from the patient before either predictive testing Your examiners will warn you when 12 minutes have
or diagnostic testing. If the patient lacks capacity to make elapsed. You have 14 minutes to communicate with the
the decision him- or herself for diagnostic testing, consent patient / subject followed by 1 minute of reflection. There
can be given by an authorised representative. will then follow 5 minutes of discussion with the examin-
ers. Do not take the history again except for details that
will help in your discussion with the patient / subject. You
Having children
are not required to examine the patient / subject.
Deciding whether or not to have children is often difficult
for people with or at risk of having an expanded HTT gene Patient / subject information
and is outside the scope of this text.
Mr Julian Lyons is a 36-year-old man who used intravenous
What are the implications of a positive heroin until 3 years ago. He admits to having shared
needles. He has been admitted to hospital with a short
gene test? respiratory illness and his doctors suspect human immuno-
There are major emotional and practical implications and deficiency virus (HIV) infection as a possible underlying
testing and counselling should be undertaken by experts cause. He has been slowly losing weight over the last few
in the condition. months. He is heterosexual, currently unemployed, and

729
Station |4| Communication skills and ethics

lives with a dog. He smokes 20 cigarettes a day and drinks


a moderate amount of alcohol. He has never had an HIV Box 4.44  Counselling with respect to human
test. The possibility of HIV has crossed his mind and he is immunodeficiency virus (HIV) testing
willing to undergo an HIV test but first would like to know • Ask if he has ever had an HIV test.
more about implications of the test. • Ask if he would consider one, reassuring him that the test is
entirely confidential. Emphasise that it is a test for the presence
How to approach the case of HIV, and not an ‘AIDS test’.
• Explain how the test is performed – a simple blood test, the
Communication skills (conduct of interview, result usually available the next day.
exploration and problem negotiation) and • Ask what impact he suspects a positive result would have on him.
ethics and law • Emphasise the benefits of the test (early treatment, knowledge
of infectivity to others).
1.  Introduction • Ascertain if he has a regular sexual partner. Explain that, if
Introduce yourself, confirm his identity and ask how he is. positive, it would be essential to tell his partner and desirable
that his general practitioner be informed. He may ask whether
2.  Explain the results of tests life assurance policies would be affected. Generally, if negative,
individuals are not discriminated against. If positive, all previous
Explain that the chest X-ray suggests pneumonia, and that policies should be honoured, provided all relevant information
there are a number of possibilities underlying it. was faithfully disclosed at the time. Future policies may be
difficult take out.
3.  Explore risk factors, suggesting possible • He may ask if he has acquired immunodeficiency syndrome
implications of the results (AIDS). Explain that the test is for HIV, but that if positive and
tests confirm Pneumocystis pneumonia, then that is an
Ask about his heroin use. Did he share needles? Were any
AIDS-defining illness. Tell him that he does not have to decide
of his associates known to be HIV positive? When did he immediately as to whether or not to proceed, but you will
stop using intravenous drugs? Ask about other risk factors continue investigating and treating his pneumonia.
for HIV acquisition, especially the sexual history. These
questions suggest that HIV is on your mind and if he does
not respond suggest that HIV is something that must 10.  Discuss what would happen after the test
always be considered. Post-test counselling is mandatory, whether a positive or
negative result, and the pre- and post-test counsellor
4.  Explore ideas, concerns and expectations should really be the same person.
Ask him if this is something that has crossed his mind, or
if he has ever considered the possibility of HIV infection. Discussion
Does a negative test exclude HIV infection?
5.  Respond to ideas, concerns and expectations
No. The window period means that testing must be
Explain what HIV and acquired immunodeficiency syn-
repeated if there is a possibility that infection has been
drome (AIDS) are, and how HIV is transmitted (and how
acquired in the last few months. The management of HIV
transmission can be reduced).
infection is further discussed in Case 2.39.
6.  Counsel with respect to HIV testing
Pre-test counselling should prepare an individual for
receiving, understanding and managing a result. Discus-
CASE 4.47  NEEDLESTICK INJURY
sion should embrace the consequences of a positive result,
including impact on partners, pregnancy, employment Candidate information
and insurance (Box 4.44).
Role
7.  Give an assurance of confidentiality You are a doctor in the emergency medical admissions
This is essential. unit.
Please read this summary.
8.  Invite questions Scenario
Ask if he has any other questions or concerns that have
not been addressed. Re: Dr Tess Martin, aged 25 years
Dr Martin is an F1 doctor (house officer) working with you. She
9.  Seek consent to proceed with the HIV test comes to you extremely upset because she has sustained a
Agree that he should decide whether or not to go ahead needlestick injury while taking blood from a patient she has just
clerked, who is an intravenous drug user.
with the test, and if he goes ahead who will inform him Your tasks are to calm the situation and advise about
of and explain the result, and that you will continue to managing the needlestick injury.
investigate and treat his pneumonia.

730
Case 4.47  Needlestick injury

Your examiners will warn you when 12 minutes have 6.  Explain what you will do next
elapsed. You have 14 minutes to communicate with the
Explain that you will counsel and take a blood sample
patient / subject followed by 1 minute of reflection. There
from the patient for HIV and hepatitis B and C (this
will then follow 5 minutes of discussion with the examin-
should not be done by the individual who was exposed).
ers. Do not take the history again except for details that
Explain to Dr Martin that she should be encouraged to
will help in your discussion with the patient / subject. You
have a blood test also for HIV and hepatitis B and C and
are not required to examine the patient / subject.
offer to counsel her to do this (or, as is common these
days, these duties are performed by the emergency depart-
Patient / subject information ment staff, who will arrange counselling and testing of
Dr Tess Martin is an F1 doctor (house officer) working on patient and staff member for baseline bloods and repeat
the emergency medical admissions unit with her senior testing of staff member 3 months later; if the patient lacks
(the candidate). She has just sustained a needlestick injury capacity blood may be taken and stored).
from a patient who is a known intravenous drug user,
admitted with a soft-tissue infection. The patient told her 7.  Respect confidentiality and consent
that tests for human immunodeficiency virus (HIV) and Assure her of confidentiality. Advise that, without a test
hepatitis 2 years previously were negative, but that his status now, if she were later found to be positive it might be dif-
had not been checked since and he has continued to use ficult to claim compensation (industrial disablement
intravenous drugs. Dr Martin wore two pairs of gloves for benefit). Confidentiality for both parties is essential and
the procedure. After withdrawing the green needle it punc- of course counselling and consent for HIV testing are
tured her gloves and ‘pricked the end’ of her left index mandatory.
finger, drawing blood. She immediately, quite correctly,
washed the area with soap and water without scrubbing,
and allowed the injury site to bleed freely. She is not sure 8.  Explore outstanding concerns
about the risks of HIV and hepatitis C in this situation, nor Explore any other concerns she may have or if there is
what she should do next. She is fully immunised against anything she is not sure about.
hepatitis B with an excellent antibody response.
9.  Invite questions
How to approach the case Ask if she has any other questions.
Communication skills (conduct of interview,
exploration and problem negotiation) and 10.  Consider the remainder of the shift
ethics and law Suggest that she might wish to go off duty and that you
will arrange the necessary cover.
1.  Introduction
Ask Dr Martin exactly what happened and reassure her
that you will help and follow the hospital guidelines for
Discussion
dealing with a needlestick injury. What is the risk of HIV transmission
2.  Advise on immediate management from a positive patient following
Advise her to wash the contaminated area thoroughly and needlestick injury?
allow the wound to bleed freely (if she has not already Around 0.3%, but individual risk depends upon the
done so). nature of the inoculation and viral load transmitted,
together with the HIV subtype and immunological factors
3.  Establish details of the incident in the person sustaining the injury.
Establish the nature of the injury, the type of needle used
and the time of the incident.
What action is indicated in the setting of
4.  Establish details about the patient and the needlestick injury if a patient is known to
staff member be positive for HIV?
Establish whether HIV and hepatitis B and C status are Action points are listed in Box 4.45.
known. Ask if Dr Martin has been immunised against
hepatitis B. What factors increase the risk of
5.  Remain calm occupationally acquired HIV transmission?
Explain immediately that the risk of infectious disease These include deep injury, visible blood on the device
transmission is not all that high from a positive patient, causing the injury, injury with a needle that has been
and that she has acted quite correctly so far to reduce any inserted in the source patient’s vein or artery and terminal
risk even further. HIV-related illness in the source patient.

731
Station |4| Communication skills and ethics

Your examiners will warn you when 12 minutes have


Box 4.45  Action for needlestick injury from a elapsed. You have 14 minutes to communicate with the
human immunodeficiency virus (HIV) infected patient / subject followed by 1 minute of reflection. There
individual will then follow 5 minutes of discussion with the examin-
ers. Do not take the history again except for details that
• An anti-HIV triple therapy starter pack should be given within the
first hour. The occupational health department can advise on up- will help in your discussion with the patient / subject. You
to-date local guidelines (national guidelines have been developed are not required to examine the patient / subject.
on post-exposure prophylaxis for health-care workers by the Chief
Medical Officer’s Expert Advisory Group on AIDS), and if outside
normal working hours the hospital senior nurse should know
Patient / subject information
where the latest guidelines are and where treatments are kept. Mrs Beth Evans is an 80-year-old widow, hitherto living
• Health-care workers exposed to infectious patients should be alone in a bungalow. She was admitted to hospital 10 days
given the opportunity to discuss the balance of risks with ago following a fall on her steps (there are three steps up
appropriate psychological support, particularly allowing
informed decisions about post-exposure prophylaxis. to the front door of her bungalow). She has fractured her
• Post-exposure prophylaxis employs combination therapy for 4 weeks. left neck of femur and this requires surgery. Surgery has,
• Pending follow-up and absence of seroconversion, health-care however, been postponed so far because of pneumonia,
workers need not modify their work. confirmed when she came into hospital. This has been
• All exposed individuals should be given advice on safe sex and treated successfully, and now the orthopaedic team would
avoiding blood donation. like a medical assessment, mostly to see whether Mrs Ever-
• Follow-up is through the occupational health department and dene is fit to undergo surgery. Her past medical history
HIV retesting is performed at 6 months. comprises osteoporosis confirmed by a bone densitometry
scan, angina (only on exertion) and heart failure, and her
medications are aspirin, furosemide and a nitrate spray.
Would your management be any different if She also takes a hormone replacement patch, and has done
your house officer were pregnant? for 20 years, as it relieves hot flushes and she was once told
She should tell her obstetric consultant of the incident, that it helped osteoporosis. Her general practitioner once
and this is essential if the incident involves an infected suggested stopping it but she does not think ‘she could live
patient. Zidovudine and lamivudine are considered safe in without it’. Mrs Evans will accept the opinion of the medical
the second and third trimesters of pregnancy but experi- team as to whether she is yet fit for surgery but is worried
ence with other treatments is limited. However, the advan- about her heart and will ask the doctor for an honest view.
tages are generally deemed to outweigh the risks and
reduce the risk of vertical transmission. How to approach the case
Communication skills (conduct of interview,
CASE 4.48  MEDICAL OPINION ON exploration and problem negotiation) and
FITNESS FOR ANAESTHESIA ethics and law
1.  Introduction
Candidate information Introduce yourself, confirm her identity and ask how 
she is.
Role
You are a doctor on the medical ward. 2.  Clarify the task
Please read this summary.
Explain that you are a physician and are here to advise on
Scenario her medical problems and help optimise her treatment
before any operation. Explain that you are not the anaes-
Re: Mrs Beth Evans, aged 80 years thetist, who would be responsible for deciding whether or
Mrs Evans was admitted to hospital 10 days ago under the not she is fit for an anaesthetic.
care of the orthopaedic surgeons following a fractured neck of
femur. On admission her course was complicated by right lower 3.  Explore symptoms
lobe pneumonia from which she is recovering well on your ward.
Her past medical history includes osteoporosis, ischaemic heart Explore the symptoms of ischaemic heart disease and
disease and heart failure and current medications are antibiotics, heart failure and how these may limit her and what medi-
aspirin, furosemide, a nitrate spray and topical hormone cations she has taken in the past.
replacement therapy. Blood pressure is currently 160 / 90, heart
sounds are normal with no added sounds or murmurs and there 4.  Explore the past medical history
are bibasal crackles. Renal function and blood haemato-
biochemistry are normal, and her total cholesterol is 5.6 mmol / l. Explore the history of osteoporosis, how it was first diag-
Your task is to assess the lady’s medical condition with a view nosed (e.g. previous fractures, strong family history of
to whether she might undergo hip surgery. osteoporosis or fractures) and enquire about the hormone
replacement therapy – how long she has been on it, and

732
Case 4.48  Medical opinion on fitness for anaesthesia

for what reason (osteoporosis, symptoms of oestrogen Discussion


deficiency such as flushes). Ask about other medical prob-
lems or medications. Ask about previous operations and List some risk factors for anaesthesia
anaesthesia. Ask about any allergies. Age is an independent risk factor (a strong predictor for
adverse cardiorespiratory outcomes such as arrhythmias,
5.  Explore ideas, concerns and expectations hypotension and death) but anaesthetists look more
Explore any concerns she may have about her medical closely at factors such as comorbidity, obesity, reflux and
problems or the operation. a difficult airway. Exercise tolerance in daily life reflects
biological age and functional status can be measured in
6.  Explain and advise as clearly as possible metabolic equivalents or METs (Table 4.11): 1 MET cor-
Suggest that you would like to examine her, especially her responds to oxygen consumption of 3.5 ml / kg / min
heart and lungs, and review the electrocardiogram (ECG) (70 kg 40-year-old man at rest). Excellent functional
and chest X-ray. Suggest that you may recommend optimis- reserve is > 7, moderate 4–7 and poor < 4 (Table 4.11).
ing treatment of her problems, notably with a view to
reducing the future risk of heart attacks and fluid on the Why are surgery and anaesthesia
lungs. Ask if she is willing for you to adjust her medication such a threat?
regimen, and that the adjustments will probably include Surgery causes blood loss and fluid shifts. Post-operatively
adding in a drug known as an ‘ACE inhibitor’ (beware pre- surgery causes pain (with reduced coughing and increased
operatively) and a statin (to reduce cholesterol and stabilise risk of respiratory infection), limits movement and triggers
endothelium) and possibly another diuretic (spironolac- the stress response of sympathetic activity, glucocorticoid
tone). A beta blocker may be considered later. An echocar- activity and fluid and salt retention. There is also reduced
diogram would be useful. Explore the past diagnosis of enteral absorption, altered immune function and altered
osteoporosis (presumptive or confirmed by bone densit- coagulation. These changes peak in the first 2 days but
ometry) and the reason for her taking hormone replace- linger for weeks.
ment therapy (osteoporosis or symptoms of oestrogen Most intravenous anaesthetic drugs have adverse cardio-
deficiency such as flushes and atrophic vaginitis). Suggest vascular effects, reducing afterload and having negative
that you would strongly advise the hormone replacement ionotropic effects. They cause apnoea necessitating posi-
therapy being withdrawn, as the risks (venous thromboem- tive pressure ventilation and inhibit mucociliary function
bolism, arterial disease, breast cancer) now very much for 24 hours. They adversely affect renal function. Propo-
outweigh any potential benefits (osteoporosis prophylaxis) fol, an induction agent, reduces systolic pressure by 40%
and that there are now better treatments for osteoporosis on average in elderly patients. Regional anaesthesia vasodi-
than hormone replacement therapy (a bisphosphonate lates the affected area and interrupts sympathetic innerva-
with calcium and vitamin D supplementation). tion of the heart and can result in marked bradycardia and
hypotension if near the higher thoracic outflow.
7.  Repeat important information Anaesthesia affects thermoregulation. A drop of 2°C
This can be an effective way of emphasising ‘take home’ centrally in the first 20 minutes is normal but can take an
messages.’ hour or two to recover with active warming. This hypo-
thermia has a profound effect on cardiorespiratory, cere-
8.  Confirm understanding bral, clotting and immune function.
Ensure that she understands your advice.
What cardiovascular problems are provoked
9.  Encourage feedback and invite questions by anaesthesia?
I appreciate we’ve covered rather a lot there. Is there anything Increased arterial stiffness with age increases afterload
you would like me to go over again or are there any such that there is little change in cardiac output with an
questions you would like to ask me?
Table 4.11  Exercise tolerance measured in metabolic equivalents
10.  Agree a way forward (METs)
Physicians are frequently asked to give an opinion on
whether or not a patient is ‘fit for surgery’ or ‘fit for anaes- MET Activity
thesia’, and the crucial decisions as to whether the patient 1 Self-care
would tolerate anaesthesia and the appropriate mode of Using toilet
anaesthesia must be made by an anaesthetist. Explain to Walking at 2–3 mph
patients (and the team making such a request) that you 4 Light housework
are not an expert in anaesthesia or the right person to Climbing a flight of stairs or a hill
judge anaesthetic fitness, but that you are delighted to Walking at 4 mph
advise on optimising medical care. Your role is to give a Running a short distance
> 10 Dancing, golf
medical opinion that can be used to optimise a patient’s Highly strenuous sports – swimming, football
health prior to surgery.

733
Station |4| Communication skills and ethics

increase in as the left-ventriclar end-diastolic volume Your examiners will warn you when 12 minutes have
increases. Consequently: elapsed. You have 14 minutes to communicate with the
• Atrial systole is even more important in situations patient / subject followed by 1 minute of reflection. There
where it is less likely (!) and anaesthetists do not will then follow 5 minutes of discussion with the examin-
like to anaesthetise patients with fast or variably ers. Do not take the history again except for details that
controlled atrial fibrillation except in emergencies. will help in your discussion with the patient / subject. You
• Reduced diastolic time reduces coronary flow and ST are not required to examine the patient / subject.
changes on the ECG are common, especially at
induction. These changes, often associated with Patient / subject information
hypotension and tachycardia, may respond to fluid
Mrs Julie Blyth is a 36-year-old single mother of three
challenges but the sting in the tail is that elderly
children. She has type 1 diabetes and takes the combina-
patients are less tolerant of this.
tion of long- and short-acting analogues glargine and
Post-operative myocardial ischaemia remains the biggest Novorapid respectively. She is very afraid of long-term
cause of death and morbidity in elderly patients. Myocar- diabetes complications and runs her blood glucoses very
dial infarction tends (anecdotally) to occur on the second tightly between 3 and 4 mmol / l, which she believes to be
post-operative night. Systolic murmurs are extremely the normal range. She has frequent hypoglycaemic symp-
common. A good exercise tolerance and ECG are usually toms (sweats, light-headedness), but does not recognise
reassuring pre-operatively, but significant aortic stenosis these as such. She feels she must drive. She drives a car to
combined with anaesthesia can lead to a disastrous drop pick her children up from school and a small delivery van
in afterload. Heart failure does not blend well with anaes- for an income. She is very concerned that the doctor in the
thesia for all of the reasons outlined. diabetes clinic could ‘take her licence away.’

Is hypertension a risk factor for How to approach the case


anaesthesia?
Communication skills (conduct of interview,
For every 10 mmHg in systolic pressure above 140, there
is an odds ratio of 1.2, such that a systolic pressure of 180 exploration and problem negotiation) and
doubles the risk of infarction. ethics and law
1.  Introduction
What medical problems are relevant to
Introduce yourself and confirm her identity.
anaesthesia?
Other important problems include respiratory problems 2.  Provide initial reassurance
with reduced functional residual capacity or low oxygen Reassure her that you wish to try to establish why she is
saturation (higher inspired oxygen is needed and positive having symptoms and aim to correct these, rather than
pressure ventilation increases ventilation / perfusion mis- recommend that she does not drive.
match), respiratory tract infection, reflux, confusion,
alcohol misuse, electrolyte disturbance, obesity, hypothy- 3.  Review her diabetes treatment and any
roidism and drugs.
complications to date
Be aware that insulin is a contraindication only to driving
CASE 4.49  FITNESS TO DRIVE a heavy goods vehicle and not to other driving provided
she is not having hypoglycaemic episodes.

Candidate information 4.  Be alert to ideas, concerns  


and expectations
Role
Establish the discrepancy between her idea of normal
You are a doctor in the diabetes clinic. blood glucose readings and what is in fact normal.
Please read this summary.
5.  Reframe her ideas
Scenario
Explain that she should be aiming for a blood glucose
reading of 4–7 mmol / l, and that she is currently aiming
Re: Mrs Julie Blyth, aged 36 years too low and this is likely to be giving rise to symptoms.
Mrs Blyth has type 1 diabetes and takes insulin. She has Reassure her that resetting her target should not increase
problems with hypoglycaemic unawareness. She is still driving and her risk of long-term complications.
you are immediately concerned by this.
Your task is to explore the issue of driving with her and make
some recommendations.
6.  Keep it clear
Keep your explanation as clear as possible.

734
Case 4.50  Industrial injury benefits

7.  Repeat important information • A relevant disability is a condition that probably


If she is anxious this may be especially important. renders a person unsafe to drive, e.g. visual field
defect.
8.  Confirm understanding • A prospective disability is a progressive or
intermittent condition that may in future lead to a
Ensure that she understands that her blood glucose target relevant or prescribed disability, e.g. type 1 diabetes.
was too low.
May a patient with a prospective
9.  Encourage feedback and invite questions disability drive?
I appreciate we’ve covered rather a lot there. Is there anything
you would like me to go over again or are there any Yes, subject to medical review at defined intervals.
questions you would like to ask me?
What are the visual requirements for driving?
10.  Agree a way forward The visual acuity requirement for driving a private car is
Assure her that she will be able to drive, but only when based on the number plate test. A person should be able to
her blood glucoses are normal. Ensure that there are read in good light a number plate of letters or numbers
arrangements for treatment and follow-up. 79.4 mm high (binocular vision with correction for any
refractive error) at 20.5 m. This equates to a visual acuity of
between 6 / 9 and 6 / 12 on a Snellen chart. Failing to meet
Discussion the requirement constitutes a prescribed disability and a
Who is legally responsible for deciding licence must be refused or revoked. The minimum field of
vision for safe driving is at least 120° on the horizontal
whether a patient is unfit to drive?
measured by the Goldmann perimeter.
The Driver and Vehicle Licensing Agency (DVLA) is respon-
sible. The DVLA should be informed if a patient who holds What are the rules on driving after
a driving licence has a medical condition that now or at a seizure?
some future date may affect driving safety. The legal basis
Current legal rules for drivers (of non-heavy goods or
of fitness to drive lies in the Road Traffic Act of 1988 and
public transport vehicles) are that they must be seizure free
subsequent regulations
for more than 12 months or free of daytime seizures for
more than 3 years if seizures are purely nocturnal. These
When might it be a doctor’s responsibility rules apply on or off anti-epileptic drugs. Patients must not
to inform the DVLA? drive for 6 months following any dose adjustment. Patients
These are outlined in Box 4.46. have a legal duty to inform the DVLA of their condition.

Do you know the differences between a


prescribed disability, a relevant disability CASE 4.50  INDUSTRIAL INJURY
and a prospective disability? BENEFITS
• A prescribed disability is condition that legally bars
the holding of a driving licence unless certain Candidate information
conditions are met, e.g. epilepsy.
Role
Box 4.46  Situations when a doctor might need You are a doctor in the respiratory clinic.
to inform the Driver and Vehicle Licensing Please read this summary.
Agency (DVLA) Scenario
• If a patient lacks capacity to understand advice not to drive,
e.g. dementia. Re: Mr Andrew Hopkins, aged 77 years
• If a doctor is aware that a patient continues to drive against Mr Hopkins is a retired shipyard worker, and has been
medical advice, every reasonable step may be made to progressively breathless. His chest X-ray suggests pulmonary fibrosis
persuade the patient to stop, and this may include breaching and there is pleural thickening with calcified pleural plaques.
confidentially and speaking with a relative. If a doctor is unable Pulmonary function tests reveal moderate restrictive lung disease
to persuade, or becomes aware that such a patient is and a thoracic computed tomography scan confirms interstitial lung
continuing to drive, then the relevant medical information disease. All of this suggests asbestos-related lung disease. Mr
should be disclosed immediately to a medical adviser at the Hopkins is here for results and says that a friend told him he might
DVLA. Before disclosure, a patient should be informed of the claim industrial disablement benefit were asbestos the cause.
decision to do so, and afterwards informed by letter. The Your task is to explain the results and discuss the way forward
patient’s general practitioner should also be informed. for industrial disablement benefit.

735
Station |4| Communication skills and ethics

Your examiners will warn you when 12 minutes  information would help determine the level of benefit.
have elapsed. You have 14 minutes to communicate  Explain that benefit may be backdated up to 3 months.
with the patient / subject followed by 1 minute of reflec- Explain that he could appeal within 3 months, if he disa-
tion. There will then follow 5 minutes of discussion with greed with the verdict
the examiners. Do not take the history again except for
details that will help in your discussion with the 7.  Explain what to do in practice
patient / subject. You are not required to examine the
patient / subject. Explain that he should go to his local social security office
and collect form B1 (100Pn). This covers occupational
lung diseases. Say that you will write this down for him
Patient / subject information later.
Mr Andrew Hopkins is a 77-year-old retired shipyard
worker, exposed to asbestos for at least 25 years of his 8.  Explore any outstanding concerns
working life during handling of construction materials. He Ensure that you have addressed all of his concerns.
has been progressively breathless over the last 18 months
and a chest X-ray, pulmonary function tests and thoracic
9.  Invite questions
computed tomography scan have confirmed asbestos-
related lung disease. He is aware that industrial disable- Ask if he has any other questions.
ment benefit is available from the Department of Social
Security (DSS) and wishes to discuss this. 10.  Agree a way forward
Agree a clear course of action, ensuring that medical 
How to approach the case follow-up at clinic or with his general practitioner is in
place.
Communication skills (conduct of interview,
exploration and problem negotiation) and
ethics and law Discussion
1.  Introduction What types of asbestos lung disease
Introduce yourself and ask how he is. make a patient eligible for compensation?
These are listed in Box 4.47.
2.  Explain the results of tests
Explain that there is thickening of the lining of the lungs
Can compensation be considered for any
with plaques (hard patches) due to asbestos exposure, and
also scarring of the lungs almost certainly due to asbestos other lung diseases?
(asbestosis). Patients with a wide range of pneumoconioses, including
coal worker’s pneumoconiosis and silicosis, are eligible.
3.  Explore occupational history
Obtain details of different occupations and length of expo- Can patients sue previous employers?
sure with each. Yes, but many patients will have been exposed to asbestos
from various workplaces and it may be difficult to prove
4.  Explain the implications of the results that exposure in one workplace caused disease. Further,
Explain that treatment options are limited and that asbes- many employers no longer exist and legal expenses make
tosis tends to progress, albeit slowly, but reassure him that legal action untenable.
there is no evidence of lung cancer or cancer to the lining
of the lung (mesothelioma) which can occur in asbestos
lung disease.

5.  Establish knowledge of eligibility   Box 4.47  Asbestos lung disease eligible
for compensation for compensation
Ask if he aware of his eligibility to claim for industrial
• Mesothelioma
disablement benefit.
• Asbestosis
• Bilateral diffuse pleural thickening to a thickness of 5 mm or
6.  Explain the legal position more at any point on chest X-ray
Explain that people with asbestos lung disease are entitled • Primary lung cancer with evidence of asbestosis or diffuse
to compensation. Explain that a DSS medical officer pleural thickening
would visit to assess the degree of disability and that this

736
Further reading

FURTHER READING
Stroke Prevention in Atrial Fibrillation
Communication skills and ethics concerns about patients to the
II Study, 1994. Warfarin versus DVLA, disclosing records for
British Medical Association. Online. aspirin for prevention of
Available: www.bma.org.uk. financial and administrative
thromboembolism in atrial purposes, reporting gunshot and
Chochinov, H.M., 2007. Dignity and the fibrillation. Lancet 343, 687–691. knife wounds, disclosing
essence of medicine: the A, B, C and Wynne, H.A., 2003. Sensible prescribing information about serious
D of dignity conserving care. BMJ for older people. Clin Med 3, communicable diseases, disclosing
335, 184–187. 409–412. information for insurance,
General Medical Council. Online. employment and similar purposes,
Available: www.gmc.org. Communication in special disclosing information for education
Hope, T., 2004. Medical ethics; a very circumstances and training purposes, responding to
short introduction. Oxford criticism in the press.
Delayed discharge
University Press, Oxford. Lockwood, G., 2005. Confidentiality.
Hope, T., Savulescu, J., Hendrick, J., Langhorne, P., Taylor, G., Murray, G.,  
Medicine 33 (2), 8–11.
2008. Medical ethics and law: the et al., 2005. Early supported discharge
services for stroke patients: a Mental Capacity Act 2005. Department
core curriculum, revised second ed. of Health, London, Online.
Churchill Livingstone, Edinburgh. meta-analysis of individual patients’
data. Lancet 365 (9458), 501–506. Available: www.dh.gov.uk/.
Neighbour, R., 1987. The inner Mental Health Act 2006/8. Department
consultation. MTP Press, Lancaster Breaking bad news of Health, London. Online.
(revised 2nd edn, Radcliffe Available: www.dh.gov.uk/.
Publishing Ltd; 2004). Buckman, R., 1992. How to break bad
news. A guide for healthcare NHS Act 2006. Department of Health,
Tate, P., 1984. The doctor’s professionals. Pan Books, London. London. Online. Available:
communication handbook. Radcliffe www.dh.gov.uk/.../Legislation/
Buckman, R.A., 2005. Breaking bad
Press, Oxford (6th revised edn, Actsandbills/DH_064103.
news: the S-P-I-K-E-S strategy.
Radcliffe Publishing Ltd; 2009).
Commun Oncol 2, 138–142.
The Oxford Centre for Ethics and Kaye, P., 1996. Breaking bad news. A ten
End-of-life issues
Communication in Health Care step approach. EPL Publications, General Medical Council, 2010.
Practice. Online. Available: Northampton. Treatment and care towards the end
www.ethox.org.uk. of life: good practice in decision
Maguire, P., Faulkner, A., 1998. How to
The UK Clinical Ethics Network. Online. do it – communicate with cancer making. GMC, London. This GMC
Available: www.ethics- patients: handling bad news and guidance is based on long-
network.org.uk. difficult questions. BMJ 297, established ethical principles and
907–909. expands on the principles of good
practice in the GMC’s Good Medical
Discussing clinical management Maguire, P., Faulkner, A., Booth, K.,  
Practice and Consent: Patients and
et al., 1996. Helping cancer patients
Explaining an investigation Doctors Making Decisions Together
disclose their concerns. Eur J Cancer
(2008), and replaces Withholding
Adler, B.J., Stott, D.J., 2003. How far to 32A, 78–81.
and Withdrawing Life-Prolonging
investigate older people? Clin Med
Confidentiality, consent Treatments (2002). The guidance
3, 418–422.
takes account of, and is consistent
and capacity with, current law across the UK,
Discussing treatment Data Protection Act 1998. Online. including the laws on decision
Davies, E.C., Green, C.F., Taylor, S.,   Available: webarchive. making for patients who lack
et al., 2009. Adverse drug reactions nationalarchives.gov.uk/&plus;/ capacity (the Adults with Incapacity
in hospital in-patients: a prospective www.dh.gov.uk/en/. (Scotland) Act 2000 and the Mental
analysis of 3695 patient-episodes. General Medical Council, 2001. Consent Capacity Act 2005); the law
PLoS ONE 4 (2), e4439. Online. and confidentiality. GMC, London. prohibiting killing (including
Available: www.plosone.org/doi/ General Medical Council, 2004. euthanasia) and assisting suicide;
pone.0004439. Confidentiality: protecting and and the requirements of the Human
Medical Research Council Centre for providing information. GMC, Rights Act 1998.
Drug Safety Science. Online. London.
Available: www.mrc.ac.uk. General Medical Council, 2008. End of life and palliative care
Medicines and Healthcare products Consent: patients and doctors Department of Health, 2008. End of  
Regulatory Agency (MHRA). Online. making decisions together. GMC, life care strategy – promoting high
Available: www.mhra.gov.uk. London. quality care for all adults at the end
Pirmohamed, M., James, S., Meakin, S., General Medical Council, 2009. of life. Department of Health,
et al., 2004. Adverse drug reactions Confidentiality. GMC, London. London. Online. Available:
as a cause of admission to hospital: General Medical Council, 2009. www.dh.gov.uk/.
a prospective analysis of 18,820 Confidentiality: supplementary Jeffrey, D., 2005. The Assisted Dying for
patients. BMJ 329, 15–19. guidance. GMC, London. Reporting the Terminally Ill Bill: an

737
Station |4| Communication skills and ethics

inappropriate response to the Regnard, C., Randall, F., 2005. A Royal Liverpool Children’s Inquiry,
challenge of caring for the dying. J R framework for making advance 2001. HMSO, London.
Coll Physicians Edinb 35, 195–198. decisions on resuscitation. Clin Med
Kite, S., 2010. Advance care planning. 5, 354–360. Clinical governance
Clin Med 3, 275–278. General Medical Council, 2006.  
Living Matters. Dying Matters: A Palliative Withholding and withdrawing Good medical practice. GMC,
and End of Life Care Strategy for life-prolonging treatments London.
Adults in Northern Ireland (2010) British Medical Association, 2007. General Medical Council, 2006.
DHSSPS. Online. Available: www. Withholding and withdrawing Maintaining boundaries –  
pallcareni.net; www.dhsspsni.gov. life-prolonging medical treatment: guidance for doctors. GMC,  
uk/8555_palliative_final.pdf. guidance for decision making. BMA, London.
NHS End of life Care Programme, 2007. London. Online. Available: General Medical Council, 2006. Raising
Advance care planning: a guide for www.bma.org.uk. concerns about patient safety
health and social care staff. NHS, National Institute for Health and – guidance for doctors. GMC,
London, Online. Available: Clinical Excellence, 2006. Nutrition London.
www.endoflifecareforadults. support in adults: oral nutrition General Medical Council, 2006. The
Rachels, J., 1986. Active and passive support, enteral tube feeding and duties of a doctor registered with the
euthanasia. In: Singer, P. (Ed.), parenteral nutrition. NICE, London. General Medical Council. GMC,
Appliedethics. Oxford University Royal College of Physicians, London. Online. Available:
Press, Oxford. co-published with the British Society www.gmc-uk.org/...medical_practice/
Report to Minister for Health and Social of Gastroenterology, 2010. Oral duties_of_a_doctor.asp.
Services on Palliative Care Services, feeding difficulties and dilemmas: A General Medical Council, 2008. Good
2008. Online. Available: guide to practical care, particularly practice in prescribing medicines
www.wales.nhs.uk. towards the end of life. RCP, – guidance for doctors. GMC,
Royal College of Physicians, 2009. London. Online. Available: London.
Advance care planning. National www.rcplondon.ac.uk. General Medical Council, 2008.
guideline. RCP, London. Personal beliefs and medical practice
Scottish Government, 2008. Living and Percutaneous endoscopic – guidance for doctor. GMC,
dying well – a national action plan gastrostomy feeding London.
for end of life care in Scotland. Kurien, M., McAlindon, M.E., Westaby, General Medical Council, 2009.
Online. Available: www.scotland.  D., et al., 2010. Percutaneous Conflicts of interests. GMC,  
gov.uk. endoscopic gastrostomy (PEG) London.
Thorns, A., 2010. Ethical and legal issues feeding. BMJ 340, c2414. General Medical Council, 2010. Good
in end of life care. Clin Med 3, Nehanna, H.M., Moledina, J., Travis, J., practice in research and consent to
282–285. 2008. Refeeding syndrome: what it research. GMC, London.
is, and how to prevent and treat it. Greenhaugh, T., 1997. How to read a
Resuscitation status BMJ 333, 1495–1498. paper: the basics of evidence based
decision-making medicine. BMJ Books, London (4th
Vegetative state edn; Wiley-Blackwell; 2010).
A joint statement from the British
Medical Association, the Monti, M.M., Laureys, S., Owen, A.M.,
Resuscitation Council (UK) and the 2010. The vegetative state. BMJ 341, Other communication, ethical
Royal College of Nursing (October 292–296. and legal scenarios
2007). Online. Available: Genetic testing
www.bma.org.uk. Requesting an autopsy
Bristol Royal Infirmary, 2000. Interim Novak, M.J.U., Tabrizi, S.J., 2010.
A joint statement from the Royal
report: removal and retention of Huntington’s disease. BMJ 341,
College of Anaesthetists, the Royal
human material. Central Office of 34–40.
College of Physicians of London,  
the Intensive Care Society and   Information, London.
the Resuscitation Council (UK). National Institute for Health and Fitness to drive
Cardiopulmonary resuscitation Clinical Excellence, 2012. Donor Driver and Vehicle Licensing Agency. At
– standards for clinical practice   identification and consent for a glance guide to the current medical
and training. Online. Available: deceased organ donation. NICE, standards of fitness to drive.
www.resus.org.uk. London. www.dvla.gov.uk.

738

You might also like